Pre-Medical Expert Answers

From Student Doctor Network Wiki
Revision as of 20:39, 9 December 2019 by SDN Team 1 (talk | contribs)
Jump to navigation Jump to search

Below are some frequently asked questions that have been answered by SDN Pre-Medical experts.

If you have a question that is not answered here, please visit the SDN Confidential Consult forum to ask your question of our expert panel.

Contents

How do I find a doctor who would be willing to let me shadow them?

I am a freshman pre-medical student attending school at the University of Central Florida. I recently began volunteering at the Nemours Children's Hospital in the surgical unit as I hope to become a pediatric surgeon or neurosurgeon. Based on the advice of my counselor, I was hoping I would have some form of interaction with the surgeons and perhaps be able to ask them to allow me to shadow them. However, I rarely see surgeons where I volunteer (post-op recovery) and there aren't any other areas that would allow me more access to them. I don't know any surgeons or doctors I can shadow in my area. I am really worried about gaining a competitive amount of shadowing hours before I apply to medical schools. Do you have any suggestions on how to find a doctor that would be willing to let me shadow them? Do they require certain classes to be taken before shadowing? How would be the best way to approach them? Any help would be greatly appreciated!

mark-ER

Shadowing docs is a good idea, though shadowing neurosurgeons while an undergrad, may be too much too early. I have heard of undergrads who do physiology/joint experiments shadow orthopods, or general surgeons or even OB/Gyn, but cardiothorasic and neurosurg is typically off limits to undergrads. You have to know how to scrub in, know these cases in & out to get anything out of the shadowing experience. These are typically very intense procedures, with a lot at state, where time under anesthesia is carefully accounted for and there is less time to teach or grasp complicated concepts even for residents, or medical students, much less undergrads.

What I would recommend instead, if you are truly interested in peds/neurosurg, is to shadow a pediatric neurologist or a pediatric geneticist with interest in neurological diseases. Peds docs are generally more amendable to be shadowed, are generally quite friendly and responsive to a request for teaching. Some might even let you come in and with some preparation to a brief SOAP note as/before they are seeing the patient -- perhaps with a resident (?), a useful experience and something to talk about during med school interviews.

My approach to this would be to look for a residency program in pediatric neurology in your area, and email the program director asking to point you to a doc who would be amendable to having an undergraduate with an interest in neuroscience (don't say anything about neurosurgery, at least an the outset) to shadow you.

Oh and a few tips for shadowing: be humble, be on time (which means early) and be very careful/gentle in attempts to help/chime in with answers. Medicine has some things in common with the military hierarchy in this aspect -- speak when spoken to, and generally it is best to stay out of the way. It is better not to be remembered, or thought of as bland/unoffensive, than overeager/annoying or a know-it-all. Please remember, you know next to nothing at this point, and if you happen to know some esoteric factoid, do not chime in obtrusively, to avoid to be perceived to overshadow senior trainees like med students/residents. Again, not necessarily directed at the original poster, but rather to other ambitious undergrads who might be reading this reply further on down the line.


Do I still have a chance?

I am a sophomore undergrad student at uconn who just finished her third semester. I'm not doing so well in my science classes. First semester I took a genetics class and gen chem I for which I got an A- and a B respectively. Second semester I took gen chem II and got a B-. This semester I'm ending with a C+ for orgo I. I'm doing pretty well in all my other classes. I was talking to a paid advisor about game plans and at the beginning of this semester he said this orgo class was my make or break. Now that I've told him that I got a C+, he is saying that I still have a chance if I really apply myself. I can't tell if he's being serious or if he is just trying to get more money out of me. If I were somehow able to get my crap together, would I still have a chance at med school or should I look into other career options? Thanks!

tantacles

It is absolutely possible that you could still get into medical school. However, I would say that your performance in physics (the last class in the pre-med series) will be very important. In addition, if you are able to do well in upper level science courses, this will also serve you well and show that you are able to handle medical school.

That being said, I think it is ALWAYS a good idea to have a backup plan. Think about what you would do if not medicine as even many qualified applicants are not able to gain admission to medical school.


Can you get away without showing all the transcripts in the medical school application?

So my story goes like this: I completed my associates degree in Biology through a program in High school called Running Start, where I attained college credits during high school. I got a 3.41 GPA and a 2.9 Science GPA. Once I graduated from High school with my diploma and Associates degree, I went to a 4 year institution, WSU - Pullman campus. My first semester was terrible, got an F in organic chemistry, a C in Genetics and A- in General Physics, (Phy 101). Knowing that Medical schools average out GPA in the medical school application, how would medical schools know if I just started fresh from another university next semester and retook the same classes and got a better GPA? And I would pretend nothing happened, and finish the rest of my degree over at the new University. And I would send my community college transcripts and the new University i went to start fresh from. Yes, I take Financial aid from every where I went. But would I have to write down that I took classes at WSU for one semester? What if I dont report and just started fresh at a newer university?

tantacles

You are required to show your transcript for every undergraduate institution you attended. Medical schools use a service to determine if you have submitted all transcripts, so if you were not to submit this transcript, it is likely that no medical school would interview you as they would see that you had omitted a transcript.


Am I done for? How can I improve?

Hi everyone! I just finished my first semester of college as a pre-medical student. I went through so many family problems this semester that it was difficult for me to focus entirely on classes. There were deaths in the family and my mom needed caring for due to health issues, so I would have to frequently go back home. My grades suffered and I ended up getting a 3.25 GPA (B- in gen chem 1, B in Calculus 1, B+ in writing, A in Spanish III). I know that none of this is an excuse for bad grades because Med school will be a hundred times harder, but I want to know if it’s time to rule out med school for the future. As I said, this was my first semester of college, so I don’t know if a lot of this had to do with having bad study habits as well.

Also, how can I improve my studying habits and be better at subjects that I know for a fact I could’ve performed so much better in?

tantacles

A 3.25 GPA is not a bad GPA and leaves a lot of room for improvement. If you are able to improve your GPA and do well on the MCAT, your performance this semester should not hold you back.


MD chances... DO?

Hi All,

I am posting just out of curiosity... I am a first generation student with plans on attending MD/DO school. Currently I am just finishing up my fall semester of my junior year with a sci gpa of 3.54 and an overall gpa of 3.46. With 2 pre reqs to take ( Physics I and Calc), as well as numerous sci classes to fulfill my major for my spring semester and senior year (Bio emphasis on A&P/Minor in chem) what are my chances pending a decent MCAT score (taking this summer) on getting into a mid- lower tier MD school or decent DO school? I should also point out that I have 100+ hours of community service, 100+ clinical research hrs in the ED, multiple leadership experiences, and 20+ hrs of shadowing currently.

Side not question- Does being a dual national effect my chances?

tantacles

If you get a good MCAT score, you have a good chance of admission. My suggestion is that you purchase the MSAR and take a look at GPA and MCAT averages for many schools and try to target those schools (excluding state schools where few out of state students are accepted). My biggest suggestion is that you do your absolute best for the MCAT and not take it until you are truly ready to do well; that may mean taking a gap year so that you have time to study.

Being a dual national typically does not affect the chance of getting into medical school.


Asking for a letter of recommendation from a professor who was a postdoc when they taught me? Is that a bad idea or fine?

Hi, I was wondering if it would be bad to ask for a letter of recommendation from a professor who was a post doc when they taught me (they also taught me in a class that is highly recommended you take before applying to med school)?

mark-ER

Short answer = yes, it's a perfectly appropriate source for a letter of recommendation. Couple of caveats: is the person now a professor in the sciences (esp. biomedical sciences)? Is he/she employed by a reputable school (i.e. accredited college, university of medical school, preferrably in the USA)? How long ago did you closely work together and is that person going to remember you and write you a STRONG letter? The questions are more to spark your thinking bout "is this appropriate person to write a letter" (most likely yes), but is this the best person to serve as a reference (i.e. maybe there is someone better).


Medical School Application Advice

Hello, I have what I would consider a bit of a unique situation and am in need of some advice on how to proceed with applying to medical school.

I am graduating undergrad with a BS in Biomedical Sciences with a roundabout 3.3-3.4 GPA and a 520 MCAT score.

During my second and third years as an undergraduate, four of my immediate family members were diagnosed with incurable cancers that required extensive care. I was a part of my family's care team, and part of my duties was to transport those family members to chemo appointments located out of state. My family also maked their living from multiple family businesses, so part of my time was also used to assist in the upkeep of these businesses until additional help could be found.

I have also worked a full-time job in healthcare for all 4 of my years as an undergraduate. During this time, I logged nearly 6,000 clinical hours. In addition to working full time, I currently lead a research team of RN's and PharmD's with a focus on diabetic education that will lead to a change in hospital protocol of patient diabetic education and in-house insulin dispensing guidelines. This research will be published by mid-2019. Over the past two years, I have spent my Spring Breaks in third world countries, volunteering my acquired skills to help treat the severely medically underserved communities.

Given this rundown of my college history, I would like advice on how to go about applying for medical school. Am I a good candidate for MD schools, or should I just apply to DO?

Any advice is greatly appreciated.

mark-ER

I think as you explain your situation in a thoughtful, well-organized personal statement and make a solid, but not overaggresive efforts to contact admissions offices so your application is seen, you have a solid shot at MD schools. Your biggest worry should be that you get 'screened out' due to low GPA, either thru a computer algorithm or by someone pretty low on the totem pole of Admin offices. You have a solid MCAT score, a lot of good experiences related to medicine, and significant life challenges that helped you to grow -- these are many attributes we all want among our future docs. That being said, you should apply to quite a few schools (15-25 allopathic, at least 5 more DO schools), to make sure you get in somewhere. Good luck.


If I get my Associates in high school via Dual enrollment. Can I still get into med school?

My high school offers dual enrollment courses that I could take at the community college and earn my AA. Once I do that I can “transfer” to a university and start off as a junior. If I complete some pre-req courses at the community college instead of university will med schools accept the credits? Or see me as a weak candidate.

I’m also concerned about the MCAT and all my clinical experience in undergrad. Because I’ll be started as a junior... I’ll have to take the MCAT that year and will only have basically 1 year to complete clinical experience.

Anything will help! Thank you.

Pathdocmd

Community college credit is handled differently from school to school. It should not hurt you. Yes, not having as much time as a person that did a more traditional college route gives you less time to get clinical experience. For most medical schools, clinical experience is quality over quantity. Also, there is no rush to get into medical school. You will be doing medicine for the rest of your life. You might want to take some time and do something else after graduation.


What can I do after an interview to tell a school I would really like to go there?

I interviewed at a medical school on October 1st. They are my number one pick, and I'd love to go there because I feel like I fit well with their program. Additionally, attending this school wouldn't require me to move, I'd have family and friends close by, and it wouldn't force me into a long distance relationship.

I haven't heard anything back from them yet, and I believe they don't accept most students until spring. I really just hate sitting around waiting if there's something I could do to help impact the decision that will be fairly life-changing for me.

What, if anything, can I do after an interview to tell a school I would really like to go there/ improve my odds of getting in?

Pathdocmd

Sit tight. Calling or e-mailing them frequently will not help. Look at their policy for updates and follow it.


retake 512?

I am really contemplating a retake... I scored a 512 (127/128/127/130) when I took it the first time. I have a lower GPA (around 3.5) but I went to Stanford, had a strong upward trajectory, and was a varsity athlete there. I am currently doing full time cancer research and hope to have a publication soon. I have good ECs and recs. I know my score is pretty decent, but I really want to go to Baylor (TX resident) or another top 20 school. Any help would be greatly appreciated!! :)

Pathdocmd

With your MCAT, GPA, undergrad school with the time commitment of a varsity athlete, and state residency I would say no if you want to stay in TX (all schools). If you want to get into a "Top 20" (whatever that means) then you might want to try. If it was me, then I would not. You have a lot of good choices in TX.


Undergraduate degree

I'm currently in school pursuing my bachelors in nutrition. Will medical schools consider a nutrition degree? I am interested in med-school but wondering if my undergraduate would be accepted or not.

mark-ER

Sure, any degree from an accredited, 4 year degree-granting university should be OK. As long as you take the required pre-requisites, do well (high GPA, especially science GPA) and do well on the MCAT. You can certainly do a lot with a degree with nutrition (GI, endocrinology comes to mind). Good luck.


MDPHD application - Am I stronger after a gap year?

Hello All! I am reaching out to SDN because I am struggling to decide whether I should take a gap year before applying MD/PhD. I am unsure whether I have a strong enough application to apply following the completion of my Junior year, and after having read forum after forum here, I have decided to reach out to the community! I thank anyone in advance for taking the time to skim through this info - I have spent months researching this topic and have no-one to turn to for concrete advice, and thought it could help to reach out to the experts and see what feedback I get!

A little background on me:

   I am a white male who attends a small private liberal arts college (NOT University, meaning we have no graduate school), which is decently well-known (top 70 school - nothing too crazy).  I am a pre-med student completing a double major in Chemistry and Spanish Language & Literature, and double minoring in Math and Biochemistry. By the end of my junior year, my stats should look as follows:

3.85 cGPA 3.80 BCPM 4.00 Non-BCPM

  • Haven’t taken MCAT yet: planning for the spring of 2019, as I am currently studying abroad in Spain.

Research Experience: By the time of graduation, I will have 2 co-authored publications from an NIH funded laboratory, and will have 2.5 years of research experience in that lab.

Clinical Experience: - 1000+ hours working as a Certified Nurses Assistant, particularly working in Dementia Care Unit (1.5 years as of now) - Hospice Volunteer - Dental Intern (150 hours) assisted in filling cavities and procedures.

EC’s: too many to really list, but I will try - Student government (100+ Hours) - Manage a volunteer program in an elementary school and coordinate and manage volunteer participation. (500+ hours) - Started a newspaper and started an article writing mentorship program in an underprivileged district - Chemistry Teaching Assistant: Assist in lecture - Tutor for Chemistry (College and High School) - Completed an internship in Spain whilst studying abroad (clinical setting) - Deans award for Outstanding performance in Chemistry - Deans Award for Spanish - Alpha Lambda Delta Member - Health Professions Society and American Chemical Society

Essentially, I am wondering whether I should aim to take the MCAT in the spring and submit my application in June of 2019 (finishing my Junior Year), or if I should shoot for option 2:

Option 2:

Apply MD/PhD at the end of my senior year of college, and take a gap year afterwards. I will have the opportunity to apply to a related lab following graduation, where all of the graduates of my lab go to do research until they begin graduate school, at a top 30 school with a well established professor. By the end of graduation, my stats are likely to look something as follows:

cGPA: 3.90 BCPM: 3.85 Non-BCPM: 4.00

Further, I will have more research experience from the lab, however it will not be continuous experience. I go to a college that does not have a graduate school, so I will have 2.5 years of research at my college upon graduation with 2 publications, and will then have 1 year of research in the top 30 university lab.

I know this is difficult without an MCAT score to really gauge my competitively, but I get the impression that my application would not be competitive if I were to apply following the completion of my Junior year. Any thoughts are genuinely appreciated, because I don’t really know who to turn to for advice. Thank you very much - I really appreciate any genuine thoughts on the subject!

Mr.Smile12

I can't say that either option is a bad one, but in my opinion, I would personally want to understand why I would want to sacrifice 8 or more years of my life to get the MD/PhD (or DO/PhD). Research is an option without having to do both, so I would want to get a really good idea of why both would be essential. The usual path for people pursuing this route is to develop academic physician leaders, but you lose a lot in terms of potential lost income. In addition, seeking a career path outside of academia with both degrees -- while desirable -- is not often a common topic that is discussed (though that can change). If you are applying MD/PhD, you need to understand that while you may be applying to a few schools, all of the program directors know each other well. And once you join the MD/PhD ranks, you'll know so many of your peers going to other MD/PhD programs, so you have to be on top with your interview skills and selective on which institutions/departments you want to do your research work. Sure getting the 97th percentile across the board on your MCAT is a daunting task, but you need to be sure this option is the one you want, knowing you can probably do the same with just an MD/DO at a medical school that requires a research thesis or a Ph.D. without all the debt.


Should I retake MCAT? Score, 514 (BB 130, CP 131, CARS 127, PS 126)

Many people have told me not to retake, but I feel as if my score is skewed in PS. However, I did average a 514 on my three AAMC exams, and a 515 on my Next Step exams. I have a 4.0, from a small school, and I have three years of both clinical and research experience. Planning to apply next cycle, should I retake?

mark-ER

Short answer = no. Not worth the time/hassle. Yes your score is a bit skewed, but not overwhelmingly so and re-taking an exam always means a risk of a lower score. In short, I think you have a solid stats and a well-rounded application, and with a well-thought out application that is submitted early in next cycle, you should be able to get in. Good luck.


Should I double major in Biomedical Sciences and Psychology for Pre-med?

I am a junior in college with a major in Biomedical Sciences and a minor in Psychology. Should I up my minor to a major in psychology? I can handle it credits wise and was wondering how much of an impact it would make on my application to medical schools.

mark-ER

Short answer = minimal (likely none) impact. If you want to double major, then do it, but don't do it solely for a greater chance of getting accepted. Both bio and psych undergrad majors are pretty 'vanilla', and frankly your grades (esp. in science) and MCAT score and your passion/evidence of commitement to medicine is what carries weight, not a specific major.


Sufficient application for this cycle or improve application for next cycle?

I was complete to every medical school September 19. I have a 505 MCAT 127,127,126,125 with a 3.8 cGPA and a 3.85 sGPA. I have had one interview at CUSOM and was not offered on the seat. I double majored in Biology and Spanish and double minored in Chemistry and General Science at Bowling Green State University in Ohio. I studied abroad in Costa Rica for 9 months while also completing some volunteer work teaching English to local elementary school children for 30 hours. I achieved 160 hours of clinical experience by volunteering in a local clinic and hospital in La Ceiba, Honduras. I do not have any research nor do I have any further volunteer/clinical experience although I did shadow a plastic surgeon for 10 hours.

I completely believed my statistics were good enough to receive many interviews from DO schools and was being optimistic about receiving applications from MD schools. My question is do you think my application was strong or weak. I believe the thing that hurt me the most was submitting in September. I am already planning on improving my application for next cycle as I just received a full-time job as an inpatient pharmacy technician at UC medical center level 1 trauma. I also will try to shadow a DO doctor for 40 hours and volunteer to teach english to local latinos.

My overall question, because I have incorporated a lot of information in this post, is am I overreacting already from not receiving an acceptance? Is my application strong enough to receive an acceptance this cycle? and most importantly, if I do not get accepted, are the changes I plan to make strong enough to get me an acceptance if I apply June 1???????? If not what would you recommend I do to strengthen my application for the following cycle.

Thanks

mark-ER

I think submitting a bit late did hurt you, and overall you have a reasonably well-rounded application and you seem to show very strong commitment to a career in medicine. However (probably no surprise), my advice would be to re-focus and put all of your energy to studying and re-taking MCAT. Even a few extra point (say getting to a 510) would make a huge difference in your application. As it is, you will get screened-out by a lot of adcoms, with either a staff lower in the decision making process or no humans (algorithms) screening your application out.

So in short, make sure you get a good year of experience, study hard & re-take the MCAT, put together a solid personal statements and letters and submit on time to a range of schools (both DO and MD) and you should have a reasonable shot. Good luck.


whether I should continue in the pre-med track in college

I am a freshman in college right now and I am currently failing gen chem 1 with a 60%. In high school I took AP chem and passed the class but didn't pass the AP exam, I also took IB chem and did not pass that exam either. Should I drop this class and take it again next semester? Should I stay in the class and try to study harder? Or am I just not cut out for chemistry?

MusicDOc124

If your aim is for med school, it would be better to drop a class, even if it means a W, than to continue on and risk an F, a D, or even a C or low B.

With that said, retaking it later should only happen when you realize what's going wrong. You're in college taking gen chem and failing. You previously did not pass the AP exam. You MUST absolutely figure out what is inhibiting passing first. Are you not putting in the time? Are you not focused while studying and only studying passively? Are you studying the incorrect info? Do you need a tutor but haven't gotten one? Do you utilize office hours? These are all questions you need to ask yourself and then some. Retaking it would be pointless if you don't correct the error because you'll be right back in the same boat, but with 2 Ws for the same class, which is not good.

Depending on how much time you have before you can still drop for a W (assuming its past the point to drop without it on your transcript), consider getting a tutor who can help with the material and possibly study habits for that class and see if your grade starts to turn around while in the class currently.


Research for during undergrad

I am going to be applying for medical schools this coming Spring. My application is pretty solid throughout with one exception: I have no research. Research has never been something that I was passionate about, so I never pursued and opportunities. I was wondering how critical research is for medical schools, and whether it will make or break my application.

Mr.Smile12

It will depend on the schools where you are sending applications. You may want to talk with admissions staff at the schools on how important research is for applicants and whether they have experience with similar candidates with no research when it comes to getting them into residencies they desire. Some schools are built for students to do research as part of their education, but research doesn't mean just bench research.


3.8 cGPA from state school, 517 MCAT, low clinical experience

I have a 3.8 GPA and 517 MCAT, but not much clinical experience. What schools could be realistic with this?

mark-ER

Are you applying this cycle or next cycle? What's the tier of your undergrad/major combination (some schools notoriously grade inflate, and some majors are harder than others)... that being said, for a middle-of-the road state school U, and a typical biology major those are reasonable stats that should get you into at least a middle-tier allopathic (MD) school -- so for most places, that means your state school, or mid-lower tier private school, perhaps with the right letters/personal statement/motivation even a top tier school (one caveat being california, which is notoriously more difficult).

If you're applying for this cycle (to start in 2019), you are behind the ball -- apply NOW, as in TODAY, since most medical schools have rolling admissions and apply broadly to 25-35 schools, because you are so late. If you are applying for the next cycle (2020), then you can improve your profile with clinical volunteering/research and could be more selective, applying to 3-5 top tier (top 20 USNWR) schools, 5-10 middle tier schools and 5 or so lower tier and in-state schools, for a total of about 15-18 schools. Good luck.


Choosing Universities and Medical Schools

Hi! I'm a high school junior starting to examine and research specific universities and colleges more. I'm not 100% sold on becoming a doctor, but it is one of my career ideas. That said, I would love to study abroad and then return to get my medical degree, or even get both of my degrees abroad as I would love to live in Germany or Switzerland. But, how would I go about this? For instance, I'd like to attend Oxford for biochemistry, but is that a sufficient "pre-med" qualification for American universities, or other European universities? Thanks for all your help!

Mr.Smile12

For the US system, first things first: get your undergraduate education but keep your eyes open about where you can apply in the US for medical school. A word of warning that university education outside the US and Canada is different than within, so many of the admissions committees may require you to get your foreign degree evaluated to see if you have fulfilled requirements and desired courses. Now if you're that good that you can get a top-merit scholarship at a US university that could include foreign study abroad at Oxford or Germany, that's another option that might work better for you if you intend to apply to US medical schools.


i have the option to take biostats and also to take ellementary statistics. what is the difference between them? which one is easier to learn and which should be better for mcat and medical school

Mr.Smile12

To answer this question, check out the syllabi of both courses and compare to the desired competencies tested for on the MCAT and for entry to medical school. I suspect that there is no difference except in the types of examples you will be exposed to in the classes.


Should I put on my resume that I am currently donating to UNICEF USA & UNICEF WORLD for 3 years?

I have been constantly donating to the Unicef USA and Unicef World Wide. Should I put that on my resume or application to apply for medical school? Or Should I only put the volunteer and community service hours on my application?

Thank you

mark-ER

No, personal monetary donations to charities are not relevant and simply reflect your financial means to do so. But your hands-on activities, volunteering your time and effort are meaningful.


Is 11-12 hrs of class time + 10 hours of volunteer, community service, research, ect. (Per week) enough for Post Bac?

Hi there!

I am wondering my my semester load is competitive enough?

11 credits (16 hours of class time due to labs) +10 hours total of outside class volunteer, community service and volunteer, ect.

Per week.

Any insight would be so appreciated. Thanks!!!

mark-ER

Good start. Whether you do more and what you do depends on other aspects of your applicant profile. Have you already attempted MCAT and did you do well? If not, perhaps focusing extra outside energy on the MCAT may be your best best. If you have a good MCAT score and/or are generally confident/capable of doing well on standardized exams, consider working for $, perhaps getting a paramedic license, something medically relevant. Also, it would be helpful to do something medically relevant in your volunteer activities, perhaps serving as a pharmacy tech/organizer in a free clinic, medical translator if you speak another language, or psych/suicide helpline if you enjoy/intend to go into counseling. Otherwise, do what you enjoy -- if you like research, do research; if you like community outreach, do that.


Will an LOR from a med school clinical professor improve admission chances?

Will having a letter of recommendation from a clinical professor at a medical school help me get an interview or improve my admissions chances?

Long story short I did some shadowing under a physician alongside M3s doing their family practice rotation. The physician has been teaching clinical rotations for a top university for years and let me know that he wrote me a strong letter of recommendation for grad school. I've done about 60 hours of shadowing with him and will hopefully at least double that by the time I apply to med school. While I obviously know the letter wont hurt me, I'm curious if (and how much) it will help me. I've had a hard time finding information in this area.

Other relevant information about me: My GPA sucks (3.3), I have ADHD and I've worked basically full time throughout undergrad while being in school full time. I'm in grad school now, working slightly less, and doing a masters of Bio in pre-med. I haven't taken my MCAT but I think it'll be about ~510. I took intentionally VERY difficult classes in my undergrad that were specifically designed to emulate med school courses, and did decently well in them. (Think pathophys, cadaver anatomy, and medical physiology. After pathophys I was able to answer about 40% of the USMLE1 questions without having taken the rest of the M1 and M2 curriculum.) I don't think getting through med school will be a problem, but I do think getting in will be due to my GPA. BUT, I have a REALLY good LOR from the clinical professor I shadowed with for two weeks. He knows I'm capable, based on seeing me in a clinical setting alongside M3s and being able to keep up with (and occasionally teach) them. I am currently in grad school and I will be applying for med school in summer 2019 or 2020. If its 2020, I'll likely have a second physician letter very similar to the first, but with a different university affiliation.


1) Will an LOR like that be enough to get me an interview at the university he's affiliated with? Other less-prestigious universities?

2) Would asking him to contact the admission committee recommending that they grant me an interview, based on his belief that I would be a good fit for the university, be appropriate? (He's volunteered that has a very favorable opinion of me and my ability to do well in this field.) He is a professor at that school...

3) Will the letter help in general with attesting to my competence to do well in med school, despite my sub par GPA?

4) Any general advice for my situation?

TLDR: Will having a great letter of recommendation from a clinical professor at a medical school get an interview, even though my grades suck?

mark-ER

Short answer -- nope, letter of recommendation from a physician will NOT help to compensate for poor undergraduate grades. 3.3 GPA will get your 'filtered out' unless your MCAT is amazing (518 or above), and possibly with the exception of your in-state school, some private allopathic schools on the lower end of the prestige spectrum and DO schools. You will have to do a lot of legwork, contacting admissions to make sure you are not automatically screened out. Regardless my advice is to take a couple more science undergrad courses at school where you are currently doing your grad school coursework, to improve your GPA, study hard for MCAT and rock it.

Last (& this is more for others, than just yourself): little humility please. Putting thoughts in writing/speaking out of turn like " don't think getting through med school will be a problem" or "in a clinical setting alongside M3s and being able to keep up with (and occasionally teach) them" is not great. I have been both thru grad school and med school, and medicine is a lot more hierarchical, almost army-like atmosphere with a more-or-less clearly defined pecking order. You have to realize early you don't know anything, ask questions, maintain a very humble attitude. Showing off, or worse yet denigrating someone's intelligence, even to someone with "just" a high school diploma like a lab tech or nursing aid, can get you a bad eval and tank your chances at a top residency. Tough words, but better come now in an anonymous forum, than later when it actually matters.


What options do I have for pre-med?

I am a US citizen currently studying Adult Nursing (BSc) in the UK. During my nursing studies, I've found a keen interest in medicine, and I realize that I may or may not be too late to get into the medical field. I plan to finish my degree and move back to the US and hopefully get into a medical school there. I have a few options and a few questions.

Options: 1) Either work in the UK for 1 year as a nurse (to note down experience for med school application) and move back to the US, or apply straight to any pre-med programs in the US as soon as I graduate.

and

2) Attend a formal post-bacc pre-med program, take the MCAT, and apply to a medical school

or

3) Attend a regular bachelors program and make a DIY post-bacc and possibly drop out of the program after fulfilling all medicine prereq's

Questions: 1) When I apply for med schools, would they accept my UK BSc degree along with the US prereq courses? Or do I need to complete another bachelors in the US for me to qualify for admission into medicine?

2) For option 1, would having that 1 year of work experience increase my chances of admission into a med school? Or would it be a waste of time? Should I just move back to the US as soon as I graduate to do my med prereq's?

3) For option 3, is it necessary to finish the bachelors, or would I be able to drop out of the program as soon as I finish all prereq courses?

4) What other options do I have?

mark-ER

Great questions, good way to sum things up... Perhaps a bigger question is: does your UK degree in nursing translate (or can translate) into a nursing certificate in the USA? If so, it's (almost) a no-brainer: come back to the US, get experience in the US system and if you are not 'picky' geographically you can also earn a very solid salary. The most flexible approach would be to do a DYI post-bacc at your own pace, while working part time. But if you think you need the structure and the support, both from admissions counseling and MCAT prep standpoint, you could go the formal post-bacc route -- more expensive, less flexible, but a lot more structured. You know yourself. Last option you should not completely take off the table: clearly you have connections and work permit for Europe. Have you considered a US-style (4 year) European medical school like Trinity college (Ireland) or Jagiellonian University (Poland)? Would be a faster way to get thru medical school, especially if you intend to go into a primary care field, not uber-competitive subspecialty.


Could I matriculate to a US medical school from Canadian undergrad?

I am living abroad and I plan to attend a premedical program at a Canadian university, due to its vastly cheaper cost. As a US citizen, would I struggle to return to the States for medical school?

Similarly, if I, as a US citizen, attends a Canadian medical school, would it be more difficult for me to match into a residency program in the US as compared to a US medical school graduate? In essence, would my citizenship aid me in coming back home or would it not make a difference?

Thank you for your help.

mark-ER

Good questions -- first the short answer. As a US citizen, you will be absolutely fine, with either option you describe.

Now a bit longer: You should be able to get into a good medical school with a Canadian undergrad, as long as you take the requisite premed courses and do well on the MCAT. In fact some higher ranked Canadian schools like UBC, or McGill hold enough prestige to be a better option than most US schools (IVY leage, Stanford and a few top-notch big state schools like Michigan and UNC notwithstanding). Second option of doing both undergrad and medical school, then transfer to residency in the states is a bit more difficult, but only a little bit. You will technically be a US-citizen FMG, but (maybe with the exception of Australia, UK and Ireland), you will be heads and shoulders above the rest of FMGs and with appropriate grades, step1 scores and extracurriculars, you will NOT be held back from any residency, even competitive subspecialties. The best way to check is look at the match lists for canadian schools. Either way, I commend you on your approach -- costs ARE very important and if you reside in Canada and want to live there at least for the duration of your training, why not take advantage of the lower cost of training?


Post Bacc Choices to fit my situation

My GPA coming out of undergrad is a flat 3.0 as a biochemistry major with a 508 mCAT. I had a large upward trend of dipping below 2.0 my freshman year to bringing it up from there. I have over 200+ hours of volunteering and about 50-60 hours being clinical. I got accepted to post baccs in SFSU, Elms College, Chatham University and CSUEB, but I am not sure which one would best suit my needs. I want this post-bacc to simply improve my GPA to show to medical schools as continuation in my upward trend and be able to have some clinical experiences so that I can get a good letter of recommendation/committee letter of recommendation.

Mr.Smile12

I hope you asked the program directors at each institution what you want. I'm not sure about clinical experiences each program offers, so hopefully you picked to apply to those programs specifically for that reason. You should ask what the process is to get very strong letters of recommendation or program letters for your intended application.


Kaiser med school?

Hey y'all, I'm applying to med school next cycle (2020) and I'm interested in Kaiser permanente. I know they were planning to have their first class matriculate in 2019. Anyone know anything about this? Their website is silent and AAMC doesn't have any data on them. Just curious! Thanks!

mark-ER

According to modernhealthcare.com, they do not seem to be LCME accredited (yet), but Kaiser is a very well-respected medical institution, should not be an issue. They have a board of directors, and I am sure the facilities are being prepared. It should be a good choice for the very crowded California medical school 'market', to start with at least, they should have a similar profile as say Loma Linda, or as a better comp Geisinger which is openining a new medical school in Scranton, PA. You can always 'tweet' at them to ask or just keep checking for updates, their twitter seems active.


I need help forming a school list; I am open to both MD and DO

Here are my current stats, and I need help forming a school list that I will be competitive in. I am currently a Tennessee resident.

GPA: 3.85 Mcat hasn't been taken yet, but hoping for at least 508

Extracurriculars: -200 hours ram clinic, 10 different locations -350 hours at VA working with physical therapy -3500 hours as paid medical transporter at hospital for 3.5 years (does this count as clinical experience?) -250 hours at Second Harvest Food Distribution -100 Hours as a Children's Ministry Leader at church -50 hours at a homeless shelter -100 hrs tutoring underprivileged youth -Underprivileged youth Mentor Program for two years -Tutoring For College underprivileged students 2 years (paid) -250 hours Shadowing Experience 8 specialties Involved in three clubs and working on research I have had memorable experiences and good talking points about all activities above.

mark-ER

Thanks for a relatively thorough run down. Couple of things that may influence the suggested list: are you URM (or conversely overrepresented)? What's the pedigree of your undergrad (ivy/top 10, top 50 USNWR college, major known liberal arts college, state schools, etc)? Also, I am not sure what you mean by "working on" research -- do you have any independent project(s) that you can easily talk about to an expert, and potentially publications (or en-route to publication)? And of course actual MCAT score. With a balanced 515 or above, you will get in somewhere and you should be more than OK with applying to a range of allopathic schools only, including in-state public, regional private and a few 'reaches'. Unless your MCAT score is absolutely stellar (520 and above) and you have research publications and some other major accomplishments (Rhodes scholar, amazing athlete, or overcame major life issues), I'd probably stay out of top-10 schools (Harvards, Hopkins, Stanfords) U Tennesee is hard to beat in terms of cost/value equation, but the triad of Tennessee, Vandy and Pittsburgh is realistically what you should be aiming for. Create a list of 15-20 schools, put those on the top. Good luck with your applications (presumably next cycle).


pre med Post Bac program or another bachelors degree?

Hi all,

I have been accepted into a pre-med program at Fordham University but now I am contemplating attending. I’m not sure if I should do the pre med program or get another bachelors degree.

My previous BFA is in fashion design (8 years ago) and I graduated with a 3.2. I am wondering if I would be better off doing another 4 year program (maybe majoring in biology) since my gpa was so low. A pre med Post Bac program will only raise my gpa so much.

Any advice on what would be the best path to increase my chances of getting into med school would be great. Thank you!!!

mark-ER

Good questions -- Post-bac adds on to your existing grades, so it would average out with your 3.2. 3.2 is low, but fixable. If you get straight As or close to it, you can bring up your average to 3.5 and with a good MCAT score (above 510) you ought to be able to get in. So a 2 year post-bacc is probably the way to go for you, not to mention the support and guidance you receive, which is not always a given in a traditional 4-year biology or even pre-med pathway at many colleges. So if you are truly dedicated to becoming a physician (though please keep an open mind to other allied health professions -- genetic counselor, PA, podiatrist, nutritionist), the abbreviated 2-year pathway thru a formal post-bacc may be the best route for a non-traditional student coming from a non-science background, such as yourself. Good luck.


Should I withdraw from my SMP? Horrible second exam

Can’t really say much. hurricane Florence really held me back. I had to travel across the country and it led me to do terrible on my second exam. It was a straight F. I was already borderline C+ to begin with after the first exam but this F really set me back. I ask you guys should I keep on co tinhig and hope for a B which at this point looks impossible with two exams to go or should I drop out and retry elsewhere ? I get that SMP are dress rehearsal for medical schools and they predict success but I sincerely get the material and am interested. I just got wrecked by this second exam and nd now it seems like my entire dream is in jeopardy. (U cum gpa- 3.03, sc gpa - 2.89 MCAT -507

I applied for 10 different DO schools. Would getting a C in one of the classes in SMP end me ? Should I withdraw before I do more damage ?

Mr.Smile12

I am sorry to hear things didn't go so well with you. I think withdrawing is going to do a lot of damage to any timeline that you may want to get you into medical school in the near future while staying with the SMP and getting support from the faculty about your situation would give you some chance. You have to shape up or else there really isn't much more hope to give you outside of getting into medical school at a Caribbean program.


Should I retake my class or withdraw? Pt. 2

Hey, I just wanted to let mark-ER that the class is not combined or averaged. Is it bad to think a C in the lab is okay? Cause if I end up with the A in the lecture and a C in the lab, that's still a 3.5 gpa overall. I would rather just go into physics 2 with a pretty solid gpa, then have to withdraw and just take the physics 1 lab in the spring. My school doesn't allow me to go into physics 2 lecture without the physics 1 lab (makes sense). So I'll talk to my lab professor in a few weeks about my grades in his lab. Anyways, Thank you for the advice.

mark-ER

Yes, your thinking is right. As long as you are learning the material and you are getting what you want from the class, just stick with it. Also, if you are seeing bad grades in the lab component, try to address it NOW not in a couple of weeks. The rest of my advice still applies -- read ahead, work extra hard... it might mean more time dedicated to this class than you might want, but that's how the cookie crumbles sometimes.


Should I retake my class or withdraw?

Hey guys, I am a little more than halfway through the semester and am questioning if I should withdraw from my physics I lab. I am doing very well in my lecture, I got a 95 on my last exam, but the lab has been really difficult. The professor is pretty bad. He is a few weeks ahead of lecture, so its kind of difficult to understand what the material is and how to apply it. His grade for the entire lab is 70% lab reports (I have an 85 average on.) and 30% quizzes (like a 63 average on.). I can probably get at least a C in the lab, but it seems to be kind of a bummer especially if I get an A in the lecture. Does it look weird to the adcoms if I get a C in lab but an A in lecture? should I suck it up and take C or withdraw from the lab? Thank you

mark-ER

Does lab & lecture components of the course get averaged together? Basically what do you see on your transcript? Double check that, and go to the prof who teaches the lab and/or TA office hours and gently/carefully try to understand what they want you to get out of the lab component of the course. No grade-grubbing just a calm, rational conversation. If that fails, you should adjust and focus more energy on the class perhaps by reading ahead, if the lab component is ahead of the lecture component. Others may be doing better on the quizzes for the lab component, because they may have had the course material covered perviously (i.e. AP Physics in high school). You will see this plenty both in college & in real life and it is not always fair. Bottom line is: you are still earning solid if not stellar grades. If you are getting something out of the class (i.e. learning the material) and are just frustrated with the lab component/arbitrary nature of the quizzes, stick thru it, adjust and control what you can. Yes, you can withdraw and it's better to have a W than a C. But whether a W is better than a B is questionable and then you have to repeat the course again.


3.53 GPA 3.48 Science GPA 508 MCAT I haev no interviews yet and have not heard back from 18 MD schools

18 MD Schools have not replied, does that mean I am still being considered or do some not tell you you are rejected? DO you think I have a chance for an MD program with my numbers? I applied mainly in NY, Pa & NJ. Is there anything I can do now to help my current application? thank you

mark-ER

Hard to say. When did you apply and did you get/quickly fill out secondary applications (?)... I would not worry yet if you applied in September, but perhaps be somewhat concerned if you did so promptly on the first possible day (or week).

Your stats and MCAT are (slightly) below average, I also hope you applied to the right tier of medical schools (mid-lower prestige, local to your residency status) and perhaps a few DO schools as well. Perhaps other red flags (academic, something in the letters of recommendation or not enough exposure to healthcare thru volunteering or research)? Either way, do not panic, but act to control what you can control.

Briefly, gently and VERY respectfully inquire about your status at a handful (max 5) of medical schools where you most hope to attend. Depending on the feedback (almost regardless, unless you happen to get 3+ interviews pending), I would expand your application list by another 10-12 schools. Time is of the essence, most medical schools have rolling admissions. Last but not least, if one of your recommenders has strong ties to the medical school where you are applying, it may now be time to pull on that lever. Good luck.

Other things you can do to help (since you asked): did you have a major accomplishment in the meanwhile -- higher MCAT score on a retake, a 1st author publication, a Rhodes/Marshall scholarship or something of that nature? Then definitely, as you reach out to adcoms, update them/your profile.


Trying to make decisions about whether it would be worth it to still apply to medical school in the cycle or wait

I am currently a junior majoring in Neuroscience and History with a minor in Chemistry and possible Religious Studies Minor.

My GPA through my university is a 3.56

Going by AMCAS GPA policies: My BCPM GPA is a 3.24 My All Other GPA is a 3.7 My Running Overall GPA is 3.43

I have yet to take the MCAT, but I am still planning to do that in early May 2019. I am currently trying to decide if I have a chance at getting into medical school in the next cycle or if I should wait and do a post-bacc year-long program and apply in 2020. I am attempting to get into a clinical research lab right now and waiting to hear back on a clinical volunteer program I've applied to where I sit and comfort people who are close to dying and do not want to die alone. I have been extremely active in my service fraternity and have an excess of volunteer hours through them. I am working on gaining shadowing hours. I currently attend an OOS school for undergrad and would like to stay up north for medical school.

I would like to know if it is worth it for me to even consider applying next cycle for medical school? Like do people get into medical school with GPA's like mine? Also any alternative suggestions for how to make myself a better applicant especially if you believe I should wait a year or two?

mark-ER

Ultimately it depends on your MCAT score (aim for at least 515) and what your ultimate career goal happens to be.

If your ultimate career goal is to get into a competitive subspecialty, you probably ought to spend another year or more in a post-bacc to improve your science and overall GPA to 3.5-3.7, respectively. Postbac would also have the benefit of dedicated support and prep time for the MCAT.

Your alternative would be to apply to A LOT (and I mean 25-35) lower tier private allopathic schools and all your in-state schools as well as all DO schools. If you continue your exposure to volunteering/healthcare exposure opportunities, continue to earn excellent grades (all As from now on, especially in science) and get an excellent MCAT score and apply early you ought to get in somewhere. So if your ultimate goal is a less competitive subspecialty, just be on top of things and apply early next year; don't forget opportunity cost -- you 'save' a year in training, less expenses (tuition/living expenses) and earn an extra year of physician salary.


cGPA 3.2, sGPA 3.0, MCAT 509 - Plan of action advise please

I graduated in May of 2018 with cGPA 3.2, sGPA 3.0 from a top 5 liberal arts college with a non-science major. I applied with GRE scores [95% in verbal, Quantitative 55%, Writing 95%] in June for post bacc. I was rejected from Loyola [both programs], and Tufts post bacc.

I took my MCAT and got 509 [127,127,127,128]. Please help me with suggestions on what to do next. I am currently shadowing doctors, volunteering in hospitals. I have applied for a data coordinator job with a professor in a medical university - this includes clinical research and some shadowing].

My current plan is:

1) Apply for post bacc to start Fall of 2019. If so what programs would you recommend? 2) Should I apply for DO programs next June as I am doing post bacc [provided I get in to one]? 3) Should I wait until I finish my post bacc and them apply for DO and a few MD programs?

Or should I apply for a few DO programs now [is it too late]?

Please advise. Thanks in advance.

Mr.Smile12

Hopefully your institution has a health professions advisor or office. It's tough to figure out what your options are just based on the information you provide. I would say that you should look into DO programs in general but it's not clear what you know about osteopathic medicine or why that route would best suit why you want to go into medicine in the first place.


Low GPA/High MCAT Applicant; MD or focus on DO

Hey all, thanks in advance for taking the time to look at my question!

I am a current Master's student with a low undergrad GPA (3.26 cGPA, 3.24sGPA) and a high MCAT (520; 130/129/131/130). I am applying next cycle and want to get an idea of where I should be applying and if I should spend the time applying MD or just put maximum effort into DO schools.

I went to a SUNY undergrad school and graduated with a B.S. in Biochem with the above GPAs and am going to another SUNY school to do my MS in biochem as well. My Master's is research focused with a thesis requirement and I have a 3.9 in the classes I have taken there (25 credits worth, all biochem/analytical chem/ cell bio/ stuff like that).

I have ~3,000 hours direct patient care experience as a volunteer critical care technician on an ambulance (A bit lower on the totem pole than a paramedic but can do most of the same treatments and skills). Also significant leadership and teaching experience here, was elected to office and in charge of all training.

I have also been doing research in cancer biology for a little over 3 years now and that has resulted in 2 second author literature reviews and one second author manuscript in a mid-level journal. Plus I'll have my thesis to point to by the time I apply if that is taken into account?

My shadowing experience is scarce, only ~35 hours with a plastic surgeon whom I met serendipitously and he wrote me a great LOR. 

For non-clinical volunteering I began a social-support group on campus for students battling drug addiction (not in recovery myself but sibling died of an overdose so it is something I'm passionate about). That's it for my non-clinical volunteering though.

I guess I just want to know if my undergrad GPA will cause me to get tossed at MD programs and if I'm better off focusing my time/energy to DO programs . I was thinking maybe I can just apply to SUNY MD schools and have the rest of my apps be to DO programs I'm interested in. Any insight from experts would be greatly appreciated. Thank you all in advance!!

mark-ER

If your ultimate career goal is to get into a competitive subspecialty, I think you ought to spend another year or more in a post-bacc or in your case (with excellent MCAT score already in hand), a self-designed post-bacc, taking several science classes at a local college (above a community college, but does not have to be USNWR ranked). You don't have to be in school full time, maybe 3 classes in the fall, 2 in the spring should do it, but to get there given how late it is, you will have to spend 2018 and 2019 doing this, and apply 2020. In the background you can continue to work (and cover your expenses) in your current laboratory, maybe even get another manuscript published (1st author would help a lot, 2nd/3rd authors don't account for as much). This may be the best approach if your ultimate career goal is a competitive subspecialty practice.

Your alternative would be to apply to A LOT (and I mean 25-35) lower tier private allopathic schools and all your in-state schools as well as all DO schools. Given your background and MCAT score, if you asked this question in early August/late July I would have said pretty confidently with enough applications you could get in somewhere (likely DO, but perhaps at least 1 lower tier MD ??) But at this point with rolling admissions, it may be too late.

So my suggestion is to perhaps split the difference between the 2 plans, particularly if your ultimate goal is to be an internist, peds, psych or family doc (something less competitive when residency applications roll around). Enroll in 1 undergraduate science class (ASAP, like today), then a 2-3 undergraduate classes in the spring. With those, assuming A's, you should boost your GPA to 3.4 or so, which will help and probably be OK to get into a lower tier allopathic school next cycle (2019), given your application includes enough schools (20-25). Remember it is the undergraduate GPA that counts (& at this point might lead to a 'screen out' at a lot of allopathic schools) Just be on top of things and apply early next year.


Career advice: aspiring scientist interested cancer, cognition, and nutrition

My background:

I am currently a junior in college majoring in Neuroscience and Philosophy. I am heavily involved in basic science research (neurodegenerative diseases) and have been published in peer-reviewed journals/presented many abstracts. Currently, I intend to go to medical school for an MD/PhD and am studying for the MCAT (medical college admission test) in preparation for next year's application cycle.

My Question(s):

I am writing for advice on potential career options for someone who is interested in working at the intersection of neuroscience/cognition, nutrition, and cancer. In particular, my interests lie in how certain dietary changes/habits can enhance cognition and/or sensitize cancer to better, more efficient treatment modalities. Is there anyone who has/is currently working in this area? Is it possible to be productive in something that is so niche-specific?

As a follow up question, I have been working as an assistant in my current lab for 2 years (freshman/sophomore year) and have developed a strong relationship with my PI. She has written me countless letters for various scholarships/fellowships and will continue to do so as I progress in my career. That being said, my research interests have definitely changed over the course of my undergraduate career, and I am more interested in nutrition then I am Parkinson's/Alzheimer's. Would it be unethical to search for other labs this "deep" into college? I am currently working on a large project in the lab and am independent, so I would be leaving a lot behind.

Any advice and insight would be greatly appreciated!!

mark-ER

Short answer: it is productive to be very focused and "niche", but not so early in your career. Keep an open mind, do not restrict yourself to one line of thinking. In fact, vast majority (myself included) of research-oriented physicians benefit from multiple lines of investigation, often changing throughout their careers. What you are interested in undergraduate != interests in grad school != interests as a pos-doc/resident and may not necessarily = research you pursue as faculty.

So that also answers your second questions = no, stick with your current lab. Be productive, get something out of it (poster, publication and most of all learn how to do science; and large part of science is persistence). You can search for a lab that fits your niche during your grad-school phase interviews and lab rotations that are a part of MD/PhD.

Overall, interesting major combo & sounds like MD/PhD (esp. neuroscience/psych) may be the right track for you. Just remember it is a LOOOOONG road. Most people do not get established as an independent (though still junior) investigator until their late 30s/early 40s, whereas your MD-only colleagues will be earning a 'real doctor' salary with real doctor medical responsibilities for 8-10 years by then. Make sure you are OK with that; check out physician-scientist forum here on SDN.


Math and the MCATS

Hi there!

I will be attending a Post Bac pre med program at Fordham university next fall.

This is a huge career change for me as I graduated with a BFA in fashion design 8 years ago. The only math I had to take for my degree was algebra 1 and 2 but since it has been so long I have forgotten everything. To help I am receiving private tutoring in both prior to my program start at Fordham to prepare me better for Calculus 1. BUT I came across a list of all the math I must know for the MCATS, which involves geometry and trigonometry. I never learned either. I was thinking of maybe getting my BA in biology instead that way I can take all these math classes as cutting corners scares me. I guess my question is if I should continue with my plan to attend the pre med program or apply to a four year program? Or is the math I will be using for all my sciences classes and the MCAT simple enough to learn without having to take geometry and trigonometry?

Any guidance would be so appreciated. Thanks!!!!

mark-ER

Ok, difficult question -- it depends on where you are headed and what you want both out of your preparation and your ultimate career. Yes, many (some argue most) medical subspecialties do NOT require calculus and or more advanced maths in general, but some (say critical care, nephrology) may. If your goal is to be a general practitioner, you can be OK or more than OK with "just" calculus at a post-bacc, perhaps supplemented with a statistics class. Overall, I think your approach is right -- hire a tutor and take a "remedial" (I hate that word) pre-calculus class on your own to fill out your holes, and do your best in calc in post-bacc. If you are just worried about the entrance exam, relax; there are VERY, VERY few questions on the MCAT that require snap thinking/top of your head trig, geometry or even calc.


Nurse to D.O. with WGU MSN

I am a registered nurse who has been practicing for eight years. I've worked in SICU settings and medical/surgical settings. I am currently working as a nurse educator and finishing my MSN in nursing education from Western Governors University. I am curious with WGU being a competency based school and my odd background if there were any medical school (osteopathic or allopathic) that would be interested in me. I plan on starting to study for the MCAT after I am done with my MSN degree but I didn't want to waste my time if it's unlikely to get into a school.

Mr.Smile12

The best thing to do is connect with admissions staff at some of the medical schools you are interested in. The fact you have been pursuing nursing (graduate degree) would interest some medical schools just because you'd have an insight into clinic that is hardly covered in medical education. I'm sure that you may find some of the schools more accepting of your journey.


3.9 GPA from Cornell and a 513 MCAT?

I ended up doing a lot worse on my MCAT (129,126,130,128) than all my practice scores but have a 3.9 total and science GPA in addition to a lot of clinical, community service, and research hours. Is my MCAT okay for this cycle?

Experience: 1,200 hours of research 1,200 clinical work/exposure orgo tutor and a few other clubs/jobs Honor society for top 10% of Cornell students

mark-ER

You are still OK, but (as you suspect) it will now be a bit more of an uphill climb. You should NOT be screened out, except among the most competitive medical schools. As such, you ought to avoid the top 10, though you could throw your hat into the ring at one or two places in the top 25 (for instance for Cornell Medical school). In your shoes I would focus on solid in-state institutions where you maintain home residency (that may or may not be New York), geographically close to your undergrad/home (maybe UBuffalo, Pitt), schools where you have some personal attachment to and mentors who can advocate for you. You also ought to put in an application into a few lower tier schools, and perhaps up your number of applications to 25 or so.

You still really should be able to get in somewhere, but time is now of the essence -- remember most medical schools have rolling admissions and interviews have started.


3.96 cGPA, 4.00 sGPA, 501 MCAT, nontraditional applicant, Asian: Chances for MD school?

Hello, So I applied to 15 MD schools for the 2018-2019 cycle... here are my stats:

cGPA 3.96, 4.00 sGPA, 501 (128/123/127/123) MCAT (I know, it's horrible)

I am Asian American; Registered Nurse with over two years of experience working in post-surgical care and intermediate critical care; have shadowed two MDs; Participated and currently participating in an Organic Chemistry research lab, Private tutor for the biochemical sciences; volunteered for 5 years as a Mandarin-English translator; Participated in Medical/Nursing Mission Trips to Taiwan & Honduras.

I know my MCAT score is a very low score, which was lower than what I scored on my practice MCATs. Is there any possibility that I would stand a chance of acceptance at any MD schools?

mark-ER

There is always a chance, but due to your low MCAT score you will be "screened out" at a lot of places. At this point, best chance is with DO schools, though you can try a few lower tier allopathic schools as well.

However, my overall recommendation would be to retake the MCAT. If you scored well in prior MCAT practice tests and there is a reason why you think you performed poorly (got sick, really nervous), I would re-schedule MCAT and just retake it as soon as possible. Even a few point improvement (say to 510) would do wonders for your chances. Otherwise your application is quite good.


How to approach a doctor when asking for clinical observation opportunities?

Hello!

I recently reached out to two doctors (in different specialties) asking if I could contact them and ask questions about the medical field and their experiences. I would eventually like to ask them if they would be open to being shadowed or if they may refer me to someone. However, I would like to go about this in the correct manner and do not know how to approach the situation or how to ask. I have both of their contact phone numbers and email addresses. They both told me calling is the best way to get in touch. Should I ask over the phone or is this too informal? I have never before approached or had contact with doctors in this manner before and this could potentially be my first clinical observation if all works out.

My primary questions are these: When I call, what sorts of questions should I ask the doctors? Should I ask about clinical observation/shadowing over the phone (and in the first conversation)? Or should I simply introduce myself ask a few questions and then ask them if I can call them again in the future (ie build the relationship a little more before asking)?

mark-ER

Very good questions, I wish more students in your shoes would ask. First point is to find someone who would be amendable. Probably the best way to do so is under the auspices of a formal internship program (my hospital has one of these for undergraduates over the summer). Alternatively, perhaps you know someone in your family or a friend who would be amendable, or who can recommend a colleague, perhaps even in a specialty of particular interest to you. It is always best to have an 'in' rather than call or email a doc out of the blue. If you do not have any other connections, I recommend singing up to volunteer with the hospital's volunteer department... it does not have to be an onerous commitment, maybe every other week x 6 months. You could be delivering flowers, be a mobile library, greeter, etc. and can get contacts with docs and other allied medical professionals (podiatrists, etc) that way.

In terms of how to ask and what to ask -- informal phone call or conversation can be OK, as long as you have actually previously met the person. If not, email is probably best at least for the younger docs. The few questions you may need to know or ask ahead of time -- do I need a short white coat or is business casual attire ok? Should I furnish you with a HIPAA training certificate? (you can do a quick 30min training course online, google it). Mention that you are low-mainenance, will keep in the shadow with not a peep out (as you should), but then ask if this is a specific clinic day (say endocrinology, child neuro, etc) and if so if they recommend a brief topical review you could read ahead of time so you can get as much as possible out of your day.

Those are my recommendations; they are on a bit more ambitious/outgoing side, but still within line of not being too overeager (it's a fine line to walk, as you will discover in your medical training).


Grad work in sciences or something non-science?

I am trying to decide on what post-bacc education I should explore. I am considering two things, on different sides of the spectrum and I'm not sure which would look better on a med-school application. On the one hand, I am considering doing an online masters program in biology. On the other hand, I'm considering getting an associates degree in American Sign Language (ASL) Interpreting and becoming a nationally certified interpreter, and potentially going on to do some work in medical interpreting. The former I know is good because it would show that I can handle the upper level science courses, but I've also heard that you should have something on your application/resume that most everyone else wouldn't have, which is where I think the ASL work could be beneficial. I have taken ASL classes in the past, and have been told by a Deaf instructor that I am very good and should considering getting into interpreting. Note: In the meantime, I will hopefully be working as an EMT no matter what (I just got my certification and have been applying) so that will give me some hands on experience in the health field.

mark-ER

You are right, both options can be appealing. Remember, masters (no matter if it's online or not) will not help your undergraduate GPA, but may improve your knowledge base and help you to have the time to study for the MCAT. On the flip side, unless you are interested in ASL in and of itself, that is not necessarily going to be helpful there. So it's more of a matter of what you find more interesting and how you plan to study for the MCAT. Good luck.


Pre-Med from PR where to apply?

I'm from the University of Puerto Rico. Most of the students who pursue their undergrad education in PR don't apply to US schools. I don't want to limit myself to the schools on the Island. I received my MCAT score and got a 512 (129/125/129/129), I have a 4.0 GPA, research experience and volunteer work. I'm Mostly lacking clinical exposure but am currently volunteering at a hospital. I've worked since my second year. I want to apply to US schools but don't know which are best in terms of considering out of state and Hispanic students.

I've applied to 2 schools in PR and 3 in the US: UCONN Health (did a summer program there), University of Central Florida and Penn State (long shot). Was thinking about applying to Quinnipiac or Miami Miller School. Want to apply to a really great school even if my chances are low but don't know which school is the best fit given my application and background. I'm still waiting for the AMCAS verification so I can't apply to any school with a deadline in October; yes, I'm late for applying I know.

mark-ER

Short answer apply where you feel comfortable living (if that's florida, then great) and where you have a solid chance to get in. That means where they take good number of out-of-state students and where they value diversity/Spanish-speaking skills. I would focus more on the former, than the latter. So no in-state public university (no University of Michigan, no UNC, no UIowa, etc), but perhaps mostly mid-lower tier private schools and maybe a sprinkling of higher tier/reach schools, though perhaps not the very tip, top-10. Among mid-lower tier think along the lines of Rosalind Franklin, Loyola, Wayne State, Jefferson, etc. Among reach schools maybe Vanderbilt or UChicago. Overall, I think you have the right attitude -- apply broadly, you never know where you are going to get in. Your stats and experiences are solid, though not super-star level, hence my recommendations. Read and research SDN forums and again as you point out do so quickly, as it is already nearly October and most medical schools have rolling admissions, so you are behind. Good luck.


Looking to hear postbacc stories

Hello - I am a current medical student at the University of Southern California. I mentor undergraduate students and have realized that it's really challenging to find information on post-bacc programs.

I would love you hear your perspective: +What's the value of post-bacc? Does it really help your odds of getting into med school? +What do students do after post-bacc if they don't get into medical school? +How should students think about picking a postbacc program that's right for them?


tantacles

Post-baccalaureate programs tend to be valuable for students who have not taken their pre-requisite courses for medical school as well as for students who want to boost their undergraduate GPAs after a poor performance.

Many students who do these programs go into other health professional fields if they do not get into medical school, like dentistry, PA school, or nursing.

The best post-baccalaureate program is one in which you have the appropriate support to get the highest grades possible. For many people, location plays a role in this as having family close by helps them to do better in their programs. Some post-baccalaureate programs also offer automatic interviews based on grades in the program and MCAT score.

Ultimately, all post-baccalaureate programs serve the same purpose - allowing students to take science courses for undergraduate credit to bolster their applications. The decision of which post-bacc program to choose is personal, but if you do well, the post-bacc program will be a help and not a hindrance to getting into medical school.


Help with school list

I have a 3.9 gpa and a 517 mcat. I am from Pennsylvania. Can you help me generate an appropriate school list?

tantacles

I would suggest that you purchase the MSAR:

https://apps.aamc.org/msar-ui/#/landing

This app will help you choose schools that have similar GPAs and MCAT scores to your own, and it will also provide information on how many out of state students are accepted to state schools. SDN also has myriad resources. I would suggest that you purchase the MSAR, choose 15-20 schools that seem in range with some schools that have averages below yours and some above yours and then repost either here or on the forums. Generating a school list will be quite difficult without knowing more of your preferences, but your MCAT score and GPA will not lock you out of any schools.


Secondary App Mistake

Hello! I was checking up on my application statuses on the different schools I applied to and I realized that I filled out the residency statement for UVM incorrectly. Is it okay for me to email their admissions with the corrected statement as an application update or should I just leave it as is? (The app asks for a specific format for this question that could only be found in the FAQ, which I obviously did not look at prior to submitting my app. It asks you to write the town you lived in at every age and the population of the town, whereas I simply wrote I lived in x state for my entire life.) Thanks for your help!

tantacles

I would honestly let this be. Don't draw attention to the fact that you filled out this portion of the application incorrectly. They likely know that you filled out this part incorrectly. I would imagine that if the rest of your application is up to snuff, the school will reach out to you and have you correct the information if they are interested. If your application is not up to snuff, they will likely not seek you out to update this information.


Complete secondary even though course requirements aren't met?

So, shame on me, I applied to a medical school without doing the proper research and I didn't realize that I didn't meet all of their course requirements.

Weirdly, however, I received a secondary from them (though it took much longer for it come than the other ones), and when filling out the course requirements tab on their secondary is when I found out I am missing some courses!

Is it worth it to still submit that secondary? Or am I literally just throwing money down the drain?

mark-ER

Yes, worth it IF (big if), you are only missing one or two pre-reqs that you can finish up in the next semester or two, before your matriculation into medical school. You took the MCAT, right? -- you must have had most of the pre-reqs, so you are missing one or two (?) Another approach would be to reach out to the admissions office for the medical school where you did receive a secondary.


Is it late to apply to DO schools now ( mid september) ?

I have a GPA of 3.5 and MCAT score of 506 and graduated from UCLA with a BS in Psychobiology . I have multiple volunteering and work experience however my application lacks research publication although I have worked as an RA in 2 different labs.

Mr.Smile12

While I don't think it is technically too late, you are playing against the odds. Many schools have already set up their interview dates through the end of November. Now you can still apply because schools are always trying to fill their classes up through the spring, but you should make sure everything is in order with your letters and transcripts to AACOMAS.


What are my chances?

I am a 5th year health sciences student. I have been through a lot since beginning undergrad. I have lost both of my parents, a grandparent, and went through a bad breakup and have suffered with anxiety and depression. My grades are not the best. I am going to have to retake at least 3 science courses I scored D's in. I am willing to apply to DO schools, my overall GPA is above a 3.0. It's hard to know exactly what it is because I transferred and my new school calculates my GPA based on the courses I've taken there, so right now according to their calculation I have a 3.5. I have took some courses at community college and have attended 5 different institutions due to everything I've been through. I have not taken the MCAT as yet. I am hoping to raise my science GPA to a ~3.3 by the time I graduate next fall. Will this be enough if I do well on the MCAT?

Mr.Smile12

I guess it all depends on what you mean by doing well on the MCAT and the schools you are trying to apply to. In general though you should probably focus on those courses you need to repeat and do your own AACOMAS calculations. You should also ask each school about their GPA considerations before you even apply.


Finishing undergrad overseas

My husband is in the military and we’re going to be moving to Germany. I have one year of my undergrad done- is it possible to finish a bachelors in biology in Germany and still have a good chance at getting into a US med school?

mark-ER

Yes (meaning you can proceed this way) and yes (Western European schools with teaching in English are viewed well by US med school ADComs). I answered a similar question, so search archives, but briefly: search out a school where teaching is done in English; make sure the school is appropriately accredited and you can transfer credits (confirm this BEFORE enrolling); ask if anyone else before you with a similar background (American Expat) was able to successfully navigate the medical school admission process with a degree from their university, then reach out to him/her. You should be able to get answers to these questions from the admissions office at most European universities. From what I read (I have not been back to Europe for 10+ years), German universities are becoming more desirable to international students, so much so that they are becoming crowded with higher student:faculty ratio and a lot of burn-out among faculty. If you are a type of person who benefits from more individualized attention (rather than self-learner), seek out solid but lower profile/prestige schools (for instance Frankfurt am Oder. Ultimately, how the whole thing plays out is largely up to you -- you will have to be very self-motivated and do well on the MCAT. Overall, the experience can be very good, but it is what you make of it and preparation is a big part of setting yourself up for success -- so kudos to you for asking the right questions.


Post Bacc Pre-Med program as an International Student

I have always wanted to become a doctor. Couldn't go to med school then owing to a couple of reasons. So pursued electronics engineering (2014 pass out) then did my MBA(2016 pass out) worked as a business analyst for a year and half. Now decided to do what I 've always wanted to. And now Post bacc Pre Med is my only way in. I would like to know if its a plausible thing as an International student? and what are my chances? If there is a fighting chance, how and where do I get things started? Has anybody taken the route of Post Bacc Pre-Med ->MCAT -> Med school as an International student?

Mr.Smile12

Without really addressing your citizenship status (US permanent resident or citizen or other eligible status for financial aid and residency standing), it could be a waste of your time and money until you have a timeline established for your status change. Otherwise, I know there should be a forum full of international students who have gotten US status and gotten into medical school (or there should be) whose advice you can seek.


Connections to Med School's communities

On many secondaries, medical schools often ask you to describe your connection to their school, their community, the state that it's in, etc (beyond "why do you want to go here?"). If you actually have NO connection (didn't go to school there, not born there, no relatives there, never visited, parents didn't go to school there, literally not a single connection), is there a strategic way to respond to these questions? Just to emphasize, the question isn't why do you want to come here, where you could do research on the school, the area, and the state to come up with your reason, the question is what connections do you have. If you don't have any, do you leave it blank? Or is there a way to go about this?

Thanks.

Mr.Smile12

Most applicants are upfront with their selection strategies when it comes to picking medical schools. After all there are 140-180+ different choices. Maybe the word "connection" is the semantic issue; usually I find that schools want to know why you would be attracted to their program, and so that would be the way I would suggest anyone think about their answer to such a question. You don't want to come across that you are just doing a random search.


Should I go to school to become an MA before going to medical school in order to get clinical experience?

Right now, I attend a community college, have a 3.7 GPA with 60+ credit hours. I am at a cross-roads in regards to where I go after I graduate in the upcoming spring semester, and I am researching universities to transfer to. One college that I've been seriously looking at, has a fantastic medical school but to increase my chances of getting in, I need to do undergraduate research. To get that opportunity, I'll need to get into the honors college. The good news is, I qualify because of my GPA so far, the bad news, only 22 students are accepted. This is path 1.

This is path 2: I will also need clinical experience to increase my chances of getting in. Medical assistants are needed badly in the U.S. right now. I could go to medical assistant school, become an MA, and get about a year's worth of clinical experience under my belt before I apply to medical school. But I can't do both college and MA school concurrently; I don't have enough time or money. I'm tempted to choose the first option, because my education was already delayed about a year and a half due to two surgeries. Clinical experience would be nice, but it's not a requirement to get into medical school. However, going to MA school might be the more logical option. What should I do? Go to MA school first, or continue my education as a pre-med student?

Thanks in advance.

mark-ER

The first option is better. Not enormously better, but better. 3.7 GPA is pretty solid, though you did not break out your science GPA (hopefully that's higher) and you do not have MCAT score on file yet. Many medical school AdComs will like to see you succeed academically at a higher-tier 4-year institution. Yes, research and clinical experiences are helpful, but you can get hit those checkboxes as extracurricular activities and during summers. Research experience can be done outside of an honors program (google UROP, there are plenty of undergrad research experiences likely within your desired undergraduate institution and outside). So yes, try to get into the honors program, but your option #2 may be to apply more broadly to 4-year colleges within your desired geographic area, rather than MA school. Of course if you have other extenuating circumstances (need to earn a salary or support a family), the calculus might change; but if your main goal at this time is to succeed in school, try for the best 4-year undergraduate institution you can get into, at a reasonable cost (that is often your state's flagship, i.e. "University of ").


could i get into medschool?

currently just became a junior, looking to see how i stack up and how/ where i can make improvements in my overall resume

-GPA: 3.62 (deans list 4 semesters) 1 year microbiology research 2 years bioengineering research (presented nationally) 6 months pharmacoepidemiology research (to be published) 1 year neuroscience research (to be presented this year)

club president, newsletter founder +150 hours shadowing experience +200 hours volunteering

just became a hospice volunteer

lost? could i be doing more? should i be?

Mr.Smile12

Hopefully you have been in touch with your health professions advisors to give you a good idea of where you stand for the moment. It is always too challenging to gauge where anyone stands without knowing how well you do on the MCAT and what schools you would be targeting.


Lots of clinical experience no volunteering

I have a 3.7 gpa I’m taking the mcat in the spring. About 1000 hours of direct patient care, 4 semesters of research in a psychology lab, 8 hours of shadowing and no volunteering. This summer I will be applying and I am wondering what kind of numbers I should strive for with my shadowing and volunteering? I will still be taking classes and mcat prepping. I am not sure how much schools value volunteering when you have a decent amount of medical experience so if you could shed some light on that I would appreciate it.

Pathdocmd

I would try to get something in, but your 1000 hours of patient care is more impressive to most committees. Try to find something you are interested in and it does not have to be medically related. Showing is good, but a lot isn't better. Besides, the patient care (active) work trumps shadowing (passive).


What should I do? I have lost all hope!

I am very worried about my upcoming school year going into second year, I did not do as well as I had hoped to do in my first year of University. My GPA was only a 3.1, making it seem next to impossible to raise it up. I am an extreme procrastinator with a touch of depression, that makes it difficult to succeed in my courses. If I study I know that I am capable of achieving high 80s and 90s, but the issue is commitment for me. I was wondering if anyone had a similar undergraduate GPA for their freshman year, and how did you bring it up? I want to raise my overall GPA to a 3.9 by the end of my 3rd year, and do not know how to achieve this with the current grades that I have!

mark-ER

Certainly not all hope is lost, some people have a harder time adjusting to college than others. You may also have very real, legitimate mental health issues (please strongly consider counseling, a lot of colleges have professional, free or nearly free, anonymous services). Couple lines of approach/advice: lower down your course load (you can still finish in 4 years, even if it may require an extra class in the summer), see if there is an academic help consult service available (my undergrad had a service like that; I used it my first semester, and by year 3 I became a tutor), and/or find a group of like-minded individuals both to help in studying/understanding concepts and motivation. Good luck, you can do it!


Help with School List

I have a 3.9 GPA (science and overall) and I recently scored a 516 mcat and I am from New York. Please help me make a school list of target schools

mark-ER

Very strong numbers (congrats) and quick question right to the point (I like it).

In your position, I would put schools into 3 tiers: likely acceptance, mid-tier and reach. The number of schools on the list may depend on how much you can spend on application/travel to interviews & how sure you want to be of the 'right fit'. I think anything more than 25 is overkill -- after a certain point you will get tired of the process and basically all US medical allopathic schools will get you where you want to be (albeit at a different costs (#1 consideration for many), geographic location and training environment). For likely acceptance tier, include all instate schools and maybe 2-3 lower tier private schools that take out of state residents in a geographic location that is desirable to you (SUNY Downstate, SUNY Stony Brook, SUNY Buffalo, Albany and for private schools maybe RUSH if you like big cities). Middle tier should be your largest group, and here again mid-higher prestige in-state and private schools in your general area (Einstein, Icahn, Thomas Jefferson, University of Rochester, Brown, Hofstra, Tufts, Rutgers, Drexel, University of Rochester, Pitt) and maybe a couple where you might want to live geographically (Georgetown, Uchicago, Vandy). Then last group is high tier/prestige schools or places where you really want to live geographically but have not ties to and/or you are out of state (very low chance). That's for you to decide, but keep that last group to fewer than 5 -- it's hard to get rejections and though your stats are very strong they are not absolutely stellar, so places like Harvard, Hopkins or Stanford are going to be quite a reach (on the other hand, you only live once).

The only schools I would really avoid, because it is a waste of your time are in-state schools that only take 2-3 out of state candidates per class and even then only with VERY strong family ties (alumni, lived there their whole lives), places like UHawaii or UC San Diego. If you want those locales, you can try them as a resident, much fewer geographic consideration in the selection process.


Should I do a post-bac or should I retake my science courses in my undergrad years?

Before I get into my current crisis, I am going to talk about my background.

My ultimate goal is to become a physician. In my freshman and sophomore year at my university, I was declared as a Chemistry major and it has been a difficult road. I took some prerequisites for medical school and my experience has caused me to stumble a lot. I passed one year of physics, one year of Calculus, and the first general chemistry class. However, I also took Gen Bio I & II, and Gen Chem II. I withdrew out of all of those courses (I was around a D in all of them, so I just took a W).

Now, I'm going into my junior year. I decided to major in Religious Studies & Spanish so I can still be on track to graduate on time. The Religious Studies major will help me as a prospective physician because each culture has their own beliefs, especially relating to medicine. In addition, Spanish is the second most-spoken language in the world. It's significant to have a connection with my patients rather than having a translator in the room.

I only have two options:

Option A: Should I still fit in one science course each semester? This method could work because I would only focus on one science course. I took two sciences in a semester twice and I had to drop out one other science course. I can complete my two majors on time and do this. Maybe I'll understand the science courses better since I have already taken them.

-OR-

Option B: I recently went to a Pre-Health conference at Stanford University and I was introduced to a post-bac program. It's available to those who have an interest in medicine and already have a university degree. It's a one year program where I can strictly focus on and retake my science courses, prepare for the MCAT, and shadow physicians at hospitals/clinics. It's about $30,000 for an entire year but it guarantees admission to medical school. Would this be a better option so I don't have to stress about retaking all of my prereqs?


I already took these courses and here are my grades: Physics I (B+) & Physics II (B+) Calculus I (C) & Calculus II (C) General Chemistry I (C)

I'm missing the following courses in order to complete the prerequisites: General Biology I (W) General Biology II (W) General Chemistry II (W) Organic Chem I (not taken) Organic Chem II (not taken)

Basically, My science GPA is 2.58 and my overall is 3.2

Besides school, I work a part-time job to support my family and I volunteer every week at Kaiser Permanente to gain experience in a healthcare setting.

Any thoughts? All comments will be appreciated! I just need a sense of direction. Thanks in advance!!!!

mark-ER

First, I commend you for taking the time to "stop" and re-evaluate your position, even if it required taking withdrawals. If you were to continue along this trend, you could have done irreparable damage to your grades and dashed your hopes of pursuing your career track in medicine. That being said, you still have quite a steep hill to climb. If lowering your course work and taking one class in science at a time, focusing on your other courses as well (it does not seem you have had straight As in your major either, given your overall GPA is also quite low), then that is exactly what you should do.

You will likely have to follow BOTH plan A and plan B, since you will have to do some re-mediation. I would suggest really putting your head down to the grindstone and trying to aim for AT LEAST a B+ in General chemistry and O-chem and one or two advanced biology courses, aiming to finish college in 4 years with your selected major. Then retake Calculus (at least calc 1), and take an additional biology course (perhaps a genetics) or a stats course in a post bacc -- either specifically designed post doc (like the one you mention for stanford), or self-designed one, while studying for MCAT and doing some research.

So I know that is a lot, but that is what is expected from successful US medical applicants. So also please seek help and academic counseling, talk to your classmates when choosing courses/professors and if you see yourself struggling drop the course BEFORE you have to take a W. Lastly, please do not despair or think of yourself as a failure if you cannot reach these lofty 'ideal' goals -- consider DO schools and consider shadowing other allied health professionals -- genetic counselors, PAs, CRNPs, etc. They all do important meaningful work, with a lot of autonomy but less hoops to jump through and a lot lower student loan debt than physicians.

Last, but not least upon re-reading your longish question -- NO pre-health post-bacc program can "guarantee" admission into medical school. You still have to have the grades, MCAT score and show yourself to be capable to handle the workload from both intellectual (and perhaps more importantly) self-motivation/delayed gratification standpoints.


Will it matter if the total units at my UC are different than the total units I receive at the CC?

I am planning to retake an organic chemistry class at a community college since my institution will only allow me to retake the class if I got below a C-. However, the total units I received at my college for the entire course will be different than the total units I'll receive from my community college, even though those classes are equivalent.

Will this cause any complications? The organic chemistry course at my quarter school was 15 units (5 units each quarter) total but at my community college it'll be 12 units (6 units each semester).

Also, If I don't finish the entire organic chemistry course on time before I apply, will this hurt my application since organic chemistry is a pre-requisite? I have taken it already but I just want to retake it since I didn't do so well.

Pathdocmd

You have to check with the individual medical school, but if it is a "full year" of either semester or quarter hours, it should be fine. The other thing to look into is if one school includes the lab in the lecture course or if the lab is a separate course. This may be the cause of the credit hour discrepancy. I would take the lab with the course


C's in Organic Chemistry but retake policy is only for anything below a C-. Can I still retake at a community college?

Hello, I have recently graduated from undergrad from a UC earning a B.S. and am planning to apply to med school next year in 2019. My GPA is on the lower side because I had accumulated 3 Cs in organic chemistry (with a C- in one of the courses in the series). These Cs are the only Cs I have gotten during my undergrad and I would love to retake the entire series to prove that I can handle the course load.

In my upper division biochemistry courses I had gotten all B's and above. I would like to retake the entire Ochem series so that I can redeem myself. Additionally, I understand that some medical schools require a C and above so my C- would be dragging my whole application down. However, at my institution you can only retake a class if you had gotten below a C- so I couldn't retake the course at my UC. With this policy though, can I still retake the course at a community college? Also, my UC was on a quarter system and the community colleges around me are all semester schools so would that cause any complications? How would I submit my grades on the AMCAS/AAMCOAS if I did retake the course would this class be part of a "post-bacc?"

If I cannot retake the course because of my UC's policy, then what are some options I have in terms of this C-. I would still be able to apply possibly to some schools with my C+'s but my C- would really be the limiting factor here since it might not even be accepted at all. What would I do in that case?

Any guidance or insight towards my problem would be of great help and appreciation. Thank you!

tantacles

You can absolutely retake these courses at a community college. AMCAS has no rules about which courses you retake, though all grades are factored into your GPA, which means your C- and the new grade you got would be averaged together. AAMCOAS, however, allows for grade replacement. The course would simply be factored into your GPA as an undergraduate course taken after you graduated.

If a school has a policy that they do not accept a C-and you choose not to retake the course, it is unlikely you will receive an interview from that school.


How does grade inflation/deflation affect medical school admissions?

I am currently a California HS senior enrolled full-time at a local community college, trying to decide where to transfer. I have been told GPA, MCAT, and extra curriculars are the most important things to admission committees, but do they consider the rigor of the undergrad curriculum? For example, would an applied mathematics major at UCLA with a 3.8 GPA be viewed in the same light as a gender studies major at a small private school with a 3.8 GPA?

Mr.Smile12

Most medical schools will consider where classes were taken or where the student was enrolled. I'm not sure to what extent they consider majors, though I suspect they will focus on common core courses that are generally required or preferred.


Pre med dance major

Hello, I am currently signed up to be a biology and dance major. However, due to my dance schedule it looks like the double major at my school will be impossible unless I take 18 credit hours a semester and take a fifth year. I don’t have money for a fifth year. I am on scholarship for dance so I would like to not drop the dance major. Will I still be able to get into medical school as a dance major? Also, due to my dance schedule, I will have to take some of my pre requisite courses for medical school over the summer at a community college. Will medical schools accept these credits?

mark-ER

Yes, you can get in with a dance major. As long as you do your prerequisites (talk with the pre-health advisers at your school), do well in your science classes and on the MCAT you will be OK. In fact, keeping up with a busy dance schedule would be a testament to your tenacity and following your passion and add diversity to the medical school class. As another example, per recent match list at harvard medical school, one person decided to forego residency for a year to pursue a stint with a professional dance company.


Adding more medical schools

If my AMCAS application has been verified since June and I have added additional schools, how long will it take me to receive those secondaries?

mark-ER

Pretty quick at this point. Expect within 1-2 weeks (remember, medical schools have rolling admissions).


Should I do a post-bacc?

Hello!

I was wondering if you had any information that would help me decide whether or not I should look into post-bacc programs (aimed toward academic record enhancers) or retaking 2-3 classes while working full time.

I graduated college with a 3.46 cGPA, 3.42 sGPA. I have 3.5 years of biomedical research experience (2 yrs full time since graduating undergrad). I am taking the MCATs in early 2019, so I can't comment on my performance on that exam.

I given this information, I wanted to know if I should just focus on post-bacc programs, or look into re-taking or taking additional classes to boost my low GPA. Given that a post-bacc is a significant financial commitment, I was hoping to get some advice or guidance.

Thank you in advance!

mark-ER

Your GPA is low, but something you can fix/improve/overcome with a couple of good, upper level classes and a very good MCAT score. So it really comes to knowing yourself -- if you think you can stick to a regular, strong study schedule every evening (for the MCAT), while working and taking classes part time, go for it. Also do you have access to your undergraduate schools med school advising/admissions staff? Choosing a do-it-yourself (DIY) route you would be saving yourself a good chunk of change in tuition, but loosing out on potential structure, help with admissions, exam and potentially link-in into medical schools. Good luck.


Obtained a unique experience after primary submission- should I send a letter to update schools?

This September I am taking a course to be a certified level 1 wine sommelier. While this is completely unrelated to medicine, I am unsure if schools might find it interesting enough to distinguish me from other applicants. What do you think?

mark-ER

No, not worth a formal letter update ,but it is something you could update as part of a larger update process for secondaries (plus publications, new sets of grades, etc). It also makes for an interesting snippet of conversation at interviews, particularly in & around wine country (California)? It certainly doesn't hurt, but also doesn't help that much (sorry to burst your bubble; there are a LOT of very interesting/dedicated people applying to medical schools.


Forgot an Institutional Action AMCAS Application

I just realized I forgot that I have an institutional action and did not mark "yes" on my AMCAS application. I was talking with a friend when the subject came up and it sparked my memory that I was written up the first week of college freshman year for alcohol under the age of 21 occurring in the dorms. It is on my conduct records under ResLife. I went to go change the answer on my application but it is "Ready for Review" so I can't change that section. The incident is not on my official transcript and I was never legally in trouble. I am not sure what I should do

Mr.Smile12

I don't know if you have begun to answer secondary applications yet, but you probably will have another opportunity to properly disclose this institutional action there. If you are interviewed, you should probably make sure you disclose it again to the admissions staff. At any rate, you can probably email the schools you have applied to now and let them know of the oversight.


Should I gain experience in a lab or work on an animal farm?

I am going into my senior year at IU and a professor offered me a job at a sheep and cow farm not to far away from campus, but she can also put me in touch with some professors who could put me in an animal behavior lab that studies the aggression of male hamsters when testing their territorial boundaries (run by the head of the Biology department at IU), or one that studies dung beetles in microbiomes.

I'm not sure which one would be more worth getting experience because lab experience with the head of the biology department would be great, but also working on a farm would be nice to relieve some stress and I believe it would make me stand out from other applicants.

tantacles

The best thing to do is to do the thing that you are most interested in. Generally, this will give you better topics to speak about at interviews and will make you stand out from other applicants.


Pre-Med Post bacc for International applicants

Has anybody taken the route of pre-medical post-baccalaureate, then MCAT, and finally make it to medical school in the States as an international applicant? If so help me with how and where do I start?

tantacles

I would strongly suggest that you ask about this in the pre-medical forums; there is a large number of applicants from all walks of life who may be able to share their stories and help you find a solution. You can find this forum at the below link:

https://forums.studentdoctor.net/forums/pre-medical-md.10/


Will DO schools or post Bachs accept this?

(Im a PA resident btw) So I took most of my prereqs and I’m headed into senior year, just have physics 2 and biochem left. I want to focus on my EC’s since I’m gonna be taking a Gap year. I had a lot going on during the spring semester junior year and got a C- in orgo. I really really don’t want to retake it because I absolutely hate my school’s chemistry department and it makes me super depressed whenever I’m in there. I want to apply mainly to both post Bach’s with linkages and DO schools after my senior year and go with whatever I can get into. My question is.... can I get into programs with that C- or do I have to retake it? If I can, please name which programs I have a chance at?

tantacles

It is possible that some programs will overlook a C-, but I would check individual programs' websites. Many programs state that a C or better is required for admission, so you may want to consider retaking organic chemistry regardless.


extra semester community college

I am currently in my third semester at my local CC. I could technically finish my AS degree next semester, but I am thinking about taking an extra semester before I transfer. This would be for personal reasons not academic. If I did take an extra semester I would complete all the necessary prerequitsities before transferring. My major at the university would either be neuroscience or physics. Considering I would be taking a ton of upper level science courses to finish my degree, would it hurt me to spend an extra semester at CC before transferring?

mark-ER

You are asking if it would hurt in terms of attaining medical school admission, right? If so, no extra 1 semester of CC is not going to hurt. You are just finishing your associates and transfering to a full 4-year university, so you are just completing your course of study. But please be advised, many medical school admissions committee do take CC grades/credits with a grain of salt, so please excel in your advanced coursework at the 4 year university and ace your MCAT. That being said, for many (? most) people, the financial/social benefits of 2 years of CC arguably outweigh doing full 4 years at a university, so good for you.


Technical MCAT Scoring Question & Advice on my MCAT Situation

1) Is it possible that by virtue of how AAMC standardizes MCAT scores that, say, if your cohort of test takers does worse on average than other groups that you get a higher score? Like, when I take my exam, who all is considered in scoring? Taking in August. Is it possible that summer test takers are more/less prepared than say January takers and so scores are different just by who I test with? AAMC Official Guide doesn't make it clear when the groups "reset" or if it depends on which version of the test you had, etc., other than alluding to scores resetting in 2018. In that case, if I have being standardized against a smaller sample, won't it be more likely that my score is de-/inflated compared to what it would be if it were standardized against years and years of this particular administration of the exam?

2) When I took the first time I didn't study & got a 501. I only practiced by taking AAMC FL 1 & 2, and got 501 & 502 resp. but w/a **** ton of variance in section scores (123-128). Now, I retook FL 1 & 2 and got more consistent, higher scores, 512, 515 resp. with only one 126 & the rest >127....

BUT... when I took FL 3 I only got a 511 (128/127/130/126). So I'm worried, were the scores inflated bc/I had taken FL1&2 before? (I didn't remember the answers or anything, only was familiar with some of the CARS passages but that's it). I test Friday and want a 510 and am nervous AF. Just gonna be memorizing and reviewing my weakest concepts until then I guess. Any advice? Esp. from FL3 vs. real scores?

mark-ER

Also, why are you taking the exam so many times? I can understand re-taking, maybe even twice, but what makes you think you can in a great way improve your score taking it 4-5 times? Ultimately it all evens out & you are putting yourself thru unnecessary stress (& expense). You have a decent score, go ahead and apply with what scores you have.

tantacles

Taking more practice tests is always a good idea to get a better idea of how you will do on the real exam. As far as how scores are standardized, it is possible that a poor cohort compared to you could mean a better score, but there are a large number of people on any given day taking the test, and thus it is unlikely.

It is hard to say how you will perform on this specific exam, but if you are unsure, I would suggest doing another practice test to see where you stand.


When to start worrying

Hello!

I've been watching many people who were complete after me receive interviews already. It's extremely discouraging because I feel like I have a strong application and come from a top 10 undergrad school. Furthermore, all of my friends who have gone through this process told me to submit early. I'm wondering if the time I submitted is correlated at all to when I'll receive interviews and if I should start worrying.

Thank you!

tantacles

It is hard to say exactly when schools will send out interviews. Some of this depends on which schools you have applied to, and some of it depends on how good a fit those specific schools feel you are for their program. That being said, many schools interview until April. If you do not have any interviews by October, I would start worrying about your application and thinking about a backup plan.


Does it matter what online certificate I do?

I graduated undergrad with a cGPA 2.5 and mostly C's and a couple of D's in my science courses. After uni I joined the military, EMT, attempting to go to medical school with no debt. But running into the same problem that my GPA is low, I'm attempting to fix it now after being in denial for bit. I am deploying right now so that limits me to online courses. While deployed I'm planning on redoing some of my prereq courses online and I'm trying to figure out what certificate to do while I do my prereqs. When I come back I'm going to try to enroll into a physical college and do badass on my prereqs. I'm obviously going to be trying to study for MCAT and do super well on that too. So if you could give me any advice on any online certs or advice to my plan that'd be great.

Thank you!

Mr.Smile12

The first thing to do is to contact medical school admissions officers at programs where you have a high interest attending and ask them for their advice about your situation. Obviously a certificate program isn't going to address or alter your GPA, so I'm not sure how much an online program is going to help at face value. (You likely already have a lot of desirable qualifications and experiences from your time as an EMT.)


Community College Coursework and Transferring

I am currently in my third semester at my local community college. Technically, I could finish my associates degree next semester and transfer, but I am thinking about taking an extra semester at CC. I have already taken multiple prerequisite courses. If I do decide to take an extra semester before transferring, I would complete all of my medical school prerequisites at CC. I am thinking about doing the neuroscience major at UofL where I would take some anatomy and physiology courses through their medical school. I would also be taking upper level biology and psychological sciences courses having to do with neuroscience. So my question is would it hurt my application to take an extra semester and finish the prerequisites before I transfer even though I would be taking dozens of upper level sciences courses at UofL? Also, UofL medical school is my number one choice, so I might get to have some of the professors that teach there. My current GPA is 3.77. Thanks for taking the time to answer me!

Mr.Smile12

I am hoping you have talked with admissions staff at U of L about your decision. Your upper-level science courses there are going to more likely dictate your chances about being considered for admission, but you also want to do what you can to network with students and faculty there during your time as an undergraduate (or even now), including opportunities for research or community service. Work carefully with your transfer student counselors to be sure that all your prerequisites count when you transfer (that is to say, help satisfy any entry requirements to take upper-level science courses).


How Much Damage is Done?

Hello Everyone,

I am currently in the summer of my 1st year of my Undergraduate career.

In a Bio Lab class I took my 1st quarter, my professor told us that "Nothing can erase a C in Ochem".

The way Ochem is split in my school is that there are 3 separate classes, each with their own labs. Each lecture is 3 units, while the lab is just 1. Unfortunately, I got a C in 1 lab, and A's in all the lectures and labs (the other 2).

My GPA currently is a 3.7 because I received a B+ in a general chemistry class and a B- in Bio (1 out of 3). (I've received either A's or A-'s in the rest of my classes).

I am also doing research for a lab (hoping to get published in the next year or so) and I have been shadowing a doctor for roughly 2 months now (will try to do this for as long as possible). I am doing this along with other extracurriculars. Best case scenario, if I get straight A's from here on out for the rest of my 3 years (would be ~3.85 maybe) and an excellent MCAT score, how much damage has this 1st year done for hopes of being accepted into a top tier medical school?

Mr.Smile12

I would want to know what you define as a "top tier" medical school and why you are insistent going to one. If you want to go there, how much research have you done (I do acknowledge your answer)? How much leadership and community service have you done? Have you considered a gap year, and what options are you thinking about? How much networking have you done with students attending those medical schools and alumni who graduated from there? And just as important, how much have you done working with your prehealth advisors on campus? I don't think one C is going to undermine your application, but having blinders on to just focus on the top tier may prevent you from seeing a lot of other unique opportunities many other medical schools have developed to help with the clinical education of its students.


Personal Statements

Hey, guys. I know this is rather late, but would anyone be willing to give me feedback on a draft of personal statement, please? Any help would be greatly appreciated. I'll PM you a link to the google document.

Thank you.

Mr.Smile12

The forums have specific individuals who volunteer to help with personal statements, so I hope you posted your request there.


Sending a CV or Resume post secondary submission

I am applying to both MD and DO schools for the 2018-2019 cycle. Most of my applications are considered complete. I noticed that many of my schools have the option to submit a CV or resume, and some have options to submit anything as an update. I am thinking of submitting my CV mainly because it contains the citations for many poster presentations I have given, but were not able to squeeze into my AMCAS or ACOMAS applications.

Is it a good idea to submit a CV to the schools that list it as an option? And what about the schools that don't specifically list CVs as an option, but allow you to submit an "update?"

Thanks in advance!

Mr.Smile12

If you feel that there is important information that is not included in your original application, I don't see it hurting you to take advantage of sending a CV that lists significant items such as publication/poster references/citations. I would think carefully about what you want to submit because you should be efficient and not send an update every month.


Is there hope?

I need all the answers I can get.

So I have a 2.4 gpa with only one class left for me to graduate my undergraduate studies as a biology major. I have yet to take the mcat. I have done research, volunteered in the hospital, and I've been a Ta for Organic chemistry. I really would like to be a doctor and have always dreamed of doing so, but I have clearly went off track. I would really like to fix my life and get back on some type of track. Do you have any advise for me? At this point I am not a competitive applicant for Medical schools in the U.S . nor even the accredited schools in the Caribbean . Please advice me on what to do at this moment. Also it would be great if you can estimate the time frame before I make it to medical school. I know post-bacc programs, and possibly SMPs are an option. If you have any programs in mind for me, also let me know. Is there hope?

tantacles

Right now, you are not a competitive applicant for medical school regardless of your MCAT score. If you want to be accepted to medical school, you have several options.

1. Wait several years and pursue a post-baccalaureate program after this period. This will allow schools to see that you've taken time and improved.

2. Do an SMP program. The SMP, if you can get accepted to it, may offer an interview to the school it is connected to. That will likely be your only interview given your GPA.

That being said, at this point, your GPA at this point may keep you out of even an SMP. I would also look into other options besides medical school.


non trad, low gpa, high mcat

More than 10 years ago I was a terrible undergrad, I've since had several careers, and recently became a paramedic, which feels like a calling, so I've done the pre-med courses. cGPA 3.2 sGPA 4.0 MCAT 524

I have no idea if or where I might be competitive with those stats. Any advice would be really appreciated.

tantacles

Based on your redemption and the fact that you've been able to do well in all of your science courses and perform excellently on the MCAT, you have a great chance of getting into medical school. It's hard to completely predict your chance of success, but provided you've done everything you can to get the appropriate experience (including being a a paramedic), you should absolutely receive interviews based on your statistics.


I worked in my Uncle's lab for a summer after freshman year, he is one of my reference letters and i am applying to the school he works at.

I worked in my Uncle's lab for a summer after freshman year, he is one of my reference letters and I am applying to the school he works at. I worked very hard doing research there. Will the adcomms look down on this? It's too late to retract the letter, and I have to disclose family members working at the school on my secondary.

mark-ER

You should be fine. Does your uncle have the same last name? Regardless, he should know better and mention you are related, plus he should make sure to incorporate verbatim "in quotes" positive comments from the senior people you worked with at the lab, grad students, technicians, postdocs. Ask your uncle if he already sent the letter (if he's like most academics, probably not) and briefly mention that to him, just to be extra safe.


Kicked out: failed 1 class due to grief, waiting for appeal any advice?

Kicked out of my program: failed a class due to grief. My mom passed away unexpectedly. I have a wife and child on the way, immersed in student loans. Waiting for the appeal. Any advice? My program didn’t let me take a year off. They won’t let me re-apply.

mark-ER

First of all, my deep condolences, very difficult situation. That is a lot on your plate. Please, first of all take care of yourself and your family. There are a lot of resources for free and confidential psychiatric help and counseling -- someone who will listen and help you to process. Take a deep breath and a pause. Your life and well being are VERY VERY important and I know it's hard to see being directly in this situation, but this one misstep will not drastically alter your trajectory in life.

Onto your actual question -- it looks like you are still a pre-med (undergrad), hopefully junior or below. It is not clear how this failure happened -- was it academic performance, academic dishonesty, tempter at a professor/TA? Regardless, but especially if it is the latter, please with support of your wife and a senior academic mentor (guidance counselor/another professor), write the professor teaching the class you failed a short succinct but very conciliatory note to smooth things over and so that the situation does not escalate. E-mail may be OK if it is a younger professors, but in my experience older academics are typically touched by a hand-written note. Then set up an appointment with the dean of your division or undergraduate, perhaps where your mentor/supporter can also attend. Some schools in your specific very difficult circumstances may be willing to offer a "W" (withdrawal) rather than a fail and allow you to retake the class. Especially if you set yourself up and get buy in from the professor teaching the class. Then ROCK the class on a retake, and you will have a wonderful turn-around story (worst case scenario) and a supporter/reference letter from the professor whose class you failed (best case scenario).


Chances of Acceptance Into md/do School/HELP

I am currently a senior. I wanted some help/opinions on what you guys think my odds are of getting into either MD or DO school. My current major is Molecular and Cell Biology, however I decided to stay an extra year to get a degree in Psych to help boost my cum gpa. Currently my cumgpa sits at a 2.44 and my sgpa is at about a 2.7. I have some volunteering experience doing MedLife in Peru and also helping out some food shelters in my area as well as ALOT of research experience working in a microbiology lab in addition to a regenerative engineering lab(program that I completed this summer). I have written a thesis in my microbiology lab as well as a review paper for the regenerative engineering program. In addition to this I have also done SMDEP (Summer Medical and Dental Program) in the Summer at Rutgers as well. What has caused me to do so poorly is I have had various extenuating family/domestic circumstances that I have been placed in and has effected my grades throughout my 4 years in college. I still havent taken the MCAT which i believe will hold a significant amount of weight in my application just wanted to get your thoughts on if I should retake some of the science courses I did poorly in or just try to finish the Psyc Degree. I am looking to take MCAT in Jan. and apply Spring 2019 so any type of feedback would help and be greatly appreciated. :) (Side Note: I am really trying to avoid having to do a Post-Bacc or Masters Program hence why I am trying to add the Psych degree for an extra year to boost my GPA)

mark-ER

To even consider applying to US medical schools (MD and/or DO), you will have to absolutely get straight As in all your remaining science courses, to bring your GPA to at least 3.4, science GPA to 3.2 and score at least 515 to make up for those very low grades. That is quite an uphill battle. Please seriously consider 'plan B'... you can still stay in healthcare, but perhaps a field that's a little bit more understanding/forgiving of academic difficulties: DPM, RN (and if you do well, then DNP or anesthesia), or maybe PA. You will still have a good degree of autonomy and professional respect, with a silver lining of a lot less educational debt and shorter training.


Getting into University of Iowa Carver College of Medicine

What are my chances of getting into Iowa if I have a 3.88 GPA, lots of research experience, around 100 hours of volunteering, around 30 hours of shadowing, and a low MCAT score of 504 (126/124/126/128). I am an Iowa resident and attended the U of I for undergrad. I know my MCAT score is very low for Iowa but curious if there is still a chance?

mark-ER

There is a chance, but a low chance (spitballing here, but 20%, maybe a tad bit higher if you are a URM). The best advice I have is please retake MCAT at your earliest possible slot. Even a few points of improvement like a 508 might greatly improve your chances. It may not be fair, but MCATs predict step 1 score, which in turn greatly influences residency match. So even with fantastic grades and good ECs/leadership you still need a solid MCAT score. If you are applying this cycle my best advice would be to expand your application to 30-40 schools (all in-state, many if not most low-tier allopathic programs, and many DO programs), and at the same time schedule to retake the MCAT. Do it today (!), time is of the essence, medical schools have rolling admissions.


University of Miami pre-med first semester course load.

I am taking General Bio with lab, General Chem with lab, Beginners Calculus 1 and Accelerated Elementary Spanish for a total of 16 credit hours. Is this too much?

mark-ER

Nope, that is not too heavy of a coursework, but of course that depends. On one hand, if you want to be a strong medical school candidate, you should be able to handle this coursework. Some high achievers do even more, do sports & work part time on the side. But you know yourself and your high school background -- did you already have calc in high school? Did you take AP bio and AP chem? If you think you would have trouble with this, you can substitute a sociology/history class for the bio class. I would NOT remove chem or calc -- you need calculus and it's easier to do it in continuity with HS maths. You need chemistry and it's best to do gen chem freshman year and O-chem sophomore year and perhaps P-chem (if you like chem) or biochem (if you like biology), so you have that background before MCAT in the winter/spring of junior year. It's good that you are thinking about it, though.


Med school app grammar

I'm at a loss for whether or not using "they" as a single, gender-neutral pronoun is appropriate in med school essays. Online grammar sites say that "they" in the singular form is only used in informal writing, and med school essays would probably count as formal writing. But at my undergraduate university, using the singular "they" was the default to be inclusive.

This issue has come up a few times in my secondaries. One school includes "him/her" in the scenario-based question itself, but it sounds really repetitive writing "him/her" over and over in response. In another case, the question asks "what would you do if a student..." and in my response I'm tempted to just use the singular "they" instead of "him/her." What should I do?

I know a simple grammar issue probably won't make or break my application, but an older professor reading over one of my essays circled every instance I used a singular "they" as incorrect. I think physicians are probably trained to avoid misgendering patients and so may understand my usage of "they," but I also don't want a similarly negative reaction from the admissions committee.

Thanks for any advice!

mark-ER

Use "he/she". By the letter, that is grammatically correct (University of Chicago manual). There is no time/space to explain the nuances of your pronoun choice in personal statements. This diction choice may arrive as fully acceptable shortly. For situation where you think you are using he/she too much, alternate and use forms such as "the student" or "the individual". Good luck.

Edit: if the personal statement question deals specifically with transgender issues, "they" is now considered acceptable. In this specific situation only, I would then suggest using he/she/they initially, and then using 'they'. Otherwise, safest bet is to use he/she.


Help with Texas University Choice

I want to know what school is good to go for pre med in Texas. I really like the UT Austin campus but I’ve heard that the research opportunities and shadowing opportunities there are bad. The classes are also really big making it hard to get close to the professor. I want to know what other good schools there are in Texas. I don’t know if it’ll help with finding more research and shadowing opportunities at UT Austin being trilingual in Italian, Spanish, and English. The other good school that I know of are Baylor and of course Rice. Also with the addition of the new Dell Seton hospital there should be more opportunities at UT (right?).

mark-ER

It is less important to find an undergraduate with an associated medical school, than a school that is a GOOD FIT for you and where you are going to excel. So kudos to you for considering the setup of the school, class size, who teaches the courses (TAs vs. tenured professors), and perhaps curricular strengths. Yes, UT-Austin may be "the best" (i.e. highest ranked per USNWR) state undergraduate institution, but it may not be the best choice for you. Still, consider it, Austin is a great place to spend 4 years. Otherwise, good schools in TX for undergraduate on a pre-health track are private Baylor & Rice (latter, particularly if you would consider engineering), University of Houston and among UT in-state schools are UT Galveston and UT El-Paso. Last, but certainly not least, consider the costs, but also take into the account that the stated tuition cost for an expensive, private school like Baylor & Rice may not be what you pay, so apply regardless, put in your FAFSA and even if the cost is still higher than you want, you can challenge the assessment.


509(123 CARS) 3.86/s4.0 does it kill my chances

509(123 CARS) 3.86/s4.0 does it kill my chances? I am already applying but I want to be realistic, still awaiting verification

I have ok EC, with one publication URM

my list, please as truthful as possible

Tuft

Penn State

Loyola

Central Michigan


U of Miami

New York medical college

Oakland University

Tulane

U of Vermont

Cincinnati(reach)

Thomas Jeffersons

Umass medical

Quinnipiac

Rosalind Franklin

University of Arizona College of Medicine Tuscon

Indiana

Virginia Commonwealth University School of Medicine

University of Minnesota Medical Scho

WVU

Creighton

mark-ER

Your GPA is great, EC and a publications are a 'cherry on top'. If you are indeed a URM, you should be good to go (despite probably for you) disappointing MCAT score. I think your list looks good, and perhaps a few more 'reach' places could be considered. Some 'reach' places to consider, based on your current geographic preferences may be Pitt, Case Western, Medical college of Wisconsin. Perhaps instead of some in-state schools like Indiana, Penn state and Vermont (look carefully at admission rates for in-state vs. out of state candidates). Good luck.


How bad is institutional action due to marijuana use?

I started undergraduate early, taking summer classes. I received a disciplinary probation for being found with marijuana and upon further search a couple alprazolam pills that were not mine, alcohol, and a fake id. This will not result in a criminal record, however, it has resulted in institutional action. I have full scholarships to a couple universities still standing for the fall. There is a chance my current school will expunge the record once the sanctions have been completed, yet I still may be compelled to report it when I apply to medical school. Should I withdraw from my current university and go to another, less prestigious, university to avoid this record being seen in my medical application?

mark-ER

It sounds like you are trying to avoid owning up to this. Do you consider this mistake, indiscretion and do you plan to continue to use? Regardless, I would stick with your current undergrad, try to learn from this experience and be open about it. It won't trail you forever -- once in medical school, it won't be on your record and won't follow you into residency and/or practice. Unless you happen to be a repeat offender... so treat it as lesson learned; do not repeat the mistake, own up to it and you should be fine. It won't prevent you from getting into medical school, but it will probably make things a bit harder. So overcompensate -- rock your GPA and MCAT.


Do I still have a chance to get into Med School?

I'm concerned about the introductory science classes I took as pre reqs for pre med last semester. I became ill in the middle of the semester due to my health condition. I managed to pass with C’s but I wanted to know when applying for med school, is there an option to explain why I got C’s in these classes? Currently I’m considered a Junior in my college, however I have not completed majority of the classes within my major. I just wanted to be aware before applying whenever I have completed my degree.

tantacles

There are options to explain C's on your application. That being said, many students are accepted to medical school with C's on their transcripts. It is important, however, that you do well in all of your other courses to improve your GPA so that you can be taken seriously as an applicant.


Adding Admissions Recruiters to your LinkedIn?? Professional or not?

Is it professional to send an invitation to connect to an admissions recruiter (LinkedIn, currently applying to this school)? We have exchanged several emails, and I think that there is a good communication link. This school is very important to me, and I definitely want to seem invested, especially since the recruiter has been emailing me, but I do not want to seem unprofessional or too eager (to the point when it is annoying).

What should I do?

mark-ER

Not while you are applying; maybe after you are done with the process.

(too many people are too loose with their facebook/linked in contacts, personal point of view but one that is shared by many professionals).


Past alcohol criminal history

Hi there,

When I was 15 I was charged with possession of alcohol. I went through a diversion program and my record was sealed. When I was 18 I again was arrested for underaged consumption of alcohol. Both of my records have been sealed but I'm wondering how/if I would say this on an application. I know that doesn't look good because it's like I didn't learn my lesson. Do I even need to talk about the one that happened when I was 15? Both records don't show up on BCI background checks but I know they run an FBI background check too. Any help is appreciated. Thanks.

tantacles

You must report these infractions when you apply. That being said, many medical schools are lenient with regard to charges related to alcohol, so the best way to approach this is to be honest and state that you have learned something from the citations. Several years will have passed since these events, so medical schools may be willing to overlook them if you are honest.


MCAT 508, GPA 3.83, SCRIBE 2500 HRS, COMM-LETTER, C-SERV. 300 HRS, RESEARCH, FLORIDA-RES, WHAT ARE MY CHANCES AND WHERE

tantacles

Given your MCAT and GPA, you have a good chance of getting interviews at mid-tier schools and possibly some mid-high tier schools. I would apply to all schools in Florida and use the MSAR to select schools that fit your MCAT and GPA combinations at around the median.


Multiple MCAT - (491<498<500<511), WAMC?

I did something stupid, and took MCAT in 2015-2016 several times before finishing all the prerequisites (no idea what I was thinking). Only the last MCAT 511 was taking after organic chem and biochem was finished.

My GPA is both 3.65 (over 250 credits, - 2 bachelor's and master's), I have military service (was a medical technician), other stats:

research: 500 hours (NIH) paid medical employment (not military): 2300 Learning Assistant (chemistry) : 480 hours Leadership (was manager of supply in medical unit, military): 1320 hours, clinical direct patient care experience: 250 hours.

What I my chances? I know that multiple MCAT score is bad, but how bad? do you think it will automatically ruin my chances of being accepted?

FYI: already submitted all secondaries (within the first two days of getting them, in the secondaries mentioned multiple MCAT as an experience that taught me to not rush, plan, etc.)

Drexel Katz Kimmel EVMS (in state) GW GT Howard MCW MUSC U vermont Tulane U. COnn U Mass UVA U Wisconsin VA tech Creighton Wake Forest WVU.

Should I add any more schools to increase my chances? (VA in state)

Thank you

mark-ER

Your chances are NOT ruined. Taking MCAT early is an uncommon mistake, but not completely unheard of. However taking it so many times is very unusual, and you should preemptively do something to address it. Pre-meds are often very ambitious driven individuals who like to test themselves. Often these individuals are used to taking exams early, think PSAT or SAT while still in elementary school or early high school. The problem with MCAT as opposed to SAT is that MCAT is much more content based than intelligence/critical thinking test. So no wonder not having formally had the material it's easy to do poorly. You did not realize it -- decent enough explanation. Put a similar respectful and succinct explanation of your situation as an addition to your application and I think some (though probably not all) schools will disregard your early attempts.

If you already submitted primaries and secondaries, please followup with the schools you have applied to and send them a formal letter to clarify this situation. Ask kindly for it to be included following your personal statement.

In terms of schools/school choice, it is a decent list, but perhaps short. With your stats you should consider adding 4-5 additional schools and if you are not getting a solid response (several interview offers by late October), then another 5-10 schools on top of that, perhaps even some osteopathic. You are a Virginia resident, so clearly you focused your applications to in-state VA, and generally mid-lower prestige tier private schools on the east coast (good choice). But I also see some schools in the midwest, particularly wisconsin -- do you have some personal ties to the area? If so, adding several more schools in the midwest might be helpful (perhaps both some 'reach' schools like Northwestern/Uchicago, Mayo but also some lower tier schools like Rosalind Franklin, Loyola, Rush, Michigan state). Oh and I think you are missing Virginia Commonwealth? Good luck.


Low Overall & Science GPA, can a high MCAT score allow my application to be considered?

I graduated from a CUNY institution (located in NYC) last year with a 3.05 overall GPA last year. My science GPA would be close to a 2.9 and my other GPA would be 3.6+. I am a female, first generation college student and I am Asian.

Since then I have been studying for the MCAT in the hopes that my application will be considered for an MD program?

I have participated in a number of programs to expose myself to the field of medicine. I have shadowed physicians, volunteered countless hours and organized community health fairs. I have also conducted research and I am a coauthor on a publication.

mark-ER

At this point, despite reasonable overall GPA -- presumably post-postBacc, you are bluntly a marginal candidate. A high (and I mean HIGH) MCAT score may ameliorate things a bit and help you to reach some lower tier allopathic and/or (many) osteopathic schools. MCAT is by far the biggest determinant, but do you have time/room for more classwork? Can you load it up with (relatively easy) science classes to improve your sGPA? Your overall GPA isn't too bad, but unless you had straight Bs in all prereqs, those Cs and below in science classes are going to hurt you.

As a note of encouragement: at this early stage of your career it's difficult NOT to have a tunnel vision, but there are a ton of rewarding careers within medicine that are as rewarding professionally -- DPM, masters of genetic counseling, optometry, PharmD... That being said, you *can* still reach for medicine, with some HARD study. Good luck.


HELP! Should I drop Pre-med?

Hello, I am about to be a 3rd-year student at FSU. My major is Women Studies and I am on the pre-med track. (As if that's not obvious) I struggled my two years in college and failed pre-calculus. I am fighting to get my GPA to a 3.0 and honestly I do not know if pre-med is for me. I have not done any research or volunteer yet. I'm involved on campus but not with organizations that are medically based. My peers are super involved, done research and volunteer work. And I have done nothing. I question if I even wanna be a Doctor cause it seems like so much work and my regular and science GPA is not that good. I have no motivation whatsoever. And my advisors do not help what so ever. I honestly feel like their as confused as I am. I've looked at other career fields and the only one of interest to me is Marketing. But I'm conflicted. This is my 3rd year... There is no more time for games. HELP?

tantacles

If you are truly motivated to become a physician, it is possible that you could redeem yourself. However, if you have no motivation and don't feel that you will be able to do well, I would consider another career option. Most medical schools will not even offer an interview with a 3.0 GPA, so you would need to get all A's if you wanted to have a chance at acceptance.


Where could I get accepted

Hello all,

So i am currently applying and would like to know where my best chance of getting accepted is. I am a Florida resident with a sGPA of 3.1 and overall GPA of 3.6 with a pretty low MCAT score. I will be retaking it this August but wont get scores back till September. I already applied to NOVA since it is in state and have already submitted my secondary.

Also does anyone have a list of the schools with master bridge programs that guarantee an interview? I know LECOM does but am not sure of any others.

Thanks!

tantacles

There is no current comprehensive list of programs that offer a guaranteed interview, but programs' websites usually give a good idea of whether or not they offer this as a feature of their program. If that does not work, you can also contact the admissions department with an e-mail address. If you do not feel comfortable using your real name, you can create an e-mail address with a false name.


Submit applications now or wait for MCAT retake?

So I took my first MCAT and got a 502, 127/123/126/126, I know its not good and I'm going to retake it on august 4th, I know its a little late but thats beside the fact. What I am wondering is if I should submit my applications (already verified) to the schools I want to apply to now with that bad MCAT score and just have on the app that I will be taking it again, or will the school just look at that MCAT and say no. Also, I'm applying both MD and DO, Florida resident, Hispanic, Male, I know the score is really bad for MD but I figure I still gotta try, at least my in state schools (i.e. FSU).

Mr.Smile12

It always depends on the schools where you are applying, and no single school behaves like another. If the MCAT is the only issue (assuming your GPA's are good), then I would encourage you to begin the application process now and let them know of your retake.


Secondary Applications

I was hoping to get some advice on when to submit secondary applications. I recently took the MCAT will not get my score back for another few weeks. I submitted the primary application the first week it opened and now I am receiving invitations for the secondary applications. Would you recommend waiting to submit these until I get my score back or submitting them anyway? Honestly I feel that my score could go either way. Any advice is appreciated.

Pathdocmd

Do not wait. Medical schools know if you are waiting on scores, so your secondary will be put on hold until the scores arrive. That how is it done at my school, so I can't speak for every school. If you do well, things will not be delayed. If you do poorly, then you are out the secondary fees.


Chances of getting accepted

Im a junior at UCF I'm majoring in pre-clinical health sciences and I am unsure about my future and am looking for guidance. I have 57 credits and a 3.33gpa after a rough 2 years. I have finally got myself together and I realize if I get a 4.0 for the next two years I will only have a 3.66 gpa which isn't a great gpa for med school acceptance. I volunteer at a free clinic, I start volunteering at a children's hospital in august, I start volunteering at a human milk lab in august, I am working towards getting my CNA license, and I plan on studying abroad (South America) shadowing doctors for a week during spring break and a month in the summer. What are my chances of getting into med school? Why mcat score should I get and what other options do I have if med school isn't in my range?

Pathdocmd

The upward trend is great and most schools will notice that. You really can't predict chances until you have your MCAT scores are available. Also, there are many more factors like state residency, the type of school you are applying to, etc.


Best Choice for Gap Year

1. Do an MBS program (SMP) to increase my GPA, get better letters of recommendation, and more research programs. I could also work on campus for convenience and study time.

2. Work at a pain management clinic where I’d be working alongside an anesthesiologist, shadowing him at BCM, and who I would be publishing papers with, and maybe take some classes on the side.

I can only do one option, and I'm torn between the two! Thanks!

mark-ER

I think either choice is OK... if your GPA is at least in the OK range (say 3.5-3.7), and you have a good MCAT score or you know yourself enough to think you can stick to an structured, independent study schedule, then go for option #2. My thoughts -- MBS costs $ (tuition, room/board), whereas for #2 you would be making a salary and potentially have time to get a few publications under your belt. If you do MBS, you will not have time to take classes, study for MCAT and do any meaningful research, but if research is "baked into" your job, then you are better off. But again, it's not a slam dunk decision, more to do with your pre-existing stats and how well you do in a structured vs. less structured study environment.


Already submitted primary but received low mcat score...what now

I was told by multiple people to submit my primary application as soon as possible even though my mcat date was not until June, meaning my score came out in July. So I listened and my primary has been completed and approved. I received my score and it is not competitive enough to get in and its unlikely I will get an interview even. I considered retaking it in late August but am afraid I will still not have enough time to get my score where I want it (I want to take a prep class before the next one). I have accepted that I will not be getting in this cycle with my current score and want to reapply next cycle. That being said, do I still complete the secondary application so the school knows I am interested and sees I turned it in when I apply next cycle? Or do I not send it in and hope it doesn't hurt my chances for next cycle with that school. I feel like I have been left in the dark and have received little or false information from my advisors so any feedback would be greatly appreciated.

Mr.Smile12

I guess if you have decided to delay your application until you get a better MCAT score, I don't see why you should submit secondary applications and fees. I would check with admissions staff and even some medical students you may know from the "inside" who have gone through the process and can help encourage you if it's appropriate. I don't think most medical schools pay that much attention if you withdrew from the process before you sent them a secondary application (and fee).


Advice for College- Switching Majors to Bio?

Hi,

I am currently going into my second year of university (undergrad). I am thinking about switching majors from music to bio for med school. I know med schools don't really care what your major is but my science background isn't too strong so I think it may help me prep for the MCAT.

Is this a smart move on my part? I really don't want to hurt my GPA but at the same time, I feel like I need these science courses to prep properly for my MCAT, rather than just doing the book courses.

mark-ER

You are correct in stating that the major does not matter; but you DO have to have a certain number of science courses, and specific type (math, chemistry, biology, physics) -- make sure you understand the prereqs you need before you apply and the background courses you would need before taking the MCAT. A good place to start:

https://students-residents.aamc.org/applying-medical-school/applying-medical-school-process/

For many people, there is just so much overlap between biology and med school prerequisites that they do a second major. If you are a rising 2nd year (i.e. just completed your first year of college), you will be fine, but if you are a rising 3rd year you will have a hard time to squeeze all these in 4 years, taking MCAT and applying w/o taking a year off.


Advice needed

I am currently a Biochemistry major with a 3.85 overall GPA (idk what my science GPA is but it is higher than that I'd say 3.88). I have junior standing as of right now. What I have so far is:

-50 hours of shadowing a doctor with 38 of those being an ER doctor and the other 12 with an urology surgeon. -53 hours of volunteering at a food bank and the local hospice combined -I was an Undergraduate TA last semester for a Cell biology and genetics class -Currently am in a research lab in which I started this Summer 2018. I should get published (3rd author?) by the beginning of next year from what I have been told. I put a lot of time in the lab. - Pre-med club member

Unrelated that might affect: I am fluent in Portuguese and English and I also am moderately good in Spanish. I am also a permanent resident card holder in the USA (I have lived here since I was 9, I am now 20). The school I go to is Washington State University.

I was wondering in what areas would be good for me to work on in order to increase my chances of being able to go to med school?

mark-ER

You are on a very solid footing. You have enough volunteer and medical shadowing, you dipped your toes in research, your grades are very good. Your main (if not only) concern is doing the best you possibly can on MCAT. The fantastic MCAT score vs. run-of-the-mill above average one can mean a difference between one or a couple of matriculation options versus several admits and perhaps even a merit-based scholarship.

A very, very minor point -- if really liked research and you intend to pursue grant-funded research in the future (perhaps as a physician-scientist) a US citizenship would be an asset. If you are already a permanent resident with a green card (that how I understand your status), you are all-set from a medical school admissions standpoint, but for some scholarships and many federally-funded grants it is beneficial if not required to have US citizenship.

Mr.Smile12

Congratulations on your hard work. So that said, what outreach efforts have you made with the school at Washington State? Have you met their students and know their AMSA officers or other club leaders? Do you know what they do when it comes to clinical and community service experience while in medical school? You can always shadow more physicians in different areas and volunteer more, but you want to keep your focus on the missions of the schools you want to get into and why. Think of your recommendations and doing what you can to have the strongest champions for you.


Advice needed

I am currently a Biochemistry major with a 3.85 overall GPA (idk what my science GPA is but it is higher than that I'd say 3.88). I have junior standing as of right now. What I have so far is:

-50 hours of shadowing a doctor with 38 of those being an ER doctor and the other 12 with an urology surgeon. -53 hours of volunteering at a food bank and the local hospice combined -I was an Undergraduate TA last semester for a Cell biology and genetics class -Currently am in a research lab in which I started this Summer 2018. I should get published (3rd author?) by the beginning of next year from what I have been told. I put a lot of time in the lab. - Pre-med club member

Unrelated that might affect: I am fluent in Portuguese and English and I also am moderately good in Spanish. I am also a permanent resident card holder in the USA (I have lived here since I was 9, I am now 20). The school I go to is Washington State University.

I was wondering in what areas would be good for me to work on in order to increase my chances of being able to go to med school?

mark-ER

You are on a very solid footing. You have enough volunteer and medical shadowing, you dipped your toes in research, your grades are very good. Your main (if not only) concern is doing the best you possibly can on MCAT. The fantastic MCAT score vs. run-of-the-mill above average one can mean a difference between one or a couple of matriculation options versus several admits and perhaps even a merit-based scholarship.

A very, very minor point -- if really liked research and you intend to pursue grant-funded research in the future (perhaps as a physician-scientist) a US citizenship would be an asset. If you are already a permanent resident with a green card (that how I understand your status), you are all-set from a medical school admissions standpoint, but for some scholarships and many federally-funded grants it is beneficial if not required to have US citizenship.

Mr.Smile12

Congratulations on your hard work. So that said, what outreach efforts have you made with the school at Washington State? Have you met their students and know their AMSA officers or other club leaders? Do you know what they do when it comes to clinical and community service experience while in medical school? You can always shadow more physicians in different areas and volunteer more, but you want to keep your focus on the missions of the schools you want to get into and why. Think of your recommendations and doing what you can to have the strongest champions for you.


Med schools asking about college finances?

In both the primary AMCAS application and a couple of my secondaries, med schools are asking how I paid for college. Why are they so interested in this and how do they use this information? Is it frowned upon to already have a lot of debt? Thanks!

Mr.Smile12

While I cannot speak for admissions committees' interest in this question, there is general recognition that higher education is costly, and medical education is going to add to that. Some of your peers may already have found a lot of grants or scholarships to offset their undergraduate debts, including those doing ROTC or perhaps a job taken at the same time as taking classes. Though ability to pay is not a factor in offering admission, it may help when it comes to "closing the deal" if you wind up being eligible for special financial aid opportunities.


Is it too late or how can I get on the right track when I am entering my fourth year of undergrad and the only aspect of med school i'm ready for is my GPA? (Little to no EC)

I am currently entering my fourth year of undergraduate studies in Biological Sciences and I have a GPA of 3.95. I was not sure what I wanted to do throughout university and just made sure I had good grades so that wouldn't hold me back when I graduated. Looking into careers I want to pursue I realized that all the summers and the last three years of my undergrad have been wasted as I currently have little to no ECs (I volunteered short-term with children with disabilities, church) and no experience doing research or abroad. I am considering extending my degree to 5 years so I have a little more time to gain experience during undergrad but I am worried it is too late. What can I do when there isn't much of this summer left and I feel like I've already wasted so much time?

mark-ER

First of, you are still very much on track. Most admission officers do not mind a candidate who 'discovers' medicine as a career option later on, but with a stellar academic record. So onwards to 4th year -- as you point out, you are very unlikely to be able to apply this cycle, but maybe not with the main reason you list (lack of ECs): with a biology major you might (but maybe not) have all the prerequisites, but you still have to prepare for and take the MCAT, getting letters, writing a PS and putting together an application is no joke, and remember medical school has rolling admissions. So you are behind the 8-ball here.

That being said, if you have enough courses to graduate (and please re-check into your medical school prereqs, adjusting your next 2 semester coursework accordingly) you should be able to graduate in 4 years. No real reason to shell out more $ for tuition/room & board. You can then take a year doing what YOU want to do, that you find interesting and useful. Several options: MPH (stats and community health), medical mission abroad, work as a tech in a research lab, post-bacc NIH program, or most likely a combination of several things. I would really, really recommend the last option -- perhaps you'd get the research bug and getting the experience of living in/around DC is fantastic. Whatever you decide to do for "5th" year, the key is to start now -- many masters or year-off program applications are due summer or fall of the preceeding year, some may require a GRE. Good luck.


Emory Secondary Prompt

I'm unsure about the "Briefly describe your health-related experiences. Be sure to include important experiences that are in your AMCAS application, as well as any recent experiences. (200 Words)" prompt.

My understanding is that it is only about clinical experiences, such as volunteering and shadowing, but my father says that I should include my basic research as well? Can someone shine some light on this dilemma?

Mr.Smile12

I would ask Emory about the prompt on whether research experiences are appropriate. Be prepared to talk in brief about your specific research if it is bench-related, field-related, or survey-related.


Deferring

I applied to medical school this year but I recently obtained a research position through Harvard Medical School and Brigham and Women's Hospital. It's possible that this position might be a two-year commitment. I am wondering how likely it would be that medical schools would accept this as a reason for deferment of attendance if I was accepted.

Mr.Smile12

Each school has a policy on deferrals, but many will not grant you that deferral, especially if you were aware of this opportunity ahead of time.


High ranked IMG vs Low Ranked US MD

Hey, I'm a non-US citizen looking at Neurology as a career choice. I currently have a 3.4 GPA (not taken the MCAT yet) and am in a position to get into a top-ranked medical school in my home country in Ireland. Given that I am not aiming for a very desirable residency program (Neurology) what do you all think about picking that school and being an IMG vs struggling to get into a low-ranked US school with my credentials?? I know that being a US MD is >>>>>> than a non-citizen IMG, but will the caliber of the medical schools cross things out when being evaluated by residencies? Or is even an unranked US medical program better than say an Oxford or UCL MBBS for purposes of matching into a less than desirable speciality?

I know there is always the chance that I will change my mind and then be stuck with a speciality where only 19% would do it again if given a chance....so yeah not sure mates

mark-ER

Let me reverse your question a little bit -- you have NOT yet taken MCAT and your GPA is pretty low. On top of that, you are NOT a US citizen. Your chances of getting into US medical school are not great, actually it's quite an uphill battle. I knew quite a few non-US citizens on a student visa (i.e. not a citizen and not a permanent legal citizen, i.e. w/o a green card), and even from top-notch undergrad, with top notch GPA and MCAT getting into medical school as a non-US citizen/permanent resident was still a MAJOR struggle. Only a handful of non-citizens each year get admitted w/o legal status/permanent residency, either DACA or special circumstances (MSTP/researchers, uber-talented individuals).

On the flip side -- I think the plan to matriculate in a well-known, western medical school, esp. if it's trinity college Dublin, is a great one. Tuition cost = low, chances to match into US low-competitive program like neurology is great, with the caveat (big if) that you do well on USMLE step 1 and do well in medical school. You are also not 'stuck' with neurology -- you are probably out of luck for the very tip-top competitive specialties (think derm or combined surg-plastics), but other than that it depends on how well you do on USMLE and med school (sorry to repeat myself, but that is the key if you pursue this path). In fact, you have a major leg-up on most IMGs, because: (1) native English speaker (2) you have lived or have connections in the US (??). Good luck.


Is it better to Void MCAT or get a low score then retake later?

I'm planning on applying to the 2019 cycle. My MCAT is in a week and I haven't been scoring a 500 or better on my practice exams. I was wondering, should I take the MCAT and Void it or should I see what I get and just retake it next year?

mark-ER

Short answer -- if you do not feel ready, do not take the exam. It sounds like you are not ready and not comfortable taking it yet. Have you had all of the prerequisite courses? Perhaps a dedicated study course is in order.


Undergrad questions about GPA.

I am currently attending a community college then transferring to a university, my freshman year first semester I got a D in psychology and contemporary mathematics the classes were not even hard I was just being lazy and unfocused which I regret. I got all A's & B's my second semester & took 3 summer classes which I received A's in to help boost my GPA. I'm about to start my sophomore year and was wondering should I retake the psychology & math course at some point maybe the upcoming summer? And would it replace the D's in my GPA if I did or just average the new grade I receive with the D? I am a biology major planning to attend med school so I want my GPA almost perfect. Thanks!

Mr.Smile12

You should check with your academic advisor about retaking those classes as they may affect your progress towards your degree. Your institution may have a grade replacement policy for calculating GPA's, but most of the application services do NOT replace grades when calculating GPA.


3.7 sGPA 3.8 cGPA MCAT 505 Competitive for MD/DO School(s)?

Hello!

I am going into my senior year and would like to gauge how competitive I am for the schools below. Any advice/insight is incredibly appreciated!

Degree: M.S. Biology with Distinction w/ minor in Community Health Sciences Caucasian/Not economically disadvantaged or first generation college

sGPA 3.7 cGPA 3.8 MCAT 505

  • No Clinical Employment*

Clinical Volunteering Medical Volunteer/Shadowing (2 weeks - Guatemala) 80 hours Medical Interpreter (Spanish) 200 hours (4 years) Hospital Volunteer (Surgical Services) 300 hours (2 years) Peer Health Educator/STD Clinic 200 hours (4 years) Trauma Intervention Volunteer 150 hours (3 years) Non-Clinical Volunteering Boys' and Girls' Club 500 hours (6 years) -Governor's Service Award, Presidential Service Award, Summer Camp Counselor for underprivileged, organized summer camp for teen leaders, volunteer services and teen internship (nonpaid)

Extracurricular Army National Guard | Motor Transportation Operator (8000 hours) - Basic Training Honor Graduate - Advanced Individual Training Distinguished Honor Graduate - E-5 Sergeant (non-commissioned officer) - Combat Life Saver - Master Driver - Air Assault Qualified - Basic Leaders Course - Soldier of Year Board Winner - Other army schools (HAZMAT, Ammo Handler, Resiliency Trainer, etc.)

Leadership Basic Training - Was selected by Cadre to be Platoon Guide where I lead 70 soldiers on a daily basis through first aid, marksmanship, physical training and land navigation. Advanced Individual Training - Was selected by Cadre to be Platoon Guide for 60 soldiers on a daily basis through physical training and specialized transportation training. Army Non-Commissioned Officer - Operated as a leader on several convoys. - Delivered several briefings to 50+ soldiers. Student Government Health Sciences Senator - Represented 6,400 college students - Chair of the Committee on Public Affairs - Sat on Academics, Budget and Finance, and Oversight Committees. Other: Anatomy Dissection Leader and Genetics Teaching Assistant

Research Ecology Based 500 hours Undergraduate Research Award Winner Senior Thesis

Misc. Toast Masters Skiing Work part time as a restaurant busser Fluent in Spanish I also received the Army Health Professions Scholarship Program Award (full-4 Year tuition) - should I indicate this on AMCAS?

Applying to many DO schools, some State MD schools, and a military medical school (Uniformed Services University of Health Sciences).

Again any help is greatly appreciated!

mark-ER

Your activities and leadership are outstanding, though your grades and MCAT are on the left edge of the bell-curve .. so I think your approach is right. Apply broadly (30-35 schools): many DO schools, state MD and lower-mid tier private schools that take people from out of state. A few 'flyers' or reach schools are OK, though I would not send in 5 applications to Stanford/Harvard/Hopkins/Penn (but many one or a few to mid-upper tier places like Columbia, Brown or Vanderbilt?) Other questions you have to answer for yourself -- do you have a desire to do military further? Do you specifically aim for DO vs. allopathic? If not (i.e. you are more motivated to get into the best possible medical school you want), then apply broadly and early and control what you can control -- good support letters, great personal statement, finalize things as early as possible -- remember medical schools have rolling admissions.


What are my chances for getting into Med School?

Hello! I have a few concerns regarding my chances of gettings in to medical school. I have taken Organic Chem. 1 three times, and I just passed it with a C. I haven't been the best student when it comes to Organic, because I had a high school mindset of not having to study for things, so I just gave up the first 2 times. However, I have matured, and I really am starting to see what it takes to become a doctor. I spoke to my advisor about the issue, and I told him that I am in a period of academic growth and development. I know I have messed up, but I know I can turn my direction back into the right one. I feel that my chances are horrible, but I am trying to retain some type of hope towards medical school.

Pathdocmd

One class will not make or break you; HOWEVER, you will have to shine in all your other science classes and do well on your MCAT.


Should I submit secondaries before I have all of my recommendation letters?

I set a goal to return my secondaries a week after I received them. I am on track to submit 3 of my secondaries now. However, I am still waiting on one letter of recommendation. The writer said the letter would be submitted by the end of next week. Should I wait for this letter to be submitted or should I submit the completed secondaries now?

mark-ER

Yes, submit secondaries now. Be polite, but persistent with the faculty writing the letter... it sounds like he/she is willing to do it, but might have a lot on his/her plate. Remember, medical school has rolling admissions, so sooner you finish your secondaries = better.


Foreign transcript

Hi, I’ve a Bachelors in Nursing degree from out of country and have done a masters and a certificate course in the US in Anatomical and translational sciences. Currently prep for the MCAT. Went through the application and found that foreign transcripts are waived. Please I’m in need of advice or betterstill are they schools that can deal with foreign transcripts or do I️ need to take a post bac course to augment my application.Also I’ve been a nurse and currently working in the US for past 5yrs I’ll appreciate your advice. Thanks

Pathdocmd

If you are applying to a US MD school (AMCAS), the your non-US/Canada grades will not count. Only your masters and certificate will count in the GPA calculation. I can't speak for DO or Texas schools.


civil infraction

I am applying for medical school 2018. I know I received civil infraction when I was 19 years old when I try to get into bar with my friend's ID. The application does not ask anything about civil infraction. Do I need to disclose anyway or wait till they ask? Not sure what I need to do. Anyone who can help me answer this question? thank you

mark-ER

Was it a misdemeanor, or worse yet a felony? If it was not classified as such, and given the rather innocuous nature of the infraction, probably just keep it to yourself. You can check your own criminal record (several good ways to do so online, landlord do it all the time)...


Should I retake the MCAT?

Although I consistently score a 509-510 on practices tests, I bombed the actual MCAT and scored at a 505. I have a 4.0 GPA, 7 peer-reviewed publications, over 200 shadowing hours, participated in activities like student government, and have lots of volunteer hours. I worry that if I opt to retake it now (which would mean me taking it in August), that my application wouldn't be considered until it is too late as I am applying to schools with rolling admission. Is it worth the risk of not improving or should I just apply this application cycle with the score I have now?

mark-ER

The short answer is "yes", if you think you can do significantly better (at least +2-3 points), then retake it. I think your grades and activities will compensate to some extent, but a better MCAT score will boost your chances of multiple acceptances and perhaps better financial aid offer (? scholarship). If you retake it in August, you can still update your application and it won't be too late. There is always a risk of a lower score, but weighing the upside & downside, it seems like value play in your situation would be to retest.


What are my chances? Could you guys recommend me a school list?

I'm a rising senior. I have 100 hours clinical volunteering, 2260 hours labwork (with leadership within lab and employment over the summers, and an honors capstone senior year), interim leadership for a club, and decent ECs. 160+ hours of varied shadowing.

3.94 cGPA, 3.90 sGPA

508 MCAT, committee letter not certain (my MCAT is one point below their minimum to write a letter so there is a possibility I won't get a committee letter, although I have a valid reason for a score this low. I was nauseous and threw up during my MCAT, my score was 3-4 points below what it was on every single practice test I had. I will be petitioning my committee to allow me to still get an interview).

Submitted my primary yesterday, and I have documentation of throwing up during my MCAT.

[More details on this: My father said it’s my last chance to take the MCAT and if I voided the exam this time, they wouldn’t let me take it again. I don’t want to tell med schools about this, though. He says I can retake it if I want now but this relied on me not voiding the exam I took. I will apply to more schools once I can make sure I can get a committee letter. I voided it once before due to food poisoning, so I knew my whole family would be devastated if I voided again because I was vomiting. Now they’re just sad I got a 508... I did my best while absurdly lightheaded, lol... If I can’t get a committee letter, I will ask a pre-med advisor what steps I should take (Retake MCAT? Apply to schools in the South? Etc). I really want to go to a school in the Northeast so as long as that’s possible with a 508, we’ll see. I really would love advice from you people online :). I’m happy about my GPA but have no idea how far I can go with this MCAT score.]

I've applied to

Robert Wood Johnson (Rutgers) Penn state NYMS

Do I have a shot at these schools, and which ones I should add?

mark-ER

My take: with your stats, extracurriulars, etc. you probably can get in somewhere with a 508 MCAT, though it is an uphill battle. My recommendation would be to submit early, submit to 30-35 medical schools (including a handful of DO schools), and still plan to retake MCAT. My best guess with your MCAT score as it stands: you will probably get in 1-2 schools, you will probably get on a couple additional waiting lists and if you retake the MCAT and get a better score, you should get off waiting lists and possibly get better financial aid offers (maybe even a scholarship, if you ROCK it, above 520).

Sorry to hear about the unfortunate event @ prior MCAT attempt; you can pursue remediation action, but it is just difficult issue to prove/ameliorate, easiest course of action is just to retake).  In terms of schools, there is close to enough in the NE, but you will probably want to expand into SE and midwest.  Apply to all in-state schools, all lower and mid tier private schools that take significant % out of state in your preferred geographic area and as previously stated several DO schools.  For now, worry about things you CAN control.  Study hard for retake, prepare for interviews, get great letters and personal statement.  Good luck.

Should I give up on my dream of becoming a doctor?

My main concern is whether I still have a chance at going to medical school? My transcript consist of A's and one B, but the catch is that I have a D in biology(retook it and got a C , it was a summer course which was much harder), C- in Physics. But I plan on retaking it next summer. I am currently a 1st term junior undergrad doing pre-med. I work as a trauma technician and as a volunteer EMT on my free time. I plan getting my GPA to 3.5-3.7 and take my mcat after graduation. I just want to know whether having those low grades in such an important courses will ruin my chances of going to medical school? I would appreciate any kind of advice, thank you.

mark-ER

Not necessarily, though as you point out it might take you longer -- an extra post-bacc year to push up your grades, studying extra, extra hard for MCAT, and even then you might have to be happy with either low tier MD or DO school, or maybe even going international. That being said, as long as you persist and work hard at it, there is still a chance. See how next year goes, if you can truly dedicate yourself and get all As in your science classes and get your GPA to 3.5 or above, then definitely still pursue it. If it winds up that however hard you try, you cannot get higher than a B, B- in science classes, then perhaps it's time to consider plan B (maybe still in medicine, perhaps PA school, optometry, podiatry, clinical genetics/genetic counseling there's lots of options).


DYI Post Bacc or SMP - Please advise

Hi Just graduated with a non-science major but I did the pre reqs for med school. My final GPA is cGPA 3.2, sGPA 3.0 [Went to a top 5 Liberal Arts School]. I do not have many shadowing hours or clinical volunteering hours. I did one summer of bench research, I worked with social justice programs for almost 2 years and worked in college library.

I am taking my MCAT this August.

Please advise which option is better:

1) Do science upper level or non-attempted science courses in local Community college or state school to build up the sGPA? If this is an option how many hours do I have to attempt [assuming I get staright As in all of them] to boost my sGPA from 3.0 to 3.5 in one year?

If I choose this option I can volunteer in hospitals, shadow doctors and build up my application. Assuming I can get a new cSGPA of 3.5, [my cGPA will also increase from 3.2 to 3.5+] and get MCAT over 510, will I be a reasonable candidate for application cycle starting June 2019?

2) I have applied to SMPs [very late application 2 weeks ago, so not sure if I will get in]. If I do get in to SMPs [Loyola, Tufts], will this be a better option that option 1? If I go this route and do well [get GPA over 3.6 in post bacc], then I can apply for the Jun 2020 application cycle [provided my MCAT scores are above 210].

3) Or is it better to go to Carribean?

Please advise, thanks in advance for your help.


Mark - Thanks for your advise. Appreciate it.

mark-ER

To address your questions directly -- this will be a hard road. Your GPA is pretty low (even if from top-notch school) and you are trying to cram in a year of science and to study for the MCAT all at the same time. I would NOT advise you to try to take the MCAT before you finish the vast majority of your coursework -- that is a recipe for a low score. I also would NOT advise you to do community college -- you already have a degree from a liberal art university, so CC is a step down. Some schools don't 'count' community college science classes, some discount the grade when figuring it into GPA. As you know SMPs are both fairly expensive and fairly selective. There is a third way -- is there a post-bacc (not necessarily SMP) at your local college(s)? It does not have to be a big-name college, just so it is a 4 year degree-granting institution. I suggest you do it thru a post-bacc, because of the camraderie and advice/support such a program will provide. SMPs are typically better, but as discussed more expensive and more selective... ultimately how/if you make it into medical school is up to YOU and (typically) cream will rise to the top.


Retaking Classes

Currently a senior with a 3.83 GPA and was curious on how medical schools view retaking classes. I retook GenChem 1 and my grade went from a B to an A. Also retook OChem 1 and went from a B+ to an A. Helped my GPA out but I heard conflicting info about if med school average those two attempts and make their own GPA or if they just take my schools GPA. DO medical schools look down on retaking classes?

mark-ER

Retaking to up your grades from a B to an A is OK, but I question why you decided to do it (perfectionist? want an absolute rock-solid handle on the foundational material? Make sure you have an answer to that question). Your GPA is well within norm (maybe even above average) for accepted medical school candidates. So do well on MCAT and take more advanced coursework, don't worry about retaking anything unless you get a C, C- or below.


LOW MCAT (501) and LOW SCIENCE GPA (3.3)- any hope for MD or DO school? Best chances?

Hi,

Here are my VERY low stats........

MCAT- 501 (took it one time this May) GPA- 3.4 Science GPA- 3.3

I am trying to apply for MD and DO schools this summer so these are the stats I will be submitting.

Any advice on MD or DO schools that accept these low stats? What are my best chance schools? Is there any hope for me? :(

mark-ER

Of course there is hope, but for now unless you have MAJOR extenuating circumstances (by major think - "I was an olympic athelete, and my training got in the way of studying for MCAT"), unfortunately you need to re-take the MCAT, if you hope to get any interview offers. That goes for both DO and MD schools. You need to aim for at least 507 and preferably much higher to compensate for your low GPA too. Talk to your pre-health adviser, get some more feedback from the forums here on SDN, but my strong recommendation is to take another year after getting your degree, do something to strengthen your application (research tech year, volunteer), while taking one or two science classes of interest to you (genetics, statistics) that would be useful for your future career, and studying HARD for the MCAT (think 4-5 hours a day for 3 months). Your road is not unusual (I am mentoring 3 undergrads in a similar position), and YOU can do it. But if you don't think you can handle that intense of a year, perhaps it's time to think of a 'plan B' for your career, maybe still in the medical field, but maybe not medical school.


UWO vs UTSC for pre med

Heyyyy! So as my name indicates, I have a really hard time making decisions & & I'm currently facing a severe dilemma lol. I am enrolled in the neuroscience specialist co-op program right now at UTSC and so far for first year, my marks have been around 76% (except for chem whoops). I am really afraid that my grades aren't gonna enable me to med (or rather, I KNOW that my grades aren't on par with med school standards). I was thinking of either switching my program and doing a double major in something like neuroscience & health studies (but then that might cause problems with getting all my prerequisites in-- which btw, as a sub question, what are the necessary prerequisites? & can one fulfill the prerequisites in a post-bacc program if not fulfilled during undergrad-- reason I ask is because what if someone had done an English major and then had a later epiphany in life that they wanted to do medicine but never actually took the necessary classes?) to avoid the really intense upper year science courses at UTSC that could cause my GPA to drop even more. That, or I was thinking of transferring over to Western University for their neuroscience program as well. However, before making any decision, I wanted some advice (preferably from someone who's taking courses at UTSC -- not UTSG (because I've found their system aslo runs a bit different, for example first year chem in UTSG was orgo in half a credit which isn't done here at UTSC until second year under a whole credit (two courses for the year) , AND WESTERN Uni. I currently live downtown with family and I love being in the city, so that's something I would miss if I were to go to Western. I also find the city helpful in that there's a lot of job opportunities as well as events/clubs/just life really to participate in whereas London (from what I hear) is mainly composed of the university & not much else? In terms of commute, it's been alright as some of my courses have been web-optioned & it's easier to catch up without physically attending some of the classes. I also am afraid of Western as the reputation I hear of it is that there's a lot of distractions. etc. Is this true? Regardless, my biggest priority are my grades & I was wondering if I am right in switching over to Western for attaining higher grades? Now, I will admit-- first year I have been dealing with personal issues, including bipolar disorder which has made it difficult for me at times to concentrate during exam season-- I find myself contemplating life more than studying & I've also skipped numerous classes during manic periods and assumed I could teach myself the whole curriculum a week before the exam. However, I AM working on fixing those issues with my therapist/psychiatrist. Do you think if I were to alter my studying habits and attend/watch every class, I could possible attain grades that are good enough for medical school? Or should I switch universities completely???? I can't determine the exact issue at hand. Thanks a bunch in advance! Xxx

Mr.Smile12

If you are dealing with mental health issues, that should be among your highest priorities. Get yourself the help you need at your home institution, and find out what would happen if you changed institutions and thus campus providers. It's important to note that if you change schools, you need to have a plan to transfer the counseling notes to the other institution.

Your prehealth advisors at both institutions should be able to give you a sense of the critical questions you need to ask before you make a decision to transfer out. Academic rigor does count a bit with medical school admissions as shown by grades and solid recommendations from faculty who know you well. You also have to consider what work you would have to do if you transfer out of your home institution, take classes at other institutions and try to transfer them into your transcript (because there is usually a process for this), and support from peers. Why don't you ask the advisors at the institutions you mention to find other students who can answer your questions?


Advice Needed on what I should do

Hello,

I am a 23 year old newly college graduate. During my undergrad year, I went to a Community College and graduated with a 3.7 gpa overall. When I transferred to my University, I left home and lived on campus. My first semester as a biology major my gpa was a 3.3 and it was my first time getting a C grade. As my semesters continued, I began to get ill almost every week. I was in a car accident when I was younger that flipped my life upside down, causing me to lose my spleen, as well as become very depressed and somewhat limited with mobility. Once I was on my own and got older my depression and health became worse and ultimately started to show in my school work. I started to do horrible and even had to drop classes due to being ill. I finished school with a 2.1 overall average, a 2.2 in biology, a 2.6 in my psychology minor and a 3.6 in my African studies minor. My plan is to go to medical school and become a doctor. But, I know with my grades my chances are not very high unless I bring them up.

I am taking a gap year, maybe even longer, but I am going back and forth with taking classes to go to nursing school and then attempting to go to medical school because I do feel very discouraged. I know my options are very slim but is there anything I can do to bring up my grades? I haven’t taken my MCATs yet but my goal is to take them in a year. I don’t know how long it would take for me to bring my grades up but I don’t want to give up on my dream of becoming a surgeon.

I know I’m not unintelligent but as time progressed my health and depression just won. But I have started taking care of myself and now I am ready to make a change.

Any suggestions help!

Mr.Smile12

Whatever you wind up doing, make sure you connect with the health professions advisor at the four-year institution/postbac program you decide to enroll in. You also should talk to a few admissions officers way in advance and make connections with practitioners as you shadow them. Nursing school is totally different from medical school and has its own pressures that you may not appreciate, especially if you want to be a surgeon. You should consider surgery assistants as a career (or other physician assistants, anesthesiology assistants, etc.) and see if that is something you want to do. Being a surgeon is really stressful, and anything you can do to avoid relapsing in your mental health problems is going to help you in the long run serve as many patients as you can.


Extracurr activities, volunteer experience: How many hrs is competitive for matriculation to medical school ? Advise on specific activities?

Volunteered in long term care hospital once (2-4hrs)/week for over a year Volunteered with a community organization helping food insecurity seasonal approx. 4 hrs/week for 16-20 weeks over 2 years Volunteered with a seasonal sporting competition as a safety team member 9hrs/day for approx a week per year over 3 years Volunteered tutoring high school students 2-3 hrs/ week. seasonal for 10 weeks over a year

How many hours is considered competitive by med school admission committees? And what do I do now?

I have enjoyed what I have done so far, but, I may want to explore more areas. Is that bad in the eyes of med school admission committees? Should I continue with the same activities? Some people pick one or 2 activities and continue for years; others like myself like a wide range of activities and enjoy new experiences and challenges.

Please advise.

mark-ER

Just to be clear/blunt: volunteer activities are pretty low on the list. Clear-cut stats (your major, GPA, MCAT score, the university you attended) are just much easier to evaluate and according to many academic articles at least somewhat predictive of your success in medical school & beyond. So the key is having done "something" to expose you to clinical medicine/clinical practice/sick patients. The minimum is ~50-60 hours on a regular basis -- 1-2hours a week for a year or so. You have way more than that, and more is better, but with rapidly declining "returns". Bottom line is: do what you want to do & do activities YOU enjoy. If I had to give more specific advice -- for someone with your extracurriculars the one thing that may be missing is exposure to research. Experience asking a scientific question, collecting & analyzing data, interpreting literature and maybe writing an article is of great value in medicine & science in general. Good Luck.


Chances of Acdceptance and Avg MCAT for potential target schools?

Am I a good applicant for this cycle with a 3.9cGPA 3.7+sGPA and a 508 MCAT (129/125/127/127). Having trouble determining the avg MCAT for my target schools URM: Puerto Rican and Peruvian

Shadowing

-Anesthesiologist: 55 hours
-Atlantis Project (shadowing doctors abroad in spain, utilizing my knowledge of Spanish): 72 hours split into 4 departments (Anesthesiology, ENT, Urology, Cardiology), received leadership scholarship for this program
-Orthopedic Surgeon: 50 hours
Research
-Patient Contact: Assisted people with their workout routines to test power vs. strength training in the Elderly, focusing on the effects to their ADLs. Over 84 hours in the one semester the lab ran
-Wet Lab: Working with Schwann cells and their ability to produce myelin to help with the repair of damage axons following a spinal cord injury (3 semesters: 400+ hours)
-Summer Undergraduate Research program @ Sloan Kettering (IP): Testing the ability of 77 different compounds to reduce cell viability in Diffuse Intrinsic Pontine Gliomas (poster presentation at the end) (So far: 130 hours, anticipated by the end: 380-400 hours)


Extracurricular
-Working within Biology Advising Office (3 semesters)
-Workshop leader for Intro biology courses (2 semesters)
-Tutor for student athletes (3 semesters)
-Tour guide for the university (House Leader for my specific group: entire organization split into 4 houses)
-Secretary for my fraternity (highest GPA on campus)
-Biochemistry tutor/proctor for professor (wrote LOR
Volunteering
- Note Taker for the office of disabilities (receive 60 hours of volunteering per class per semester: accumulated 300 hours)
-Blood Bank volunteering when I return from the atlantis project --> 30 hours
- Local Rescue Mission that I have been a part of since 2007. Volunteering for holiday dinners, was able to accumulate majority of the hours while in middle and high school, but had to slow down in college because I am not always home for the holidays (150: 100 before college, 50 since)
LOC
- All professors thought highly of me and spoke with them almost every class. No hesitation when I asked for LOR. A or A+ in their classes
GPA trend
-hard to say because I had a 4.0 my freshman year. Dipped down slighty when I took orgo 1 (B+) but got an A in orgo 2. Multiple semesters with a 4.0. Non science GPA is a 4.0. Mainly got non-As (few) in labs that ended up having significantly hard finals that had content we didn't learn in the class associated with it.
State of Residence: NJ

mark-ER

Your combination of grades, classes, extracurriculars make you a very solid candidate. You can likely get into medical school as is. But if your goal is to get into one of the top 10-20, elite schools, you should try to retake MCAT and score at least in the 512-515 range. Based on the experience and drive you seem to exhibit thus far, and some SERIOUS prep, that score should be achievable. Talk to your school's pre-health adviser and consider the pros and cons of applying now (with updated MCAT to come later) versus waiting to retake the MCAT. Besides having the choice of schools/potentially a higher-ranked medical school, testing up to a higher MCAT may mean better financial aid package (esp. scholarships). So think hard about it, but strongly consider retaking the exam.


Low Stats - Advice Needed!

Hi there,

I need advice about my GPA. I have terrible stats right now, with a 2.8 science GPA and 3.1 overall cummulative GPA. I am Biology major and intend to take the MCAT next summer, and have already started to take a course that my college offers for the biochemistry section of the MCAT. I have reflected and thought about my current situation for Medical School and I am getting extremely worried. With the appilcation process more competitive now than ever before, I dont even think there is a .1% chance I'll get in anywhere. Should I take a Gap year and take a couple of more classes to fix my GPA? Or should I just power through, try to make straight A's and hope for someone to look at my applicaion? I also have just finished my sophomore year of college, and I am so worried that even if I do decently well, say 3.6+, There still would be doubt if the ADCOMS look at my application - I have other extra curriculars ( research, leadership ,shadowing/volunteering), I felt like I needed to make up for what my GPA lacked, but without decent stats I'm afraid it would be all for nothing. I just have no idea what to do and this dream about becomig a doctor is slipping away. Your input and opinion on what would be the right course of action ( what you think) would be more than enough.

Thanks.

Mr.Smile12

You should have already gone to your prehealth advisor. If you are a sophomore, you are quite a bit early to discuss gap years especially with that GPA as it stands. You should do a lot of networking to be sure you know why you want to be a doctor in the first place. But no doubt your grades are very important and you need to change some of your habits and priorities if you're going to discipline yourself to be ready for the rigors of medical school and training.


Discouraged/depressed after mcat scores

I got my MCAT scores back yesterday, and i’m devastated to have gotten a 506. I took 4 practice exams, and scored above a 508 on all of them so I don’t know what happened. I have a 3.71 GPA, am a medical scribe, have done biomedica research at a hospital, and computational molecular biophysics research at my university. I have traveled the country presenting my research and am the president of a volunteer ACT tutoring organization in addition to significant other volunteering. Do you think I still have a chance to get into an in state school at least (i am from kansas). Beyond that, how do I forgive myself for this mistake and move on? I feel like such a failure, and like I dont deserve anything good after this.

mark-ER

With your MCAT score and GPA, and unless you have serious extenuating circumstances, you will have a very hard time with applications. You can try and I would submit multiple (25+ applications to MD schools, and many, if not all, DO schools). But my strong recommendation would be to unfortunately fight the war on 2 fronts -- redouble your efforts and retake MCAT as soon as possible, try to score 510 or above. If your scores come out in September-October, then release new score to schools you are applying to, and depending on how many interview offers you got, perhaps expand your application window. Good luck.


What to do after this?

I finished freshman year with an okay GPA (3.7) at a school knowing for its low GPAs, and I was happy with it (this included about 4-5 science classes, including Orgo 1 and both bio 1 and 2). I am pretty good at the sciences, but I wanted to study abroad junior year, so I decided to take physics 1 and 2 (calc based) at my home university this summer. However, I turned out to have really bad luck, and (while I'm not trying to make excuses), I mean I had a 100 on the last day of class and the final was just 10 MCQs and 50% of my grade, and I failed it so I ended up with a C (an A in the lab component but that doesn't affect the overall grade).

I don't know how to come up from that, because the logical answer would be to get an A in physics 2, but it's taught by the same professor, and while I know I could try harder, I don't think an A is possible at all! (or maybe that's the fear talking, idk... but seems risky to take a chance, since I thought I was going to get an A in physics 1  like 3 days ago because that's what I got on all other tests) 

Now I have two options - 1- I was wondering if I could take the 2nd physics class of the algebra based sequence. but idk how bad that looks? I mean I'll def get an A (bc a better teacher and an easier course overall), but does that look like a really bad idea? I mean med schools don't require calc-based physics courses anyways so why should I stay in one if it's killing my GPA. Ideas? 2- Go back to my college and take physics 2 there. This is also going to be a little tricky, as it will probably mean I can't do my minor in chemistry because I don't want to be taking 3 lab courses at the same time (I'm a bio major too, so I take like one bio course and one chem course each semester). This is also not going to give me an A (from what I know about the physics courses at my college) but there's no chance of a C or anything. But it ruins so many of my planned out classes and it's just very frustrating but seems like the best bet...?

any advice would be appreciated! I'm really upset because I think overall I'm a very strong candidate and would have become a really strong applicant with a high GPA and a high MCAT score (which is usually something my liberal arts college boasts about). I don't know what to do anymore.

Mr.Smile12

You should always check with your prehealth advisor when it comes to course selection. I'm not sure if your major required you to take calculus-based physics, but you should check if algebra-based physics would be satisfactory to your degree requirements. Otherwise, most medical schools won't care.


Drug screening requirement for incoming students at AUC

I am aware that drug screening takes place prior to, and during, clinical medicine/rotations (years 2-4). Does AUC require a drug screening during the first two years of medical sciences/upon matriculation? Just want to know the facts.

Mr.Smile12

AUC should be able to give you that information directly. I'm sure that you probably have to be tested or get a background check before starting clinic rotations as that is pretty standard protocol nowadays.


U.S. medical school pre-reqs in Europe?

I am a recent graduate with a bachelors degree from an accredited U.S. university. While there, I’ve completed some of my medical school pre-req but still need a couple (I.e. organic chemistry and physics). There is a strong possibility that I will be traveling to Europe (as a military spouse), but still want to pursue medicine. So, I was wondering would I be able to complete the pre-reqs abroad and have them be recognized by U.S. medical schools?

mark-ER

The short answer is 'yes', for many European universities, US schools will accept your classes and grades in lieu of course requirements. Couple of things to consider: make sure the classes are in English and the university is accredited by a body recognized in the USA. If you know where you will be stationed and what possible universities are in the area, you can even gently approach the AdCom for your in-state medical school to confirm (preferably in writing). The disadvantage is you might not be used to the way courses are structured and graded, plus there is considerable logistical issues with getting transcripts and figuring out grades/GPA esp. when combined with American coursework; advantages are cultural enrichment and (probably) very low and/or military-subsidized tuition.

Besides enrolling in classes at a local university, there is always an option of doing online courses, and many reputable universities (Drexler and New Hampshire come to mind) offer such programs, or if you are close to a major metropolitan area, a lot of US colleges/universities have study-abroad programs that offer credits for that university, which is again based in the USA... with US professors, grading, etc. The downside is likely expense and many of these away courses are in language/history/sociology not in sciences. Good luck.


What not to write in med school apps - geography?

Do med schools consider where their students want to practice medicine in the admissions process?

My school adviser told me to make sure I focus on wanting to practice in the US since I talked a lot about my international experience in my PS, so I'm aware of that. But I also read an off-hand comment on SDN about how med schools like their students to practice in the same general area after they graduate. I've been drafting some of my secondaries and focusing a lot on my home state, which is well known for its medical inequality, and how I'd want to eventually go back and work on these issues as a physician. The only problem is that most of the schools I'm applying to are across the nation from where I live, so after reading that comment I'm worried that admissions committees would be put-off by my forceful dedication to leaving the area I'm applying to. Should I change my essays to talk about dedication to patient-centered care (which I'm also passionate about) or something else?

Thanks for any advice!

Mr.Smile12

When picking schools to apply to, you have to read the mission statements to see if they are focused on providing service to the local community and the activities they organized are focused to that degree. If you are trying to apply across the nation, then you should be very smart in reading these mission statements, curricular activities, and state residency composition to help you select your preferred schools. Don't try to err to being more generic (patient-centered care... every medical school should be teaching this by now) if that is not what you want to do. Otherwise, most schools I don't think really have a strong preference about where you want to practice. Many of them know that students probably have a strong desire to return to their communities where they and their families were established ultimately.


Rejected an MD acceptance

In the 2017-2018 cycle, I applied to 24 schools, of which I interviewed at 2 and was accepted at 1. The acceptance came from USUHS (the military medical school). After a lot of deliberating, I decided to decline the offer (further explanation below). I am in the process of reapplying, however I began reading that schools to which I am applying this year may know that I was previously accepted and thereafter declined the offer to an MD program. Can anyone (ideally people on admissions committees/with other experience) confirm or deny this?

Additionally, because I am already in the process of applying, would it be reasonable to get out ahead of the story and make it abundantly clear in whatever way I can why I chose to decline the offer? This would have to be in the secondaries, seeing as I already submitted my primary.

Decision to reject: First and foremost, my decision came as a result of seeing two of my siblings (who are current officers in the military) become frustrated with the bureaucracy of the military (long story there, but basically one was a victim in a car accident and the process of reinstitution into his unit was extremely tedious and time-consuming). Another reason is that I became aware that my ability to pursue some specialties may be limited based on what the military match has to offer (USUHS graduates apply to a different match cycle rather than the citizen match). For example, I may be unable to pursue ENT as a specialty if the military does not need any more ENTs and thus does not create spots for postgraduate training. Additionally, the turbulent political climate dissuaded me.

About me: Graduated in 2017 with B.S. in biology from my state university. GPA 3.83. MCAT 516. Some college athletics. Some research experience. ~400 volunteer hours. Currently working as medical scribe and tutor. Child of MD. ORM.

Mr.Smile12

I'll admit that takes some courage to turn down an offer, and you will have to disclose that in your reapplication. I would talk with your prehealth advisor about how you should try to address this in your application. The problem is that you weren't that successful getting 1 of 24 schools you applied to accept you, so there's a lot more that you have to overcome.


Chances of Acceptance to American Medical School

Originally from NJ. Graduate of Wash U with Drama major/Bio minor, 3.4 GPA, 3.07 science GPA (with upward trend), and 499 MCAT. Spent two years scribing, doing clinical research, and taking upper level bio courses, in which maintained 4.0 as non degree student and now with 3.42 overall GPA and 3.25 or 3.28 science GPA. Applied to MD (20 schools) and DO (8 schools) and did not get in, with the exception of one interview from DO school. Re-took MCAT and scored 512 and now looking to reapply. Also have many hours of shadowing and volunteering. Want to know my chances of getting into an MD school and DO school? What would be a good list of MD schools to apply to?

Mr.Smile12

I can't quantify your chances, but I think you would get a bit more attention with the better MCAT result. It still depends on where you have an in-state residency advantage, but I would start there and apply regionally.

mark-ER

I second the other expert. Strictly numerically you are a borderline candidate, though your determination to improve your grades, do research/get clinical medicine exposure and improved MCAT speak well for you. If you think you can do better on the MCAT (at least 515), I also agree that retaking the test for a third time may be your best option. If you think this is the best you can do +/- 5 points, then apply this cycle and see how it goes. A list of 25 MD schools, focusing on all schools where you have a reasonable shot -- all in-state school, lower-tier private schools and schools where you have strong personal links to (grew up in the area, have family there) & take substantial % out of state, are the place to start. Several (8-10 DO) schools are a good idea too, as is a strong 'plan B' in case you don't get in anywhere. Sorry, I wish I could be more encouraging ... maximize what you can control (great support letters, great personal statement, practice/practice/practice your interview skills) and be on time with your application -- remember medical schools have rolling admissions. Good luck.


Choosing a Major fro Pre-Med with incoming Dual Enrollment credits HELP

Hello, I am going to a University this fall and am bringing in over 62 credits from dual enrollment. I really want to knock out the rest of my prerequisite courses for medical school by the spring of 2019 and take my MCAT that summer. However, the problem I am running into is choosing a major that would be "not too difficult" and allow me to give the MCAT my all. So my question is should I extend my University time to 3 years instead of 2? and if so should I go for a more challenging degree. Or should I stick with 2 years and possibly rush my MCAT and extracurriculars? I mean if I can get my MCAT and everything finished in 2 years (even if it is difficult) why shouldn't I.

mark-ER

Big question: why are you in such a rush? 2 years vs. 2.5-3 years is pretty trivial, and would allow you to get a bit more well-rounded college experience. If you are concerned about costs (appropriate) and/or are just a very focused, single minded individual (that's good too), then with your credits it sounds like you could put your head down, get a biology major and get thru college in 2 years, taking MCAT next summer. But that's a pretty intense goal, and a goal, which I strongly believe with the advantage of hindsight, you might eventually regret. If you are indeed that very goal-focused, single-minded individual even stretching things out a bit to 2.5 instead of 2 years, would give you added perspective and perhaps allow to take useful/enriching classes to your future career like molecular genetics, biochemistry, with an additional half year working as a tech in a lab on a project (perhaps present at a national conference or even publish).

Lastly, if your goal is getting into medical school, you should not worry about choosing "easy" major, but one that you have an interest in, that will enrich you as a person and give you insight into your patient's lives and difficult questions/situations you will run into in your own life. So venture a bit further afield; take some history, linguistics, philosophy, music... it might be your last chance to do so formally in an educational setting.


Should I retake the MCAT if I already sent in application?

I took the MCAT in May and just received the score back and I got a 502 (124 C/P, 126 CARs, 127 B, 125 P/S). This is disappointing because I was used to scoring 510 on practice exams, but I was extremely nervous that day and knew that I didn't do well on the C/P section so it threw me off for the rest of the exam. I submitted my primary application to only one school and it was processed already.

Should I retake the MCAT at the end of July and still be able to apply this cycle? If so, should I wait to add more schools to AMCAS until I get my scores back late August?

The rest of my application is as follows: I go to a top 10 public university with a 3.8 cumulative GPA, 3.68 science. I have solid letters of rec from 4 professors and a research professor, 2 full years as a research assistant while being first author on 2 conference abstracts, 150+ hours of clinical volunteer experience, 2.5 years of being a Chemistry TA, and 30+ hours of shadowing. I am willing to take a gap year if needed.

Mr.Smile12

If you know you can focus your energy on preparing for an MCAT retake, I would try to see about retaking the exam again, especially if you feel nerves got you. I don' know what other universities you may be looking at, especially ones close to you where you may have a state residency advantage, but I would consider expanding your net to include any such schools before your retake your exam. I don't think with your GPA that you would need to take a gap year, unless you're trying to really shoot for high-reputation high-metrics schools.


How can I best prepare my self for pre med courses as an incoming college freshman?

Hello, I am an incoming Pre Med Student as I will be a freshman this fall. Although I am majoring in Biochemistry and Molecular Biology, I do not have a good background in science. My Biology Honors teacher freshman year went on maternity leave one month into the school year and never returned, our substitute was a computer science teacher that volunteered. We hardly covered any material that year and were all just given completion grades. However, in Chemistry, I had the highest average in my class the entire year and I truly love the subject. I transferred schools midway through high school. My new school didn't allow me to take AP courses in Biology and Chemistry because of a weird policy regarding taking college-level courses. So heading into one of the most rigorous tacts of study in the sciences, I was wondering if I should try and self-study Biology this summer before attending college to make up for my lack of knowledge in the subject. Or do entry-level college bio courses build that foundation? Thank you!

mark-ER

Overall, I agree with the sentiment in your last sentence -- that's what entry level bio class is for. That being said, if you feel your HS preparation was very sub-par and you feel extra motivated, you can read a bit at night (1-2 hours couple of times a week) for the summer, you will get a bit of a flavor for college level classes. The book I most recommend, that's basic but well written with online modules is "Molecular Biology of the Cell" by Alberts. Now, if you are a super-duper-overachiever, you can even look for a free, online course (such as those offered by MIT), that uses this specific book to keep yourself on the schedule.

I would strongly recommend the middle-of-the-road approach of just setting yourself up on a schedule, and reading a chapter a week from the book I recommended above -- you should be able to find it in a library. Make sure you take some time in the summer to relax & recharge, too. I remember I worked 2 jobs in the summer between HS and college (60+ hours a week) and was quite tired by the end of the summer! Don't repeat my mistake, of being tired not enjoying freshman orientation as much as I should and having a hard time starting up.


Should I take a gap year before applying to medical school?

I am currently in the summer between my second and third years of undergraduate studies at a public university, majoring in Biology with a minor in Philosophy. Currently, I have a 3.95 cumulative GPA and a 3.96 cumulative Science/Math GPA that does include a year of upper-level coursework. Regarding my extra-curricular activities and experiences, I have completed approximately 200 hours of volunteering in under-served areas near my university, abroad, and inner-city neighborhoods around the state. This volunteer experience includes clinical and non-clinical work. I've additionally accrued over 600 hours of experience as a medical scribe in the Emergency Department, though I have no other clinical exposure. I also was elected as President of a Biological honor society chapter, participated in club sports, and will be a TA for a General Chemistry course in the upcoming academic year. I have not completed summer research yet, but I plan to do so next year; however, my Organic Chemistry course's laboratory component included a semester of collaborative research as part of an ongoing project by the professor.

Based on the information presented here, and assuming I continue to gain volunteer hours and research experience, is it reasonable to continue my plan of applying traditionally? I would take the MCAT in late May/ Early June 2019.

I apologize for the length and specificity, as my academic adviser recently retired and I have not been assigned a new one. Thank you for any input!

mark-ER

With minor caveats**, you will be fine, applying in a standard cycle w/o taking an additional year off. Interesting major-minor combo, lots of activities, top-notch GPA... if your MCAT score is in line with your other accomplishments, you will be in very good shape to matriculate into medical school, likely with multiple acceptances.

Caveats: you will need to have more research experience if you want to apply MD/PhD, or potentially more work/enterpreneurial experience if you want to go MD/MBA route.


Should I retake the MCAT?

I am in the process of applying to med school right now. I have a 3.90 overall GPA and a 3.97 BCPM GPA. Plenty of volunteering, shadowing, some leadership, research, etc.

I just got my MCAT score back today and I got a 512. This is extremely disappointing as I had been scoring 515-520 on the AAMC full length tests. It was basically just the CARS section that I did bad in (125) that pulled me down so far. Anyways, should I retake? This is probably the only thing that is holding me back. I have no desire to go to a top 20 school, and I really just want to go to my state school, which has a median MCAT score of 511.

mark-ER

You will be fine in terms of admissions. With a broad enough application pool (20-30 schools), focused on schools that are more likely to accept you (in-state, ties to mentors and family/geography), you ought to get at least one matriculation offer (likely multiple).

Yes, it may be disappointing to receive MCAT score lower than you expected; if you really want to (in part to prove yourself, in part to torture yourself), you can retake it again. If so, I would encourage you to do so later in the application process once you already have a few interview invites in hand, just in case something goes completely off.


Microbiology and Immunology together

I am considering on taking Microbiology (5 credit) , Immunology Lec (3 credit), Immunology lab (1 credit), and a 3 credit history class this fall 2018. I am wondering if it is a goof idea to take Immunology with Microbiology? Any advice? I would like to get an A in Microbiology, but I don't want to have too heavy of a load.

Your help will be gladly appreciated.

mark-ER

Do-able. Immunology and microbiology (somewhat) complement each other. They are both interesting courses and with history on the non-science side you have a bit of balance. This should not be considered a 'heavy' class load for somebody who is trying to get into a medical school (** I am assuming you are a rising junior, not a freshman with AP credits only; if you are a freshman that is a bit foolhardy).


How can I apply for Post-Baccalaureate Course for MD?

I am looking for some suggestions. I am living in Chicago. I want to apply for post-baccalaureate course (Pre-Med). I am a foreign pharmacist and I obtained my whole education outside of the United States. I have science background. Can someone guide me?

Mr.Smile12

There is an online directory of postbaccalaureate programs organized by the AAMC which you should use. Contact the program officers for those programs that are best suited for your situation.


College major

Hello,

I'm an incoming Freshman at UC Davis. My major is currently Biological sciences. I'm considering changing it to Human Development or Psychology. My senior year of high school, I really enjoyed studying AP psychology. My main concern is that when I'm applying to med school, will it be better to have a bio science or a psychology major? I've been told that majors do not really matter but I want to take the path that will prepare me for med school.

mark-ER

College major is less important than doing well in school (high GPA), enjoying what you do (taking classes you like) and getting a well-rounded education that will prepare you well for medicine, but life in general, too -- plenty of people change their mind about doing medical school in the process, so you want to keep in mind plan B (and C and D). That being said, if you do decide to change away from biology/physical sciences, you still want to take challenging science courses to fulfill medical school requirement, and then some. Make sure you connect well with faculty in your major, so you can get strong letters of evaluation and opportunities to interact with faculty/graduate students academically (get involved in a research project or two). Good luck.


MD or DO? And Post-bac or not?

I’m 27 years old and a mom, so I’m definitely a non-traditional applicant. I’ve been really struggling with the decision of what path to commit to, and I would love to get a bit of input.

The basics: I got my degree in biochemistry/cell biology with a 3.54 gpa. I did retake one class though, so if both grades get counted then it’s really a 3.48 (bummer). I got a masters in biochem with another meh gpa of 3.57. I have about 4 years of lab research experience and a couple publications where I’m 4th/5th author.

I can get good letters from professors I TAed for and from my most recent job (from a physician specifically). Aside from those relationships though, I don’t really have any undergrad professors to ask for a LOR.

I’ve always thought about being a doctor. I got distracted by the PhD research route and it took me a long time to realize that basic science research isn’t my strength. It also took a lot of time and attention away from getting more A’s honestly. Now I’m sure I want to be a doctor, but I’m torn on what path to take.

I haven’t taken the mcat yet but I’m confident in my ability to do great on it.

If my interest is in holistic, integrative medicine and I most likely want to do internal medicine, is it best to go for DO or MD? I’m not really interested in OMM (although I wouldn’t mind learning it either). I AM concerned about DO’s not being regarded as highly as MD’s in some settings, and about not having the best chances at working in a good hospital at the end of all this. I’m afraid to think “eh, it’ll be fine” like I did with the PhD route, and then realize I’m in the wrong place (again).

Is my gpa good enough to try for some DO schools, like Western University in Pomona? Should I do a post bac for a decent chance at DO and/or MD schools? If I do a post bac and I would like to finish it in a year (since I’m not working and I’m not getting any younger), what are good options in California? So far UCSD extension seems like a great choice but I might be missing some other solid options.

Really any guidance on any aspect of this (md, do, post-bac, mom applicants) would be awesome.

Renee_MD

Hi there,

Thanks for your question. I think the best way to for you to answer this question is to spend some time with an osteopathic physician. I recommend setting up a time to shadow an osteopathic physician and see for yourself if this is the type of medicine you would like to practice. Also, if you choose to apply to DO schools, most of them require you to have some experience with osteopathy so this will help in that aspect as well. In my own opinion, it sounds like osteopathy may be the best option for you. You will be more competitive at DO schools given your GPAs. And since you are interested in more holistic, integrative medicine then the DO philosophy would fit your preferences more. One other thing to consider is that in 2020, all the DO and MD residencies will be combined so you will receive the same post graduate training as MDs, and when you are in your practice, then you will be practicing essentially the same as MDs. Finally, you can check out all good post-bacc options by looking at the AAMC post bacc search engines: https://apps.aamc.org/postbac/#/index This is a great place to start to see which programs would be the best fit for you!

Good luck!

Dr. Marinelli


Chances of getting into med school low GPA

Hi, I'm 23 and graduated last year 2017 from University of Colorado

GPA is 2.9 past MCAT was 488, taking it again in July. My GPA tanked because I could only get C's in Genchem, Ochem, Biochem Worked at a hospital in Africa for 3 months Volunteered 200 hrs in undergrad at a hospital Worked as an athletic trainer Latina woman Colorado resident

I was thinking of applying this year to medical schools, but I am not sure if I should since I am retaking my mcat in July. I am not sure if it is too late. Also I am not sure what would be suggested for my GPA. I am currently at a cross roads don't know if to apply for a masters or go back to school to get a better GPA.

Bottom line. Should I apply to medical school this year or wait another year (which would give me 3 gap years) by trying to increase my GPA.

mark-ER

I know it might be painful, but take a year, a few post-bac courses (& do well), study hard-core for MCAT. To have a chance at med school (even DO/osteopathic, and even as a URM), you will likely need to get to at least 3.1 GPA (3.3 would be ideal) and get your MCAT score to as close as possible if not above 500. No, it is not an absolute rule, there are always people on the far left-end of the curve as far as undergraduate stats get in every year, but your chances of getting into medical school (and frankly even getting interviews) right now are very low, <<10%. Also, masters is not going to help your undergrad GPA, though as I found out on SDN forums, there are MPH/posbac programs, where you can get you a useful masters in addition to being able to take undergrad courses & study for MCAT. Talk at length with multiple people who have insight into medical school admission and seek guidance from a pre-medical counselor at your undergrad, but you will have a very hard road ahead to make up for where you stand academically right now. Not impossible, but very, very hard. Good luck.

If you want another opinion, for somebody in a very similar situation, search these expert forums for the following post "Low cGPA (3.0), MCAT 484 chances of USA or CANADIAN Med School?" The individual received similar advice


How can I fix my current situation and get into a Medical School?

I am presently 24 years old and I have been out of college for approximately 18 months. I have worked different jobs and have not went back to school. I am looking to matriculate into any reputable medical school as soon as possible. I am open to Caribbean medical schools that are reputable that would give me the opportunity to start in the Fall or at the worst in the Spring semester. I have had some life events that have resulted in me having a horrible GPA and low MCAT score. My Gpa is a 3.11 and my MCAT is a 479. I don't know what to do at this point but I still would like to become a doctor. I am willing to give it one more chance.

I am open to special masters programs (that will take me and whose deadlines have not yet expired) that will allow me an opportunity to "link" into a medical school without wasting additional time. I am not getting any younger, I would really appreciate it if someone who has been in a similar situation could help me out or guide me in the right direction.

I presently have a job working at an Orthopedic surgeon's office, but it is a low-paying job and not nearly enough to sustain me. If someone could please reach out to me or reply to this thread and help me. I would greatly appreciate it. I really need help, I feel like I am wandering aimlessly and I want to fix the situation that I'm in right now. Please help! (I'm seeking the path of least resistance)

Mr.Smile12

I think that if you're going for the path of least resistance, Caribbean schools are going to be the option you would explore. That said, it's going to be a very dangerous path if you wind up failing medical school entirely, and the numbers you have shown wouldn't give many admissions staff (at least for MD or DO in the US) much confidence. I would do a lot of homework with your options with coursework, MCAT prep, and admissions chances.


Where do I go from here?

I am going into my senior year of college and currently have a 3.5 cGPA and a science GPA of 3.17. My first three semesters I did very well and kept a GPA around a 3.8. However, I began struggling with some mental health issues in the middle of my sophomore year and have performed poorly my last three semesters, leading to the decrease in my GPA. I am planning on retaking my three worst classes ( I got two C's and a D) this year and then completing a graduate program before applying to medical school. In addition, I have extensive research experience and extracurricular activities. My school offers a graduate program in physiology which would be ideal because they have many avenues to pay for it without loans. However, I am concerned this will not be enough and that I need to complete a SMP, which are quite expensive. In my scenario, do I need to complete a SMP or will another graduate program be viewed equally? In addition, should I disclose my struggle with mental health when I apply?

mark-ER

I think masters vs. postbacc (either stand-alone or SMP) are both a reasonable choice, as further discussed below, with the big caveat that masters level courses are NOT included in your undergrad GPA. Unless your MCAT is absolutely stellar (515 or above), and even with that MCAT score your GPA (especially sGPA) is your biggest hurdle to getting into medical school. So, I agree with the approach of re-taking the courses where you got a C or below. Retaking and getting an A or A- would show you can handle the work, and at the same time increase your GPA.

In terms of masters in physiology in your institution vs. posbacc/SMP, I would advocate for postbacc for folks who are non-traditional, have been out of college for some time, do not have pre-med prerequisites in place and/or already have another postgraduate degree (MS, MPH, PhD, MBA, JD, etc). It sounds like you do NOT fall into any of these categories. So perhaps a masters would be the best course of action, especially accounting for cost of education (it sounds like you have an option to get into your local masters programs with minimal additional expenses/ student loans), it would make you potentially more marketable for future jobs (plan B), and again show that you can handle upper level science classes. Lastly, I always think of a "third way", out of the box solution... as a concrete example, University of Michigan has a masters level physiology program, that's geared to help people figure out their career path and potentially apply to professional health degree programs (DPM, MD, DO, DNP, etc). In the evenings (1 course/semester) one could then take an additional science class at a cheaper (& easier) local university, such as Eastern Michigan, Concordia, or similar. It would be a very, very tough year but also a good prep for medical school.


Can I become a Doctor?

I am currently entering my sophomore year in college as a Electrical and Computer Engineering major. Meaning, I'm more of a math major than a science. I love my major and school, I have an academic scholarship that covers my 4 years, I have a 3.7 GPA, but I don't see any bio courses and barely any chem. I am not sure what all the requirements are to get into med school are either. What would be recommended in my situation?

mark-ER

You are only a sophomore, you have time. Indeed, there is a core curriculum required to apply to medical school -- that includes calculus, biology, chemistry, organic chemistry + biochemistry, statistics and physics. You likely already have physics under your belt, same for math and maybe stats. You have at least a year, maybe more to squeeze those other courses into your schedule. You could double major; alternatively, you can do a post-bac or take summer courses, though that's extra time & cost. As you will hear elsewhere, medical schools want students in all majors/concentrations, not just 'vanilla' biology or biochem. I think an individual with strong mathematics or engineering background, good GPA and (most importantly) strong logical problem solving skills and communications would make a great doctor. Good luck.


Can you take the MCAT in high school?

I am interested in applying for a BS/MD program. The school I would like to apply to requires you to take the MCAT. The website said that your MCAT score would be one of the things that they look at during the application process, and as it is a program that high school students are able to apply to, I am taking this to mean that you would have to take the test in high school. How would this be possible?

Edit- I am doing a full diploma IB program, if that would help me.

mark-ER

You could, but I would NOT recommend it. I think you could be OK in chemistry and physics with very good AP classes, but even with AP bio you are way behind in terms of breadth of background (others with 3-10 college biology courses) and not taking O-chem. Plus however you do, that test will stay on your record; yes you can explain it away, but why? But, at least back when I was prepping for MCAT almost 20 years ago there was an opportunity to take a full exam on a trial basis, with an actual score but not recorded and not validated. So if you are really insistent about trying it out, consider that possibility.


Withdrawing from a summer class - multiple “curveballs”

I need some advice. I just finished my junior year of pre-med, and decided to take my physics courses during the summer before my senior year. I am not doing well in the course - my first exam was a 49% and my second a 52%. He drops the lowest exam. I am considering withdrawing so as to avoid a low GPA. I am taking this class a different college than the one that I go to, so my overall GPA isn’t affected. However, when they analyze my scores on my AMCAS application it will show up. I did running start, and then went to a 4 year college. Over the course of these 5 years, I have had 4 withdrawals (two during my running start and two during my 4-year college). All of the classes were repeated with a fairly good result besides one that was not repeated. If I withdrew, I would graduate, but I wouldn’t get my degree until later. Contrarily, if I don’t withdraw, I do not have any time to study for my MCAT. I plan to take the MCAT during January of the next year, and this is my last chunck of time to study.

My college will only accept a C grade or higher, and I don’t know that I can pass it like that based on my trends so far. If I withdraw, I already spoke to the professor and he said that am I allowed to keep going to the classes (which I will do) but I won’t be there for a grade.

My GPA is a 3.689 currently, with a science GPA of 3.61. Will another withdrawal be very bad? What do you all think I should do?

mark-ER

If you strongly think you cannot get above a C (at least a C+), I would consider withdrawing. If it was an upper level physics class, especially graduate-level course, you could consider taking it pass/fail, but it sounds like this is a basic/preReq class. Having trouble in a basic pre-req physics class is not a good sign, and to be blunt 4 Ws (even with relatively good grades on repeats) is also a red flag. And unfortunately, yes your AMCAS application listed GPA will be affected -- no matter when/where you take your undergrad courses, they count toward your GPA. So even if you took an undergrad courses in high school, not in the college you eventually attend, yes these count. Ultimately, MCAT is the 'great equalizer' so if you do very, very well on MCAT, especially in the physical sciences section (assuming those are the courses where you had Ws), that will mitigate things a great deal. But again though not a dealbreaker, multiple (3+) withdrawals are a potentially serious red flag, so be ready for questions about this in your interviews. Bottom line: if you can bring up your grade, that should be your approach first (talk to your professor and/or TA about it & work your butt off).


Please help me with the school list, GPA 3.7, MCAT 511 (128/126/129/128), NYS resident.

GPA 3.7, MCAT 511 (128/126/129/128), average ECs, NYS Resident.

Hi, I’m trying to finalize my list. Plan to apply to 30ish schools. This is a list of 40 schools. Can you please help me narrow the list to 30? Is there any other school that I missed and should be added? Do I need to apply for DOs, too? Really appreciate your input.

1. Icahn 2. Weill Cornell 3. Pitt 4. Emory 5. Albert Einstein 6. Hofstra 7. Brown 8. University of Rochester 9. Ohio State University 10. SUNY Downstate 11. University of Maryland 12. Dartmouth 13. Thomas Jefferson 14. Georgetown 15. UMass 16. Tufts 17. Rutgers 18. Chapel Hill 19. University of Colorado 20. Temple University 21. SUNY Stony Brook 22. SUNY Buffalo 23. Robert Wood Johnson 24. New York Medical College 25. Drexel 26. Albany 27. SUNY Upstate 28. Tulane 29. George Washington 30. Rush 31. Virginia Tech 32. Virginia Commonwealth 33. University of Washington 34. University of Tennessee 35. Wayne State University 36. Chicago Medical School at Rosalind Franklin University 37. Loyola University Chicago Stritch 38. Toledo 39. Michigan State University 40. Howard

Mr.Smile12

This is definitely a lot of schools. What you're not doing is giving yourself a chance to see if you fit with the educational mission of each school. I suggest this article to help you with the process: https://students-residents.aamc.org/applying-medical-school/article/make-sure-you-fit-schools-mission/ .

mark-ER

I wanted to 'second' the other expert opinion. I have seen many applicants with your very solid, if not spectacular stats. Unless there are other major red flags you are not disclosing and/or you have issues with nerves while interviewing, you should get in somewhere. I would also recommend narrowing down the list to about 25 schools. If nothing else, it is a lot to pay upfront. I think the top of your list looks good for your geographic location/in-state status, you could flip some of the schools on the lower end of the spectrum for some mid-upper tier places (perhaps UChicago or Vandy)? Unless you have geographic ties to the area (such as close extended family, grew up there), exclude state schools that do not take too many applicants from out of state (Ohio State, U of Maryland, Michigan State, etc) and again put in a few mid-upper tier places. So bottom line is go down to 25, narrow down your list as recommended by me & the other expert and IF you get tepid response (1-2 interviews only), you can always expand the list later.


LORs with AAMC/AMCAS IDs being sent to AACOMAS

So I already applied to MD schools this cycle but am considering DO schools as a back up (my GPA isn't stellar). I have 9 letter writers that submitted LORs to Interfolio, but 7/9 put my AAMC and AMCAS ID numbers on their letters. I'm now in the process of completing my AACOMAS application but don't know whether it is a *terrible* decision to forward the letters with the IDs to DO schools.

I'm kinda in a sticky situation because some of my profs/physicians took *forever* to compose their letters and I'm sure it'll take another month for them to send a revised version. The AACOMAS application is well into the application cycle too. If I were on DO AdCom's, I'd be salty about reading something with an AMCAS ID on it.. but is it going to really reduce my chances? I'm an above average applicant for DO schools (cGPA 3.6, sGPA 3.5, MCAT 509), so I'm hoping that counts for something.

Bascially... to send or not to send, that is the question.

Mr.Smile12

I'm not sure there's any major disadvantage if you submit your application now. As for giving your ID numbers, your AAMC ID number is what is used to access your MCAT scores (I think), so it is fine if you had listed your AAMC ID. I'm not as familiar with whether you have different ID's for AMCAS vs. AACOMAS, but your AAMC ID should be fine.

That said, I'm not sure DO admissions committee members really have that much of a problem if you included your AMCAS ID. I'm sure many realize that applicants are casting a wide net, and the MD vs. DO stigma is not what it used to be.


Junior in undergrad, will I get into med school with bad grades freshman year

I'm a student at PSU and I'm at a 3.1. I'm certain that I'd like to go to medical school. My freshman year I withdrew from Biology. I also got an F in statistics (due to not spending time on this class and studying for calculus and getting an A). I retook the statistics class and also got an A. In both gen ed chemistries I got a C and retook both and got As. Every other class has been A or B. I suffered from depression my first semester at this school due to a suicide of one of my close friends during my childhood. Should I continue to pursue medicine or is my transcript already way too bad. I've done a lot of volunteer but I'm having some serious doubts.

Mr.Smile12

I'm sorry to hear about your friend. If you haven't actually shadowed physicians, you shouldn't stress out right now about applying to medical school. Make sure you know you're going to enjoy the lifestyle of a physician because it's going to be extremely stressful depending on what you wind up being attracted to in the field. Mental wellness is a big deal, and a lot of medical students and residents are saying that it's very important to keep your perspective and health while you're in school due to the stresses you'll experience. I'd talk with your prehealth advisor, and I'd envision your goal over a number of years beyond your graduation from college so that you are truly ready and mature for professional education.


HELP! Chance, or no? URM!

I've been wanting to type all of my info out for a long time, but finally have the energy and time. I've been majorly sick throughout my undergrad years (can provide medical documentation/proof), if needed. I am a psychology & addiction studies major. I've been wanting to be a doctor ever since I can remember (cliche, right?)

my gpa and mcat are HORRIBLE. I mean compared to my friends who have 4.0 gpas and such. To get offered an interview at my home state med school, you need a 496. My current gpa is 3.1. I do have an upward trend in my grades though. I have 7 years of healthcare work experience, amazing letters of recs, and volunteer hours. One big thing-I am from a reservation, and my state school is highly interested in Native American students and improving rural health care. Do I even try to apply to med school for 2019 or do a post-bacc program? I currently feel school burn-out, so I AM considering a gap year, or two.

I plan on retaking my MCAT, but do NOT see it improving. I think it was the hardest test of my life, and I hate how it determines how you'd do in med school. Any advice? Thoughts? Anything is helpful at this point. I'm smart, but not smart "enough" for med school I feel.

Mr.Smile12

I would talk with the representatives from your state school. Why are they interested in Native American students? What types of programs do they have where students and faculty help other Native Americans with health issues? Why do you think your impact in these problems is best suited if you are a physician and not any other health care professional?

I'm hopeful that you can do well on the MCAT, but know that it's the first of many additional high-stakes exams you will have to take in order to remain a physician. The MCAT is only the hardest test of your life until the next one comes along, and they will for medicine.


How will a dismissed drug charge affect my medical school application?

I got caught with lsd and the charge was dismissed through a first offender diversion program. I plan to be completely honest on my application about what happened and how I have learned from the experience. I am hoping that any adcoms or individuals that have been in a similar situation in the past can give me some insight on how much this hurts my application and if I should give up on medicine entirely.

Mr.Smile12

If you are totally honest in your disclosure, it shouldn't hurt you. You may have to answer a question or two on it in an application or an interview, but as long as you explain the situation and how you remediated, you should be fine.


What goes into my science gpa?

My major is exercise physiology and a few of the courses I have to take include: Exercise Physiology and Anatomy and Physiology 1&2 with labs. I was wondering if these go into my science GPA or not

mark-ER

Short answer -- they should count into sGPA. Longer answer -- I believe they should both cross-list within biology. Biology = science => sGPA.


Caribbean SGU or Post-Bacc SMP?

Here are my stats: 3.1+ cGPA, 2.9+ sGPA, 490+ MCAT (1st time), varied shadowing opps since I was young (ex. trauma, epidem, neuro, gen. surgery, etc.)

St George's Univ (SGU) seems to be one of the best Caribbean medical schools out there and it seems to be getting better...

An admissions counselor from SGU encouraged me to apply with my stats because they thought I would have a decent shot of getting into SGU, so I applied. I applied for SGU specifically because a family friend of mine went to SGU, completed their two year program there, and was able to match in the U.S. in Indiana for a residency program with family medicine. This individual is now finishing up their third year in the residency program in the U.S. and seems to be doing well.

I also applied for a U.S. Post-Bacc SMP program, which I have a good shot of getting into. The SMP program also provides MCAT-prep courses included with the curriculum. I could retake the MCAT at the end of the SMP, try for a higher score, and reapply to U.S. MD programs. This may be too preemptive, but I may reach a point where I have a choice to make: start the MD program right away at SGU or start the SMP program and retake the MCAT later for a higher score.

With a weak GPA (cGPA and sGPA) and low stats in general, I don't want to jump into the MD program too soon and unprepared, despite pressure from my parents to catch up with my HS peers who are already in the MD program. However, I have been reading a lot about the problems that seem to befall U.S. Med Residency programs as of late (not enough residency slots open, increasingly poor outlook for IMGs/FMGs w/ U.S. residency matching, etc) and I want to get into a neurology residency program, based on my past shadowing experience. Caribbean MD schools fail rates are typically abysmal and I don't want to become a statistic if I go to SGU before I'm ready.

The uncertainty of U.S. residency matching landscape in the next 2-5+ years plus the fact that U.S. Residency programs do not view students of Caribbean schools well leaves me concerned.

I know I want to become a physician. Given all this (lengthy) information that I presented, what would be best for me to do? Stick with SMP so I can retake my MCAT or go with MD in SGU? Or is there a third option you would recommend? Thanks.

Mr.Smile12

Among the Caribbean programs, SGU has a solid track record, but I am not sure what's going on with residencies in the future, since there are some significant changes that may make it challenging for most internationally-trained medical graduates. At least with most Caribbean programs, you have multiple entry dates, so you can go when it's convenient for you.

Now whether you should do SGU vs. a postbac, that's up to you. I don't know what your academic habits have been in succeeding well enough in medical school to get into neurology. Do you need resources to do medical student research to give yourself a strong shot?

You have a lot of questions you can ask on whether anyone from SGU made it into neurology, and what clinical rotations in the US in neurology SGU has agreements with. In the end it's your decision with your prehealth advisors and your evaluators (not your parents).


so what exactly *is* a post-bacc?

hello!! After messing around with my degree audit, I realized that if I took out all the pre-med reqs I have left, I can graduate from my university in a year, making me a senior as of right now. woo! buuut I still have eight pre-req classes left and a not too hot science GPA (3.2) (yikes, I know but as a liberal arts major that wasn't sure about medicine until a couple months ago, I'm not too mad about it). There's room for growth and I'd like to improve my GPA overall, so my advisor reccomended I look into post-bacc programs. I've been researching and they seem interesting but I'm still not sure if it's the right choice for me. I know I have a few options but right now I'm debating between finishing up my degree and finding a post bacc program/ taking my pre-reqs at my homeschool as a non-degree seeking student, OR go at a snails pace these next two years and graduate with my peers and with all my reqs done. I've found some interesting programs, like PRIME at Case Western Reserve, but I'm not sure if it's worth that giant price tag. Any information or advice helps

Mr.Smile12

The plain answer to your question in the header can be found online at https://students-residents.aamc.org/content/article/considering-postbaccalaureate-premedical-program/ . I hope you talk with your institution's prehealth advisors to help you with your choice.


Should I apply now or next cycle to better my ECs?

I graduated from my undergrad college in May of 2017. I took a gap year primarily to up my volunteer hours and work as much as possible to make sure the burden of my loans didn't fall upon my parents. As of right now I have everything filled out on my AMCAS except my ECs, and the more I reflect on it the more I feel I am lacking the correct amount of ECs to be considered a competitive applicant.

My ECs are as follows:

~24 hours of shadowing (planning on many more hours to come) ~177 hrs of non clinical volunteering (patient transport volunteer, feeding homeless, proctoring /grading science competition exams, etc). ~ 10 hours of clinical volunteering (participating in hospital emergency epidemic drills, participating in physician ultrasound training courses). ~2.5 years of D2 collegiate rugby ~ 7200 hrs paid nonclinical work

  • currently interviewing for CNA positions so factor in future paid hours of clinical work.

- 0 research - 0 significant leadership roles

Obviously my clinical volunteer hours are largely inferior and need some work. I applied to volunteer with my local hospice center so I should have future hours in that. But I don't want it to come off as shotgunning some hours into my clinical volunteering.

The other factors of my app would be my MCAT and GPA which are a 507 and 3.67c/3.69s respectively.

Should I take the time to apply this cycle or should I hold off and bolster my clinical hours? Any advice at all would be of great assistance.

mark-ER

I think your focus is a bit off -- you have plenty of activities, and a bit of extra volunteer work might help, but not very much (just a smidge, and just at the margins). Plus if you are interviewing for CNA positions, you will get plenty of patient experience so that's good enough to substitute. The bigger concern is your stats --- GPA is respectable, not stellar but OK (esp. if you went to a top-notch college and had a very hard major, think engineering). The bigger issue is the MCAT. 507 is on the left tail of the curve. What would BY FAR improve your application most is to retake the MCAT. Study hard, day in and day out for at least 2-3 months, do whatever you need to do to get at least 509-510 (above 512 would be ideal). You can still apply this cycle and take your chances, but it may be a tough road to plough. If you do plan to apply this cycle, apply to every single allopathic school where you have a chance (all in-state schools, all private schools at the lower end of the spectrum and even a few out of state schools that take substantial out of state applicants, esp. if you have geographic ties to the area). If you are interested in going Osteopathic, apply to all DO schools as well. Also, don't undersell yourself as having no significant leadership roles -- I am sure you did something that can be considered leadership.

Good luck.

Do I stand a chance?

Hello,

So I'm currently having a mental breakdown because I think I majorly screwed up my chances for med school. I graduated with a BA in Biochemistry and BS in Molecular & Cellular Biology. I graduated early, in 3 years instead of 4 (most people advised me to graduate rather than stay.) My cumulative GPA is 3.49 and my BCPM GPA is 3.36. I'm so disappointed and frustrated with myself because I let myself slip in my last semester and missed the cum laude and honors accreditation by dropping 0.01 points in my GPA. I wrote an honors thesis for the honors college at my school, which I thought would be a good addition to my app but now all my work for it is gone. I will be taking a gap year to gain more clinical experience/shadowing.

For my GPA, I feel like it could be attributed to many reasons. The number one main reason is that I should have worked harder. I'm hesitant to say this, but looking back, I think I had a lot of depressive episodes where I just felt demotivated and horrible, and let my grades slip as a result. I don't think the admissions board will accept this, which is understandable, but I truly think this did play a big part. Especially after my low score on the MCAT, I just felt inferior to everyone. Another reason is that my grandma's health has been declining with dementia and I took a couple short trips to see her. My father was also unemployed for a large part of my undergraduate years (I live at home), and the financial instability further contributed to my stress.

For more background info:

I took the MCAT once and my score was 503. Again, I'm angry with myself for this score. However, I only ran through a full practice one time, and an advisor told me that if I was able to get that score by practicing only once, I had a high probability of scoring higher with more practice exams. I plan on focusing and working hard to hopefully compensate somewhat for my GPA.

I have 2.5 years of research (I started the second semester of my freshman year and continued on to graduation). I will be listed as a primary author in a soon to be published paper. But because I did a lot of work for research, I feel like I didn't balance it well with my academics.

I was in the biochem club, pre-health club, biochem ambassador and mentor program at my school. I mentored new freshman and met with them monthly to provide advice.

I have 100 hours of clinical volunteer experience, as well as more social work volunteer experience (I counseled young children going through traumatic/emotional times after the loss of a loved one).

I'm planning on a volunteer abroad experience in Peru, where I will be shadowing a doctor. Plus it's been something I've always wanted to do. I will also be shadowing a pathologist.

Please let me know of any recommendations or if you think I still have the chance to turn my app around into a good one. Thank you!

Mr.Smile12

If you have a mindset that you are open to continuous improvement, then you always have a chance to improve your application. The problem is making sure you're focused on the right things, and it seems that right now you're not really focused on the obvious areas of improvement. Not knowing what other classes you had taken, you may have a chance to take some upper-level biomedical coursework either as a non-degree postbac or as a master-degree-seeking postbac. Your GPA's aren't that bad, and your MCAT may need more rigorous study.

That said, presuming you have regained some control over your mental health and your family situation, you need to show how you have been able to adjust and overcome these challenges and still do what you need to do to be the best physician you can be. Only then will one be able to give yourself a strong chance to getting in and staying in.


WAMC at Medical School and MCAT tips?

Hi everyone, I'm new to SDN and created my account specifically to get input on what you think my chances are for medical school! Here are my stats.

academics: I graduated from UVA with a BA Psychology degree and Sociology minor cGPA: 3.55, sGPA: ~3.21 -my sGPA somewhat inflated because I took a summer physics course at a 4 year university (that is "less prestigious") near home and got an A and A-. -sGPA is also low because I got a C in my first semester of chemistry, and a D+ in my first semester of orgo. I retook orgo and only got a B+ -also had a W my first semester of freshman year for calc 1 because I was naive thinking I could handle 17 credits the first semester -not a strong upward trend -did not take biochem so it puts me at a disadvantage for applying to one of my med schools (VCUSOM)

extracurriculars: -financial chair for premed organization for female students -asian student union for a year -peer advising group for a year -participated in a semester long leadership program for female students -undergraduate research assistant for a 1.5 years in the psychology department (no publications or presentations, but i did take an upper level psychology class where we were required to do a poster presentation for research we did during a semester) - volunteered my first 3 years of undergrad

-shadowed pediatrician for 5 hours and ob/gyn for 15 hours at the UVA hospital. General medicine in a private practice for 15 hours

my credentials aren't out of this world so I was wondering what your thoughts are. I am currently doing a gap year, thinking of taking on a scribing position since i don't have direct clinical experience. I also plan on taking the MCAT around March 2019 to apply for the 2019-2020 cycle, and any tips for studying and my time frame would be much appreciated.

also I plan to apply to mainly virginia MD and DO schools because I'm trying to get in state benefits - wanted to know what i should and shouldn't do.

THANK YOU SO MUCH!

Mr.Smile12

As an in-state applicant, I would think you may have a decent chance among the schools in your state depending on your MCAT performance, though your science GPA is a bit lower than desired. Network with the admissions officers at all the schools and get an idea of their treatment of your early grades. I would have taken biochemistry if you really wanted to have a shot at VCU if it is a requirement, especially if a significant amount of biochem content on the MCAT.


Medical School List: Request for Help!

Hi everyone!

I need some help with my school list. I am abroad right now, so it is not possible for me to have a one-on-one with my premed advisor. I came up with the list on my own, after doing a lot of research. What I want to do is study medicine so that I can participate in humanitarian efforts. Hence, I looked for school with strong global health tracks (that is the case for most of the schools anyway). Here is some information about me:

I am an NY resident. Female.

Academics: sGPA: 3.8 cGPA: 3.9 MCAT: 514. I was not too happy with my MCAT score, but considering the circumstances I took the exam in, I have been told to be satisfied hahaha! I have also been told not to retake. I have a Bachelor of Science from a top 20 school in Biology. I got some at least 3 research related scholarships.

Research: I have done research for two years in a cardiothoracic surgery lab. Unfortunately though, I have not been published. But I did present our poster a few times.

I have also done field research abroad for a summer. This is the experience I am most proud of. This was not published either, but I am currently working on getting it to be published.

Medical ECs: About 50 hours shadowing a pediatrician. About 80 hours volunteering in a cancer ward and ER. About 20 hours volunteering in a hospital abroad.

Nonmedical ECs: Multiple TA positions. Summer camp counselor for a year. I started a branch of ASBMB in our school. Multiple mentoring positions. Teaching abroad during my gap year, but doing so much more than teaching (volunteering, leading clubs, planning and leading camps, etc).

I hate making it sound like I am bragging, so if I am doing that, I apologize.

Anyway, here is my list.

Stanford ( I REALLY want to do something about the Cholera Crisis in Yemen, and Stanford seems to be the only school I can find that is involved in Syria. I know it is not the same, but I want to be involved in the program so that I can learn how I can get involved elsehwere). Hopkins Columbia Cornell Einstein Pritzker Duke Brown (There is a doctor here that I really, really want to work under. He has led humanitarian missions during the ebola crisis, which is simply incredible to me). Dartmouth Tulane George Washington Temple Chapel Hill Buffalo Stony Brook New York Medical College Drexel Albany

Any advice whatsoever, would be really appreciated! Thank you!

mark-ER

I think you have very solid stats (GPA, MCAT) and extracurriculars and you should be aiming to have a difficult choice -- acceptances at multiple schools. No need to worry about bragging -- your profile is VERY solid, but not absolutely top-notch stellar (i.e. not Rhodes scholar, etc), so you can and probably will get into one or more top medical schools, but unlikely to get into every single one. For instance, Stanford or almost anywhere in California will be a long-shot for anyone not from California, or without just absolutely amazing stats/story & extenuating circumstances (MD/PhD not withstanding). At the same time, that doesn't mean you shouldn't try. But overall, I am glad to see you have a range of prestige among medical schools you plan to apply to, with focus on ones you have geographic ties to (New York and Northeast). I would even encourage you to expand your list a bit, from 18 to maybe 25, so perhaps you have a bit more options. You seem to be interested in epidemiology/ID in developing countries (as your profile seems to suggest), so consider Emory (ties to CDC) and Vanderbilt as well. Also why not Sinai (due to your NYC ties), Pittsburgh (great hospital system, international outreach) and Mayo.

Lastly, do NOT completely focus on international/outreach work in your interviews and personal statement, to the exclusion of everything else. There is a still a LOT of need in the USA. As an example, Hopkins has an amazing school of public health with strong international outreach, but it is also in a rather 'rough' area of Baltimore, and wants to be a good player in the community. So they also look for medical students who aim to make their local communities better, practice in the USA, rather than necessarily far-flung yet poorly defined dreams of contributing to international missions. So be strong, express your passions/opinions, but not to the exclusion of everything else. Good luck.


Chances at med school?

Very worried about applying to med school and wonder if I have a strong enough application? About me: I'm currently still a California resident and moved to Reno for UNR Stayed close to Ca due to my father's diagnosis with dementia Grew up shadowing him as a prison doctor Worked in the entertainment business since the age of 7 till my junior year in college Last of 7 children and have had a hard time with my father diagnosis so a few bad grades are on my transcript: D first time around in calc repeated and got an A and a few Cs one in ochem2 Major: Biology Current GPA entering Senior year is 3.37 with hopes to come out around 3.4-3.5 Work as a full-time caregiver at a retirement home in lockdown/memory clinic Most recent work in the entertainment business was 5 billboards for Nugget Casino Hotel in Reno Volunteer as Big Brothers Big Sisters match to a little girl for last 2 years Shadowed in general surgery and internal medicine for a month in Trieste Italy Volunteer t Animal Shelter for 4 years

mark-ER

You have an interesting life story & some difficult challenges (seems ongoing, so best of luck). Your GPA will hold you back, but the biggest unknown is MCAT score. You need to get 510 or above to have a realistic chance at an allopathic (MD) school, 500-510 you can consider DO as well as a few MD schools (in-state, locations where you have geographic tie-ins). Otherwise, however painful, perhaps a year to re-group, post-Bac to improve your GPA (esp. sGPA), and really study up for the MCAT vs. year as a research tech at a university. Good luck.


Do osteopathic schools view scribing for a DO the same as shadowing a DO?

I am currently in the process of applying to osteopathic medical schools for the 2019 cycle. I know that DO schools highly value experience shadowing an osteopathic physician and submitting a letter of rec from a DO. I have extensive experience working as a medical scribe (1000+ hours) for family practice physicians who are DO's, and feel I can express my motivation for DO based on these experiences. I also have a great letter of rec from a DO that I scribe with . Am I at a disadvantage since I don't technically have any formal experience shadowing a DO? Or will they adcoms likely view the scribing as essentially "paid shadowing"? For secondaries that ask applicants about their experience shadowing osteopathic physicians (western U secondary, for example), would it be appropriate to discuss my scribing experience? In my opinion, scribing is simply a more active form of shadowing. I'm probably overthinking this, but just wanted to get some advice!

Mr.Smile12

I would need more information about how you are scribing for a DO. If this is an emergency department scribing experience, it won't matter what degree your physicians have... in all likelihood, osteopathic manipulation isn't likely to happen.

Nowadays shadowing a DO is a little more important for some schools than others, but most are looking for your understanding of osteopathic holistic practice and why it would be important to you as a physician. I doubt many application readers care if you scribe versus shadow, so do the best you can describing your experiences.


Future Doctor Needs Assistance

I’ve already decided that I want to go into the medical field someday. I've been seriously thinking about becoming a pediatrician. I absolutely love little kids However, 12+ years of school/training does seem discouraging... I'm not choosing this career simply for the salary, as I'm sure I want to have an occupation in which I can help others, especially if they're children.

So, is going into pediatrics a rewarding experience? Any pediatricians that can tell me how they love/hate their jobs? Also, any tips for a (maybe) future pediatrician would be much appreciated. I'm currently a high school student entering Junior year with a 2.9 GPA (Feel very insecure about it, I was slacking on my freshman year) and I know I should excel in science, math and English; but other than that, what else can I do now to prepare for the future? (ie, what colleges or majors should I be looking at, what SAT score should I aim for?) Any doctors that would like to share their high-school GPAs?

Thank you!

Mr.Smile12

High school GPA's are meaningless when it comes to entry to medical school (unless you're trying to get into a BA/MD program, which your GPA seems too low to do that). You need to be fully prepared for college science/math courses and the critical analysis that comes with becoming a physician. You should also excel in coursework that truly interests you, but be thoughtful about some of the classes you may take that may help you work with children, such as psychology and child development. So push yourself and see how you do with college-level AP/IB courses while you can.


Does chiropractic assistant count as clinical patient hours

I was recently offered a job as a chiropractic's assistant. Would working as his assistant count toward patient clinical hours?

mark-ER

Did you have direct interaction with patients? If so, I think it does count... I would include it, though I would downplay it a bit (list it lower on the list) and include another experience or two shadowing a physician (MD or DO), and volunteering in a hospital. Some in medicine, higher level academics in particular, may not think of chiropractors highly. On the flip side, there is also perhaps an overrepresentation of questionable advice coming from chiropractors, and as we all know a few bad apples can spoil the bunch (several personal anecdotes come to mind).


Personal Statement

In my PS, I talked about how I initially had self-confidence doubts about entering a medical caeer but through my essay I talk about how my experiences with healthcare have changed that. Is it bad to even mention those doubts, even if I feel they are important to my story?

mark-ER

I think you will get multiple point of view on this question. Some people don't ever have doubts and wanted to be a doctor since age 4; those are in a distinct minority (<20%). Having doubts is human, but therefore also NOT unique. In your PS you want to special/unique about your experiences, overcoming a challenge, perhaps relating to a specific anecdote/episode in your life. There are plenty of references/exemplars out there, including on SDN as well as books you can get from your library. Good Luck.


Will medical school care about two Ws on my transcript?

I have two Ws on my transcript. The was in physics lab during sophomore year. I was dealing with a medical condition at the time and wanted to reduce my course load from 17 credits to 16. The second W was in a computational biology class that I took during spring of my senior year. I got a concussion halfway through the semester, which caused me to get unbearable migraines whenever I looked at a computer screen for too long. This was obviously an issue in my comp bio class, where every assignment involved coding.

Will these two withdrawals negatively affect my medical school application? I have a 3.97 GPA and 524 MCAT score.

mark-ER

Short answer -- no, it shouldn't affect your application. But there is a good chance someone on the interview trail will ask. Short, clear-cut answer straight to the point, probably with only the most pertinent detail, if any, about personal health or stressor is best. I think your academic record otherwise speaks for itself, so you should be more than OK, with multiple acceptances. FYI -- while a junior in undergrad, I took a graduate class Pass/Fail and someone asked if I converted to P/F because "I couldn't cut it". Somewhat of a rude question, but: (a) there is very blunt people out there in every profession, including medicine & (b) sometimes the interviewer is just prodding to see how you answer a challenging question.


Questioning whether or not medical school is the right choice for me

I have been dealing with a tough decision. After graduating with my undergrad, I took a gap year to spend some time in a beautiful area loving life and to apply to focus on my MCAT and applying to medical school. I did well on my MCAT (512), and I have an sGPA of 4.0 and overall GPA of 3.9. I received admission to my dream medical school, but I decided to turn down the decision and give myself an extra year to decide what I want to do. So here I am.

I absolutely love the academic side of medicine, and learning about the different body systems etc., and I enjoyed all of my shadowing and internship experiences. My internship was in cardiology, but I did multiple rotations through surgeries, including heart surgery, and through the emergency room. While I loved my experiences, I never really felt that "Oh my god this is what I want to do the rest of my life" feeling. Being an EMT, I thought the emergency room would be the place for me, but I realized that it isn't the exciting action I was expecting. I love medicine, but I also love the adrenaline rush that being an EMT brings.

In my gap year, I've sen how much I love hiking and enjoying life, and I fear that becoming an MD will prevent me from continuing these adventures, and that my whole life will revolve around medicine. This brings me to the PA route, in which I could pursue what I am passionate about, while also having time for said activities. However, if one qualifies for medical school and has he potential to become an MD, am I being stupid not just going to medical school?

Then my last option, is to pursue the route of a paramedic/firefighter. During my ride-outs for EMT, I rode on an engine for a fire department, and the rush of excitement and happiness I felt every time I heard the tone and hopped on the truck was far greater than anything I had ever experienced in the hospital. The idea of being the first responder and the person who is there for the patient first on their worst day is so appealing to me. Being a college athlete, I love the physical aspect that being a firefighter entails, as well as the teamwork and camaraderie that come with the job. I worry that taking this route is a waste of a great MCAT score, and that in years to come I'll find it isn't as intellectually challenging and fulfilling. I am scheduled for a ride-out tomorrow with my local FD just to make sure that I still get that excitement and feeling I used to get.

I also know there is quite a pay difference between the three routes, but at this point in time I am focused on living my best life, loving my career, and doing the best I can for my community.

Renee_MD

Hi there,

I think if you are having doubt and there are alternative career paths that you are considering, then becoming a physician may not be the best route for you. Since it requires so much time (4 years of medical school, 3-7 for residency), I think you should be fully committed to this field before embarking on the journey. It sounds like you would thrive as a paramedic/firefighter, which does have a lot of cross over as a physician. Go check it out on your ride along and see how this goes and maybe you will find your calling here. Also, as you mentioned, I know that you want to have hobbies outside of your career, and in medical school and residency, you will not have a lot of free time for these other things. Once you are done with your training you will have more flexibility, but for the next 7+ years, you will be very limited.

I hope this helps weigh out your options somewhat! I think you are doing the right thing by thinking it through very carefully, and I'm sure the right decision will come your way!

Dr. Marinelli


I have recently gained admission to two Masters of Science in Biomedical Sciences and I do not know which to pick. Tufts MBS/MPH Program or Drexel University MBS

I am not sure what program to commit to because at Tufts University I will be getting 2 degrees and still get MCAT PREP, but no guarantees ( interviews, admissions, etc.), just going to get 2 degrees and hopefully do better on the MCAT. At Drexel, the program is also two years and the first year is all classes and an MCAT course and the second year is all med school classes, but I am promised interviews with several schools. My end goal is medical school, but along the way I have loved the idea of Public Health and doing something for the LGBT/HIV community and in general. I am very conflicted on what school.

mark-ER

Overall, either one is a reasonable choice. Based on your interests in community health, I think you would benefit from MPH & its strong emphasis on clinical research and statistics. You do NOT need a year dedicated to MCAT prep. With appropriate motivation, study tools and perhaps some equally motivated/gifted colleagues you should be fine. Another important aspect you did not mention that may be a decision maker is geography -- where would you prefer to live? Does your family and/or significant other play a role in the decision? Is there a difference in tuition/cost of living/financial aid?


MD reapplicant hoping to defer after being accepted

I'm reapplying but I will be doing a two year graduate program in the meantime. I want to apply now though because I don't want my MCAT to expire next cycle. I have thought of transferring those credits from the grad program to the medical school and doing a dual degree when matriculated but worst case scenario I would have to defer a year (meaning entering class of 2020). Worth still applying? If so, how should I report this on AMCAS/secondary applications? Thank you!

mark-ER

You have already been accepted into medical school, presumably at a great cost (monetary & effort), right? Why not go the route you suggested -- try to transfer into the grad program of your choice at the medical school and work with the admissions office there to get a deferment. Yes, you may loose some credits, but likely not entire year. Alternatively, you could do a year or two of medical school, then do a gap year and finish up your grad school. First year of grad school you generally just take classes. What I am trying to say, don't jeopardize your acceptance and go thru the process again unless you are really, really unhappy with the medical school where you were accepted, and/or are having serious second thoughts about your career.


Advice for medical school applications

Hello. I'm currently in the process of applying to medical school. Here is a summary of my experiences/academic info:

Biology B.S. w/ a neuroscience and french minor CumGPA: 3.5 SciGPA: 3.27 MCAT: I'm taking it in early August I have shadowed an MD for 20 hours, have completed 100 hours of community service, am a lead tutor for biology, do research in neuroscience/have developed a poster, and am currently completing a course to become an EMT. I also have been shadowing this summer.

I guess I'm wondering if anyone has any advice for which schools to apply to/ is there anything else I should think about doing this summer? I have a couple MD and DO schools in mind but was hoping to expand my list based on recommendations. Any advice or recommendations would be greatly appreciated!

mark-ER

The recommendation will very much depend on your MCAT score. You should be aiming for 512 and above. Unless you went to a top-notch Ivy league college (and even then), your GPA is holding you back. If you get 515 or above, I would recommend DO schools, along with all your in-state MD schools and a smattering of (at least 10) private schools that take out of state students, basically anywhere you have links to the area thru family, geography, mentors, etc. If you score 512-515, same strategy, but perhaps even more schools to give yourself a strong chance. Below 512, I would plan to re-take MCAT while making a contingency plan for a year -- MPH or a research. Good luck.


Chances of Early admission in KU-med

Hello, I am a 41 yrs-old medical student currently taking pre-requisite in Wichita state U,Kansas. I had 2 MS and one BS, of which the latest MS in Biology track Molecular Biology done in 2017 at Wichita st. I got my permanent residency in 2016 Dec which allowed me to start the pre-med UG classes in 2017. I am done with gen chem,org chem-1,phy-1, biochem,molecular biology, human phys,soc with sGPA 4.0 and currently taking org chem-2,phy-2,Pre-calc-2, psy and public speaking. My bio and english requirements are waived by Univ as I had MS. My MS GPA 3.887. My volunteering experience ~200 hrs till date, shadowing ~150 hrs, teaching experience 8 yrs ( more than 5000 hrs), research experience~7yrs and clinical experience as a CNA for 6 months (will be continued). I never served any executive board though. My plan to apply KU-Med wichita campus as an early decision applicant. I am curious to know what is my chance to get into the Med school and what are the areas I can improve on? I will be applying to Med school in 2019 summer. KU-Med is my only option as I am married with 4 yrs old kid.

mark-ER

Best answer would be: apply and keep an open channel with the admissions committee. A priori, based on your stats and, prior experience and medically-relevant background, your chances appear to be pretty good. Main element that is missing -- what about MCAT? Also, as you allude to -- do not put all your eggs in one KU basket, apply broadly. As for encouragement -- you can get in as a non-traditional student at age 41, having a residency status (green card) is a huge plus for a US born individual, and many medical schools will value the diversity you represent. Good luck!


MCAT Test Date & AMCAS application

I was hoping to get some advice on this plan. I would like to take the MCAT mid-July (meaning my scores come back mid-August). I have my AMCAS application ready to be submitted and I found out that you can add schools to your application even after submission. Should I submit the application with only a few schools attached and then add more after I get my scores back?. That way the application will already be verified by AMCAS. My quandary is whether to take the MCAT earlier or in mid-July (I think I could use the extra time to study), however I do not want to decrease my chances with time. Any advice is helpful. Thank you

Mr.Smile12

You should be able to submit an application with a minimum number of schools before you take the MCAT and then add once you have your scores. As the process is rolling admissions, getting your MCAT scores earlier is always more desirable especially when you need this information to decide whether you should submit to more or different schools.


AMCAS Application

I have my AMCAS application complete and could submit it in a few days (6/1), however I do not plan on taking the MCAT until mid-July. I know that it takes a while for them to verify your application, but should I submit it this early if I wouldn't get my scores until August if I take it in July? Also, do you think early-mid July is too late to take the MCAT?

mark-ER

So just to be clear, it seems like you have not yet taken MCAT (please correct me if I misunderstood). If so, there is a bit of a risk to submitting the application now, as you will have to pay now and you are now obligated to go thru the application regardless of your MCAT score (yes, you can pull out, but then you loose your fees). Conversely, some people find that 'externality' motivating, but according to the more traditional school of thought, perhaps if you are not motivated enough to study for MCAT regardless you should not be applying (??).

So in short, you could submit at or around June 1st, but it may be better to wait -- at the very least to see how your test prep is going and you are confident you will do well on the MCAT, or after you get your score. July is not too late, though you will have to step on it (no procrastination with finishing up the paperwork) as many schools have rolling admissions.


My chances for getting into the MD/DO progam

I'm in a very difficult position in my life. I want an honest expert opinion on my chances of getting into an MD/DO school. I have a 3.11 GPA sgpa of 3.3 without counting the D for bio and F I got for my genetics class. Yes, an F I was definitely was not counting on getting that F but unfortunately due to severe family issues I was not able to concentrate in class. I also changed my college that same year. These are not my excuses I know people who are in med school might have gone through the same things yet got good grades but every person is different. I made a mistake and I would like to improve it. I'm currently a senior about to take the MCAT but I'm not too confident on my application at this moment. I have clinical volunteer experience more than 100 hours, I did community volunteering, research and shadowing this summer. I am passionate about helping people that is the sole reason I really want to be a physician. If med school is not an opinion can someone give me any advice on what I should do like post-bac or master for improving my application and grades?

Mr.Smile12

You should make sure you talk with a prehealth advisor at your undergraduate program, but I would suspect that a master's granting postbac program is likely your best shot to show you can do the work for medical school. Ask your advisor and check the postbac program database on AAMC for help.


Parent with mental illness personal statement

Hi, My mother was diagnosed with bipolar disorder when I was very young, and her diagnosis and how it has shaped me growing up is one of the reasons I would like to become a doctor, so I would like to write about it in my personal statement. However, I know that adcom's may see this and think that perhaps I have inherited this condition, and it will affect my ability to complete med school. I obviously don't think this is the case but wanted to know your opinion. I have heard mixed opinions. Please give me your honest thoughts about this (I am okay with not writing about it but want to be sure).

Pathdocmd

I have seen this subject in an essay several times. In my experience, it has never been an issue.


AMCAS 1 Year Biology and Lab Requirement

I'll be graduating as a chemical engineer with a biotechnology concentration from UC Berkeley. They counted AP Bio and let me take a bunch of upper division biochemistry and bioengineering courses:

MCB 102 (Molecular and Cellular Biology) Nanoscience Engineering and Biotechnology BioEngineering 111

I'm wondering if I can count these courses as biology on AMCAS, or if I need to play it safe and take an entry bio class in college.

Also, I know you need one full year, two semesters of biology lab, which I don't explicitly have. I've done summer internships at biotech firms as a research associate, doing CRISPR, flow cytometry, PCR, etc, but I'm not sure if I can necessarily count that on the AMCAS as credit. Can you use work experience as a biology lab qualification?

Thanks!

tantacles

Most schools will accept upper level courses as substitutes for basic level courses. However, what schools will accept as substitutions for lab work differs by the school. I would suggest that you check the websites of some of the specific medical schools you are thinking of applying to to see how they count AP credit and try to adjust your plan accordingly.

mark-ER

I think you will be fine with MCB 102 to satisfy the requirements. If asked, you can show them AP bio credits on your transcript. I am sure you did well in college & on the MCAT (tough major & good school). But kudos to you for asking to double check, if you want to be extra-extra sure you can also ask the adcom office for your local medical school (presumably UCSF).


Low MCAT/High GPA, good experience should I apply to any MD schools or focus on DO?

I am a non-traditional applicant applying this cycle. Originally, I planned to apply to both MD and DO programs, but after receiving my MCAT score (500), I am unsure of my chances. I graduated Summa Cum Laude in 2017 from a public research university with a degree in Health Sciences. I have completed all of my premedical courses except biochemistry and have a cumulative GPA of 3.96 (Sci GPA of 3.9). I have over 600 hours of clinical exposure, 900 hours of research including a publication, served as president of my pre-health committee, and have received awards in leadership, community service, and athletics.

I was a two-time captain of my collegiate hockey team competing at the NCAA division-III level and a two-time academic All-American. As a student interested in orthopedics, I realize the MD track is a better way to go. In summary, my question is if (at all) I would get a look from allopathic schools with such a low score or if I am better suited to shift my focus to osteopathic programs. I appreciate any advice or suggestions regarding my situation. Thank you.


mark-ER

Do you have a good reason for wanting to go DO route? If you are only aiming DO because of your stats and thinking osteopathic schools are easier to get into, then instead really focus your application toward mid-lower tier MD schools and apply to EVERY SINGLE ONE that you think you have a shot of getting into. That means every one of your in-state schools, all schools where you have a geographic tie-in (family, lived there, attended college), and all lower tier private schools that take sufficient number of students from out of state. It will be expensive, but you can't win the lottery if you do not buy a ticket. Secondarily apply to a handful of DO schools, and again make sure you give them a reason why you want to do DO, instead of MD. Look and play with LizzyM score here on this site, and always remember, stats like GPA and MCAT are a bell curve. Yes, you want to be above mean, but plenty of people get in on the left end of the bell curve too, and if you apply to enough places you increase your chances. I think your high GPA, good (if not ivy league) college, and compelling story of being a high level college athelete and extensive volunteer experience are all a plus. You ought to get in somewhere with a broad enough application strategy, but even if does not happen, not all is lost. (apply again next year, perhaps while doing MPH and retaking the MCAT).


MCAT Prep (Again): Are Kaplan In Person or Princeton Review's MCAT Strategy worth the cost?

This will be my third time taking the MCAT (494 and 497). While being optimistic about the waitlist, I have not had studied so I plan on taking my scribe offer and taking it again in March-April 2019. I took a school offered prep course but I focus more on my GPA than my future. I finished with a 3.8 cGPA, and I tried self-paced Kaplan with the books and limited online. I'm trying to make the third time the charm. I know both the Kaplan and Princeton Review prep courses are $2,000+. Are they actually worth the cost? And which one has a higher success right? In need of advice before spending the cash.

tantacles

This question is difficult to answer. For many people, the Kaplan and Princeton Review prep courses are extremely worthwhile. Depending on your learning style and level of motivation, these courses may help you improve your score. That being said, some find that they do better with self-study. It sounds like self-study has not worked well, so these courses may be a worthwhile investment to give you a better framework with which to succeed.


I'm asking for a suggestion as to which program and medical school to apply to based on my credentials.

I am an undergraduate Biology major at the University of the District of Columbia with a 3.6 GPA. I have also done various internships at places like Georgetown University, Johns Hopkins University, and NIH. I would appreciate it if someone could suggest to me what program (MD/MD-PHD/DO) at which medical school would be best suited for me.

Mr.Smile12

Credentials can get you through the door in an admissions process, but you still have to pick your own doors to unlock. Do you want to stay close? Do you want more of a career in rural communities? Do you want an intensive research environment? I'm guessing AAMC and AACOM have resources with simple questions to ask yourself when it comes to identifying what are the key things you are looking for.


I have 3 C's (2 freshmen year and one sophomore) and grades have all been As and Bs from there with a GPA of 3.65. Do I have any chance on getting in to MD schools?

What is an MCAT score I should aim for to have a good chance at MD?

mark-ER

Clearly, you want to avoid Cs (or below), but what's done is done. You are showing improvement, plus your overall GPA is reasonable. If you can bump it up above 3.7 and your science GPA also in that range, together with a good MCAT score you should be OK. Those 2 aspects are key; anything interesting about yourself (personal background, leadership & extracurricular activities, research & publications) are certainly nice but more of a 'cherry on top'. In terms of MCAT score, don't think of a specific goal target, aim for the highest you can get. When you get the score back, you can use LizzyM calculator on this site (SDN) to figure out the number and prestige range as far as med school applications. Good luck!


Should I apply?

Hello,

This is my first time posting. I am very concerned mostly about my MCAT score. I completed my undergraduate as a double major in forensic science and biology premedical with a minor in chemistry. I graduated magma cum laude a year early. I then went on to my masters of science in cellular and molecular biology. My sGPA in undergrad was a 3.78 and my masters is a 3.97.

I have one publication and a lot of research. I should have 3 more publications by the end of 2019. I have presented at international conferences. I have shadowed 3 doctors and have been an emergency room medical scribe for a year. I worked in a forensic pathology morgue for 2 years. I would say my clinical hours are approximately 2500 hours. I have also been accepted for 2 competitive grants and have completed research with those. I am also now a research assistant at my university performing molecular research on Lyme Disease.

I have had multiple leadership positions in extra cirricular activities as president and various other chair positions. I am currently President of my graduating class. I was also in a sorority for women in technical studies where we did a ton of community service. I was in 5 honor societies in undergraduate.

I have (what I think) is a good PS, as it speaks about my clinical experiences and my journey and also into my sister, who is special needs. She is my main motivating force to become a physician. I also have great letters of recommendation from physicians and professors.

Here is where my downfall is: I got a 501 on the MCAT.

Should I even bother applying this year? I feel that everything is wonderful, except I have this HUGE flaw on my application, which will present itself first, most likely before medical schools see anything else.

Suggestions? Be nice.

Mr.Smile12

In my opinion, remediating a low MCAT score is easy; just retake it. You know better what may have distracted you from getting a better score. You may have done super well on practices but didn't do as well when you sat down to take it for the first time. The MCAT score you have may not be the downfall you predict for certain schools on your list, but certainly if you have high expectations of getting into a top prestigious program, then yes, you probably should take it again.


Should I void my June MCAT after taking it and immediately reschedule?

I'm currently scheduled to take the 6/1 MCAT.

Today I scored a 510 on my AAMC FL 2 (128/129/128/125).

My studying as a whole was admittedly poorly structured -- I took a Kaplan course over winter break with the original intention to take the exam at the end of March (my Kaplan FL scores were Diagnostic 499, FL 1 500, FL 2 504, FL 3 508, and FL 4 505), but was overwhelmed with other commitments in February and March so I pushed my date back to June. I probably studied like 30-40 hours total during the semester (I know, I'm the worst -- I just really wanted get a 4.0 this semester), and have been going ham (10-12 hours a day for the past 2.5 weeks) since I finished the semester.

I took the AAMC FL 1 a week into my going ham to gauge my progress and got a 506, breakdown 127/128/126/125. That freaked me the heck out and I've been going harder since...but today, still no improvement in Psych :'(

I'm a rising senior at an Ivy League institution who is planning on applying after one gap year (starting process senior spring). 3.79 GPA with a strong upward trend (had some personal and adjusting issues freshman year that really messed me up, but have carried a 4.0 per semester since. Hopefully will break 3.8 by end of senior fall). I'm going back to Cambridge for research this summer and will probably be working 40-50 hours/week.

I was hoping to take the MCAT around now because I have taken all of my premed courses like I would have applied directly (physics, ochem are more fresh in my brain). I also wanted to take it before I started my research so I could focus completely on that.

I am really hoping for a 518+ (in my dreams 520+ heh) so I can be competitive at top 20 medical schools. I've gotten As in most of the premed prereqs (save for the freshman year slip ups), and was admittedly overconfident in my abilities. During my content review, I did get the sense that if I spent more time reviewing, I could better applying the knowledge to practice problems/section bank/etc.

Haven't done the third AAMC FL 3 yet (was planning on taking it Saturday), but am now worried to take it and "waste" it if it is still not a good score.

I don't want to psych myself out but I do want to be realistic with myself. What should I do in my last ten days? Should I go into FL 3 this Saturday (reevaluate after) and the actual exam hoping that I'll get a 518+? Should I do as much section bank/content review as I can and take the actual exam and promise myself to void it, scheduling myself for a late July or August date after? Any advice would be appreciated.

Mr.Smile12

I have a different perspective that you have definitely been improving since your Diagnostic exam (Dx). I would consequently be very careful about adopting a mindset where you just take the exam and dump it afterwards. There are only specific instances where I would suggest you do that, and it usually involves an unexpected and perhaps sudden loss of confidence or focus during your exam. There have been many people I know who have done terribly on the practice exams only to completely nail their actual exam (with the old scoring system); there have been others that bomb it despite having done super well in the practices.

So, don't psych yourself out. Go in confident these last 10 days and take your practices. Focus on making sure you don't lose points on things you already know now. And make sure you give yourself time to relax soon before your exam day.


Advice for a Pre-Health student lost in europe?

I am a Pre-med student who has had a kind of weird college career so far and in need of some advice. long story short, I am currently in a BSN program in Norway chugging along as a B- student after completing my A.A and an EMT program stateside. I've recently decided I want to jump ship after finishing my BSN and go back to the states to complete my undergraduate degree in hopes of applying to medical school, but I want to get as much out of my time here short of a higher GPA.

So my question is, do EC's or MD/Professor LOR's from a different country (say, shadowing experience from Germany, Norway or Sweden?) actually help/matter if I'm applying to a US medical school? School related EC's dont really exist over here (clubs, frats/sorority, etc) and research isn't really available to students outside the science majors so I'm limited to volunteer work. Red Cross, Norwegian Peoples Aid (SAR/disaster relief based) or the Night Ravens (neighborhood watch group that patrols larger cities to help citizens at night) are pretty much the top 3 organizations outside of the churchy stuff this side of the pond.

I have 2 years working as a CNA for county home-nursing every 2nd weekend and 1 year as an LPN in a cardiology ward working every 2nd week. I was also a class representative to the student union for 1 year, 1 year as the student union treasurer and 1 year as the treasurer for the social activities group (plan events, apply for grants, organize people and overall keep track of money).

tantacles

Your extracurricular activities will count no matter where they come from and when they were done, but that being said, your most recent experiences are the most relevant. Your letters of recommendation from Norway will likely be helpful, but I would suggest that you at least get 1-2 from your new institution where you plan to finish your undergraduate degree.


GPA questions for med school?

I just completed my sophomore year, and I made C's in general chem. However I did well in other science and math courses. My science GPA is 3.2 and my overall is a 3.45. Do med schools round up? And also, is there hope for me to come back from this. Will they care if I retake a course?

Mr.Smile12

Every medical school will consider different calculations based on the AMCAS/AACOMAS calculations given to them. As far as I know, every calculation is taken to two decimal points, and each school may opt to recalculate GPA's if you retake courses (though most won't). Keep doing your best in your classes and MCAT, network with admissions staff and your prehealth advisors, and stay positive.


Was doing premed at Berkeley a bad decision?

Grade deflation, competition, and difficult personal events have made me question whether going to Berkeley was worth it tbh. My cGPA is around a 3.62 and my sGPA is not too good lmao. I am a freshman.

I wanted to transfer but my parents refused, saying that it'd "prepare me" for the rigors of med school and what not. Thoughts?

tantacles

A GPA of 3.62 is quite viable for medical school admission, and you are only at the end of your first year, meaning you have a chance to improve. It looks as if you're doing well, and if you're able to do well on the MCAT, you'll be in a great place to be accepted into medical school assuming your grades continue to be at a similar level.

That being said, it might be worthwhile to transfer, but transferring has downsides as well - you will have to create a different friend group and build new connections, which people sometimes find difficult. I would consider your decision carefully and ultimately try your best to do what is right for you and not for your parents.


Advice for low sGPA student: MCAT score? EC? Masters??? How important is undergrad major?

Hello,

I will be graduating undergrad in 2019 and plan on applying to med school that June. My stats are: 3.14 sGPA and will likely graduate with slightly over a 3.5 overall GPA. I will be taking the MCAT this August, and based on my preparation believe I could score about a 515.

I am a biomedical eningeering major and have research experience (one publication). I am also involved in a few student organizations (including orchestral emsables),, and am the president of one student chapter. I also volunteer at a hospital and will begin shadowing this summer.

My question is: does the fact that I am studying a difficult major compensate for my low GPA? Should I score a 515 on the MCAT, will that be enough to prove that I am capable? Basically, based on this, am I competative enough for MD programs?

Would getting a masters in my field help my application?

Thanks.

Mr.Smile12

Some schools may consider your lower GPA/major when reviewing your application, but they'll also find many engineers who do quite well in their coursework and have better GPA's, so your argument won't have much to stand on, depending on where you apply. A biomedical master's program is likely to help, especially if you want to have a broader selection of schools to apply to.


Boston University Pre-Med Undergraduate who really needs some advice...

Hey! Thank you to whoever can give me some guidance!

I am going to start my senior year at BU this fall as a Human Physiology Pre-Med major.

I really underestimated how much I would be challenged and as a result how much I would grow as a student and human during my time at BU.

Freshman year was the worst year of my life thus far, I got a D in Intro Bio and a C- in General Chemistry. These are the lowest grades I have received. Currently I have a 3.0 cGPA and I am confident it will rise in between a 3.2-3.3 by the time I graduate. I went from having a 2.3 GPA my first semester to a recent 3.75 GPA so my GPA has only been on an upward climb/trend.

Extracurriculars: - After school tutor for 1 1/2 years - Brigham & Women's Hospital Medical Career Exploration Program (180 hours) - Nationwide Children's Hospital Pediatric Intern/Research (3 months while at home for summer) - St. Vincent's Hospital Infectious Disease Intern (study abroad in Australia for a semester) - Boston Medical Center Research Assistant, currently working on 2 projects: 1. minority women who have been diagnosed with diabetes 2. writing a palliative care research paper (started about 3 months ago and will continue for at least a year) - Pediatric Shadowing Hours - Islamic Society Member 3 years, Pre-Medical & UNICEF Society 1 year

I have recently lost faith in myself when it comes to medical school admissions. I have made up my mind to take a gap year where I will continue research and work as an EMT (I am currently taking the class and will have my certification in July). It's been a tough pill to swallow when it comes to my GPA as I was a straight A student & valedictorian in high school. Freshman year was definitely a challenge but I only grew stronger from this and I really do understand how to study now (I could've gave up). I have not yet taken the MCAT but hypothetically if I get a high score, what do you think my chances are? I also have considered retaking those 2 classes but that is a lot of money. I am a very hard worker and I am so passionate about having the opportunity to add my contribution to pediatrics.

Mr.Smile12

Ultimately you'll have to address the fact that without showing consistent performance in challenging upper-level science classes, the overall/science GPA will still be an issue. Think of it this way: would you rather spend the money on retaking two undergrad classes or maybe doing a rigorous master's postbac program, or would you rather spend it on remediating a semester of med school courses considering how much higher tuition is? Challenges happen to everyone, but you can't think of shortcuts too much or else you shortchange yourself. To that end, work with your prehealth advising staff to come up with other options: MD is not the only way to be a physician.


Can you get in a medical school with D?

I am planning to go in a medical school. I just finished sophomore year of college. I started to doubt whether or not I will get accepted. My PGA is around 3.3. My chemistry grades are B+ and A-, for organic chemistry it's B-. physics C+ and (probably will end up with) D+. DO you think I have a chance of getting in a medical school? Any advice would be appreciated. good medical school that would at least consider me.

Mr.Smile12

Any D's made in prerequisite classes must be retaken, no exceptions. Otherwise, very few medical schools (I would probably say no MD or DO programs in the United States) will consider you seriously. There are too many very strong applicants in the applicant pool.


How to tie a personal life event into a personal statement

Me question is what is the best way (if at all) to incorporate a personal life event into a personal statement? I’ve been dating my fiancé for over 5 years now and will be getting married in less than two months. We just found out recently that her father has been having an affair with another women for over three years now. It was about as bad of a situation as you could imagine. There was a lot of lies and everything was going on behind our backs. It destroyed my fiancé’s family and they are having trouble grasping what all has happened. It has been especially hard for the kids especially since one is a little boy and I feel as though I am kind of acting like a father figure so-to-speak for the time being.I am having a hard time with it as well as I have grown to look up to this man as a father figure. This was something that nobody ever expected and they recently found all of the emails that have been sent back and forth over the years and it was heartbreaking to read. This has changed my life because it has helped shape me into a better person. To grow into a person who will be better because of this event. To be a better doctor someday because I know what my actions can do to others and how it can ruin their lives. How trust is so important especially in the aspect of patient-physician interaction. This has helped me to truly be a better person because of this mans infidelity. I would never wish this upon anyone. Is this somethings that I can use in a personal statement and how it changed my life for the better and how’s it’s been a life lesson to always true and caring for others? I have hundreds of hours of shadowing and volunteering in a hospital and other areas and things that I can connect to it describing why I want to be a doctor. The shadowing and volunteering experiences are why I originally wanted to be come a doctor. Interacting with doctors and seeing them make the lives of other better is what led to my passion for medicine. But will this experience help to portray how after going through this is will make me a better person and therefore a better doctor? Will it help me stand apart from other applicants/personal statements? Or will it be too off-topic. Any help would be appreciated.

Thank you

Mr.Smile12

A personal statement is your best chance at a first impression, and it's subject to any followup in interviews that you will have. I'm not sure I would want to be known as the applicant whose father-in-law has an infidelity issue. If you say you just found out about it, it may be too recent for you to be able to process and reflect in a way that you can tie it in to how you would interact with patients who you may encounter in a professional manner. Do you have an experience where a doctor truly broke confidentiality or a patient's trust in the same way? In general, I would advise against bringing this up.


DO Cycle 2018-2019: Is a June 30 MCAT re-take too late?

I took the MCAT for the first time this January and received a 502. I want to take it again to see if I can improve my score (nothing crazy, maybe a 504/505 or so), but I'm afraid that taking it June 30 will put me too late in the application cycle. I already have most of my AACOMAS Primary filled out, letters are completed, and I'm on track to have my primary submitted by early June. Of course they'll already have my first score to look at, but will having my second score not being released until July 31 significantly hurt my application?

Mr.Smile12

In all likelihood, retaking the MCAT in late June isn't going to be a problem with your application. Just make sure that the schools are notified as soon as your official scores are released.


How do med schools look at classes that you retook?

I ended up struggling greatly during my freshman year of college with many personal problems. I ended up getting C's in all of my science classes I took and ended with a 2.7 GPA. Obviously, I know I need to preform much better my next 3 years, but what if I retook them? How do med schools look at that? Should I keep my C's and just finish strong?

Mr.Smile12

You should ask the prehealth advisor on your campus about what to do. You may want to consider taking those courses again since you want to be sure you have a solid foundation for the upper-level classes you'll be taking, especially if your issue was related to coping with personal issues.


Is taking a Gap year a good idea postBac?

So right now I’m a Bio major going into my jr year. I’m in the honors college at my university and I did community college for 2 years. I’ve done this so far:

-a full summer as a research volunteer in general surgery for Trauma (150hrs) -hospice care (3months once a week) in HS -scribing for cardiology (recently started &planning to do 2 years) -bilingual -3.61 cumulative gpa (1 Withdrawal, 1 C in gen chem2)

I’m concerned I’m not in good standing to at least get into 3 medical schools. I’m hoping to get into atleast UM or UF. I haven’t taken my mcat but I’m planning to study as I finish my prerequisites(orgos and physics classes). Is it really necessary to take a gap year? I kind of don’t want to. Also my last two years of undergrad are going to be pretty overwhelming given that I’d be taking about 6 or 7 classes a semester. What are my chances and is a gap year the ideal route in my case? Thank you for the help!!!

Mr.Smile12

As a rising junior, it's too early to say whether you "need" to do a gap year. If you don't want to do a gap year, do well in your courses, network with students and admissions officers at the schools you want to get into, and develop a pre-application year plan with your prehealth advising team and your potential evaluators. If you do this and have a strong MCAT score, you may be well-positioned in your application coming out of undergraduate.

That said, you may not want to do a postbac, but if you want to become a physician, you'll have to understand that you need to embrace doing things you really don't want to do. There is a lot of that in medical school and residency, and in retrospect a postbac is going to seem like a very small inconvenience in comparison. To that end, you should determine for yourself how really passionate you are about medicine if you aren't willing to do whatever could be needed to prepare you for a health professional career.


Should I study for the MCAT this summer going into junior year even though I'm doubting the medical path?

I've been set on medicine for most of my life, but now I'm thinking about going into computer science/AI instead due to the creative freedom. This is hard for me because becoming a neurosurgeon has been my dream for a while, and I've also already completed almost all medical school prereqs. I also have a 4.0, love learning/taking classes (so don't mind the long medical trek), research experience (which is related to AI, so not a waste regardless), and am imbedded in medical "culture." I also find medicine extremely fascinating, but feel it has become too bureaucratic and also seems so specialized to the point that most medical positions seem monotonous (concluded this after shadowing many surgeons and some general physicians). I am also doubting my ability to think quickly and accurately under pressure. Even though I'm good with people and like being in control and the source of stability, I've recently questioned my own stability and also ability to actually be a good leader and take care of people properly in a medical context (albeit I've clearly not been trained yet). Furthermore, I really want to travel, and I'm not sure that will be possible as a doctor (especially a surgeon). Does it seem like I'm doubting things too much as an incoming junior in college? I do temporarily have lapses where I want to be something else, but this has been the worst one yet I believe. Usually I am very clear about entering the medical field, but lately I've forgotten why. What should I do? Continue with my plan to study for the MCAT or hold off until I decide to stick with medicine again? Perhaps shadowing another doctor might confirm my new career prospect or get me back on the medical track. Thanks in advance for any advice you may have.

mark-ER

Don't make any generalizations about medicine after shadowing generalists/hospitalists. No matter what, things can become routine but medicine provides a lot of room for creative people, especially computationally minded as you seem to be. Consider shadowing some specialists: medical genetics, molecular genetic pathology, or medical informatics (image informatics in pathology or radiology are particularly cool). Those are just some areas off the top of my head -- there are a lot of room for all types/personalities within medicine. Also consider MD/PhD, or a MD-masters pathway such as HST if you enjoy research. You can always go into iBanking or silicon valley/biotech after medicine. But it might be tough to do it the reverse route.


I wanted to know my chances to get into a direct bs/md or ba/md or bs/do program

Hi, i am currently a junior in high school and i am interested in getting into a direct medical program, but i feel as if i dont have the chances. My SAT score is 1430, my weighted gpa is 4.12. In addition to academics, im currently shadowing a doctor, i have been doing martial arts for 8 years, and i am going into a medical internship at Palomar Health in san diego. Plz tell me which direct programs i should apply to. My top md programs are gwu and uconn and my top do program is nova southeastern. thank you!

Mr.Smile12

Your first concern is to make sure you apply to the appropriate undergraduate institution who is a partner with the schools you eventually want to complete your medical education. Based on your metrics, I wish you luck in your chances to be part of those guaranteed admission tracks, so make sure you indicate your interest to the undergraduate schools in your application.


Will I get into med school?

hi I go to a pretty rigorous private university for undergrad and I made the mistake of taking biochem and organic chem II at the same time and I got a C- in organic and probably not much better in biochem. I have decided that I'm going to retake organic this summer. How will this affect my chances of getting into MD or DO school? I am a sophomore and haven't taken the MCAT yet, I am double majoring in biology and english, I've done research since my freshmen year and have one middle author publication and another coming out this fall, I am a writing tutor and voluteer at the hospital during the summer. cGPA 3.55, but all of the prereqs (except physics) are done.

Mr.Smile12

You're doing the right (and perhaps only) thing by retaking organic chemistry and any other course you got a C-minus in. It's too hard to say what your chances are until your MCAT is taken, but I would worry first about getting those grades in before you worry about the MCAT.


I am seriously torn about disclosing an alcohol related illness my freshman year on campus where I was not charged but was under university sanctions, attended alcohol counseling and stayed on probation for a year. There is NO record of this on my transcript and it is not discoverable. Are there secondary applications which will force me to release student code violations which are not documented on my transcript? There is no way for anyone outside of my university to know that I had one stupid alcohol/personal safety violation if I do not admit it. I get the honesty/integrity BS, but I also do not want this to quash my otherwise spotless and highly qualified 4.0 University President's award application. What is really the bottom line here?

I am a 4.0, scholastically rewarded, 20 year old applying to medical school for the first time with an alcohol related illness student code violation. I was transported from my dorm to the area hospital because the student health center was closed. I completed alcohol diversion education and was on probation for a year (university routine sanction). The rest of my record is spotless; my MCATS are high and everything else looks excellent. There is nothing on my transcript about this issue. It is not able to be discovered, nor is the fact that I even have a disciplinary record (much less what it says) unless I divulge it personally. Do secondary apps require releasing any/all records from the undergrad institution? Should I really cut my legs out from under me? Seems stupid. Or does it really not matter in the big picture for one stupid offense.

Mr.Smile12

You should talk with your prehealth advisor for specific issues with secondary applications, but in general, you should always side with self-disclosing. Work with your honor code administrator to review any such statements you give. There is normally a question on disclosing reasons why your education was interrupted in college (i.e., you were on probation for a year), so you have to say something because your transcript will likely show that gap.

That said, you are probably not the only one who has had alcohol issues while in college. If the offense was deemed not important to document on your record, that's better than the alternative. I would always advise on disclosure though, with appropriate notations on your remediation.


How do med school admissions look at five or fix years of undergrad?

Lets say you take more than four years to get an undergrad degree given that you do double majors and you did a fellowship/co-op for a year (gap year), how will med schools consider this?

mark-ER

You might be asked by a curious interviewer and need to have a good answer as to 'why', but overall no one will blink an eye. As long as you had a good reason to do it (many, many people take a gap year), you spent the time productively and maintained good grades/obtained good MCAT you will be fine. Many programs in the US (engineering comes to mind) are routinely 5 years. Should not be a worry in & of itself.


2.8 gpa senior year, taking a gap year. SMP or MCAT?

I am graduating in a couple of days with a low undergraduate gpa (2.8). It droped from 3.2 due to a D in biochemistry and calculus. I want to take a gap year to raise my gpa and also take the mcat/ do intervention but now I am unsure. I am moving to California and I want to apply there but idk how to go about that. Basically what do you think I should do. Should I take the MCAT first and then boost my gpa??

Mr.Smile12

Your priority at this point is to address your GPA first. You have a D in biochemistry that no medical school is going to accept unless you remediate that grade (calculus too for many schools). Go to your prehealth advisors and get some help in your questions about moving to California, registering for classes there, and applying as an in-state resident. You may have to fulfill a time requirement of residing in the state to qualify, but the medical schools there should be able to answer that question.


C's on Transcript

Hi,

I have 3 Cs on my transcript and 2 Qs (a withdraw without the grade penalty in Texas). Classes were Freshman and Sophomore yrs: 1st semester Chemistry lecture, 2nd semester Biology lecture&lab (one class), and Chemistry lab (2nd semester). I Q-dropped my statistics class freshman year and retook it sophomore year and got a B. I Q-dropped physics because I was changing majors.

Funny enough, I have a 3.73 CGPA and a 3.74 BCPM GPA. Drastic changes during Junior, Senior, and Super-Senior years - did not receive one B and got all As in OCHEM and BIOCHEM as well as some other upper-level biologies. I retook all my Cs and got As. I even tutored in chemistry and biology.

MCAT-523

I have 5 years of research in 1 lab with 2 papers, and 1 year of research at the University of Cambridge with 1 paper (all 2nd/3rd authors). I also have plenty of leadership experience and president of a club. I play on my university's soccer team too.

Did I truly bounce back? Do I still have a shot? (I need this more for peace of mind to convince me to apply this round of admissions)

Renee_MD

Hi,

Yes, you absolutely 'bounced back' and still have a shot. There is really not anymore you can do to improve those initial bad grades.

Medical schools understand that not every college transcript will be perfect, and they know that some students will have ups and downs. The point is to show them that you have grown from that, and have the academic ability to do well, and you certainly have shown that!

Great job! Good luck on this application cycle!

Dr. Marinelli


Gap Year Job

So currently I am in the process of applying for medical school. I was a computer science major and I have a offer from a tech company but I don't know if working at a tech company during the gap year might show a lack of commitment to the medical field. However, I don't have too many healthcare related activities during my 4 years in college. So would it be better if I did something medically relevant?

mark-ER

How about doing a gap year in something techy, yet medically relevant? There is a LOT of need for people with informatics/comp sci background in medicine. I could post some ideas, but they would basically be links to companies & (indirect) advertisements, and SDN frowns on that. Where are you located/what's your preferred geography? Feel free to PM me.


Extracurricular activities

I currently work as a respite worker (paid) for my autistic younger brother. I help him with various activities such as helping him take a bath, making him food, and helping him with homework. I am curious if this type of work would be considered a good extracurricular activity? I am also currently shadowing a few different doctors. I also am looking into doing an oversees missions trip with my church in the future. Are these other activities favorable for EC? I am just trying to see how these things can fit into my medical school application. I am currently a sophomore in college. Thanks for taking the time to answer my question!

Renee_MD

Hi,

Your work is definitely a significant extra-curricular activity in regards to medical school admissions. You are demonstrating true compassion and care with your work for your brother. In addition, I do recommend trying to get additional extra-curricular activities in. This includes shadowing as you mentioned, clinical volunteer work, non-clinical volunteer work, research and leadership. A missions trip can be a great activity, but to be competitive for medical school admissions, you want to try to have longitudinal experience in these different extra-curricular activity experiences as well.

Keep up the good work!

Dr. Marinelli


Two Full Time Jobs, Full Time Student, No Clinical Experience.

I grew up in a low-income family and always dreamed of becoming a doctor and help people. I am more financially stable now and was able to start college, and I realized how much closer I am to my dream. However, I am a Police Officer working two jobs averaging around 65 to 80 hours a week, I drive an hour and a half to school, and I am constantly on call with one of the two departments I work for.(I investigate crashes where a fatality or serious injury has occurred) I have been doing very well in school, and I am lined up to take the MCAT in August. I am unable to find the time to fit my clinical hours or volunteer hours in, will my Police career make up for that?

Mr.Smile12

Hello, and thank you for serving our community. It's hard to say whether one can replace your employment experience for a lack of clinical hours. (I'm more certain that many things you do off-duty could be argued for volunteer hours, but that may vary by school.) I would make sure to network with many of the schools you are looking at, presumably ones that are close to you and your family now.

mark-ER

First & most important is that your MCAT and GPA handle the hard cutoffs. Read the forums (though always with a grain of salt), adjust your application (# of schools and medical school tier) accordingly. After that, I think your life story and prior career will VERY much help. Yes, it would be helpful to do some shadowing, such as a 6 month stint doing volunteer work 1-2 hours twice a month for 6 months, just to show you are OK around sick people, but it is understandable if you absolutely cannot squeeze that in given your busy schedule. Are you by any chance interested in forensics, given your prior career track? One option to volunteer and to get a letter from your friendly pathology MD (CME at your local office), and maybe shadow there... There is usually at least 1 or 2 pathologists on AdComs; as a tangential connection and even if you wind up doing something completely different with your career, it might help with admissions.


choice of undergrad institution, choice of major

Hello, i am a 25 year old who has returned to college, i am in my sophomore year at a community college and currently majoring in Business, but 100 percent want to be a Physician.The reason i am getting my BS in Business is that i would love to use it to help hospitals run and with money being a crucial part of our healthcare system in the US figured it would work nicely. In particular when looking for positions such as chief resident. I am taking my pre recs for med school Biology Chemistry Physics and Math and have a 4.0 Gpa in all science courses as well as non science. Due to my good GPA i have the option to attend UC Berkeley as an undergrad. my question is since Berkeley does not have a med school should i choose a different school such as UC Davis or UCLA?that both have top 10 med schools? or will it ultimately not matter that Berkeley does not have a med school when i am applying to med school? would appreciate feedback thanks! just want the best advice to get into med school and fulfill my dream of being a Physician love this forum!

mark-ER

Go to the best school, in a place where you (& your family/significant other) wants to live, plus give some significant consideration to cost. So yeah, that being said if you are a california resident, UC schools are hard to beat. UC Berkley vs. UCLA is a toss-up in terms of education/exposure costs, you can suss out the advantages of one vs. the other. Your key goal is to do well in your undergrad courses maintaining GPA of at least 3.7 and to do well on the MCAT, sprinkling in activities with exposure to medicine as you go; you can do that regardless of whether there is an affiliated medical school, though it does make things easier.

Last random comment, don't mix up being a chief resident with anything to do with healthcare economics. Yes, it has some managerial components, but it is more along the lines of personnel management & HR (micro-micro economics).


Acceptance Rescinded?

I recently got accepted into an osteopathic medical school. The school states on the application prior to submission, that a student who is accepted is required to achieve a B or higher on any coursework completed after acceptance, although this was not stated in my acceptance letter. Recently, I received my grades back and had 2 of my 3 course grades (all 3 of which were pre-req courses) at a C level. I did not perform poorly due to senioritis.

My issues were 3. The first of which was significant health issues (cancer & Lupus) suffered by my mother that required attention. The second was full-time work during school hours in a city located 1.5 hours away from where I live and go to school, in order to provide for my family. The third was attending 5 interviews outside the country (I live in Canada and interviewed in the United States) in an 8 week span from early January to early March. Due to these 3 issues, I did not perform at my typical academic level.

I am afraid that when the school is informed of these grades, that they may rescind my acceptance. Do you believe the school will rescind my acceptance based on the information provided? If so, do you believe that the school will reverse rescinding my acceptance on appeal based on my extenuating circumstances? And do you have any advice for me on the matter (note: my professors are tough and did not give me the opportunity to improve my grade)?

Asking from a student who is having significant stress over the issue and does not want his hard work over the course of 5 years to go to waste based on the performance of 3 courses in 1 unusually difficult semester.

Thank you

Renee_MD

Congratulations on your acceptance! I am sorry to hear of your circumstances and some of these challenges that you faced.

I think the best thing to do would be to contact the school and explain to them the situation. I am not sure if they will make an exception or not, but if you are able to explain your circumstances and what happened, I would think they will be more than willing to work with you. This may be an exception and regular acceptance, a later start or deferment to next year so you can improve those grades. Really the answer will be based on the school and their own policies, however the best thing for you to do is to be open and honest with them.

I hope this helps. Getting an acceptance means they want you at the school! So I would be willing to bet they will work with you to make sure you matriculate!

Dr. Marinelli


Publications on the AMCAS

I am trying to list my publications in the activities section of the amcas and I have them cited in the AMA format. In this format they are over the 700 character length. What should I do to include all of them? Should I give two activities for them or not list all of the authors?

mark-ER

Plug in your Pubmed ID (PMID) and use Mick Schroeder's citation manager (free & useful online tool): https://mickschroeder.com/citation/. I double checked with one of those giant collaborative Gene Atlas publications & yes it will automatically shorten the author list to 3 authors. If you are middle author in a long string, consider listing first 2 authors, then ellipses (...) then your name, then ellipses again (...) and senior author. It is an honest way to represent your contribution. Good luck with your application. [also not sure by what you mean by 2 activities].


2.61 gpa and credit hours 141 should i proceed med school

My gpa is 2.8 and i suppose to graduate this may but i extend and i m taking two extra semester . In summer I am doing internship now what should i do I really want to be doc .. I am also planning to take mcat in summer. For extra curriculum i have 2 years work in ambulance and 4 years work ( 2 month /year ) in community center now what should i do

Renee_MD

Hello,

I think you should wait to apply and try to improve your GPA. Your extra-curricular activities are good, but unfortunately a lot of weight is placed on the GPA (and MCAT) and with a 2.8, it will be difficult to secure an acceptance. I would encourage you to consider post-bacc or extension courses and raise that GPA somewhere to around a 3.5 before you apply.

Good luck to you!

Dr. Marinelli


MD and DO personal statements

I’m going to be applying to both MD and DO programs and I have drafts for both my personal statements. I originally wrote my MD statement first and have whittled it down to fit the DO limit but I’m wondering if I submit two essays that have identical parts will they consider that plagiarism?

Mr.Smile12

By definition, you cannot plagiarize yourself. You can only plagiarize others and claim/co-opt their words as your own. To that end, I don't think you should worry about it. Good luck on your writing.


Letters of Rec

I am interested in applying MD/PhD. I have a fairly competitive application, but my biggest concern is my letter of recommendation from my senior year PI. I thought we had an amicable relationship, but she has consistently ignored all my emails requesting for a letter (4 emails, 2 calls to her office). I even reached out to the PhD student in the lab, and she told me the PI has been extremely busy and hasn't even been that responsive to her (she's currently not on campus because of medical issues or else she would've asked for me). As an MD/PhD applicant, I understand how critical it is to have a letter from every PI you've worked with. However, I am a post-bacc now and am doing far more intensive research at a top-tier school. In terms of a learning experience, my post-bacc has been incredible, and I have significantly more responsibility in lab now than I did in my entire undergrad lab experiences combined. I am getting a letter of rec from my current PI, my PI from sophomore year, and my summer PI, but I'm just not sure if I'll be able to get a letter from this senior year PI by June 5th (when application opens). I'm really worried this is going to be the reason I have a red flag on my application, and it would be really unfortunate because I've worked really hard and I'd hate to not get in just because my PI is too busy to check emails. Any suggestions? How should I prepare to explain this if I can't get the letter?

mark-ER

It won't be a big red flag if you don't have her letter. It sounds like you have plenty others. It's better to have fewer strong letters, than even one lukewarm/negative letter. Your former PI may be in the midst of a busy grant application season, family emergency, who knows... she just went dark-mode, as highlighted by the grad student's reply. And it sounds like you barraged her with a ton of emails/requests. Take a deep breath, wait for a month and check in again with that friendly grad student. See when that PI is back at normal schedule, and then either call or stop by, thank her for mentorship, and ask if she would be willing to write a STRONG recommendation letter (use those words). If you noticed hesitation, gently and casually back out of it.


I am reapplying to medical school but I know that parts of my application are not great. What are some things I can do to make myself more competitive?

I am reapplying to medical school but I know that parts of my application are not great. my GPA is 3.0 and my science GPA is a 2.67 my first MCAT was a 491 but I am retaking it. I took a prep course and studied differently. I’ve done a lot of volunteering and I also am published in research. my GPA suffered a lot because I commuted, worked 30+ hours a week during my first two years of college but I picked it up and made dean’s list for my last two years. The other reason it suffered is bc my family is not supportive in my educational goals BUT i am also the only one who is able to care for them health wise (which is why my MCAT suffered the first time - I had to take care of my family after surgery). How do I explain this in a more eloquent/acceptable way for my application? What are some things I can do to make myself more competitive?

Mr.Smile12

In my opinion, if you have access to a health professions advisor, you need to seek their advice and suggestions. You also should talk with your evaluators and see if they have any thoughts about what to do. In the end, you have to take care of the obvious problems with your GPA and your MCAT scores. Your application won't be part of any conversation in an admissions process without that. You probably should strongly consider a masters-degree postbac program, and that may come with a lot of resistance from your family and the stress that comes with taking care of them.


Strong leadership position, but below-average clinical hours. Damaged application profile?

Just to give my situation some context: I am ending my term as president of my University's student government. My GPA and MCAT scores (just around matriculant median) likely took a small hit due to this 30hr/wk time commitment, however this greatly impacted my involvement in clinical volunteering due to the more consistent time requirements. As of right now, I only have about 20 hours at the local hospital, with much more planned this summer. I got a late start to clinical volunteering because until 6 months ago, I was aiming to apply for an MD/PhD program, so I focused more on research (700+hrs, 3 poster, 1 pub pending). I also have 50 hours of clinical exposure through various shadowing experiences, and 200+ of non-clinical volunteering.

Due to my lack of clinical volunteering hours, some have advised me to take a gap year to boost my hours. How bad does this look in the grand scheme of things? Should I wait to apply next cycle? I plan to hit 150hrs by the end of the summer, but I am worried there is not much weight put into service hours completed just before or after application submissions. Thoughts? (Thank you in advance!)

tantacles

Like many others, I would suggest taking a gap year. Your application is good right now, but it could be great, and it sounds as if you've already assessed the weaknesses of your application. it is always best to apply when your application is completely ready, so take your time and get the experience you need to have the best application possible.


Bad First Year

I am currently a student a low tier state university. My high school career was beyond a wreck and so was my first few semesters at my community college due to suffering from mental illness, abusive home, and addiction (obviously not making excuses just wasn't mature enough to put the effort in to school) I left my community college with a 3.0 gpa and am currently excelling in all my pre reqs and my upper level science courses. However some of my courses from community college that have little to no meaning (mass communications, business) are Ds and there's about 2 or three of them and it's sincerely dragging down my cumGpa. I have very good letters of recommendation, from proffesor and physicians I shadow and continue to shadow. Also have a decent amount of volunteer hours and still have about 2 years left of undergrad that I will continue to do EC. Medical and Osteopathic schools are numbers games and I already know that. Best case scenario I'll be coming out with a 3.3-3.4 cum GPA and maybe a 3.5 science GPA. My question is what are my chances seriously as these are not competitive numbers and I know medical schools don't care for sob stories or excuses, just numbers. Does my upward trend hold any water? Or am I going to have to consider Carribean Schools? Any help and guidance would be amazing. Thanks

tantacles

If you've shown a great upward trend, even allopathic schools are likely within your reach, particularly if you have a great showing on your MCAT. I would suggest that you continue to do well and get your extracurricular activities and research in order so that you'll have the best shot possible.


What information do medical schools have about other acceptances

Can you explain what information medical schools have about the status of their applicants at other schools. I recently learned that once schools make an offer to an applicant, they can also see where else they have offers. This doesn't seem like a widely publicized fact and it really seems like it might hurt applicants. Does access change on May 1st such that schools can see where applicants are in even before making an offer? Do they see the schools that applicants turned down previously or just whatever current school they are holding.

Mr.Smile12

Suffice it to say, if you have multiple acceptances, you need to be fair to other applicants and give up your extra seats as soon as you can. It's not easy knowing that someone else's career hangs in the balance because you haven't decided which school to attend... but that is what is happening. Schools also need to know so they can fill their classes with people who are enthusiastic to start medical school.


Research, pressure, favoritism, and competition in undergrad

I am about to start summer research with my current organic chemistry and organic lab professor (I'm finishing up O chem 2). I don't feel qualified for this research position, since I applied almost a year ago and was accepted before this professor knew how I would do in his organic class (not too well due to some heavy personal issues that came up). Right now I'm doing just ok with a B-, but compared to my fellow straight-A researchers in my class I'm doing quite poorly. These students tend to be favorites of the professors and sometimes I feel we compete for his attention and approval, which I feel I have not earned. Anyway, recently my professor has been watching me like a hawk in lab and critiquing every point of error in my lab technique. I have made some silly mistakes in lab like spilling water all over my bench, likely due to the fact that I am often uptight and nervous for that very reason. While I appreciate the constructive criticism, I am getting stressed by the pressure and the feeling that I have disappointed my professor and that I shouldn't even have this research job. Has anyone else felt the same way? How did you deal with this competition and self-doubt? I truly want to succeed, but the doubt can be crippling.

tantacles

Every job has its difficulties, and it sounds like you are having a hard time adjusting to being directly critiqued by this individual. It is difficult to know whether your professor is critiquing you because he feels that he can improve your work and not because he is critical of you and does not want you in his lab. I would suggest simply doing your best and working hard to perfect your lab technique.


What are my chances for this application cycle?

31 brown female, Nontraditional, US resident, with a master in public health and an international master in Medical Genetics; 6+ years of research experience; 300+ leadership/ volunteer hours and 30 hrs clinical shadowing so far and will continue shadowing/scribe job this whole year. Reapplicant and Retaking MCAT in June for a better score.

Graduated from MPH program in 2015 and submitted primary to AAMC in 2016 with my international bachelor degree but was not successful in terms of course requirements and MCAT scores. So I took all the prereqs in a US university (1 year of each- physics, gen chem, O.chem, Biochem( graduate-level ), psych, Adv Anatomy and Physiology w/ lab plus graduate-level biostatistics, medical anthropology, health policy etc while in my public health program ). Post bacc cGPA= 3.9 and graduate cGPA= 3.8. Did not take general biology courses but getting them transfered.

What are my chances to get it if I applied to medical schools this year? What areas of the application should I work on to be successful? How does being a "reapplicant" affect my chances of getting in? Please help

Mr.Smile12

It's too hard to give any prediction in the absence of a solid MCAT score or specific feedback from admissions officers. I know that as a non-traditional student your access to prehealth advisors may be limited, but it is important to know what your overall GPA is when you include your graduate and postbac GPA's.


Career path for an Anesthesiologist

Currently I am a Junior in High School and I was wondering what would be the best career path to becoming an anesthesiologist. Im trying to get ahead anyway possible and I was wondering if I should take any certain classes to already be one step forward. Right now I am only one class away from finishing my A.A. and I plan on finishing it during this summer while taking a program at my local college to become a CNA. After that I'm not sure what I should do. Should I take an EMT or Phlebotomy class or should I try to become an LPN or an RN and then possibly an CRNA? Or should I just skip all that and just go to get my bachelors in pre-medical and then straight to medical school? Im so confused and unsure what to do any help or suggestions would be greatly appreciated. Thanks!

Mr.Smile12

I wanted to point out the website Explore Health Careers, which will occasionally spotlight a career area, such as anesthesiologist (https://explorehealthcareers.org/career-spotlight-anesthesiology/). One should also note there is a physician assistant career called anesthesiologist assistant that you may want to look into.

tantacles

In order to become an anesthesiologist your first order of business is to get into college. Your major in college is not important; it is simply important that you take the pre-medical requirements and get the highest GPA you possibly can. You can get your clinical experience however you want; being a CRNA may help you figure out if medicine is the right career for you.


Gap Year Plans

I'm a senior majoring in Biology and have a cGPA of 3.4 and a sGPA of 3.29. I am currently trying to figure out my gap year plans I am trying to decide between three options.

Option #1: Enrolling as a non-degree seeking student at a local university and just taking classes individually to boost my sGPA.

Option #2: Enrolling in an online post-bacc program for Medical Neuroscience

Option #3: I recently was accepted at CWRU for a Master's degree (2 year program but can be done in one year), so I would enroll here as another option.

The CWRU is quite expensive so I was hoping to stick with a cheaper option and work from home (hence option 1 and 2), but I don't want to make the wrong decision when it comes to applying for med school.

I also plan on applying this upcoming cycle to both MD and DO schools, taking these courses or doing a program would be a fallback if I do not get in this cycle to apply next cycle with a hopefully stronger GPA.

Thanks

mark-ER

Masters won't improve your uGPA. Post-bac will, and would be an asset, especially if you take at a top-notch reputable program (check forums). Stats, biochem, micro, things like that may be useful & interesting, and your plan for medical neuroscience isn't bad, especially if you have neuroscience research background and/or future career interest in neurology/neuropathology. What I would NOT do is just take community college science courses, just to improve your sGPA.

Glad you are being realistic and aiming for both MD and DO schools, and hopefully your application list is long (even with a rock-solid MCAT, unless you are from top notch ivy league university with engineering/physics major, 3.3 science GPA is quite low and a red flag). Good luck.


Cheaper Options To Raise GPA Instead of Post-Bac?

Help please!

I'll be completing my sophomore year of college in roughly a month. Currently, I have a low cGPA of 3.4 and sGPA of 2.8. My GPA dropped after making a C- in pre-calculus, and I'll most likely end up with a C in Calculus again this semester. Nothing else in my application is a concern (MCAT, extracurriculars, volunteering, etc).

I'm currently double majoring in Molecular Biology (B.S.) and Spanish (B.A.). With this being said, financially I'm not able to double major like I thought I could because it would put me graduating a semester late on top of taking full course work during every summer session until graduation.

I've been advised to choose a major and minor in the other. I love them both equally, but here are the pros and cons:

Spanish Major with Biology and Chemistry Minor: -Only have 7 more classes to graduate -More scheduling flexibility -Study abroad opportunities -Worried about not improving science GPA up (from taking less science credits) enough to be competitive and needing to do post-bac

Biology Major with Spanish: -More science classes to improve sGPA -If I do well enough I might not need a post-bac -Love the department -Behind in major, but will still graduate on time

My main priority is getting into my state medical school (ranked in top 25). I'm having such a difficult time deciding because I want to avoid doing a post-bac or masters at all costs, due to finances. Instead, I'd prefer to work in a clinical setting, do research, or volunteer abroad during my gap year(s). So, if worse comes to worse and I need to focus on improving my GPA, I'll do the post-bac if need be.

But, if there are other affordable options to focus on GPA, like retaking undergraduate classes and staying an extra semester to do that or something similar, I'd love to hear about them. I'd also love advice deciding between majors given my circumstances. 


I'd also appreciate any extra advice. Thank you!

Mr.Smile12

Having to choose your options based on finances is hard though the issue of overall debt is a serious concern. I always recommend talking to admissions officers about your situation with your C grades and whether that would be a real dealbreaker if everything is strong. That said, if you only limit yourself to opportunities due to cost, you need to have a real champion on your side (letter evaluator, admissions officer, prehealth advisor), especially if your GPA doesn't crack 3.5 to 3.6 (depending on the metrics of the school you are really focused on). There are plenty of people who don't properly appreciate the value of the GPA in the admissions process when so many other people have higher academic credentials.

The only person who will have to decide your ultimate major/minor combination or double-major combination is you. How passionate you are about both subjects and learning about both is the real important piece, not how it necessarily looks to others. You only live once and hopefully won't have to regret a road or option not taken.


Low GPA, taking MCAT soon, great resume.... Do I have a chance?

Renee_MD

Hello. Thank you for your question. I am sorry to hear that you had a hard time in college, but the most important thing is that you have really transformed yourself and have become stronger! As you mentioned, the MCAT will be a significant factor in your overall chances of acceptance. There is no 'minimum' score for admissions, but last year, the average matriculant score was a 510. So I usually advise people to shoot for a 510 or above. You can also apply to osteopathic schools whose average MCAT is about a 505, so even if you don't score as high, you definitely can still become a doctor.

In addition, don't let your history of depression discourage you. This is something you had to overcome and did, and is a great accomplishment. You will need this kind of resiliency as a physician, so developing skills to work through problems early on is incredibly valuable.

I wish you the very best! Keep up the hard work!


I'm in Pre-Med but intro to biology (at my university) is difficult for me?

I have a strong love for Chemistry and Biology, but Biology at my university is ridiculously hard. I heard that for Microbio and Animal Bio (I guess it's consider "Bio 2" or "Zoology" you name it) are not easy, however, doable I guess it is a way to put it? I'm taking those next semester. At any rate, I'm a biochem major so I would have to take some biology courses (along with microbio courses and physiology/anatomy which I took during high school, and it was doable as long as I studied by butt off for it), which I'm fine with. But with Intro to Biology it is just insanely hard... they design it to where it's inhumanely hard. I've taken MCAT Biology practice questions and I do well on those. But with the tests for Intro to Biology... it is insanely hard. Should I just give up my dream of being a doctor because intro to biology is hard? I might have a C in Biology affecting my overall GPA to go to a 3.5ish, so will med schools look down upon that? Because this spring semester of freshman year has been rough lol. Please help.

Renee_MD

Definitely don't quit on your dream of becoming a physician! Some courses will be more difficult than others and having the grit and determination to withstand that challenge is what medical schools want to see in their applicants! That being said, since biology is a pre-requisite course for medical school, I would recommend trying to get at least a B in the course. If you can not make a B now, you may want to retake it in the future. Good luck to you! Stay strong and keep working hard! This work is in preparation for the rigors of medical school!


Summer Organic Chemistry at Cal State

Hi! I'm a first-year engineering student at an Ivy League institution, and am planning to take organic chemistry I and II and the associated lab courses at a Cal State school this summer (since my major has a lot of requirements). Would going elsewhere to fulfill my Orgo requirements look bad if I decided to apply to med school?

mark-ER

It is not ideal, though the most important thing is that you do well in the class (i.e. at least a B or B+) regardless where you take it. Basically, O-chem is seen as a 'weed out' class by admin committees to see if you can cut it academically. If you are at a top-tier Ivy League level institution and you take O-chem at mid-tier or lower institution you may be asked the question 'why'. That will be particularly true if you do poorly on the physical sciences section of the MCAT. There are good reasons to do it -- having to be at a specific location for instance taking care of family member who is sick, or taking an extra class while doing an away research internship thru UROP -- and there are bad reasons to do it. Ultimately members of med school adcoms don't have time to tease things out and generally won't interrogate you, but in an otherwise pristine application it may raise an eyebrow, and again you may be asked if you have a good reason for making that choice.


If I got a B in Gen Chem, will I be ok in future sciences (ochem, physics, biochem??)

I'm a fourth year student that decided to pursue medicine after a long period of contemplation. I ultimately decided I should at least give it a shot instead of regretting it in the future. I like bio and am good at memorization but struggle with math and chemistry. I took gen chem this semester and worked the hardest I ever did throughout my four years in college. The material isn't necessarily difficult and I did all the practice problems and homework problems but the tests are really conceptual and nothing like the practice problems, which is why I haven't aced a single exam so far. Ultimately I'll be finishing the course with a B or B+ but I question whether this is the right path for me if I already struggle with general chemistry. Will I be able to do better in Physics and Biochem when I can't even get an A in gen chem? (I go to a top 20 school so I'm just hoping my sciences courses at my postbacc institution won't be as difficult). I ask because Postbacc is very expensive and I don't know if my will and effort alone will help me get into medical school if I struggle in the sciences.

Renee_MD

I absolutely think that you still have a good chance at the other courses! Switching educational paths can be difficult, and as your first chemistry course, you may struggle! You may also struggle in physics and ochem, as most pre-meds do, but the point is try as hard as you can and keep determined to succeed! That is what it takes to get into medical school! If you want to do medicine, then definitely stick to this path but you may want to consider taking only 1 of these science courses a semester so that you can fully concentrate your efforts, and do as well as possible!


Organic Chemistry (I & II) Below Average Performance

Hello. I just wanted to ask for your opinion as to how I should proceed with my upcoming application. My only area of concern on my application is my organic chemistry grades. I graduated from a Top 12 university, have an abundance of both clinical experience and community service experience, and have an above average MCAT grade both overall and within my demographic (82nd percentile, 127 in Chem/Phys, Hispanic). However, I am concerned about the C+ grades I received in both Orgo I and II. While I took those courses as an underclassman and have a significant increasing academic grade trend (finished with a 3.6 GPA and 3.45 cGPA), along with receiving higher grades in both Orgo Lab (B) and Biochemistry, amongst other advanced science courses, I am still concerned that admissions counselors may view these two grades as a major concern and thus disregard my application. Thus my question remains: am I overly concerned about these grades considering the strength of the rest of my application, or should I attempt to take a science course in the fall after submitting my secondaries (would take a course during the summer but would not be possible with my job)? Any response would be greatly appreciated, and I apologize for the lengthy paragraph!

Mr.Smile12

Have you asked anyone at the schools you have considered on how they would consider your organic chemistry grades? If you did rather well in biochemistry and on your MCAT, and other advanced science courses, I think those C+ grades may not be as critical.


International Shadowing of Doctors

As far as undergraduate and medical school admissions, how is a program viewed where students (undergrad) shadow doctors, but the program is international? I have heard international programs are not viewed well, but I am not sure? I wont be practicing any type of care, only shadowing licensed doctors. I will also be pairing this with volunteering in the United States, so my international shadowing wont be the only clinical experience I have. The program says the shadowing hours are coherent with standards from the AAMC, but I have seen writing from the AAMC saying colleges are wary of international experience. Any advice is appreciated.

Mr.Smile12

"Medical voluntourism" is a general concern, especially if you wind up having to do any sort of procedure to patients. That would be generally unethical, and you could get into a lot of trouble. If you just observe, there probably isn't as much of a problem, but you should know that the health care systems at other countries are remarkably different compared to the United States. Now, if you are able to do a proper compare/contrast and link the experience to your overall motivations to pursue medicine, you might be okay.


Can I apply to medical school with pre-med course work taken abroad?

I messed up and took Organic Chemistry I and General Physics I during my semesters abroad. When I returned to my home institution I took Orgo II and Physics II and got As in both. I have already graduated with degrees in biochemistry and math and am now serving in the Peace Corps, hoping to apply to medical school this cycle. But, is it even worth applying, given that these 2 pre-med courses were taken abroad? I have contacted the individual schools I am interested in, and all admissions offices say that pre-medical coursework must be completed in the U.S.

Given that these two courses transferred as equivalent courses to the ones at my home institution, and that I did very well in subsequent courses in these subjects, I kind of want to apply to medical school anyway. I am hoping that instead of automatically discounting my application, some schools might accept me on the pre-tense that I retake one or both of the pre-med courses taken abroad. Or do you think my application will be automatically discounted because Orgo I and Physics I were taken abroad, even though they transferred in as equivalent coursework?

tantacles

It is possible that medical schools will accept these credits. My suggestion is that you look on medical schools' websites and check to see if they accept these types of credits. If you took advanced courses in these subjects, medical schools might also be willing to use those as substitutes for the lower level courses you took.


Re-applicant Advice

I need advice on how to approach this next application cycle. This will be my third time applying. The first time I only submitted one secondary so no surprise I didn't get in. This last time I applied to 14 MD schools and did not get any interviews. I submitted my primary in the first week of June. I didn't submit secondaries till the end of August. I don't know if the long turn around time affected things. I am a white female and Colorado resident. I have a 3.96 GPA and a 509 on my MCAT (128/127/130/124). I graduated in three years from undergrad. I have worked as a CNA since my second year and have over 3000 hours of experience now. I was in a sorority and on an elected committee in the sorority for tow years. I was a TA. I did a senior thesis which was my only research experience. I was also a camp counselor, volunteered with my church, and volunteered with a community outreach program. I am planning on retaking the MCAT but would like to know what else I can do to improve my application. I just started volunteering in a hospital as well.

tantacles

On paper, your application actually looks great. It is hard without knowing all of the details, but there are many things that could be holding back your application. It is possible you did not apply broadly enough, and it is also possible that something in your personal statement or something that you wrote did not sit well with the admissions committee. It is also possible that one of your letter writers did not speak kindly of you. I would suggest that you find an adviser that you trust to divulge all of the details of your application and let them go through your materials to help figure out why you did not get any interviews.


Chiropractic student thinking of medical school

Hi there,

I am currently a chiropractic student at CMCC (Canadian Memorial Chiropractic College). I am thinking of applying to medical school in Canada however, I'm not sure how good my chances are. My cumulative GPA in undergrad was 3.29 (on 4.0 scale) and I haven't written the MCAT yet.

I plan on finishing my chiropractic degree before applying. I have also racked up quite a bit of research experience, extracurricular activities and 4 publications. My references would be strong also. Does anyone have some advice? Do I have a chance?

Thanks!

Mr.Smile12

I'm presuming you are applying to Canadian medical programs. I'm not sure how much they value grade point average, but I suspect they weigh it very heavily. There will also be a question on why you want to move to medicine, given that you chose to go to chiropractic medicine and haven't even given it a shot as a practitioner (i.e., what really is your commitment to medicine if you're going to quit a different health profession). You should talk with admissions staff at the schools you are interested in attending and see specifically what their perspectives would be.


I don't know how to start my Personal Statement

I come from a different cultural background. Being Asian, I was kinda pushed toward pursuing a path in medicine. After taking bio classes and shadowing and volunteering, I really want to pursue a career in medicine as a doctor and stay committed to it. My problem with the personal statement is that I don't know exactly why or what motivates me to become a doctor. It's just that I like science and anatomy and I can see myself becoming a doctor and helping others. I've never had a dream or a goal since I was young, I just know that if I work hard towards something, I excel at it. How do I go about finding the motivation and really just what do I want

tantacles

Some exploration into your own hobbies and goals is probably a good idea. Many people find that taking a year or two after medical school to explore interests outside of medicine helps them to cement their interest. There are many fields that have skills that are applicable to medicine, and it might be valuable to your future career and may help you figure out exactly how you want to proceed.


Lab Tech or Masters?

Hello!

I am currently a senior at BU with a cGPA of 3.2 as well as one year of coursework from UVM with cGPA of 3.63 (but I have had an upward trend for the past two semesters and maintained it at 3.6). Currently, I have 2 semesters left. This semester I am looking at a 3.8-4.0 semester GPA, and the next semester hopefully will be the same. I have yet to take my MCAT, and will plan to take it early next year.

I have recently been offered a lab/research tech job at MGH, however I was also looking at a masters program either in research or MAMS (med science).

I was talking to my current PI who is also on the board of admission of BU Med and she said that if I can maintain 3.8-4.0 for the next two semesters, my disastrous start at BU will not carry nearly as much weight as the last 4 semesters of course work. If the next two semesters go as planned, my new cGPA will be somewhere between a 3.45 - 3.6 (this is up from 2.44 my first semester at BU)

I was wondering which program would be most beneficial to me in terms of admission into an MD or MD/PhD program, trying to bring my cGPA up or working as a lab tech? There have been many variables in trying to formulate my decision and I am lost as to what I should do.

Thanks

tantacles

I would suggest working as a lab tech, particularly since if you are not able to gain admission to an MD/PhD program and you end up going to an MD program, which will likely be easier to gain admission to, you will not receive a stipend and will have to deal with more loans. Both will likely provide you with an equal chance of gaining admission to an MD/PhD program, but the research tech position may be better if only because you might be able to get a publication during your time in the lab, which would further bolster your chances of admission.


Seeking information about writing as a physician

I am an undergraduate student who is considering a career in medicine. However, I currently have limited experience working in a clinical setting. As someone who has struggled with writing in the past, I was hoping to gain some information regarding writing as a physician. Here are a few specific questions I was hoping to have answered:

1. What kind(s) of writing do physicians do on a daily basis? How does one train to do this sort of writing? 2. What kinds of information go into a patient's chart? What components do well-written charts have? Who is the audience you have in mind when writing a chart? 3. What are some of the most difficult aspects of writing for medical documentation?

Thank you for your time.

tantacles

1. and 2. Physicians regularly write notes that document the patient's story, their thought process regarding the patient, and the plan set forth by the medical team. This is what belongs in a chart. Depending on your specialty and level of training, the length, detail, and content of the individual note can change.

3. Medical documentation serves multiple purposes: It acts as a legal document, a billing document, and a reference for the people who will need to take care of the patient later. Fulfilling these three purposes altogether is difficult and the balance between them makes medical documentation an art.


Letter of Recommendation Advice--TA led class, Professor

So the non-science professor I was counting on to get a letter of recommendation suddenly is not available to write me a LOR anymore. I was wondering if a mainly TA-led class, with weekly professor office hours (that I would try to attend regularly this spring quarter) would be advisable for me to get a letter of recommendation from? The issue that I'm worried about is that the professor wouldn't really see how I do in lecture.

Renee_MD

Thank you for your question.

Obtaining letters of recommendation from professors can be difficult as they are very busy and usually receive many requests for letters from students. In some circumstances, it is appropriate to have a TA write the letter but it is crucial to have the professor of the course sign the letter as well. Most medical schools will not accept a letter signed only by the TA. In your situation, try asking the TA to draft the letter and have the professor sign it as well.

If the professor is unwilling to sign it, then I think it would be best to try to get one from another professor.

Good luck to you!


College Valedictorian

Hello,

I was hoping to get some clarity as far as what admissions committees think of college valedictorians. Does it matter at all? I was set to graduate this April two weeks before my last class and was asked to be class speaker, only to find out that I would have been valedictorian if I had all my classes finished. I could potentially wait until next year to walk, and unless someone else gets a 4.0 and has a similar extracurricular profile, I would have the valedictorian. I will not be applying to medical school until June of next year anyway, so I was wondering if it would be worth the wait. I will have my degree, though, just would not have walked.

Mr.Smile12

I think it may be personally impressive, but there are so many strong students applying for medical school it won't really matter. Furthermore, GPA doesn't predict your ability to be a good physician. Congratulations on being a student speaker, and enjoy the opportunity.


Non-traditional Student - Chances of getting in

I am currently wrapping up a Master's in Public Health this Spring. Going into medical school has always been on the back of my mind, but I have never felt prepared or ready to go until recently. I took all of the required pre-med courses in undergrad and had every intention of applying, but didn't feel ready and joined the Peace Corps instead.

After 2 and a half years in the Peace Corps, I returned home and served 2 and a half years with AmeriCorps, before finally deciding to pursue my Masters in Public Health. I'm incredibly happy with the choices I have made, and the path I have taken, but I still have the urge to give medical school a go. My grades aren't necessarily outrageously awesome, but they definitely aren't the worst. I have a lot of personal and professional experience to bolster my resume, and I have to retake my MCATs (my previous score was acceptable, but has since expired.)

Is it still worth it to apply, even though I'm getting older, and won't be starting my career until about age 40? I have also considered pursuing a Nurse Practitioner's license, but am not sure if that is of the same calibre that I am ultimately interested in. Any suggestions or help would be highly appreciated!

Mr.Smile12

With an appropriate academic record and MCAT's, I think you have a strong chance with a medical school program that values your experiences and knowledge in public health and public service. Your task is to find a program that would support students similar to your background and address whatever personal or family concerns you would have in making the transition through medical education and consider the financial support you need to make it happen.


International student from Australia applying to MD programs in the US

Just wondering if someone could give me some insight and maybe clear the road for me a little

I'm an Australian citizen wanting to study medicine in the US. I am an economics major graduating in December. I've taken single units for science pre reqs (chem bio physics and biochemistry) and in regards to extracurriculars I've done two months with the Kurdish red cross on site in Syria and Iraq and 350 hours of shadowing a physician. what do I need to improve my application in regards to extra curriculars? My top pick would be Vanderbilt as I've spoken to their medical admissions and they do accept 8-9 internationals a year so I know it's not impossible. I also messaged them and asked if I should take a year of coursework in the US before applying and they replied back saying they do not recommend it since the language of instruction was in English and I went to the University of Queensland which is ranked in the top 50. I also got a 519 on the MCAT :)

If anyone could give me any insight I would really appreciate it :)

Mr.Smile12

If you aren't interested in a research path (Ph.D.), which would be more friendly for international applicants, you will need to check with each of the medical schools you are interested in. The issue you also need to investigate is financial aid and citizenship requirements for internship/residency. I would network with other medical schools that you are interested in and find out their policies on accepting international students before beginning with the AMCAS application in May.


Is taking the MCAT mid june too late or leads to any consequences for TMDAS/Texas medical schools?

I had originally planned to take my MCAT early May, however after taking practice exams I am averaging around 500 when my goal is a 515+. I did some research and found the median acceptance score for texas medical schools is 508 to 514 depending on the school. I have 300 or so clinical shadowing/ medical internship experience hours 3.94 overall gpa, and a 3.86 science gpa. lacking in extracurriculars, and volunteering at clinics about 50 hrs. If that helps answers in any way. So due to my low scores I was considering of moving my MCAT date to mid to late June to better prepare, would there be any consequences in doing so? I will already have a gap year and I am trying to avoid having another one by not getting accepted this cycle. Also I have not been able to find whether or not and which Texas schools have rolling admissions, would taking the exam in mid june give me a large disadvantage? I plan to start working on my application as soon as I finish taking the exam and have it done by the time I get my scores back.

Mr.Smile12

You should have a lot of information networking with the Texas schools, so hopefully they can give you the answer about rescheduling your MCAT to later in June. I don't suspect it would make much of a difference since you're not moving it more than 2 months. It shouldn't affect your applications outside Texas through AMCAS or AACOMAS.


Concern about my gpa

So I might make my first C this semester in organic chemistry 2. My current gpa is a 3.82 and with a C my gpa will go down to a 3.80. I’m very concerned that the C will ruin my chances of getting into medical school. I am about to be a senior next semester, so time is running out. If anyone has any personal experiences and/or advice I would love to hear it.

Mr.Smile12

In general one C isn't going to sabotage your application, but a general pattern will raise attention. Move on as best as you can, learn from your mistakes, and excel in biochemistry.


Admission chance

Alright I'm going to be honest I heard SDN can be harsh and real and that's what I want, realism. Do not worry, I will apply no matter what is said but I also could use some motivation as I am doing well right now but not well enough, this is pretty much to give myself the feeling of being cornered so that way I perform better in a sense so please be as honest as possible. Ok so here goes... I am a junior in college right now, attending a university in Florida. Up until maybe a year ago I had no aspiration to be a doctor, mainly due to my social skills didn't want to burden anyone by being a doctor with poor bedside manners (there's enough of them already). I wanted to be a neuroscientist, I was pretty smart just no work ethic (slept through AP chemistry and got a C) and very little care for school. Basically I was a class clown in high school. I got to college same effort just more maturity, partied didn't care. Got chronic migraines and actually failed several classes, I think maybe 5-6 Fs over a period of 2 semesters (retook them got Bs and As which I know MD schools will take the Fs, that's fine no DO suggestions as I have researched into it and decided to just work harder for an MD). However somehow stars aligned in some odd way were I found myself in sales (improving my social skills) and then came down with a couple painful and scary bought of migraines with visual aura and I have always been a pretty empathetic person so I started to care for the other people who were suffering more than I was in the hospital. I then regretted my entire academic history. I grew up an orphan and I am also African American I never had stability so it kind of explains my care for school but no excuses of course. And the terms with Fs were primarily when the chronic migraines became uncontrollable. I have 2 true Cs on my transcript one in psychology and one in genetics. All my poor grades were prior to my decision to kill myself (metaphorically) in an attempt of the impossible (which I expect you to reinforce but I'm giving you a lot of ammo so won't be too hard). So here I am a Junior with a diagnostic of a 506 mcat before prepping (transferred from a community college where the Fs occurred to a university where my change occurred) and also I got mentally worn down due to my lack of discipline during the test and honestly just wanted it over 3 hours in. I plan to put myself on a tough training regiment for the mcat to push it north of 520 (my discipline from doing entrepreneurial things in cc kind of indicates that as no problem I would wake up 6:30 every day to work on business plans and stuff). Besides genetics which occurred during my first semester as I had to adjust to being a "good student" also got As in 4 other high level science classes that semester, and also the fact I have straight As in all my high level prerequisites (such as micro, molecular biology, clinical neuroscience (grad course), physiology, neuroanatomy, orgs 1 and 2, physics 1 and 2, biochem) I have been volunteering at the hospital (approx 200 hrs so far). I work as a scribe (I don't know how many hours but a bunch with 11 hour shifts AKA exhausting overnights). Also to throw out hypotheticals and to feed the fire I am about to set myself on I score lower than my diagnostic and get a 505 but lets be nice and run a hypothetical at like 513 something humble. Also I am in the process of publishing some radiology research but its slow because I am busy and worn down, and I am also planning a international trip where I will either help Haitian with like BP(tansyon wo/ba) and pulse (Batman ka) respirations (respirasyon) so basically vitals and vitamins and stuff, or i'll shadow some doctors in Columbia (undecided). What are my chances? I want to help people no matter what it takes I treat this as a life or death situation (hypothetically) don't hold back and be honest please. Sorry it's terribly long wanted to make this as comprehensive as possible. Also I am also applying for the masters in neuroscience as a back up then I will reapply, and I would prefer one of the schools in Florida.

Renee_MD

Thank you for your question.

It sounds like you have had quite a journey to get to this point of applying to medical school! And that's ok! I have worked with many applicants from different backgrounds before applying.

I do think that you have a chance for admission. I think the key to submitting a strong application will be to have a solid MCAT score (510+) and also completing the neuroscience master's program with a good GPA (3.5+). There is no exact formula to say whether these numbers will be sufficient or not to gain admission, but based on statistics and past students, you will be more competitive if you are able to have these achievements on your application.

In addition, I think you have good extra-curricular activities that you should continue your involvement in as much as possible. A radiology publication would look great on your application, and volunteering in the hospital and working as a scribe provide good clinical exposure and patient interaction.

I know the path to becoming a physician is long, but keep pushing! You are obviously doing great things and have a lot of potential, and if you want to be a doctor, then you are doing the right things to become one!

Please feel free to update me with your MCAT score in the future. I wish you the very best!


Necessity of volunteer hours for medical school?

Hi there,

I am currently a scribe in the ER at stony brook hospital. I was wondering if volunteering would be replaced by scribe hours due to the experience gained from the position? The learning curve for this job is pretty big, and I feel as if it will be something I can really speak about in an interview or personal statement. I am also currently trying to volunteer doing something I am interested in, but I am not sure if I will be able to volunteer there as they have not gotten back to me. Would scribing, research, leadership/TA experiences, solid GPA (~3.6 overall, 4.0 science GPA) and a high MCAT score trump volunteer hours? or is that something that is absolutely necessary for any applicant to get into medical school? I have some volunteer hours (~60 from a sports medicine class I took, and a club I was in). Any info is greatly appreciated.

tantacles

Certainly, given your medical experience with scribing, you need not do any medical volunteering. My suggestion is that you reach out to more organizations or non-medical volunteering that you are interested in in case the experience you are seeking does not work out.

In medical school applications, nothing "trumps" anything else. Every piece of your application is important, so I would suggest that you wait to apply until you have all of the components you need to have the best application possible.


Career change from DPT to MD: Pre-reqs and Post-Baccs as a disadvantaged student.

sGPA: ~ 2.5 cGPA: ~ 3.0 last 60 units cGPA: currently 3.4, plan to get to 3.6/3.7 by the end of the year.

I am a disadvantaged student seeking guidance in applying to medical school. Currently finishing up my last year of undergrad at a 4 year university. sGPA is low due to taking all my math and gen chem my freshman year when I was not ready to do so. Took Ochem brief 8a & 8b w/ lab instead of 118. I plan on retaking some math this summer however I am stumped on if I need to retake the entire ochem 118 series or just 118 a & c, also whether I should retake gen chem BEFORE applying to post bac programs. An advisor mentioned retaking these classes at an open campus and then applying to a post bacc would be the path to go.

While I do have clinical experience, I do not have any research experience. I am curious if I should consider research experience while retaking/doing a post bacc program or if my sole priority should be performing well in my post bacc program.

Mr.Smile12

It's not clear from your subject if you already have a DPT or were originally driving towards a career in physical therapy. Before I go into all these details of what you should do, I want to know why you changed and how that is affecting the courses already on your transcript plus others you are planning to take. Your specific questions on organic chemistry are best answered by your advisor since I wouldn't know what each of your organic chemistry classes are.

You should ask many of your questions to postbac programs you are interested in. Research experience is usually not the major issue for admission to postbac programs.


Grade Considerations for AMCAS application

My second semester of sophomore year I had a lot of family and medical issues going on, which caused my grades in two classes to suffer. I retook each of them at a later time and got significantly better grades. I'm wondering if the AMCAS will take these new grades into consideration when looking at my application/GPA because without those new grades my GPA is about a 3.0 and with them it's a 3.5. Is this something I should explain in a personal essay as well?

tantacles

Your new grades will count in your AMCAS application, and they will be averaged with your old grades in those classes.

If you are able to recover from this situation and do well, you may not need to explain two bad grades. The most important thing to do is to show a strong upward trend and do well in your classes in the future and do well on the MCAT.


Three year graduate

Hey So I'm a three year graduate, 3.87 total GPA, 3.9 science, science research experience from march of freshman year, started a club to help the underserved, about 100 hours of shadowing, volunteered two years in summer camp for disabled kids, humanities research from beginning of freshman year MCAT for July 24 should I apply this June? at the end of my second year

tantacles

If you are able to get a good score on the MCAT, you will be in a great place to apply to medical school. I would suggest you verify your application early and as long as your practice tests are going well, strongly consider applying during this cycle.


4 full semester withdrawals due to serious personal problems and applying to allopathic medical schools

Hi,

Thank you so much for reading and taking the time to answer my question! :)

I have FOUR full-semester withdrawals on my record due to serious personal problems.

On one of the withdrawals, my infant died.

The other three were because of my epilepsy...I wasn't a premed at the time and got the three withdrawals after I kept getting seriously hurt. I was having a LOC seizure a few times a week and with that kind of frequency, sometimes I would lose consciousness somewhere dangerous, hence the withdrawals after I would get really hurt, end up in the hospital, need surgery for fractures, etc.. I went to a big state university that really doesn't allow make-up work and would lose so many points after missing a few weeks of school that I would have to withdraw from my classes to prevent myself from getting a semester full of F's.

I went in for an epilepsy surgery eval after the third withdrawal where I got very hurt and was deemed to be the perfect candidate. So, I had surgery for it that same semester to get rid of the damn things and have since been fine seizure-wise.

I then took a long gap (3 years) from school for financial reasons.

1. Unless (God forbid) I were to have some sort of unforseeable tragedy, I will have two full years of full-time coursework before applying. I came up with two years/four semesters in a row because that is the time until I graduate.

1a. Will this be enough to allay any fears that I am at risk of leaving medical school? 1b. Would I need to do a post-bacc as well to be in a good position to get accepted to MD schools? (It would have to be a post-bacc and not an extra year of university for financial aid reasons. My stats are well within MD school range, which is why I asked specifically about MD-granting medical schools.)

2. Four withdrawals is ridiculous and I imagine I should say something on the "Is there anything else you would like to know?". How much should I divulge? Should I just say "Health problems" and "Death in the immediate family" and leave it at that? Or is four so many that it really does need more specifics? How specific should I really get?

tantacles

The most important piece of information for your application is your GPA. Thus, if taking the W's helped your GPA, that was a great choice.

In addition, your story is interesting, and the most important thing is to show that you are stable enough to complete medical school.

That being said, it is hard to say exactly how your application will be received. Your most important goal should be to do well in your future coursework to show you can succeed and to do well on the MCAT.


MCAT date

Hello,

I had my MCAT scheduled for May but I want to reschedule for June because I won't be ready to take it in May. I still plan to submit my primary in June without the MCAT scores so that I can at least get verified. My GPA is more on the low side cGPA 3.57 and cGPA 3.17 (Please see additional info at the end) so I am counting on a decent MCAT score to have a better chance. I am wondering if June 16 vs. June 29 would make a difference considering that the scores are released a month later which means that if I take it June 29 I wont't have a score until July 31st? Taking the MCAT in July 29 would give me almost 2 extra weeks for practice, if I take it June 16 I would have only 3 weeks to take practice exams as I would be finishing content review around May 24. Any advice will be appreciated.

Hispanic 24 y/o female with 1 year of research in a psychology lab for child anxiety and phobia at the university I attend, and 2 years of clinical experience (I work in a mental health community center doing psycho-social rehabilitation with the severely mentally ill population). My degree GPA was 3.76(Psychology BA) not sure if that will be relevant, my current cGPA is 3.57 and sGPA 3.17.

Thank you.

tantacles

Taking your MCAT June 29th is appropriate as long as your application is submitted as early as you hope. That being said, it is important that you make sure that you are completely ready to take the MCAT. Your GPA, while not unacceptable, is low for medical school, so make sure you take plenty of practice tests to make sure that your score falls in a range that will actually make you competitive for admission to medical school.


I volunteer at a hospital, how do I talk to patients?

I am a cope health scholar at a hospital. My scope of practice involves toileting, ambulating, feeding, and assisting nurses and physicians with other patient care tasks. I don’t know how to talk to the patients. I understand that they’re vulnerable and sick but all I can think of to say is “how are you?” or “how has your day been?” and they just seem like inappropriate and awkward questions to ask given their condition. I really just need some advice on how to speak to patients to make them comfortable and some topics that would make for good small talk.

tantacles

I think the approach that you are using is likely a good one. Talking with a patient can sometimes be awkward, and that's why the job that you are doing is relevant, particularly if you plant to be a physician, a career in which many patients can be difficult to interact with. Being cheerful and asking about a person's day without reflecting on their medical condition can be incredibly therapeutic, particularly since many patients spend the majority of their time thinking about the condition that brought them to the hospital. Anything appropriate topic you would talk about with a friend is fair game for these kinds of situations. Talking about tv shows the patient might watch is often a great in to a conversation as tv is often all there is to do in the hospital.


Would postponing the MCAT date affect my chances?

Hello,

I scheduled my exam for May 5 because I wanted to make sure that I get my application in as early as possible. I am a hispanic 24 y/o female with 1 year of research in a psychology lab for child anxiety and phobia at the university I attend, and 2 years of clinical experience (I work in a mental health community center doing psycho-social rehabilitation with the severely mentally ill population). My degree GPA was 3.76(Psychology BA) not sure if that will be relevant, my current cGPA is 3.57 and sGPA I think it is around 3.2 (not good I know) so I am relying on a good MCAT score to really improve my chances. I work full time and have had a few personal issues during these last couple months that have kept me from studying the amount of hours I had planned. I started in January and I am still half way through content review so I don't think(almost sure) I will be ready to take the MCAT in May. My question is; would rescheduling my MCAT for June or ealy July hurt my chances? How late can I take the MCAT considering that the scores take a month to be released and still be ok to apply this cycle? Also, I have read that I can submit my application without the scores, but how would that affect me? Should I just cancel and take it next year to allow more time for preparation?

Thanks in advance.

tantacles

Taking the MCAT in June or early July would be completely acceptable. My suggestion is that you have your application verified prior to your scores being released; that way, once your scores are released, you can immediately have your application fully submitted, and schools can evaluate you right away. Most schools do not start reviewing applications right when AMCAS opens.

That being said, if you do not feel you will be ready to take the MCAT in June or July and it will still be a rush, it is completely acceptable to postpone further and apply next year.


MCAT Score to Outweigh Low GPA?

Hello,

I am currently a junior undergrad student hoping to apply to medical school this summer. My overall GPA is currently a 3.3 and my sGPA a 3.1. I will be taking the MCAT for the first time in June and plan to send my applications in as soon as I get the score. My question is, is there an MCAT score that might "outweigh" my GPA? Also, I am a biomedical engineering major; will med schools consider that I studied a difficult major when looking at my GPA? Given a competative MCAT score, would I have a chance at acceptance or should I consider other backup plans for after graduation?

Thanks.

tantacles

It is possible that a very high MCAT score will outweigh your GPA; that being said, a great MCAT score will be necessary given your GPA.

While schools certainly do look at undergraduate majors when evaluating GPAs, the GPA itself is what truly counts as every student is able to choose what courses he or she takes and has the ability to choose courses in which he or she will do well in if courses prove too difficult.

With all that being said, all schools evaluate GPAs and MCATs differently, so I would suggest purchasing the MSAR (available online) to see just how high of an MCAT score you'd need to be competitive at your target schools. If your GPA for a particular school is below the 10th percentile, for example, I would expect that you'd need an MCAT score of that school's 90th percentile to consider yourself very competitive. However, the process of medical school application is unpredictable, so it is difficult to say exactly how things will turn out.


Biotechnology classes and SGPA

I took 2 biotechnology classes before. Intro into biotechnology and Regulations of biotechnology. Both classes have the code BTEC instead of BIOL. Would they both still be counted towards sgpa or just cumulative? AAMC has Biotechnology under the Biology list but im still not sure if every biotechnology class will be counted towards science gpa or just some of them or none at all.

tantacles

My suggestion is that when you list your courses, you do your best to simply say honestly which classes are science courses based on AMCAS' guidelines. If AMCAS disagrees with you, they may change the categorization of the course, but if the content of the course fits the bill for science, I would list the course as as a science course.


I'm taking calculus-based physics and I want to withdraw from the course to take non-calculus based physics next semester

Hello, I am a first year student with a 3.26 GPA and I am currently taking 17 credits. I am enrolled in General Physics I (calculus based), and I am having a really hard time in it. I spoke to my professor about it and he advised me to drop his course and take Physics I (non-calculus based) next semester. Add drop period is over and I would get a W in the course. Would it be better for me to take the non calc course and drop this one, knowing I would probably get an A in it compared to getting a C in this one? How bad will the W look on my record? The non calc course specifically says for pre-health majors in the description, could I make a case when applying to med school that I signed up for the wrong section?

tantacles

An A is always better than a C, and one W on your transcript will not look bad. I would suggest dropping the course and taking the non-calc based course if you believe you will do better in it.


Will this be considered an alarming downward trend and how do my metrics look?

I am getting ready to apply this upcoming cycle from a top 25 school in California.

I am Transfer applicant, and I did my first two years at a CC and my last two years at UC Berkeley. So my trend during each year of my undergrad is, 3.88-> 3.74-> 3.68-> 3.9.

-I know that during my first three years, I have a small decreasing trend but I hope it’s not too significant of a drop to be of concern. At the CC I had the 3.88 and the 3.74, and at UCLA I had the 3.68 and 3.9.

My overall gpa is a 3.82, and my Science GPA is a 3.74. My mcat is a 513. I also have two W’s on my transcript.

Thank you

tantacles

Your trend looks like you've shown steady work and done well during your undergraduate coursework. You are in a good place to apply. I would suggest you apply broadly to a mixture of reach schools, target schools, and safety schools.


How do ADCOMs look at Pos-Bac students?

Hello, Im currently finishing up my post bac year after graduating with a really low GPA (3.36).. I've managed to pull up my GPA to 3.39 solely based on last semesters work thanks to the 3.9 GPA I got last semester as a post bac. I still have one more semester to go (will be completed in May 2018)

I was just wondering how post bac students GPAs are looked at by admissions? Do they just look at your total GPA including undergrad and post-bac... and still frown upon it if its still low (3.4ish), or do they see that you got a significantly higher GPA as opposed to your undergrad GPA, even though you were taking more difficult classes as a post bac? What if as an undergrad you were finishing up a different major (chemistry), and only took the minimum medical related biology courses, but you clearly do well in the rest of your medically related courses as a post bac?

also if you were generally having an upward trend with the exception of one quarter as an undergrad, does it still count as an upward trend? I would think the one bad quarter would kill your chances of having an upward trend?

tantacles

Every medical school looks at post-baccalaureate study differently, but suffice it to say that an upward trend generally does not go unnoticed. It is hard to say exactly how your particular scenario will play out, but getting a 3.9 every semester in a post-baccalaureate course will look very good for medical school applications. Ultimately, your admission will still depend on myriad factors, like MCAT, GPA, your post bacc, and your extracurriculars, so it is hard to answer your question directly, but right now you're doing the right things to improve your application.


Are there any premed advisers who guide you step by step process in your journey into a medical school I have heard even if you have perfect GPA and MCAT score you are rejected. I know one student who has every single check mark but didn't get any interview.

How to increase your GPA by taking more Science classes ?

tantacles

The pre-medical adviser at your school is probably a good starting point. Truth be told there is no one who can guarantee your admission to medical school as there are myriad factors that go into the admissions process. I would suggest you speak to your pre-med adviser at your school as well as other students who have been previously accepted.


Bombed AAMC FL 2 help!

Okay so I've been studying off and on and my studying method is taking lots of full lengths as I believe best way to score well on the mcat is through lots of practice. I want to take me exam May 5th took NS1 and Ns2 and scored 498-500 and today decided to take AAMC full length 2 to see where I stand and my score was 489 and the weird thing is I got more questions correct in each section than on NS. SOO confused these are the number and the scoring is soo strange to me and seems so low Chem: 123: 28/59 CARS: 120 24/59 BB: 124 34/59 Psych:122: 28/59

On a EK fl I got 38/59 on CARS and got 126 and 127 on NS 1 and 2. I think i might have overanalyzed and not used to AAMC. I'm aiming for 515+. Analysis and tips please??

tantacles

My suggestion is that you continue to study and that you take more practice exams. Each practice exam is just one data point, and there is no way to know exactly where you will score based on one. It sounds as if based on your result you have work to do in every single subject.


I got rejected from the medical school that I sent a letter of intent to, so now can I send a letter of intent to a different medical school?

Hello! After I interviewed at my dream medical school, I sent them a letter of intent. Unfortunately, this dream school of mine recently rejected me. Now, if possible, I would like to send a letter of intent to another dream medical school of mine. Am I allowed to send this letter of intent to my second dream school since my first dream school rejected me? Thank you!

tantacles

Yes. You may send a letter of intent to another school now. Keep in mind that letters of intent do not guarantee acceptance, and it may not help you at the school you send your next letter of intent to.


2.7 G.P.A and about half way done with community college. Do I still have a chance at MD?

First off, let me begin by saying I am a non traditional student with a very unique story. I’m currently 26, but about 4 years ago I decided it was time to get educated so I enrolled in a C.C. At the time of enrollment I was a H.S. drop out, so I started by taking a class or two almost every quarter until I obtained a H.S. diploma. That was seriously tough for me as I never learned proper study habits and I was honestly just an all around dumb blue collar factory worker. On top of all that, I have been struggling with anxiety/depression for some years now which has certainly taken a toll on my grades. My transcript is rocky because of this, it goes up and down like a rollercoaster depending on how I’m feeling that quarter. The good news is that within the past year I have made great strides with this disability and am now starting to feel a sense of belonging. So my question is, if I can keep an upward trend from here on out, can I still be competitive? The only two science classes I have taken are general and o-chem. I still have another year until I finish my AA and transfer to university. Oh and I also left my job to completely focus on my studies. I’m just wondering if on paper I can still be a competitive candidate with an upward trend? I understand to get where I need to be it takes hard work and determination, I just want to be sure it’s still possible to get there. Anways, sorry for the short story.

tantacles

You absolutely have a chance, but you will have to do better in every single one of your courses. An upward trend will help you, but you have little room for error at this point. You will need to get almost all A's in your courses and do fantastically on the MCAT if you want to get into medical school.


Low GPA, Non-traditional Pre Med student, incomplete courses.

As of right now I am a senior Psychology student, and I currently have a 3.195 cGPA. I was a non-traditional pre med student in undergrad, meaning I took some of my pre med courses while pursuing a different major. I actually stopped taking those pre req courses after my sophomore year due to personal conflicts leading me to perform poorly in my classes. I had taken Gen Chem+ labs, Gen Bio, and Ochem+labs. As a result of that, I solely took psychology courses for the rest of my undergrad in order to raise my GPA to what it is now. I was interested in pursuing a post bacc program in order to raise my science grades, since I did poorly in those classes. However, I have not taken physics quite yet or some labs, and I know a lot of academic record enhancing post baccs require that I take physics and labs. Any thoughts?

tantacles

Currently, I agree that you need some GPA repair in order to be accepted to medical school. There are two options open to you once you've finished college:

1. You can do a formal post-baccalaureate program that will require you to take all of the science courses again.

2. You can do an "a la carte" post baccalaureate where you choose the courses one by one that you take.

I would suggest that you do whatever is less expensive and most accessible to you. That being said, if a program you want to go to requires physics, then you will need to take physics, so I would recommend you figure out how best to accomplish that goal.


MCAT Score to Outweigh Low GPA?

Hello,

I am currently in my spring semester of my junior year as an undergrad and my current GPA is a 3.3 (sGPA 3.1). I am a biomedical engineering major. I am planning on taking the MCAT in June and applying immediately after in July.

I have over 50 hours of volunteer hours at a hospital, worked as a research assistant (for credit) for one year (abstract published in journal), and have a leadership position in an academic organization. I have also been involved in my university's string ensemble.

I have no shadowing experience, but I did work in the reception in a medical office for many years.

Basically, my question is, will medical schools consider my difficult major when looking at my GPA? And how high should my MCAT be to even be considered given my GPA?

Thanks!

tantacles

Schools will consider your difficult major, but ultimately, the GPA itself is what matters. Presumably, you had a choice as to what major you wanted to choose and therefore could have chosen a major that you excelled in.

It is hard to say how high your MCAT should be to counterbalance your GPA; I would suggest you purchase the MSAR, available online, and use that to guide you as it will give you the best idea of what MCAT score will be good enough for each school you hope to apply to; each school has a different standard.


Gap Year Options

I graduated June 2017 GPA 3.7 I've applied and have one interview coming up. 2 years of undergraduate research Roughly 300 clinical volunteer hours Roughly 600 CNA hours (this was not on my application) MCAT 498 (I'm already planning on retaking this)

I'm hoping to reapply this Summer, but am not sure what to do while waiting on eventually getting accepted. I'm having a hard time deciding between:

CNA (I already have this certification and am working part-time) Medical Assisting (6 month program) Medical Scribe

I'm continuing volunteering so that's a given. I just don't know if it would be worth the time and money to pursue medical assisting since I'm not sure how long I'll have to work in that position.

Any and all advice is welcome!

tantacles

If I were you, I would not reapply this summer. Take an extra year. This will allow you to show significant improvement in your application before you reapply. It will also give you a chance to retake the MCAT.

If you do choose to reapply, I would suggest that you find a job that you enjoy in whatever field you want. If the job does not involve science or medicine, continue to volunteer and do medical service in the event you don't get accepted.


Bad Start in College, Complete 180 Academically But My GPA Will Stick With Me

Hi, I just joined SDN and have been working on my plan for getting into med school for a couple months, but want to take the opportunity to get expert advice.

I began college at a community college in Virginia in 2016. I was not ready for college at this point, I had no idea what I wanted to study, I wasn't prepared for the distractions of adult life, I had never developed good study skills in high school because I skated through classes easily. I ended the semester with a 1.8 GPA.

I took the next semester off to gain some work experience, settle into adult life a little more, and think about what career I might want to pursue. I'd always been interested in medicine so I shadowed an ER physician I know and from that point on I've definitely been hooked. I began volunteering with a rescue squad and spent the summer getting my EMT certification, as well as taking classes in technical rescue.

In fall of 2017 I resumed my studies with three classes as I finished up my training at the rescue squad. I retook one class I had failed because Virginia Community College System allows GPA forgiveness. I also took my second semester of English and first semester of human anatomy and physiology. I got an A in both English and A/P, and a B in the other class. This semester I am taking A/P part 2, general biology 1, intro to communications, and retaking the math class I failed in my first semester. As of now I have As in all classes and am on track to finish the semester with a 4.0.

I have also been accepted to transfer to Virginia Commonwealth University in fall of 2018, where I will be studying biology.

I have some questions about application considerations since my first semester will be following me for the rest of my academic career.

1. Using GPA calculators I have calculated that it might not even be possible to raise my GPA above a 3.4 (my minimum target) before I finish school. I will be working my ass off to get As, will med schools recognize my success in all semesters or should I expect to really be justifying myself to them considering my first semester?

2. I have heard stats get you to the door, ECs get you through the door. I have some excellent ECs and will be adding more once I move to Richmond, plus I have paid and volunteer experience as an EMT. Additionally I'm a professional drummer in a band that is financially self-sustaining. Will I be able to use that to diversify my application and show I have skills outside of academics and medicine?

3. I have found a real passion for biology. I know a lot of people recommend studying outside of natural sciences to set yourself apart but I truly enjoy biology and would like a degree in the field. What considerations should I make to make myself interesting even with the bread and butter premed degree? Are there biology specializations I can look into that will make my application interesting and still give me some good career options? (For example, a bachelor's in physiology may not be the most practical choice).

4. I plan to apply to all of the Virginia state schools. Am I going to be overshooting or are some of the schools achievable goals?

Thank you so much for your help, I really plan to make this happen and I want to be able to set myself up for success.

tantacles

If you are able to show a positive trend and get mostly A's, particularly in the science courses you take, it will look great on your application and may overshadow your previous difficulties.

You can absolutely include your drumming as piece of your application. Different readers will have different feelings, but the arts are generally viewed favorably.

If you love biology and will be able to get many A's in it, just study biology. It is very rare that a college major sets someone apart, so I would recommend you study what you want and do as well as you can as the quality of your performance will be more important than your actual major.

Applying to all o the virginia state schools is a good idea. I would also recommend you apply to a good variety of other medical schools to have the best chance of acceptance.


3.92 GPA, 517 MCAT but 126 in CARS. Should I still apply to top 20 schools?

Hi, there! I got my MCAT score back earlier this week: 129/126/131/131. I'm very happy with the overall score (517), but I'm a bit concerned about the 126 in CARS. I would like to apply to a couple of top 20 schools, but, browsing through MSAR, I see that a 126 in CARS is either at or below the 10th percentile for all of them (with the exception of Duke). My overall score is basically at the median of most of them though. I am definitely not retaking the exam, and I will apply broadly. Given the rest of my stats, should I even bother applying to top 20 schools? Or would it be a waste of time/money? Thank you so much!

sGPA: 3.95, cGPA: 3.92 ECs (pretty standard, nothing extraordinary): -A summer and two semesters of research (850 hours) -300 hours of clinical volunteering/patient care -120 hours of nonclinical volunteering -40 hours of shadowing -Executive Board member of two clubs

I also identify as a first-generation student and LGBTQ if that matters.

I have seen @Goro, @gyngyn, and @gonnif give awesome advice! Any help would be greatly appreciated!

tantacles

It is hard to say if you have any shot at top schools, but they may be willing to overlook one outlier on your MCAT score given the rest of your strong performance. I would suggest you invest the money to give the schools youre interested in a good shot.


Bioengineering major, GPA 3.3, MCAT 514 -- I know my stats are not great but what are my chances if I apply this cycle?

I have about 200 hrs volunteering at a hospital, I worked in a research lab for three years (wet lab, translational cardiology research), and I have strong leadership experience in student orgs. I know my GPA is very low, but do I have a chance of getting into MD programs in the US? Would a postbac/masters program be a good idea? Also, should I consider retaking the MCAT? I had scored higher on practice tests than I did on the actual day of.

tantacles

Right now you likely have a reasonable chance of acceptance. Your MCAT score is great and may actually be able to help you overcome your GPA. I would suggest that if you apply you apply both to MD and DO programs to have the best chance of acceptance.


Help in deciding next steps

I'm currently a junior at NYU studying neuroscience. I have a 3.515 cGPA and am planning on taking the MCAT in September. I'm getting worried about my GPA holding me back from getting into even mid-tier schools. It's been on an upward trend since freshman year, but still doesn't feel good enough. Should I consider a Master's program to try and boost this before I apply and broaden my chances? I have good extracurriculars and check off boxes in terms of other pre-med resume necessities (shadowing, volunteering, leadership, research, etc). Really conflicted about this and my family is playing a huge role in it too--finances are a concern and so is adding on additional years to the path for them. Any advice is appreciated!

tantacles

Your GPA at this point is acceptable for medical schools. Your MCAT score will ultimately determine your level of competitiveness.

Keep in mind that a Master's degree will NOT boost your GPA for medical school - only undergraduate level courses are counted in the calculation of your undergrad GPA.

It is completely acceptable to take several years off to work before you apply if that will make applying more financially reasonable. I would consider this as it would allow you to boost your application further, particularly if you work in clinical or basic science research and volunteer during that time. This way, your finances would be less of an issue. It is important to apply to medical school first and foremost when you are ready, so do not rush this process.


Should I give up? Low gpa/Bad academic record

Hello SDN experts, I have a 2.3 gpa. Currently, I'm taking some time off school to work. I'm technically a 7th yr now. My academic record has many holes/red flags. --Multiple course retakes with little successful attempts --Multiple academic probation, dismissals, retroactive withdrawals --Multiple bad quarters of below C grades No research/lab experience, few extracurriculars, no standout leadership/awards. I volunteered at a hospital - 813.5 hours. I'm aware of how bad my record is.. but my academic record does not reflect my true effort. I deeply considered my reasons to get into medicine, and I wish to find a way to "start over," if there even is such a thing. What are my options? Is there a chance, or should I just give up? What should/can I do at this point?

Thank you.

tantacles

While you certainly do not have to give up on your dream of working in medicine, a 2.3 GPA and your academic record will make very few medical schools, if any, interested. If you truly wish to be in medicine, you will likely need to take several years to repair your GPA and show that you are able to complete courses with good grades - show an "upward trend", if you will. Keep in mind that this is a difficult task. Medicine is not the only field of work in the world, and it would be prudent to look into other options, even if only as temporary lines of work, as you are extremely unlikely to gain entrance into medical school with your current grades.


Nitpicking Medical School Pre-Med Class Requirements/What are my chances?

Basic Information: - Graduated on time in 2016 at a private university with a BS: Biochemistry and a cGPA of 3.65 and an sGPA of 3.56 (both relatively uptrending) - MCAT: 497 (125/123/125/124); 498 (126/123/126/123); 511 (127/125/129/130) - Asian male

Clinical Experience:

- Emergency Department Medical Scribe: +1500 hours (2016-mid 2017)

Research Experience: - Chemistry research: +500 hours (2014-2016) - Poster and Powerpoint presenter at 2 separate symposiums - Pending publication

Teaching Experience: - General/Organic Chemistry Workshop Leader: +250 hours (2014-2016) - General/Organic Chemistry Tutor: +100 hours (2014-2015) - Organic Chemistry Private Tutor: +100 hours (2015-2016)

Other Experience: - Student Assistant/Supervisor of University Library: +1000 hours (combined); 2013-2016 (promoted in 2015)

Extracurricular Activities: - American Chemical Society Club Officer: +50 hours (2015-2016) - School Symphonic Orchestra Violist: +100 hours (2012-2014)

Hobbies: - Piano: Studied +9 years - Viola: Studied +6 years

Letters of Recommendation: - 3 science professors - 2 physicians (MD)

Notes: - mid-2017-2018 hiatus explained by recent house move and retaking the MCAT twice. Plan is to resume scribing and joining a community orchestra on spare time - Will plan on adding DO schools to application list

List of MD Schools (so far): - Cooper Medical School of Rowan University - Creighton University School of Medicine - Duke University School of Medicine (reach school) - Eastern Virginia Medical School - Geisinger Commonwealth School of Medicine - Howard University - Loyola University Chicago School of Medicine - Oregon Health and Science University (top choice/dream school) - Rosalind Franklin University of Medicine and Science - University of Toledo College of Medicine - University of Arizona College of Medicine - University of Minnesota Medical School (both campuses) - University of Washington School of Medicine - Wayne State University School of Medicine - Wright State University Boonshoft School of Medicine

Main Question: My goal is to get into OHSU. It's been my desire since high school and I think I have a decent shot considering my life-long Oregon residency and relatively solid application (gratefully improved with lots of help from others). Based on the MSAR, my results fit within their GPA and MCAT standard deviations and I believe that I would make a good fit for their school. My only problem is that I did not take a Genetics course in college and it is listed as a required (not recommended) course on their course list. Considering the rest of my application, I would hate for that requirement to be the only thing barring me from entering or even being considered at that school, since I don't know if I have to time to go back to school (even if it's from a community or public college). Will this requirement and my lack of taking the class really prove detrimental to my chances of getting accepted there, or am I simply just overthinking this? Thanks in advance.

Also, any recommendations regarding the list of schools are also greatly appreciated.

tantacles

At this point, you have a competitive application for medical school. That being said, if Oregon requires genetics, I would not make the assumption that they will make an exception for you. This may mean that you have to take a year off to apply to medical school. During this time, you can make yourself more competitive by getting a job in medicine or in research, and you can take just that one course to make sure that you have fulfilled the requirements.


High-School Classes to become an Anesthesiologist

From freshman to senior what would be the best classes to take for a job in the medical field as a anesthesiologist. Along with the best classes what would be the requirements to go into the field and become an anesthesiologist.

tantacles

The most important thing to do to become an anesthesiologist is to get into medical school first and foremost. To do this, you can take any classes in high school that you wish as long as you do well in them. When you are in college, you will need to fulfill all pre-medical requirements (which you can find on the website of the medical schools you wish to apply to) and take the MCAT as well as volunteer, find some experience in medicine, shadow physicians to establish your interest in medicine, and get experience in research. I would talk to your guidance counselor at school; they may be able to point you towards alumni who have chosen to go into medicine.


does my resume look competitive for DO medical schools?

gpa 3.58 no mcat yet, president of the anime club, president of the learning differently club where I tutored people with disabilities, president of the pre med social club, abroad to Nigeria for clinic volunteering, abroad to Kenya for helping people with disabilities, honors degree at my college, shadowed 200hrs, radio director at my church, participated in student leadership academy club, research for a year, clinical volunteering 250hrs

tantacles

Your resume looks competitive for both MD and DO schools. It is impossible to stratify you without an MCAT score, but if your MCAT goes well, you will be competitive for both.


I think I am good applicant, but am I missing something?

I am an older applicant. I am taking the MCAT in May and have completed the required coursework. I went to school right out of high school, but didn't get very good grades because I had no idea what I wanted to do and was just exploring different classes with no clear goal. So, I left after a couple semesters. I built a career, started my own small business, and spent a lot of time in various healthcare/mental healthcare settings. I obtained several certificates and eventually decided, at 27, that I felt called to go to med school. I returned to college and finished a degree. I feel I am a really well-rounded applicant with good experiences (academically and professionally) under my belt and surety that this is what I want and I will work hard for it. I have done a lot of really fun extracurricular things including volunteering and getting involved in my community. I have also been able to shadow a couple physicians and I have enjoyed that tremendously. But, my GPA is only a 3.6 because of the couple semesters I spent in school when I was was younger. Since I returned to school, though, I have had straight A's and achieved a 3.96 GPA in my minor, which is chemistry. I am really concerned about how these older semesters/grades look, and if the things I think add value to my application really do. I am concerned that it will look bad that I didn't finish my bachelor's degree as a younger student; I am 29 now. I think my MCAT scores will be relatively good. Am I being too hard on myself about not being younger or the "perfect" applicant? Will schools really be able to see how far I have come and how hard I have worked?

tantacles

It is difficult to say how schools will view your application. Much of that will depend on how you write it and which admissions committee member reads it.

That being said, your upward trend in your GPA will speak well to your current success and will likely be looked upon positively.

It is hard to stratify your chances without an MCAT score, but if you do well on your MCAT, you should have a great chance of being accepted to medical school.


I'm married and going to a caribbean medical school (AUC) as a female. Pros & cons?

I've been looking over these blogs and can't find exactly what I'm looking for, so I'm just going to layout what I'm going through. I'm 25 and married to a great guy. He is a mechanic and I work as an ascp certified microbiology lab technician. I applied to med school 4 years ago. I had a gpa of 3.8 and an mcat of 26. I had multiple interviews and waitlists, but didn't get in. I was crushed and tried to forget about that dream. Four years later, I realized that growing bacteria and obeying infectious disease docs wasn't going to satisfy my insatiable appetite for learning, I decided to re-apply to med school. The only problem was my MCAT expired after 3 years; I should just retake it, right? No, apparently the test has changed and added new subjects, so my $1500 of study materials are useless. Also, to reapply I'll be 1.5 years older by the time I find out if I "might" get in. I'm broke and find out Caribbean medical schools accept the MCAT for 5 years. I can't take out a loan for more MCAT study materials, but I can take out a loan for medical school. I applied to AUC and was accepted within a week. Every doc I work with is extremely supportive of this type of med school (even though the internet says something different). My husband is supportive and willing to quit his job and continue his education via online classes while I'm in school. Does anyone have any input regarding Caribbean med schools? Is it a lost cause or worth a shot? Also, what about marriage? Has anyone been married while at a Caribbean med school?

tantacles

Going to a Carribbean medical school is typically a bad idea. Carribbean medical schools inflate their match rates to make it seem as if all of their students are matching, whereas in reality, they have a high attrition rate and won't even allow many students to take Step 1. There are many posts about this available on SDN, and I would recommend you would review them.

Being married in medical school and being in a long distance relationship is difficult but not impossible. The most important piece of the puzzle is to speak to your husband about not only what he is willing to do but what he wants to do for his career.


MCAT Advice

I took my MCAT in January and just got my score back. I was not happy with the score that I got, so I think I am going to take a gap year and retake the MCAT next January. I really only studied for three weeks, so for the next time I take it, I plan on studying a ton. I want to start studying over the summer. However, I am a D1 athlete and I am in season in the fall. It's hectic enough being in season trying to manage my regular coursework and extracurriculars, so I feel like I won't be able to study much. Is it worth it to start studying over the summer or should I just take the MCAT at the end of the summer? How would you best recommend to stay fresh with the material while not studying super intensively? I also want to let you know that I will be doing research over the summer at my college, and after season ends, I will be done with college athletics and I will have a lot of extra time in November, December, and January to be able to study.

tantacles

Handling the MCAT is difficult, particularly when you have other commitments. My best advice is simply this: Make sure you have time to study, and do your best to only take the MCAT when you are ready, even if that means you need to delay your application for a year or two to be sure you'll do well.


Why do most of the pre req's for med school have little to do with medicine?

Some of the prerequisites have very little to do with medicine at all. So why not take a class like physics II or Ochem II and replace it with anatomy. I think that it is crazy that we require Ochem II and not anatomy or even crazier we require Physics II over anatomy? Can someone explain this to me?

tantacles

The pre-requisites for medical school are frustrating at times to complete. However, each does have some applicability to medicine, and each tests critical thinking skills. This is something that, assuming you become a physician and have some say over the requirements for medical school, you may wish to change as you continue your career.


Are professors all from sophomore year acceptable to ask for letters of recommendation from?

I am considering asking two of my chemistry professors from sophomore year and a professor from my major for sophomore year. I'm limited in which science professors I can ask as I am a non-science major so I have taken the bare minimum science requirements and they've all been very large. All three have agreed to write letters.

tantacles

There should be no problem asking recommenders all from one specific year as long as they can all write strong letters for you.


anyone is queue for a post bac program

I am looking for information about being the queue for a post bac program...what are my chances now since i was not invited for an interview but i was placed in the queue until May

tantacles

It is hard to say what your chances are as programs have individual selection criteria, and even a waitlist or queue varies by institution. The admissions committee is likely the only body that knows your chances.


Non-traditional applicant with PhD

I have a foreign Bachelors of Science degree (molecular biology and genetics) and just obtained a US PhD (genetics) degree and looking for a post-doc position. I have contributed to 11 peer-reviewed publications. I have close to 100 hours of volunteering and 50 hours of shadowing.

foreign BCPM GPA=2.64 (106 hours) foreign cumulative GPA=2.77 (144 hours) US Postbaccalaureate GPA=3.75 (12 hours) Cumulative undergarduate BCPM GPA=2.75 (118 hours) US PhD GPA=3.20 (41 hours)

1ST MCAT 495 2ND MCAT 505

I found out that some schools don't accept foreign courses or undergraduate degrees (even tough I had my transcript evaluated by WES and transferred courses to my current institution in US). I was also told that my GPA is very low. But the real problem is that I am determined about getting into medical school and becoming physician/scientist (even more determined after being told by a medical school admission committee chair that no matter how many A's I get after this point, I will never be able to get into medical school).

I am considering re-taking prerequisites courses here in the US to increase my overall GPA. I realize this might take a few years as I have to work and provide for myself.

What suggestions do you have for me to make myself a more competitive applicant? Best!

Mr.Smile12

Presuming that your citizenship status in the US is okay, your general GPA and MCAT scores are lower than ideal for certain MD programs. I'm not sure how much DO schools might be able to view your more recent GPA favorably. But I guess, the big question is why you want to switch from a Ph.D. genetics path to a medical pathway. Numbers and grades are important but you will at least get that question.


How bad is it to quit my pre-med job after 6 months?

I started working as a medical scribe in an elite ER 6 months ago. Although I loved it initially, working the required hours during the academic year takes too much time and energy away from my coursework and I might have to quit. How bad would it look to quit after 6 months?

I am a non-technical major and my resume is very light on science classes/traditional pre-med activities. My job as medical scribe was my one impressive pre-med activity. If I quit, does it look like I can't handle it or like I change my mind too quickly (I left 2 research positions after 2-3 months)?

Thanks for any feedback!

Mr.Smile12

Usually anyone doing a medical ER scribing job is advised that it is a full-time job, so I don't know if there are programs nowadays that allow for part-time hours. I don't think it's a great idea to be taking classes and doing scribing at the same time. I do think the pattern of you dropping something after a few months is a very concerning pattern in that it seems like you're not able to handle priority setting and be able to focus on doing a single job well. It's possible that the scribing opportunity while impressive isn't really something that really gets you up in the morning if it isn't inspiring you to reorganize your life so that you can excel in coursework and your scribing, but scribing is a really hard thing to do if it's the only thing you do.


Should I pursue medicine?

I am new here, and I can’t seem to find a post related to this question, so I apologize if it has been posted elsewhere. I will gladly take directions to that thread.

I am currently a 3rd year undergraduate student, and I am very interested in the field of medicine. I am also currently engaged, and I am looking forward to my marriage next year.

My issue stems from my own interests; I am someone who is very much wants a career that I am passionate about (and I love learning about medicine); however, I am also a very family-oriented individual, and my one major goal in life is to have a family, raise my kids, and spend time with my family. As most of you know, there are many times in the field of medicine where these two interests conflict, sometimes very intensely.

I have been looking for jobs that I could occupy once I graduate with my bachelor’s degree, just in case I cannot get into med school. My internal conflict is this: I feel as if I am not committing enough to med school to pursue it, yet I cannot see myself pursuing any other career. I want time with my family, but I would be willing to sacrifice some time for a job that I truly love.

Any advice or questions about the issue are more than welcome!

Mr.Smile12

Your topic comes up in some variation in the forums, so I would suggest that you review the feedback there. I will say that you have a lot on your plate and in your intended new married life. If your major goal in life is to have a family and spend time with them, you will be discouraged from professional education (medicine, law, etc.). There are women who do go through medical education (school, residency, etc.) and can balance starting a family, having children, and so on, but they do emphasize it is hard work, and you have to be mature about it. If you are admitting that you don't feel you're committing enough time to pursue a long-term career goal in medicine, as it seems you really cherish your future role as a family provider, you will probably want to take your time to think it through. Medicine is not the easiest to achieve work-life balance when you're just starting out, though it is possible if you have the resources and support. But those who I have talked to who have made it pretty much agree you have to have as much passion to be a health care professional as you have the passion to be an active parent. From what you wrote, I am not sure if you think you're there yet.


Will my age given my credentials and experience negatively impact my admission to medical school?

I am a Black premedical student and I will be 18 when applying for medical school and I want to know if the following would be enough to make me a qualified medical school applicant or if I would need additional experiences: - 3.68 science GPA and 3.8 overall GPA with 506-508 MCAT Score - Presenter at my university's undergraduate research symposium (2017) - A tutor for high school students in Math and History ~200 Hours - Volunteer at a hospital ~192 Hours - Member of a STEM club devoted to increasing women's representation in the field (3 years membership); we also visited visited the university's cadaver lab and simulation lab - Member of the university's Jewish club where I took part in organizing events for the community such as food drives (Leadership)

Please let me know if my age would play a significant role in negatively impacting my application or if it would at all given my credentials.

Mr.Smile12

I suggest connecting with any admissions officers and diversity officers at the schools you are most interested in. It does appear you have a lot of good items on your resume, but what cannot be determined here is your professionalism qualities when it comes to decision-making, listening and communicating, or working in very challenging situations ethically. (Not saying you don't have those qualities, but from what you have listed, one cannot really comment.) Why medicine if you have a lot of experiences that could also set you up well in a research-oriented path? What other community service experiences have you had that you have returned to over time?


Community college

I graduate in May with a BS in Psychology, I’ve taken all my prerequisites for medical school but my science gpa is trash, I’m studying for my MCATs right now, but once I’ve taken it i want to take a few courses at a CC that i have taken before like Genetics, Anatomy, etc. How will medical schools precieve that? I want to take them at a CC that’s like 5 minutes away from my house and it will be way cheaper. The bio department at my university is terrible so I refuse to take anymore classes there

tantacles

While community college courses will boost your GPA, they are not viewed as positively as courses taken at a four-year college. I would recommend that you take your pre-requisites for medical school at a four year college.


Re-applying only to number one choice before retaking MCAT

Hello, so I was wondering if I could get some advice about reapplying. I am a student from Maine and I am dedicated to the state and interested in only practicing in Maine. Because there are not any allopathic schools in Maine, the best option is Maine Track through Tufts. I applied this cycle to Maine Track and a handful of medical schools but I have not received any interview invites. This is most likely due to 67th percentile MCAT score. Otherwise, I have a great GPA, clinical experience in rural Maine, lots of research experience, leadership positions, and some volunteer work. Right now my plans are to not apply in the upcoming cycle but wait a year to take the MCAT (currently working full time plus a very long commute), and bolster my application with a publication and more clinical experience. My question is, should I apply this upcoming cycle but only to Tufts for the Maine Track option? New aspects would include a publication and more shadowing/volunteer work but I do not have enough time to improve my MCAT score. If I do decide to apply this cycle and don't get in, is it still feasible to apply a third time? I really want to do Maine track so I don't want to ruin my chances by reapplying prematurely. Thank you for any and all advice, I really appreciate it!

tantacles

I would recommend that you wait to apply again. You can absolutely apply to your top choice, but it is always best to wait until your application is fully ready. Taking your time and waiting to apply again is a good idea, and I would recommend not applying just to one school this cycle as it may hurt your chance of getting an interview during your third cycle.


When should I take the MCAT? I'm confused about the Early Decision Program (EDP).

The Early Decision Program deadline is August 1st, and I can only apply to one school. If I take the MCAT on June 29th with the scores released on July 31st is that cutting it too close?

Every week I have to study counts, since I just started my post-bacc program in July 2017 and spend ~40 hours per week studying rigorously on my own. The studying is going very very well, but the only thing is that I am banking on the MCAT since I have a finance degree with a low, 3.0 GPA due to health issues in undergrad.

This is why I feel that I need to apply to many schools, and choosing just one for the Early Decision Program is a complete and total shot in the dark. At the same time, I hear after the early decision, applicants are far less likely to get in to schools. I feel as though I will be ready by June, but 2 months is an extra 300+ hours to study. What would you do if you were in my shoes?

tantacles

I would recommend that you don't apply early decision unless there is literally only one school you could see yourself going to AND your scores are above what you would expect for a matriculant. Currently, your GPA is below average for all allopathic schools, and if you are not accepted to your early decision school you will be at a disadvantage when applying later.

Rather than applying early this year, I would suggest taking your MCAT when you are completely ready to do well and applying next year on the first possible day to multiple schools rather than pinning your dreams on an unlikely scenario that will put you in a bad position if you are not accepted.


MCAT and Application Advice

I am taking my MCAT on May 5th, one week before finals. My cGPA is 3.455, so a little low but with a good upward trend as well as decent ECs. I am currently very worried about getting a good enough MCAT score to make up for my GPA, and I am considering pushing back the date. My goal was to have applications finished and submitted by June 1st with an MCAT score release of June 5th. I wanted to get in early in the cycle so my application had more of a chance. The problem is, I work about 25 hours a week combined with volunteering, shadowing, research, and writing a thesis. I feel like I barely have time to breathe. I know that getting a good score the first time is crucial, however, so how much would my chances decrease if I moved my MCAT to May 24th (or later) to give myself a full 2 weeks with out school to give good focus on studying? (Obviously I am studying every week, but I only have about 3-4 hours to do so.) The release date would then be June 26th.

tantacles

I think the best thing you can do is take practice tests to help figure out where your score is likely to be. This will allow you to figure out the best study strategy. You are absolutely right that getting a good score is crucial, but only you can determine how much work you need to do to make that happen.


I am a US citizen, who completed her undergraduate degree in the UK in Economics, and am finishing my post-bacc in the US. Help with degree conversion?

Hello!

I am applying to medical school this spring, for admission to the 2019 intake and need some advice about degree conversion. About me:

1. I am a U.S. citizen but I finished my 4 year undergraduate degree in the UK , where I achieved a 2:1 in Economics. A 2:1 roughly equates to a 3.0-3.4 GPA. As the program was highly specialized, I finished with a Masters of Arts degree. 2. I then finished my MSc in the UK with merit. I did not take any science classes during my undergraduate or post-graduate degrees. 3. I moved back to the U.S. and started a post-bacc program. So far, I have completed Gen Bio (I and II), Gen Chem (I and II), Biochem, Organic Chemistry, Physics I, Physics II and have a 3.95 GPA.

How do I go about submitting my undergraduate and post-graduate transcripts to US medical schools? Will they acknowledge an international qualification and is this a disadvantage?

Pathdocmd

You do not send transcripts to the individual medical schools, but to AMCAS (https://students-residents.aamc.org/applying-medical-school/applying-medical-school-process/applying-medical-school-amcas/). The UK grades will not be converted in the AMCAS application, but your post bacc grades are excellent. I think the UK undergrad degree would be helpful by adding diversity to the class. A lot will be riding on your MCAT and the rest of your application (e.g., healthcare experience, volunteering, etc).


Choosing a Major Between Pathology and Virology

I am interested in the fields of virology and pathology. I aim to work at USAMRIID (United States Army Medical Research Institute of Infectious Disease). This institute focuses on protecting the warfighter as well as civilians from current and upcoming biological threats including bio-weapons and epidemics. They hire both virologists and pathologists and I was unsure of which to choose since both fields have similarities. From what I have read it seems like pathology would include a lot more than virology since it encompass parasitology, immunology, genetics, microbiology, and even autopsies (which I am actually quite interested in as well and had considered forensic pathology as a career). Due to the many fields pathology covers it seems like that would be the better option however with the goal of working with bio-weapons and epidemics would it not also be more viable to research viruses exclusively? Idk what are your thoughts?

Pathdocmd

There is no major in pathology. Pathology is a medical specialty that requires medical school, 4 years of residency, and most people do a 1 year fellowship after residency. If you are interested in infectious disease I would aim at a PhD in microbiology, which is research, or medical school, then internal medicine residency (3 years), then a fellowship in Infectious Disease (2 years).


PHD vs MD/PHD for Virology or Pathology

I am currently in my first year of college but am planning on majoring in either pathology or virology. I am unsure about weather to aim for the PHD or MD/PHD I have read a bit through the forums but still am unsure of which I should aim for. I intend to build my majors toward the goal of working at USAMRIID (United States Army Medical Research Institute of Infectious Disease). I do not much care for working with patients and I should not have to do that for the most part at USAMRIID however, from what I have read here the MD/PHD is much better anyways even if your only focus is research and lab work. Can anyone give me suggestions on which would be ideal for work at USAMRIID?

tantacles

If you do not care to work with patients, I would recommend, regardless of your future career path, not doing an MD. Doing an MD requires that you work with patients for at least two years, and you will likely be encouraged to do a clinical residency.


Low sGPA/cGPA. Crazy life story. TL;DR, what type of postbacc?

Hello SDN experts! As stated in my title, I have a low sGPA (2.7), and my expected cGPA after graduation this May 2018 will be 3.2. I've applied to a combination of SMP and course-work based post-baccalaureate programs. I've spent the majority of my sophomore-senior years in college commuting back home to deal with some intense issues with family. My story seems to carry some weight with pbacc admissions so far (got into Drexel BMS, UPenn SSP, and CWRU MS in Medical Physio). I have extensive research, volunteering, leadership, and shadowing experience as well. My application (IMO) is complete aside from taking the MCAT and nailing it, and bolstering my GPA's (A LOT). MAIN QUESTION: Which type of program will be looked upon more favorably in my case?

tantacles

If you want to raise your GPA, an SMP will not help at all; your GPA is not raised by master's courses. I would suggest that if you want to raise your GPA, you do a post-baccalaureate course to raise your science GPA before you apply.


Do I have a chance?

This is my first time posting on something like this, so please bear with me. I graduated in 2010 with a BA in psychology and achieved a modest 2.5 gpa. I transferred three times in my undergrad career and couldn’t seem to find a home during that stage of my life. Anyway, I worked for several years and now I’m back in school taking the science prerequisites. I’ve taken all of them at a community college, as money is a big factor. I’m almost done with the prereqs but I have a few upper level bio courses that I’ll take at a four year university. Never taken the mcat and so far I have about a 3.7-3.8 science gpa. I have no extracurriculars, no industry related work experience, however, I do plan to volunteer at a hospice while I study for the mcat. I suppose I should also say that I’m a minority and a Texas resident. What are my chances of getting into a medical school, preferably in Texas?

Thank you

Mr.Smile12

If you're in Texas you should connect with your admissions representatives from Texas schools who can give you better insight on how they may view your situation with undergraduate coursework. You really should try to find opportunities to observe clinical environments whether it's in hospitals or clinics to know what doctors are supposed to do. You should also find faculty or staff who advise prehealth students at your school. They should be able to give you appropriate advice or encouragement.


How can I increase my focus for the MCAT? It is so long.

So, my MCAT is going to be in May. I am applying to med school soon. I need to build endurance starting now. What is the best way that this can occr?

Mr.Smile12

Endurance is gained through practice, focus, and repetition. A runner doesn't get endurance without lots of training on a daily basis, and your mind doesn't work any differently. In general, you need to make sure you can be in a room devoid of distractions and be able to study or focus on your exam. I don't know how you have done already in classes where you needed this focus, but this should be a skill you've developed throughout your classes to date, and a computer exam won't be that much more challenging.

Among many tools would be meditation, which I think has some credibility in neuroscience research. Being able to calm yourself and focus on things for minutes to hours at a time takes practice. There are also quite a few forum posts that discuss techniques people have used to focus, especially with test preparation.

The other thing you need to have is a focused strategy when it comes to studying, and a dedication to that type of study for 90 minutes to 120 minutes (in my opinion). Focus on one or two domains of study at a time. I think the forums have suggested schedules which you can model for that.

Finally, look at the testimonials in Aspiring Docs that address test preparation: https://students-residents.aamc.org/applying-medical-school/taking-mcat-exam/how-i-prepared-mcat-exam/ .


Switched over to Biology/Pre-med, based on the stats provided, what are my chances for med school?

So as you read in the title, I have switched over to Bio/premed from a business major in the second semester of my sophomore year. I currently have a 3.3 cGPA (low I know). I have not completed any science courses yet so I believe that is a positive (so no existing sGPA, taking first science courses this semester). I am taking Bio 1 and Chem 1 this semester at my community college and then transferring to a 4 year university next fall where I will take the majority of my pre reqs for med school. I am ready to do whatever it takes in order to get into med school. I was quite lazy so far in college and did not apply myself much which shows in my GPA, but I am ready to get serious. Also because of my switch, I will not be graduating on time. So is it possible based on my situation? Any advice will help, thank you!

Mr.Smile12

With a clean slate when it comes to science courses, I think you know what you need to do: do real well. Get the science foundation and study skills that will help you with the topics that will ultimately help you in medicine and critical thinking, and gain test taking techniques to do well on high stakes exams like the MCAT and the boards. You can't be "lazy" with your learning or you will be easily overwhelmed. Your situation may be similar to those profiled in the AAMC's Aspiring Docs profiles, so I hope you can remain positive and resilient throughout the tough challenges ahead.


Options to improve application for 2nd Med Cycle

I am a 2016 UCLA graduate with a BS degree in Psychobiology.

I applied to medical school during the 2017-2018 cycle, and I submitted 16 secondary applications. I have not received any interviews. I am looking now to improve my application in hopes of reapplying, preferably in the summer of 2018. I am wondering if a better MCAT and more volunteering would be enough to make me competitive by June, even with a lower GPA profile. I would prefer not to delay another year by taking a post-bac program, but I understand I may not have a choice. I had poorer performances during my first 2 years of college, but I made improvements in my junior and senior year. My academic and extracurricular profile is below.

cGPA: 3.50 sGPA: 3.21 MCAT: 511 (April 2017)

2 Years full-time paid Research Assistant for UCLA Brain Injury Research Center (current) 1 Year volunteer Student Researcher for Spinal Cord Injury Lab at UCLA 1 Year volunteering program at the UCLA Ronald Reagan Medical Center 1 Research presentation at the annual UC Neurotrauma Conference 4 Years part-time employment as a Lifeguard at UCLA Recreation 30 Hours of physician shadowing

Thank you for your time.

Mr.Smile12

You should probably talk with many admissions counselors and your prehealth advising office, but I suspect if you think the problem is your lower GPA profile, then chances are you know what you have to improve unless you are willing to compromise a little. I don't know whether you applied to DO schools or why you wouldn't, but you should try to check whether the schools on your list are a little more selective than you would like or not. You might have to include a few MD schools that are newer and have very large class sizes on top of any DO programs you could probably fit with.


3.1 Overall gpa 510 MCAT

From Oklahoma. Have a 3.1 overall gpa and 510 mcat. Should I do a post bacc? Science gpa is 2.6 for AAcomas and 3.0 for amcas. Do I have a chance? My ecs and letters are already done.

Mr.Smile12

While I don't know where you are applying or what university you have attended is, in general terms, your chances would be better with a postbac master's degree program. I don't know if you just had a rough beginning or a bad ending, but strive for a better science GPA.


Chances at my dream?

Hello all. Ive been reading all the questions on here and I thought Id post my own considering my specific situation. I transferred schools the second semester of sophomore year from Seton Hall University to Montclair Sate University. I am now in my spring semester as a Junior. I wasn't happy at all at SHU, was underperforming there, and it was getting expensive. Here are my stats before and after transferring:

Seton Hall: Biology Major 3.0 GPA 2.Something (higher 2s) sGPA -> Haven't gotten the chance to calculate

Montclair State: Molecular Biology Major 3.9 GPA 3.7-3.8 sGPA (I haven't gotten below an A- on any course since transferring) Haven't yet taken MCAT but scheduled for June

Extra Curr: (with when completed) 300+ hours shadowing (internal medicine MD, pediatrics, internal medicine DO) - SHU+MSU 80 Hours geriatrics emergency department volunteering - MSU 120 Hours SMILE volunteering (food to the poor/homeless) - SHU+MSU Certified Phlebotomist (by time of application should have about 60 work hours) - MSU Certified Clinical Lab assistant (no hours worked yet) - MSU Summer internship cancer diagnostics lab - MSU Arabic Teacher for children - SHU

My main questions are:

1) Am I competitive for MD/DO schools? If so, can you list any? (I have a list but the more the merrier!)

2) How will my transfer affect how schools look at my application? (the averages of both GPAs aren't too flattering and its worrying me immensely!)

3) What can I do before application time to make me a more desirable applicant?

Thank you very much!

Mr.Smile12

There's really no way for anyone to determine your real chance of success in the application process until you take your MCAT. You do seem to have a lot of clinical experience and community service which should work in your favor, but what impact do those activities have on your career choice to medicine remains your responsibility to articulate on your application and in interviews. You need to do your own research on schools where you feel most comfortable and excited to learn as a student, and you can also think carefully about the individuals who will provide references for you. You should talk with your prehealth advisors before beginning the application process for sure.


Do I have a chance?

I'm a Junior at Swarthmore. I want to go to med school but the only thing holding me back is my GPA. I'm a psych major because I want to be a psychiatrist. As of now, my average is a B+, with an upward trend. The only grades in which I have below a B- are my chemistry courses. I'm going to take organic chemistry II and biochemistry somewhere else over the summer because they are so hard at Swarthmore. Hopefully I will do better prove that I am not actually that bad at chemistry. However, my biology grades are B's, which isn't great either. I have yet to take physics. Almost all of my other courses are B+'s or above. I have not taken the MCAT yet. I'm trying to weigh these disadvantages with my advantages. My college's website says since Swarthmore is so difficult, med schools bump up your GPA a little when considering your application. My psych grades are very high. I'm hoping they consider the specific field I want to go into when looking at my major. I was my high school's valedictorian and I'm in the National Honors Society. I got into a very competitive summer research program at Rockefeller University (SURF). I am listed as a co-author in 2 psychiatry journals from the work I've done in the program, have clinical volunteer experience, and might be published in a 3rd paper with a professor I'm working with in the clinical psychology department. I am also half Caribbean/Latina and my mother is an immigrant from Guyana, making me a second generation immigrant. I will take some time in between to gain clinical experience. Through connections I have, I will probably be able to get a job at some pharmaceutical company. Do I have a good chance?

Mr.Smile12

If you don't have a lot of clinical experience, including experiences outside being a psychiatrist, I think it would be very difficult to go through medical school and learn tasks that you probably won't really be using as a psychiatrist. You really should know what doctors/physicians do in as many specialties as possible, because that's what you will ultimately do in your clerkships in most of your training. Is there a reason why clinical psychology (PhD/PsyD) is not something you want to do?

You should also be sure to ask your prehealth and career advisors on campus. They can help you figure out your options.

tantacles

Most medical schools will look at your GPA in isolation without adding or subtracting points for difficulty. Thus, you still have a chance of getting into medical school. Unfortunately, the "mights' in your application will not be helpful until they are solidified. That being said, it sounds like you've done the pre-requisite things to get in but need to improve your GPA.

I can not, however, stratify your chances in any way until you have taken the MCAT. An excellent showing on the MCAT would make your application much more admissible for med schools, while a poor showing on the MCAT would seal your fate and make admission very difficult.


Road to Medical or PA school

Recent graduate with Bachelors in Biology GPA: 3.38 Science GPA: about 3.20

tantacles

It is still possible for you to get into medical school or PA school. However, you will either need to improve your grades with a post-baccalaureate degree or have a stellar showing on the MCAT to have a chance at MD school, though DO schools are still within reach.


How are my stats for med schools, and do I need more volunteering?

Here are some general stats about me:

-3.93 GPA and 3.89 sGPA at Rutgers New Brunswick -Molecular Biology & Biochemistry major and Psychology minor -100 hours volunteered at RWJ Emergency Department -80 hours shadowing an anesthesiologist/pain management specialist at Hospital for Special Surgery, NYC -70 hours of rotations shadowing in departments of oncology/hematology, OB/GYN, Surgery, and ER -I have worked around 15hrs/week in a gene silencing lab as a bench scientist since Fall 2015 -I've been awarded a research fellowship of $570 for writing a proposal for my research -I have yet to take my MCAT (additional question: what's a good score considering my other stats?)

Had a bad dream that 100 hours volunteering isn't enough-- Should I do more?

I plan on applying to med schools this June.

Mr.Smile12

I wouldn't worry too much about your 100 hours of volunteering considering you have another 150 hours in clinical observations or shadowing. You also have a lot of experience working in research labs, though I don't know if your preferred schools want more research. I do wonder what your non-clinical community service/volunteering experience consists of.

As for your MCAT's, you should make sure you have an idea from the schools you are targeting to apply to. They should have their published ranges in the Medical School Admissions Requirement resource or on their websites/recruitment materials.


I know some classes aren’t required for admission into Med School however if you had to pick the ideal schedule for freshman through senior year what classes would you take to help with the MCAT and Med school?

I am a High-School student with only a small amount of college experience and, I’m going to a private school with on a pretty large academic scholarship however, the school doesn’t have as much experience getting kids into med-school so I am trying to at least fix up a schedule that will help with the MCAT and Med-School.

Mr.Smile12

Every college you should be considering has a sample schedule made for those pursuing premedical/prehealth studies, but it also depends on the rigor of your college prep coursework and how much credit you may get before your start as a college freshman.

If you are taking about your high school classes, I think you need to be strong overall with plenty of rigorous college classes (math, science, history, psychology, arts, etc.). What is probably just as or more important is volunteering in the community whether in clinical or community service environments. You should do that volunteering consistently and over a long period of time, and perhaps continue it once you are in college. One should understand what it is like to care for someone with a chronic illness and what the current failings of our health care system are in helping patients (in my opinion).

tantacles

In college, there are pre-requisite courses for medical school: biology, organic chemistry, physics, and inorganic (general) chemistry. Many schools also require mathematics, humanities courses, and psychology courses. The best course schedule to get you into medical school is the course schedule that works for YOU; that is to say the course schedule that will allow you to get the maximum number of A's while still fulfilling requirements for medical school.


Undergraduate in US, Post-bac/Grad School in Foreign country

I am completing my pre-med requirements and my bachelors degree at a university in the US. Before going to Med-school I want to try to explore a little bit and I was thinking of applying to either a post-bac program or a grad-school in a foreign country (also possibly try doing some research there) where a lot of my family lives. I was born in the US and have done schooling up to now in the US. I am still trying to go to school in the US so I was wondering if this would look bad on the application.

tantacles

not going into medical school right away is a viable option, and many applicants explore other careers for many years before pursuing a career in medicine. This is absolutely acceptable. Similarly, doing more coursework or a graduate degree, provided you do well in that coursework, will not hurt your medical school application.


Extended undergraduate education or would a post baccalaureate program be beneficial?

I am a 21 yo currently finishing my 4th year of undergrad. I currently work full time as a scribe in the ED and a cardiology clinic. I have worked full time throughout my college career to pay as much of my way through. I have not had the smoothest undergraduate career but will be graduating next year with plans to began a post baccalaureate Pre-medical program (a.k.a “career changer”) to retake Gen. Chem. and finish the rest of my medical school pre-requisites with a much better GPA than my cGPA which will be roughly a 2.8-2.9 as a 5 year undergrad. My question to you is with now being a "non-traditional" applicant, with a lower cGPA. Would it be beneficial to take a "formal" post baccalaureate program or extend my undergraduate education an additional year to obtain these credits and potential raise my cGPA? I will have an Associate's in Psychology, BS in Individualized Studies (custom degree program at my university), and a minor in women's studies at the time of graduation. My primary reason for considering doing a post baccalaureate program is fear that an admissions committee would "look down" on such a prolonged undergraduate education.

tantacles

Either option that you mentioned is acceptable. Right now, your GPA is not great for medical school, and you definitely need GPA repair to be considered along with an excellent showing on the MCAT.

The length of your undergraduate degree is not too concerning, and should not impact your med school application significantly.


Is biology 1 and 2 required for medical school

I have taken microbiology, intro biology, anatomy and physiology, and planning on taking genetics. Will any of these courses qualify for the 8 hours of biology required for most schools?

Mr.Smile12

Recently, many medical schools have been rather explicit in noting the content that should be included in any biology requirements for medical school, so you need to be sure you read what each school says on the matter or consult each admissions office for schools you are interested in attending. To that end, you may want to focus more on the content you would need to for the MCAT and the kind of scientific or critical thinking skills you need to develop in taking the classes you are selecting as it would help you with higher-order thinking you would need for the MCAT and for medical school.


Medical School with Criminal Record?

I just graduated with Pre-med and now I'm planning on applying to medical school. I have a criminal record that has been EXPUNGED!! When I was 18 and stupid, I was involved in Grand theft which resulted in Felony. It happened in 2014. As my record is expunged, do I have chances of getting in medical school? Or should I just forget about med school?? Please help!! :(

Mr.Smile12

If you have an expunged item on your criminal background, you need to be honest and disclose it even if it is expunged from your record. A criminal background check is required for all health care workers, and failure to disclose would definitely jeopardize your chances. Be honest about your situation, and stay clean. Get your prehealth advisor or team to help you with that.


Statistics and English???

Hello everyone,

      I go to a liberal arts college that requires me to take so many useless core credits such as fine arts, philosophy, history, economics, 6 credits of religion, and this piece of work called "integrative core classes". They have the course title of "RCCs". It's terrible, because instead of taking a class that I really wanted to take like CANCER BIOLOGY; I am forced to take RCC Seminars-- which is just basically reading and writing. 
    MY QUESTION: Do med schools really, really want TWO semesters of English?

Also, I am a neuroscience major and have taken 2 semesters of upper-division PY400 classes for "research methods and behavioral statistics" with accompanying labs-- can these count towards my BCMP gpa? I recently just switched out of Statistics because I found that the class was too much review for me.


Thanks so much everyone!

Mr.Smile12

You will have to check with medical schools about their English requirements, but it is definitely important to note that usually the two semesters of English is part of your graduation requirements for your degree. If you have to take some interdisciplinary seminars, you'll have to take them regardless of your feelings about them. (I'm sure there will be classes and instructors you won't click with when you're in medical school.)

I may be mistaken but I think that your research methods class could count to BCMP if it is classified and verified as such by AMCAS. You'd need to check with them and your institutional prehealth advisors.


Sickness, grades, and MD application

I ended up getting mono & pneumonia finals week my first semester sophomore year. After making up my finals the beginning of the following semester, my grades dropped a bit (i.e, got a bunch of B's and had to retake organic chem2). Is there a place to mention my grades dropped due to illness on the AMCAS application/ if I have strong grades the following years should I be fine?

Mr.Smile12

On AMCAS, this is usually handled in the personal essay, and it may be a specific prompt that shows up in any of your school-specific/secondary applications.


Are my pursuits realistic?

I am a prior Navy corpsman with tons of hands on experience and knowledge. My primary role as a corpsman was a surgical technologist. Currently I still work as a surgical tech and just recently I decided to change my pursuits towards med school pre requisites. For the last several years I have been attending online college knocking out classes with no defined path. I thought maybe I'd be interested in cyber security. However, as it turns out I've been in healthcare too long and learning a whole different trade is just not possible for me at this point in my life. I've come the realization that I've invested so much time in healthcare and it would be foolish to abandon it. For the last 5 years I have worked side by side with surgeons of every specialty. GYN, Neuro, general, ortho, uro, ent. You name it. I have applied to more reputable schools to obtain my sciences and have stopped using my veteran education benefits for the time being. Currently I hold 95 credits from an accredited university. The only issue is as a result of my indecisiveness, my current GPA is not competitive at all. I've maintained a's b's and mostly c's with several d's and most recently a few f's. How problematic will this be seeing as none of these classes are pre requisites to medical school. Is there any work around for this? Assuming that I transfer to a reputable school, maintain a strong GPA, and score high on the MCAT, are my new found pursuits realistic? The one thing I do have going for me is my abundance of clinical experience. Please be as brutally honest with me as you can. Thank you

Mr.Smile12

Not knowing if you have connected with the prehealth advisor at the university where you have been taking classes, but definitely those grades are quite a bit lower than what most medical schools would consider, though if none of them are relevant to a biomedical science GPA, it is not clear exactly how much that would affect you except on a single school-by-school basis. To that extent, have you considered the USUHS Enlisted to Physician program track, and have you contacted them about your potential eligibility to participate? The website for this program is https://www.usuhs.edu/emdp2 .


Are my pursuits realistic?

I am a prior Navy corpsman with tons of hands on experience and knowledge. My primary role as a corpsman was a surgical technologist. Currently I still work as a surgical tech and just recently I decided to change my pursuits towards med school pre requisites. For the last several years I have been attending online college knocking out classes with no defined path. I thought maybe I'd be interested in cyber security. However, as it turns out I've been in healthcare too long and learning a whole different trade is just not possible for me at this point in my life. I've come the realization that I've invested so much time in healthcare and it would be foolish to abandon it. For the last 5 years I have worked side by side with surgeons of every specialty. GYN, Neuro, general, ortho, uro, ent. You name it. I have applied to more reputable schools to obtain my sciences and have stopped using my veteran education benefits for the time being. Currently I hold 95 credits from an accredited university. The only issue is as a result of my indecisiveness, my current GPA is not competitive at all. I've maintained a's b's and mostly c's with several d's and most recently a few f's. How problematic will this be seeing as none of these classes are pre requisites to medical school. Is there any work around for this? Assuming that I transfer to a reputable school, maintain a strong GPA, and score high on the MCAT, are my new found pursuits realistic? The one thing I do have going for me is my abundance of clinical experience. Please be as brutally honest with me as you can. Thank you

Mr.Smile12

Not knowing if you have connected with the prehealth advisor at the university where you have been taking classes, but definitely those grades are quite a bit lower than what most medical schools would consider, though if none of them are relevant to a biomedical science GPA, it is not clear exactly how much that would affect you except on a single school-by-school basis. To that extent, have you considered the USUHS Enlisted to Physician program track, and have you contacted them about your potential eligibility to participate? The website for this program is https://www.usuhs.edu/emdp2 .


Probability of Acceptance/ Ways to Improve

A little bit about myself.. I am 22 years old and in the process of completing my MPH @ George Washington University with approximately a GPA of 3.62. As for undergrad, I attended Dominican University where I earned a B.S. in Neuroscience with a concentration in cellular and molecular neurobiology with distinction, honors through coursework and a minor in chemistry at the age of 20. I was able to graduate early due to my ability to take 8 college courses each semester at once. My G.P.A. is 3.65, and it only went down due to a C in Orgo II.

As for extracurriculars, I have done about everything. Volunteered in Rush hospital for 1.5 years at a surgical unit assistant, Student Senator of student government and eventually treasurer for 2 years, committee member for educational policies at Dominican University for 1 year, clinical preceptorship internship for 1 year (volunteered with E.R. physicians, and shadowed surgeons), student ambassador for 2 years, etc. I also worked as a peer mentor where I acted as a teachers assistant tutoring students both inside and outside of the classroom (2.5 years) and as a research assistant for 1.5 years. This resulted in allowing me the ability to complete a distinction project, where I wrote a 60 page paper to defend my thesis and completed an oral defense. My paper was published in the Constellation and I won a research award. I also work as a wedding planner on the side, and through this event planning I created a organization that allows kids with chronic diseases in hospitals create interesting wedding decor (can't go into more detail).

As for MCAT, I think this is my weakest aspect. I took it the first time and got a 488, considering I did not study and sorta winged it. I am currently in the process of studying for my MCAT, this time for real. I am averaging 500 on the practice tests, and am about to hire a tutor to help me get my score up. I am planning to take it on April 20th, 2018.

I guess my question to you is, do you think I need to change or fix anything in my application? I applied before without my MPH degree and with the bad MCAT score and got denied. I think this occurred because I submitted my applications extremely late in the cycle. I am worried that I might get denied again. I applied in 2016 cycle, and have spent the last year completing my MPH, volunteering abroad in Peru (2x this year) and creating that small organization with the wedding decor. I still have time to add some more things to my application before I apply in June 2018, so what would you recommend? I really want to get in this time and can't afford to lose more time!

tantacles

Your MCAT is currently keeping you out of medical schools. It will be impossible for you to get in without improving it. the rest of your application is fantastic, but your MCAT is absolutely the reason you are not getting in. You must improve your MCAT to have a chance of getting in.


What are my chances into getting into med school and what should I do to increase my chances?

I am a senior getting a bachelor of science in biology with a overall gpa of 3.97 as of right now. I am working on my last semester graduating this May and applying this cycle. I have a Biology GPA of 4.0 but a science GPA of 3.94 due to taking general chemistry 2 in the summer and acquiring 2 B's. I have little volunteer hours , around 50 hrs total at a clinic, but i have around 400 total hours of shadowing and having an internship with a surgeon during the past 2 summers. I have no extracurriculars and little volunteering due to not having much free time working around 20-30hrs a week and caring for ailing family members. My internship included some research aspects as well as leadership with being a role model to the new and young interns. I have 1 withdrawal during the summer before starting university due to family issues occuring. however i took the class in the fall and got an A in it. I am taking the MCAT in exactly 12 weeks from today, (Is that enough? I am very nervous about it.)I just started studying as i am finally taking a lower course load to balance out responsibilities, work, school and studying for it. However I do not know what score I need to acquire I aim for a perfect score but I am not sure what score I should get and not worry about trying to retake the MCAT if I get at least that specific score that will get me into medical school. I am taking 2 difficult courses only this semester since they are all I need to graduate, I am not sure if hours taken affect my application as I have taken over 15 both my freshman and sophomore year but as my family's health declined and to save money for medical school and emergencies I clepped out of free electives, spanish, business, human development and started taking 12-14 hrs my junior and senior year and now 6. I am finishing in a total of 4 years. I do not know how clepping classes affects my application. During my gap year I intend to volunteer and work in medical related occupation. At the lowest my science gpa could drop to a 3.76 0r rise to a 3.95 at its highest while my overall could drop to its lowest at 3.86 or remain at 3.97. I have heard ethnicity and socioeconomic background plays a role as well but I do not know how being white and living in a middle class family affects it. I am knowledgeable in 3 languages fluent in english but knowledgeable to carry a conversation in the other 2. I have also shadowed a doctor for a year in high school before graduating if that aids in the application in anyway. I have tried to provide every possible detail to help. Thank you for all of your time, help, and feedback in advance it is much appreciated.

tantacles

Your application overall is great, and a GPA of 3.76 is very good for medical school applications. It is impossible to tell how you will do; your MCAT absolutely determines your success.

If you do not have much volunteering, I would suggest you make 2-3 hours per week to do some volunteering of some sort. It can be any kind of volunteering.

Unfortunately, I do not know what clepping is and I can not advise you on that topic.


Post bacc or classes at large?

I am a nurse with a passion to switch to medicine. My undergrad GPA was 3.48. I am wondering if I should do a post bacc premed program or take classes at large? What do medical schools prefer? If I did really well in classes on my own at a university as opposed to a full program would they overlook the 3.48?

tantacles

A GPA of 3.48 is likely to get you into medical school with an exceptional MCAT; no medical school will "overlook" your GPA as it will still be paired with your MCAT, but an excellent showing in a post-bacc will absolutely help your chances.. That being said, if you have not finished your pre-requisites, you will have to do those, and I would recommend doing them at any four year college. I'm not sure what you mean by "at large." I will say that there is little difference between doing a formal post-baccalaureate program vs. taking the courses a la carte from a four year university, so feel free to do either one.


What are my chances? Med school admission

Hello, I am a 24 y/o hispanic female who is only been in the states for 8 years. I got my bachelors in psychology about a year and a half ago. I graduated with honors and with a very decent GPA then I started to take pre med requirement courses and my GPA dropped down to a 3.54 which from what I have heard is not great for med school. My science GPA is even worse 3.1.

   I worked full time throughout all my undergrad and I continue to do so. I volunteered as a research assistant for a year at the anxiety and phobia program at the university I attend and by the time I apply for medical school I am going to have worked with psychiatric patients for two years which I would think is better than shadowing a doctor. I want to become a psychiatrist but I am getting a little nervous about my GPA and my chances to get into medical school.  I have mentioned what I have heard is relevant for medical school admission; ethnicity, gpa, and extracurriculars. I did not include MCAT scores because I have not taken it yet. What are my chances to get accepted? What schools should I be aiming to when the time to apply comes (I live in FL)?

Thanks a lot.

tantacles

It is extremely difficult to tell you your chances without an MCAT score. Your overall GPA is good for medical school, but what you get on your MCAT will also determine your chances.

You should apply to all of your state schools and then choose schools that fit your general stat profile if you want the best chance of getting in.


Nursing ( ADN) to MD, how ?, any advise or sugestion would be greatly appreciated

Little about me, I am 29 just graduated from Nursing school with the associate degree in nursing, with GPA 3.7. I always was interested in medicine but living in small town ( Nursing school was the only closest option) and working a full-time job to take care my parents. English is not my first language ( i speak another 3 ) but I didn't have any problem graduating from my college. I started working as LPN and was fascinated by the amount of knowledge our MD has, ( in the facility where I work.) To make this short, sometimes halfway through my nursing school I started to realize that I can do much more than nursing, not to offend anyone, my instructors always complimented me, and advised me many times that I should continue my education and become Nurse Practitioner and that was a goal. As days go by and the more I follow our amazing doctor ( i follow him on my days off just to watch him and learn from him for my own self ) my desire to become a doctor keeps growing bigger and bigger I can't stop thinking about it. Every one I have talked ( mostly nurses and NPs) suggesting take the NP route as this route would be a smarter way, but I am not sure if I will be satisfied when I do become NP, I am not as young as most med school students, I can't afford to make mistakes. Would it be wise to get my BSN in nursing and get few more additional science classes to meet pre req for med school or just switch and major in Biology or chemistry? And those who in med school, does anyone worked part-time ( to help out family) and combined school and studying? ( i know its probably not wise because you will spend most of your time studying but I just wonder ) . I would like to thank you all in advance, I just need to know if anyone els have been through this and how did you manage it to make it, what helped you, and what path you took?

tantacles

The main things you need to do to get into medical school are complete all of the pre-requisite courses and get a bachelor's degree (in any field; it does not necessarily need to be in nursing or a science field.). Followed by that, you will need to take the MCAT and do well. this will likely take several years, though many people pursue medical school later in their lives, even into their 40's.


Trying to make sure i have covered my biology pre requisites, having trouble with how my colleges general bio series

My community college offeres a bio series that is Bio1 - cell/molecular bio, Bio2-animal biology, and Bio3 - plant biology. Each of these courses is one semester. I have taken cell bio (Bio1) and am now enrolled in plant bio. i cannot take animal bio due to course conflicts. Will this cover me for one year of general bio? i have been looking here https://www.cc-seas.columbia.edu/sites/dsa/files/handbooks/Medical%20School%20Requirements%20FINAL.pdf and it seems only case western and Harvard explicitly state that botany will not count. but i have a fear that maybe the definition of general bio excludes plant bio across the board and i have heard some people here and other places say something like this. is this the case?

Mr.Smile12

The problem you will ultimately have is whether the biology you take is the foundation for the MCAT. In general, botany-based content will not be covered explicitly on the MCAT whereas the animal/systems biology would. You should check with prehealth advisors at your local universities that you may be interested in attending if you were to transfer coursework from your community college.


Advice "Mcat score vs course load"

This is my 5th year in college due to changing the plan on applying to PA program to MD. Thus I have had to take required classes that now count as electives for my major. I have been studying full time for the Mcat for the past three weeks and would like to continue doing this until April 21st to hopefully get a high score to boost my application due to my GPA being a 3.6. Though this semester I plan on taking physics 2, genetics and biology 2 while working as a CNA. Would dropping biology 2, graduating then going back and taking the class during the fall, while I applying look bad on my application. This is in hopes of focusing more time on the Mcats and getting a better score. I feel because I have a 3.6, I need to do extremely well on the Mcat and this out-weights dropping a general biology class that consumes alot of time and covers many subjects that won't be on the Mcat itself.

tantacles

Every student has a different load that works for them. That being said, if you are having trouble studying for and taking the MCAT on time, consider pushing it back. There is no race to get into medical school, and if you need another year, it might be wise to wait and prepare your application so that it is at its best when you apply.


SMP Curiosity

How does SMP work? 1. Once you get accepted to the SMP of school X and you finish are you automatically considered for going to X med school for your MD? 2. Does school X count the SMP if it was done from school Y? 3. What are some true SMPs with med schools? 4. I’m not looking to go for D.O. 5. Any other relevant information because I am absolutely clueless.

Thanks in advance

tantacles

Many SMPs offer an automatic interview to those in the program, but that is no guarantee of admission. It is completely possible to finish an SMP without being accepted to any medical school.

BU, Georgetown, SLU, Tufts, and Rosalind Franklin all have SMPs linked to their medical schools, and it might be a good idea to take a look at their programs.


Reapply immediately, or take a break (and what to improve)?

Hello,

I applied this 2018 cycle and completed my application at about 18 schools. I have received 4 rejections, and complete silence from 14 schools. Since it's almost January, I am assuming it's time to reassess and prepare to reapply.

A small snapshot: cGPA: 3.58 AMCAS sGPA: 3.36 MCAT 506: Percentile rank of score: 71% chem / phys: 124, 44% CARS: 127, 83% Bio: 128, 85% Psych: 127, 75%

Residence: California. Undergrad: UCLA: Neuroscience B.S.

I believe in undergrad, I have a significant amount of research / clinical experience, and have since been working full-time as a clinical research coordinator at Kaiser.

I have thought about improving in the following ways: 1. Retake MCAT 2. Try and get the professor letters required (I only have 1 science) 3. Pursue a SMP this coming year 4. Maybe some nonclinical volunteering

My question summarizes too: Should one reapply immediately in the following cycle or wait? Would an SMP be helpful for me? Do I really just need to up my MCAT?

Thank you for your time.

Mr.Smile12

This advice is contingent on other admissions officers and prehealth advisors giving you insight with your application, and it presumes you submitted an early application. You need to really be sure you read the specific preferences for each school you apply to when it comes to their letters. If you only had one science professor letter, you were probably excluded because you didn't have the required number of science letters (presuming 2 or more). You might check if the MCAT or sGPA is lower than par for the schools you have applied to and get advice on retaking the MCAT or pursuing an SMP. They should also help give you advice whether a reapplication is worthwhile in the next cycle or later.


help deciding what to do this upcoming semester..

hi there! i need help deciding what to do this upcoming spring semester. I am transferring from a community college to the university of miami.. I still have yet to meet with an advisor to help me in deciding my track and i am growing anxious thinking of how i can prepare for the upcoming semester.. As of now, the classes i know i will forsure be taking are bio, chem 2 and lab, (no labs available in the spring for bio). what other classes should i take to fulfill atleast 12 credits? i want to also participate in extracurriculars and maybe get a part time job and dont want to burn myself out.. any suggestion on both classes and building my resume? Much appreciated!

Mr.Smile12

I hope your meeting with an advisor will help you with your answers. The prehealth advisors will usually have a list of courses and sequence that will help you keep your coursework with other responsibilities in mind. You also may wind up meeting others who also transferred from community college who share your interests in a health professional career who can give you advice.


What can I do now as a freshman to help my app later on?

So I just finished my first quarter. I am curious to know as to what I can do now, mid-way through school year to help my app. Thank you!

tantacles

I would suggest that you start doing some volunteering once per week (a few hours is fine), continue to do well in your courses, and think about getting involved in some research.


28, clinical research scientist, stellar(daily) clinical/OR exposure/extracurr, avg MCAT (501), low grades, 4x applicant (late apps due to low $), no interviews; how should i best improve my app and do I have a shot

I am 28 years old (URM nontraditional applicant) and I currently a clinical research scientist. I work both in the lab and in the clinic on basic, translational, and clinical research projects. I love it but I know the best manifestation of my work is as a MD. It will help me fulfill my best potential in this realm.

However, Im a current 4x applicant, with no interviews. I have applied late because life circumstances have forced me to divert my money to other emergencies at the time of applications; I dont come from wealth and I fund these very expensive applications on my own. i have an average mcat (501) but my clinical/OR exposure/experience and extracurriculars are at this point, better than most students gaining admissions. Its just that my overall GPA (3.08) stinks. I am confident in saying that as I have had a better CV than my sister who is a current 4th year. She had better grades.

Do I give up? I have been working towards this for the past 10+ years and im so far in that turning around now seems fruitless. I know tons of people that turn around to go get PhDs, RNs, JDs, MPHs, etc after a failed attempt or two. Im too far in and quite frankly, more years in have validated my love and knowledge in this field. I believe I am best fit to train in this profession.

What should I do? My 2015 MCAT runs up after next cycle.

tantacles

Your MCAT and GPA together do not make you a competitive applicant for medical school. Frankly, without an enormous improvement in your MCAT and a great improvement in your GPA, you are unlikely to have success getting into medical school. I recommend that you do not apply next cycle and that instead you take time to do a post-baccalaureate program, if possible, to improve your GPA (this can be done alognside a job) and study for the MCAT. The process of getting into medical school may take several years, but please wait to apply again until your MCAT and GPA are repaired. If they are never repaired, you are simply wasting money.


Retake or...?

I just finished my first term. C- English, C+ Calc 2, and B+ Gen Chem, and A- GE. I know it's not the best, but I am definitely gonna be working my tail off from now on. I have the option to take an F on my English class and take it next quarter and that will boost my GPA .6 points. Should I? Also, during finals week there was a death of a family friend that affected my family and I. What should I do?

tantacles

An F in English will count as a 0.00 on your AMCAS GPA. Taking an F and retaking the course will not be good for your GPA. I would suggest you simply do your best to improve your grade.


Should I give up on my lifelong dream?

Hey! My name is Sou (short for Soukaina). I am Moroccan born, but raised in the USA. I am a Berber (amazigh tribe). I was raised Culturally as a Moroccan. I can speak two languages fluently- Arabic and English. I am also intermediate in French (four years). I am also an artist - painting and drawing is my forte. I painted a painting which was given to the chancellor of PSU Harrisburg and gave a speech to the board of directors based on it. I work part-time as a pharmacy technician at Rite Aid. I have been working there for 3 years as of today. I’m 21 and faced with a dilemma that has been haunting me for a long time. Ever since my 3.0 gpa my first semester I have been depressed. Therefore, that gpa continued for a while. Recently, I picked myself up and started getting 3.6+ semester gpa. I currently have a science gpa of 3.2 and a cumulative gpa of 3.1. I still have around four semesters of school still and was wondering if I should consider another field of work or keep going. I don’t see myself doing anything else. I didn’t take my Mcats yet. I am currently studying for it.

tantacles

You can still do medicine. You need to make sure that you continue to get as many A's as possible in your courses. You also need to do well on the MCAT. It is impossible to stratify your chances without your having an MCAT score and a final GPA. I would suggest that you apply to both MD and DO schools as this will give you the best chance of being a physician.


I am a sophomore undergrad and have 5 c's in my transcript.

I have 5 C's one is a C in US Government i got during high school it was a concurrent enrollment class. 2 of them were C+ In phycology and statistics. I also got a C in math 1050 and chemistry 1210. I will be retaking gen chem but i am worried if this is it for my medical school dream. How can i overcome this and how are my odds looking on getting accepted to a medical school. Im feeling extremely discouraged and sometimes i just want to give up or drop out.

tantacles

The most important thing is your overall undergraduate GPA. Aim to do your best in upcoming courses, and you can absolutely make up for these grades. You will also have to do well on the MCAT to be competitive for medical school.


Starting off my first year of pre-med with a not so perfect GPA...

This is way early, but I just completed my first semester of pre-med and my GPA is a 2.85. I know I need to work very hard for the rest of my years, but do I still have a chance for applying to medical school?

tantacles

You have a chance; you will just have to do better in the coming years and do well on your MCAT.


Near Graduation jitters.. HELP Please!

Graduating in the spring with a B.S. in biology looking to take a gap year! As I near my last winter break as an undergrad I’d love some insight to see where I’m at as an applicant for the 2018 cycle: URM Black male 3.62 cum gpa 508 mcat Clinical research in hospital setting 250 hours Health disparities research on colon cancer and prostate cancer in AA men (hopeful to get at least one publication out before June 2018) Bench research (plant bio 1 year and pharmacology 1 semester) Volunteer at local high school In a competitive stem leadership program on campus Work in a sim center where I assist in setting up class for med students at my campus’s medical school

Would also love some potential schools as to where I would have a good chance to get in. I’m lost as to where to apply to!

Thank you

Mr.Smile12

If you work this spring semester on getting your application together with references that confirm your involvement in clinic, research, coursework, and volunteering, and are on track to submit your application by May/June, you should have a good chance at some schools that you may be reasonably interested in. Work hard this spring to talk to school contacts at recruitment fairs to give you a better idea of what schools would be supportive of you as a medical student. Connect with AMSA and SNMA student members to get a better idea of what schools would fit with your goals and mentor you to success.


Finishing Ms looking to continue education but is med school an option for me? Brutal honesty appreciated

Hi all,

I was wondering if those who are more informed than I could give me some tough love advise on my chances of getting accepted. I decided to do a ms out of college to see what I wanted to pursue and now I'm possibly thinking the M.D. application route if it's a possibility.

Undergrad: 3.0 gpa bio/Neuro double majors with plenty of research. 2 years of garbage 2 years of 3.4 work.

Ms: out of a solidly ranked Neuro program with 3.45 GPA including gross anatomy with full dissection, histology, and medical neuroscience with M1s and a research thesis as well a certificate in teaching biomedical sciences.

Did not take orgo or biochem in undergrad so I took it while in grad school and did well. I'm currently shadowing and studying for the mcat as I finish up the ms at the end of this academic year. 24 yo Hispanic male from Illinois.

Assuming I do at least average (big assumption I'm aware), what is the realistic chance of being accepted somewhere?

Thanks for your time and honesty

tantacles

If you want to go to medical school, you must do well on the MCAT. Assuming that goes well, you have a chance. If you want to go to an MD school, I would suggest GPA repair which will need to happen in undergraduate and not graduate courses. Many colleges offer a la carte post-baccalaureate courses, and those would likely help your application.


Post-bacc options for Canadians?

Hi all,

I live in Canada but completed my undergraduate degree in the U.S at Penn State University (on athletic scholarship) My overall GPA was a 3.48 which is not competitive enough to get accepted to medical schools in Ontario. Additionally, I need some more pre-req courses such as biochemistry, physics 2, and organic chemistry 2 (which will also help me with the MCAT).

There are plenty of post-bacc options in the States but they seem to be really hard to come by in Canada.

Would you recommend doing a masters to improve my GPA and then also taking some of those courses to help me with the MCAT? Or do you or anyone know of programs in Canada that exist for pre-medical students trying to boost their GPA and get their pre-reqs?

I did consider going back to the U.S to complete a post-bacc, but tuition as an international student is extremely expensive.

Thanks so much, any information will be extremely helpful as I feel stuck.

tantacles

A master's degree will not improve your GPA at all; graduate level courses do not affect your AMCAS GPA.

You do not actually have to do a formal post-baccalaureate program to improve your GPA. Taking courses "a la carte" at a college or university near you (not a community college, ideally) is completely appropriate. The courses may help you with the MCAT, but you will likely still require a significant amount of study to do well.


No interview invites

I am a Asian women with 80 + lizzi M. I applied to mix of top 20 and mid tier schools . My secondaries were spread between July and late August. I have had 2 interviews ( 1 already rejected). I applied to 22 schools and nearing chirstmas have no II. I am yet to hear from 10 mostly selective schools. I am preparing myself for gap year. I have 100 + hours of volunteering in India, 2 plus years of research experience ( no publication) and 40 hours of shadowing. Any suggestions on what area I should focus on to strengthen my application next year?

tantacles

It is hard to say where you need to focus for a better application cycle without knowing your entire application. That being said, I would recommend taking 2 years; you have just interviewed for medical school and would likely benefit from a full year to improve your application before putting in a new application.

It sounds like you lack volunteering experience. I assume your 100+ hours was not over the course of a year but rather over the course of a mission trip. I would suggest you find a longitudinal experience to participate in for a few hours a week and do that over the course of the next year while continuing to improve your application with a job, either medically-related or not.


What is my best plan of action?

What SMP or Post bacc programs give you a degree that can actually help you get a job if med/D.O. school does not work out?

tantacles

Most SMP and post-bacc programs are not meant to help with finding a job and are strictly meant to help you get into medical schools. I would suggest doing a MPH or another master's degree if you are not planning to go to medical school.


Best option for non traditional student

I'm hoping to get some advice about next steps I should take in order to be admitted to a MD or DO program.

I am a 2016 UD Davis Graduate with degrees in Spanish and Biology. During my time there I had in my mind that I may want to pursue medicine, but didn't take it too seriously. -__- Regretting that now, of course. I did complete all required pre med courses with a 3.3 science avg.

      Q1. My counselor does not recommend a post bacc where I retake classes because I got all As and Bs. Essentially it would have been better if I did worse, apparently. Do you agree?
     Q2. I have 1 quarter (4 units or 2.67 semester units) of English because I was exempt from taking an upper division class. Can I take 1 semester at my local CC to fulfill requirements? Should I take two online quarter classes instead? 
     Q3.  Will a MSMHS post bacc(assuming I do exceptionally well) be helpful in applying?
     Q4. Taking into account my low GPA, is there a score I can shoot for on the MCAT that will be alluring enough to schools that they will give me a break? I feel confident I can reach into the 90+ percentile. 

Essentially I just need a lot of advice and I am having a difficult time figuring this all out on my own. Thanks in advance for the help!

tantacles

1. A post bacc is still viable to improve your GPA. If you are able to get all As in those courses, it will raise your GPA considerably. You don't have to do a formal post-bacc; you can just do an a la carte post bacc where you take the courses you want.

2. It depends on how the courses are recorded in AMCAS and how many credits the courses give you as well as how the individual schools you apply to look at courses.

3. I'm not sure what an MSMHS is, but I assume it is a master's degree. That being said, master's degrees do not increase your GPA, but it is possible that medical schools will look upon them favorably.

4. Above 510 would be a good start.


Is transferring twice going to hurt me badly?

Hello. I am a freshman at a liberal arts college. I wish to transfer from my current college into a cc, which has a guaranteed transfer to my in-state flagship, after the end of my first semester of financial and family reasons. Is this going to hurt me badly in medical school admission?

Thank you!

tantacles

The most important thing is that you receive an excellent GPA. If you are able to maintain an excellent GPA, you will be in good shape for medical school admission no matter which school you attend.


should I retake pre-PA prerequisites?

Im currently a Junior in college intending to apply for PA school this summer. I am trying to rush my pre-requisits because i need them done prior to application and I need to apply this summer to be accepted fall 2019. Unfortunately rushing isn't working out so well. Im looking at getting B's in almost all my prerequisites this semester. next semester might even be worse. im thinking I should retake my current classes next semester at the community college (its cheaper) because i know i can get an A and I still remember the material. my overall GPA now is a 3.75 at community college and 3.83 at university. I also will be applying with the minimum DPC hours, barely over 1000 by this summer so i will not be competitive there. I just dont want too many Bs on my prerequisite courses. i will list all my grades below.


Biology 1 and 2: 3.0 and 4.0 Chemistry 1 and 2: 4.0 and 3.5 physics 1 and 2: 3.0 and 4.0 nutrition: 3.7 anatomy: 4.0 psychology: 4.0 english 1 and 2: 3.0 and 4.0 (plan to retake eng 1) developmental psych: (current) looking at 4.0 Organic chemistry 1: (current) looking at 3.0 microbiology: (current) looking at 3.0 biochem: next semester physiology: next semester

im considering moving biochem and physiology to next year and retake orgo 1, microbiology, and english 1 next semester. this would be pushing back my application date by a year meaning i can not go until fall 2020. please give me advise on how to be the best applicant. would PA schools rather have somebody who has As by retaking a few or someone with Bs first attempt?

thanks!

tantacles

A GPA of 3.7 is not prohibitive. I would suggest you just continue to do well in the courses you are taking and accept the fact that your grades are not perfect. You will likely still be able to gain an acceptance to PA school with these current grades.


PA school HCE experience question

Okay, so here it is. I have close to 1600 hours working as a medical assistant in a pain management clinic. However, I wasn't a certified medical assistant. In Arkansas you can work without the certification. My first question is, do you think this will look less credible to a board if they see that I wasn't certified when working? Would you advise going back and getting certified as a phlebotomist, CNA, MA, etc and go through the whole process of getting hce experience again? Also, the clinic I worked at recently closed their doors. I most likely will not be able to get a recommendation from the clinic's staff or the doctor. How bad is this? Will I be able to list the healthcare experience without needing a recommendation from them? Thank you in advance for answering.

tantacles

I think it's most important to represent your health care experience accurately. If you worked without a certification and your employment was legal, your work was thus a valid experience.

As far as getting a recommendation, this will depend on the specific program. If no recommendation is available, then you will have to apply without one regardless.


entry requirements

Am a permanent reaident and I have a BSc degree in human biology but was obtained from africa. How are my chances of getting admission into a medical school. Am yet to take the mcat though

tantacles

As a permanent resident of the USA, your chances are entirely dependent on your MCAT score and GPA. Some medical schools also want applicants to have a degree from a US undergraduate. I would check on individual schools' websites to see what their requirements are.


Getting into DO school with ultrasound BA?

I am an echo tech and I have my BA in diagnostic ultrasound. I recently learned that this is not an academic degree (i.e. microbiology, biochem, etc) but rather a vocational degree (i.e. nursing), I am only able to go after DO rather than MD, which I more than happy with. Yet I still have a lot of pre requisites to take like the ochem/ gen chem series and others. I realized that the only way for me to take those at a university is if I enroll in another BA program and spend another 4 years getting a second degree. I was taking the pre reqs at a community college could save me some time, but it probably wouldn't better my chances of getting into med school. If anyone has suggestions on what the best route from here would be or if there are other options that I may not be aware of please let me know! Thanks :)

tantacles

If you have a bachelor's degree, you are eligible for medical school. Any major is all right. I would recommend that you take your pre-requisites at an undergraduate institution. Many places will allow you to take courses a la carte, and this is a completely appropriate way to save money so that you do not have to pay for a second degree.


Any dyslexic doctors out there?

Hello! I am a freshman pre-med student at Texas A&M university. I have struggled with severe dyslexia for quite some time, however, out of sheer determination and utilization of different tactics I have managed to compensate for the extra time that it takes me to study. I have been doing well in my classes so far and am very excited about pursuing a career in medicine. I was just wondering if there are any doctors out there that have dyslexia and have any advice on different approaches they took to getting in?

(Dyslexia does not inhibit intellectual abilities, it affects the language processing center meaning it takes me longer to read- that's it .)

Thanks and Gig'em!

tantacles

There are many physicians with dyslexia; it is a common disorder that does not inhibit one's ability to practice medicine. Many physicians with learning disabilities request extra time on standardized exams.

It might be worthwhile posting on the general pre-medical forums on SDN for more insight and thoughts. Our moderators on this site are very few and are more equipped for answering questions about medicine in general than providing advice based on our personal experiences as we represent a very small sliver of medicine.


How much time should i study for the mcat if i have only background in physics?

Hello ,

how much time do you think i need to invest in studying for the mcat to get a score above 498? I have background only in physics. so i need to study the rest from scratch..

which online course would you recommend and how would you recommend to study?

Thanks in advanced

tantacles

It is hard to say how much you should study for the MCAT; each person has a different threshhold for how much they will succeed. In addition, many sites offer courses for the MCAT, and which one is best will depend on your personal learning style. I would suggest you post in the MCAT section of the general SDN forums for more recommendations as the specifics of which course you should use are very personal.

Mr.Smile12

It doesn't really make sense if you haven't had any coursework in biology, chemistry, or social sciences to try to take a high-stakes exam like the MCAT where that content and its application/analysis are tested. You are underestimating the amount of time and understanding you need in order to be a higher-order thinker to do well. You can probably be okay with some of the online resources to get some basics, but true understanding of a topic doesn't come from just memorizing equations or organic chemistry reactions.


What are my chances of getting into medical school and what schools should I apply to?

Last year when I was about to finish my bachelors in psychology I decided I wanted to go for psychiatry instead so I finished my BA( 3.8 GPA) and I added a second bachelors in Biology just so that I could take the classes that I needed to fulfill prerequisites for med school. I am now taking the last class I need, and my GPA has gone down to 3.5. I did a year as a research assistant when I was doing my psych BA in a program for Child Anxiety and phobia at the university I attend, I did some volunteering at a hospital but it wasn't for too long as I have been working full time all along. I currently work at a mental health center as a psychosocial rehabilitation facilitator, I have daily interaction with mental health patients so I have some hands on experience. I plan to take the MCAT in May but I am worried about my GPA because I have read everywhere that a competent GPA is somewhere between 3.6 and up. What are my chances getting into medical school and which schools should I be targeting at the time of application? I live in FL and am a hispanic female(Not sure, but I've heard this has some influence too).

tantacles

Your GPA is still acceptable for medical school. I would suggest that after you take your MCAT, you follow these steps:

1. Purchase the MSAR, located here: https://students-residents.aamc.org/applying-medical-school/applying-medical-school-process/deciding-where-apply/medical-school-admission-requirements/

2. Make a list of potential schools you will apply to based on your GPA and MCAT combination.

3. Post your list along with your stats and extracurriculars in the "what are my chances" forum on SDN.

It is impossible to advise you right now based on just a GPA.


What should I do next?

I have recently graduated over the summer with a 3.0 cGPA. I have not taken the MCAT nor the pre reqs required to apply to Med school. My hope is to get all this completed now and finally get the acceptance into a program. After searching and browsing these forums for awhile, I have learned some useful information, but nothing that really feels relevant to my situation especially since many of the threads are a bit outdated. I want to get into a post bacc program with conditional acceptance to Med school, somewhere in the a state such as PA, DE, MD. I currently live in Washington state and really need to be closer to my family again. At this point, I kind of feel like I don't know what I should be looking for. I've seen suggestions for pursuing a post bacc program, a DIY post bacc, or an SMP in order to get the desired acceptance. I really just need a clear picture of where I should focus my energy to get the results that I need. Thx for taking the time to help.

Mr.Smile12

I would look for a postbac program that can provide you access to your prerequisite coursework and some upper-level biomedical courses. Some postbac programs may also include MCAT preparation so make sure you look into that.


Do I have a chance at Med School?

I have a low gpa (3.2) but excellent leadership and clinical experience and a goof mcat. will this gpa bring me down? I want to go to med school on the west coast if possible

tantacles

With your GPA and a "goof" MCAT, you have very little chance of medical school. I would suggest a post-baccalaureate program to raise your GPA.


Chemical engineering to pre-med

I am a sophomore with a chemical engineering undergrad major with an emphasis in biochemistry. I have decided that I want to go to medical school after my undergraduate, however I have a lot of questions, and need advice from people who have experience in this!

1. My GPA is 3.49/4, which isn’t horrible for chemical engineering. However I have heard that GPA is very important on the applications for medical school. How will this be taken into account? Are my chances lowered with this GPA?

2. Chemical engineering is considered one of the hardest majors at my university, and it definitely isn’t a rumor. It’s a very difficult major and it takes up a lot of my time. I am stressed a lot about my classes often, which isn’t anything unusual for me, but I know that I need to be getting in a lot of volunteering and shadowing for medical school applications. Would it benefit me to change my major to biochemistry?? (which I assume would increase my GPA and give me more time to devote to extracurriculars for applications) or is it better to have a lower (but not low) GPA in ChemE?

3. I work as a pharmaceutical technician, and just out of curiosity I was wondering how this would be viewed on a med school application?

4. What are your general “most important things to know” for pre-med students and for the med school application?

Thank you for any help! I truly appreciate your time!

tantacles

1. and 2. A 3.49 is not a great GPA for medical school. Keep in mind that your major does not matter to medical schools as long as you do the pre-requisite courses. If you will do better in a different major, it would be beneficial for medical school applications to change your major, whether you change to biochemistry, English, music, or French.

3. It will be viewed as work experience and will help your application.

4. Use SDN's pre-medical section judiciously; it can help you figure out your extracurricular activities, which are quite important.


Will I be setting myself up for bad grades with taking Anatomy, Pre Calculus, Chemistry 2, and Genetics in one semester?

Hello, I am a Sophomore/Junior in Undergrad, so far I've taken Biology, Chemistry 1, College Algebra, Nutrition and all other basic courses. I am a Pre Optometry Student and I work a Casual Part time job. I am aiming to possibly take Anatomy and lab, Pre Calculus, Chemistry 2 and lab, with Genetic and lab. I am the most worried about taking Genetics with only have taken Biology for majors. Please give me advise on the pros and cons of this schedule.

Thank you

tantacles

Every person has a different threshold for what they can handle. A normal courseload is absolutely acceptable for medical school, so if this represents more than you can handle, I would drop 1-2 courses and see if it's manageable.


Pre-Med? Nursing? Please Help!

Hi, I am currently a freshman in college. I am an Art History major on the pre med track. I have always wanted to become a surgeon. It is my dream job. But recently, I am heavily struggling with Chem 1. I have a D in the class and I am thinking about dropping the class (I will). If I can't even get through Chem 1, how am I going to get through Chem 2, Orgo, Biochem etc?... I've never been good at science and my family thinks that I shouldn't be a STEM major. But I just really want to be a doctor.

    So recently, I got interested with the nursing program at my school. I really want to work in the medical field. I talked to the advisor and all but I am not sure if I should do pre-med, nursing or just Art History. I love Art History and it's so fun but I know I'll regret it if I don't even attempt for a career within the medical field. I'm thinking maybe I should not be a STEM major. Some people told me that sometimes you can't do what you really want to do because you are not gifted in that particular area. 
    Honestly, I want to be a doctor but I know I'm not smart enough and that I'll doubt myself every moment as I start taking harder science classes. I like nursing too but I know nursing isn't easy and I may not get accepted to the nursing school at my university.  If I prepare to become a nurse, I'd like to graduate with my BSN, experience being a nurse and go to graduate school to become a PA. If I want to become a surgical nurse, what do I have to study and do?
    This is so hard because if I decide to do nursing, I'll have to give up Art History and Pre-med and totally shift gears. I really don't know who to ask because no one around me is in the medical field and my advisor only tells me what classes I need to take and all.. Please please help!
   Please excuse me if I said any wrong information or if I post this in the wrong place.

tantacles

If you feel that you are not able to do science well, you do not have to be a STEM major. I always suggest that people do what they're good at. Sometimes, we love things that we are not good at, and it creates conflict in our lives. If you want to be a nurse, all you need to do is finish a nursing degree. If you want to be a PA, you will have to finish the same pre-requisites as a pre-med student, which may get in your way. That being said, it may be that if you get the proper help you will be able to succeed. This decision is exceedingly personal, which makes it hard to give real advice. You have to figure out what you want to do and what you are able to achieve and look closely at where those goals match and where they differ. Therein is one of the hardest parts of life, but it will make your decision for you once you have the right information.


Does dropping organic chemistry alter chances of admission?

Alright, right now i am in my sophomore year 1st semester. I have organic chemistry 1, intro to psyc, genetics, med terms B and precalc. I am thinking I will have to drop organic chemistry, as I am currently at a C and seem to be trending downwards. I don't have enough time to donate to organic chemistry, as I just got a job as a laboratory tech that requires 20 hours a week without weekends. I am struggling to keep up in the class because of my job ad the amount of credits I have, so I am thinking about taking it over the summer. The only thing I am worried about is that this would be my second class that I drop, and I am worried that my application to medical school will be affected by the drop. Otherwise, my GPA is 3.58 and I have had my CNA since highschool with 1 years worth of hours, I worked with children in an afterschool program for a year, and I currently work as a lab tech. I also have a summer temp job at 3M and am in biology club. How does my application currently look, and will dropping organic chemistry affect me in the long run in regards to acceptance to medical school?

tantacles

Dropping organic chemistry, provided you take it again later, should not negatively impact your chances at medical school. You will have to do well in this course later, though, just as you will have to do well in your other courses, to get into medical school.


I have been dismissed from a previous university. I have taken a year off to reassess and better myself. I want to apply to Case Western... what are my chances?

I went to Miami University. I first had an academic dismissal for one semester. I tried my best to appeal it to the court, however, it did not work out. I came back to school and was again filled for an academic dishonesty (all because I did not site a rough draft paper). Again, I tried to appeal it, but failed. So, I took the year off. I began to work as a scribe and bettered myself. I want to finish my medical studies. Note, the dishonesties I had gotten were in Humanity/Arts classes, nothing relating to any of the science courses I took. Anyway, I am from Cleveland. I would like to return to school again as a transfer student and finish my degree. What are the chances I can get into Case or like Ohio State?

tantacles

Academic dishonesty is often a career ender for medicine. At the very least, I would recommend taking several years off to improve your application before applying to medical school. When you apply, you must report these charges of dishonesty, and I would recommend that if you apply you talk about what you learned from this experience and express your regret at having cheated.


Classes after undergrad degree

Can I take classes after I graduate with my bachelor's degree? What will happen to my undergraduate gpa? I want to go on to med school but I have to get ny gpa up and work on research. If i take classes at my university after recieving my degree will those classes be added to my undergrad classes and help raise that gpa?

tantacles

If you take undergraduate courses, those courses will be added to your GPA and help your med school application if they raise it. Graduate level courses will not count. Talk to the registrar to find out how those courses are classified.


What are my chances getting accepted to Ku med 2018?

To keep the information short and to the point. I have a 3.8 GPA, Exercise Science Bachelors with a minor in Chemistry. The rest of my application is well such as work actives, letters, etc. The only thing that was bad was my MCAT, I got a 487, but scores 125 Chem/Phys, 124 Biochem sec, 119 CARS and 121 on Psychology. I know I should have prepared more and im a Kansas resident. I am planning on becoming a Psychiatrist in Kansas, so I can help give back to my community. I would love any helpful feedback.

tantacles

It is hard to say what your chances are as our advisers do not work at KU. Your GPA is excellent, but I would recommend repeating your MCAT. This will give you the best chances of gaining acceptance.


What are my chances of receiving admission to DO (or even MD) schools here in the States?

- Will be 29 years old during expected Fall admittance 2019. I am a Black - African/American applicant. Current Peace Corps Volunteer (Community Health Extension - Paraguay) Close of service date May 2018 - Fluent in Spanish and French

Highlights - 3.43 non-science GPA / 3.14 science GPA - 3.25 overall - Most recent MCAT 506 (4th attempt - previous attempts 19 24 23 - all older than 3 years) - 150 volunteer hours in Hospital Setting (ER / hospice) - 1 year patient advocate - paid position (Hospital Setting) - Child Family Health International volunteer experience in Ecuador and Non-Profit Children 2 Children donation of clothes - 6 weeks - 250hrs + shadowing multiple MD/DO physicians - Connecticut Resident

Mr.Smile12

I would think that your service time in the Peace Corps on top of your accumulated experience shadowing or working in a health care setting is going to help you. I would hope that in-state/in-region schools would at least take a look. The only concern I may have is whether you have had any recent coursework in the sciences that you have taken; if so, how well you did in those classes could help your application.


What are my chances of getting into medical school in Canada? Ireland?

I am currently in a pre-med undergraduate degree in my 3rd year. I want to apply to medical school in the near future. Over the course of my undergrad, my marks have not been as high as I'd hoped due to some mental health issues that I am on my way to overcoming. I'm currently sitting at a 3.2 GPA. My current year is going ALOT better and I'm hoping it'll go up.

I have tried to keep my extra circulars active and I'm thinking of doing my MCAT in the summer this year. I am willing to do a few extra years after undergrad. Mainly, I'm feeling a bit discouraged with my marks and even my extra circulars seem to need some work. Is it too late in the process for med school in Canada to be an option?

tantacles

With a GPA of 3.2, it will be very difficult to get into medical school in Canada. With a great MCAT score, it is possible, but improbable. You would need to improve your GPA significantly, and many Canadian medical schools prefer Canadians. You have not stated your nationality, so it is hard for me to advise you.

I am not familiar with Ireland's requirements for medical school.


What are my chances of getting into medical school?

I wanted to know what are my chances of getting into top medical schools like John Hopkins, Indiana University Medical School and Eastern Virginia Medical School or any other schools in the US? I am a current pre- med student. I just started my third year of undergrad. I am a global health with concentration in clinical science major. My overall gpa (from 1st and 2nd year) is 3.77. But my science gpa is 3.61. This will hopefully increase to 3.8 (overall) by the end of this semester. I work at a nursing home and volunteer at a local hospital. My ethnicity is black. I have not taken my MCAT yet . It would be great if you guys can advise me/ tell me the minimum MCAT score that I would need to be competitive.

tantacles

Your chances will depend on how well you do on your MCAT. With a high score, you have a chance of getting into top schools, and with a good score, you have a good chance of getting into the other schools you have listed. I would suggest taking the MCAT and exploring your options at that point.


Post bac help

I am reaching out in regards to receiving some advice about post-bac admission. I am currently a senior who will graduate with a degree in Neuroscience in spring with a current GPA is 3.1 hoping to graduate with overall undergraduate GPA will be a 3.20. (I would have to receive all A's three quarters to boost my GPA to a 3.30, although I am trying to do that I am also realistic). I have finished Biochemistry (B+) and I am retaking a general chemistry class that I did poorly in freshmen year (C- grade). Throughout senior year I am taking upper level biology classes to prove I can handle rigorous coursework within my major, (Cellular Neurobiology, etc.) My whole entire goal for my senior year is to improve my GPA. I believe a post bac program would be a wonderful opportunity to strengthen my skills, continue with volunteer work, and improve my science GPA, which is extremely low (2.7). My largest mistake was overloading my class load (physics, orgo, cell bio, etc all in the same quarter) and receiving C's. I am a really hard worker and I try to do everything right (go see my professors every week, attend TA office hours, read the textbook, spend hours studying, never miss a day of class, etc). I plan on taking the MCAT when I feel I am ready to take it (most likely March) and applying to post-bac programs fairly early. I have around 600 hours of volunteer experience in the PACU as well as the Hospice unit of a hospital. I have authorship on a paper and involved in many academia clubs. However, I have put all of those on the back burner to really focus on strengthening my GPA. I have met with my academic advisor multiple times on my future because I feel uncertain if I will gain entrance into a post-bac programs. Furthermore, I am reaching out to see if there is any advice you can give me in regards to strengthening my overall application to receive admission into post-bac programs, I am an Illinois resident who is really interested in the Loyola MAMs program, Midwestern University masters program, Rosalind Franklin, Rush Biotech, and SLU Anatomy. I feel really lost on where to go from having such a bad performance in undergrad. I am more interested in DO rather than MD just for reference. I have thought about other career options such as quitting my dream of being a DO or MD doctor all together and perhaps become a podiatrist, pharmacist or Nurse Practitioner. Thank you kindly for taking the time to read this message.

tantacles

Improving your GPA is very important. It is hard to say, without the MCAT, what your chances will be, but with a GPA of 3.1 and a low science GPA, your chances would be poor for medical school without a stellar MCAT performance. It is worthwhile to reevaluate your goals now, as getting into medical school may not be feasible.


Low GPA

I studied abroad for a year and failed both semesters. I had a 3.18 before I left for study abroad. 4 C's in upper level science courses were dragging me down. Now I'm at a 2.57 because of the study abroad grades.

I returned to my University and retook the classes. We have a academic forgiveness policy. I got all As and Bs in the classes once I retook them but my school said the academic policy does not replace study abroad grades so they are still being calculated into my GPA.

What should I do? This is my senior year. I will be finished in December with all required courses besides Spanish 202 so I'm going to take it in the spring along with 3/4 of the classes I got a C in.

However I still don't think my gpa will be above a 2 something before graduation. && it's too low for post bacc..... is there anything else I can do to raise it or better my chances for getting into a post bacc program. I already retook the classes I failed abroad and I'm retaking the classes I have Cs in which will help some if of course I do better which I plan to, but I still don't think it will help much. Any advice????

Mr.Smile12

Did you ever get connected with your prehealth advising office or your career services advisors? It seems that you have arrived too late to seek their help, so it may take you a while to get to a medical school if that is your desired goal. You haven't even mentioned trying to take the MCAT, and I think you are wise to determine that your overall academic record needs to show more preparation for the real challenges of a medical education. You should seek the advice of your career services office and your health professions advisors to get you to determine a realistic goal, and after a few years, you may want to return to preparing for a health professional career after you have gained better insight on your academic preparation for those specific careers.

Now there are plenty of health care careers that you can also consider, and that's why seeking the insight of those advisors can help you construct a plan for what your immediate next goals should be.


Dyslexia on Adversity Essay?

I was wondering if I should mention dyslexia in the adversity/challenge essays or if it would be looked down upon. I have maintained a GPA above 3.5 in college so shouldn't it be seen as having overcome it or have figured out how to deal with it and persevere?

I'm sorry if this question was already asked. When SDN showed me some related questions that were already asked, they were general "adversity" for the headline and when I clicked on 'view more' to see what they were specifically asking, I got a 'Page Not Found'

Mr.Smile12

It's too hard to predict how any one school would view your dyslexia as an adversity or challenge, but I would think most would acknowledge any learning disability to be. That said, only admissions officers could give you specific advice how their school would view that disclosure.


Pass Failing a non science Course

I know this may seem like a very odd question...I go to a private libral arts school which requires you to take a semester of fine arts. To be completely honest im not the artistic type and im worried that this course may put a dent on my steady GPA since its a 3 credit class. My school offers a Pass/Fail option for once class every year so that it wont affect my GPA in the long run, and I recieve the credit it just shows up as a P on my transcripts. However ive been told different things about med school admissions with the Pass/Fail option. Keep in mind ive never used one of these before, any thoughts?

Mr.Smile12

Why are you interested in taking a course pass/fail? I don't think it would count towards your GPA calculation, but you shoulfd check with admissions staff.


Lab course work as lab experience?

Hi! I'm currently applying for research assistant, lab tech, and lab assistant jobs at the hospital I'm currently volunteering at. I don't really have any formal experience working under a professor or PI. So I was wondering if the labs I took throughout my undergrad coursework would count as laboratory experience? In one class we were allowed to take the techniques we learned the first half of the semester and create and analyze our own research project. So maybe that counts but regular lab course work does not? Any clarification would help and be much appreciated! I know some hospitals/labs will consider different things to count as experience as others.

tantacles

Your coursework does not count as work experience, but it does count as laboratory experience as you have done lab work.


Should I still pursue a medical school admission?

I was dismissed from my university a year ago for poor grades. My GPA was 0.96 I retook my general Chem at a local Community college along with Physics and general ed. courses My advisor is telling me that I've ruined my chances for a US medical school; even with a post bacc or a Master's, my undergrad will be a red flag. Is she right? Can I get an upward trend going from this point on to my upper div courses and hopefully redeem myself? If I can get an upward trend going, can I land a medical school admission without a SMP or post-bacc?

tantacles

It will be very difficult to get into medical school after being dismissed from college. I would suggest reevaluating your career choices as it's important to be realistic. You also have to pass undergraduate to get into US medical school.

It is possible that with a stellar record in the future, you could gain admission, but right now there is a black mark on your record that must be remedied, and this will take time.


Will a B or C in Calc 2 kill my chances for medschool?

I did a lot of dual credit and ap classes in high school so when I got to college I got thrown into upper-level classes I wasn't completely ready for. I am retaking Principals of Chem despite my AP credit because I was hoping an A would look good on my transcript. Yet my main issue is I was told I wasn't able to retake Calc 1 for this benefit. I went into college with 67 hours and at the time I thought I was helping myself by taking these classes early but being a freshman/junior at the same time has been a strain on my adjustment period. So I am struggling pretty harshly in Calc 2 and I got a C on my first exam. I am obviously going to strive for better but I honestly don't know if I can achieve higher than a B at this point. I am taking Stats next semester and I am way more confident about being able to get an A there. I'm just wondering how it looks coming from being very successful in high school to having trouble in my first semester and how to maybe use this experience to my advantage and/or recover from this grade. Thank you!

tantacles

A B or C in calculus will not kill your chances for medical school as long as you do well in your other courses.


pre-med course work from accelerated school?

I have a BS in accounting, but am now considering medical school as a viable opportunity. I am a 23yo, single mother with a disabled child and just do not have it in me to go a traditional brick mortar. Besides most of the other implications, I am wondering if anybody has any advice on nontraditional/online/accelerated schools that I could attain my premed sciences from?

Any comments or information would greatly be appreciated!

tantacles

If you wish to pursue medical school, you need not go back and do a full degree, but rather you can take your courses a la carte. However, the majority of your courses should come from a 4-year college as the rigor from those courses is higher than those from online courses. That being said, if you are able to excel and get A's in all of your courses, it is possible that medical schools will overlook the fact that your school is an online one, provided this is not denoted in the name of the school.


How good are my chances of getting into a PA program?

Hello! I am planning on applying to PA programs next summer 2018. My current overall GPA is a 3.7, but my science GPA is only a 3.4. The thing that worries me is that I'm having so much trouble in my chemistry class and I'm afraid that my science GPA will drop. I did the math and if I make a B or C in this class, my new science GPA will be at a 3.3 or 3.2. I'm wondering if it is still possible to get into a PA program with my GPA being that low? I plan on shadowing a PA this semester and I worked as an optometry technician for almost 2 years, equaling about 1500 hours of direct patient contact. It's not the traditional patient contact (CNA. EMT, MA, etc.), but I've emailed some schools and they accept that form of HCE. Does anyone know if my application seems weak at this point? What else can I do to improve my chances? Any input is greatly appreciated!

tantacles

It is hard to say what your chances are. Each program is different, and your entire application will likely be looked at. I imagine you will still have a reasonable chance of admission as long as you have a strong application otherwise.


Will student background screening/check show expunged misdemeanor?

A couple years ago I was charged with a misdemeanor. I have turned my life around since then and haven't gotten in anymore legal trouble. But I was wondering if this would show up on my student background screening when applying for medical schools if I got the misdemeanor expunged? And if so what are the chances of still getting accepted into medical school? The misdemeanor was falsification.

tantacles

It is unclear as to whether this will show up on your record. Regardless, read the AMCAS rules carefully. You may be required to disclose this misdemeanor even if it it not mentioned based on AMCAS' rules. Whether it will impact your chances of admission to medical school depend on the misdemeanor itself and how you speak about it on your application.


Please Help! Lost my pre-med advisor and have no guidance

So, this past June my Pre-Med advisor started teaching and left his position. They have yet to replace him, and as a result me and many other pre-med students are rather lost. Start of fall semester I was instructed to make an appointment with him so he could basically get me into volunteer organizations and under a physician to shadow. With him being gone, I ended up screwed over. I was told to hold off on volunteering as I have been taking 5-6 courses with labs during spring and fall semesters and 4 courses during the summer. I have been desperately trying and contacting multiple hospitals, filling out volunteer applications, going in, in attempt to talk to someone, trying to get into research opportunities, etc. Either no one responds or I get an automated response email wise. I am in my last year of pre-med. I take my MCAT May of 2018. Because of the course load I don't have very much volunteer work. I have no idea how to get under a physician for shadowing. Everyone keeps saying that, hey you have two bachelor degrees, a high gpa, and will get a good MCAT score so that's enough, but I know admissions don't necessarily care about that as most students have that. So what do I do??? Will I even get into med school at this rate? As doing the bulk of volunteer work in your last year doesn't exactly look good... Any advice is greatly appreciated. Thank you.

tantacles

I would suggest that you work on finding some shadowing or volunteering experience before you apply to medical school. This may mean delaying your application, but you should wait until your application is ready before you apply. If you are getting an automated response, keep asking around as to who the right person to contact is; there is likely someone who will respond to you, but you may need to wait as many hospitals have waiting lists for volunteering.


What are my chances?

I am in my Junior year of my Undergraduate Career at the University of Iowa. I have a University of Iowa cGPA of 3.31 and an overall GPA including some college credits from high school of 3.55. I have 4 years experience as an EMT and 1-year experience as a Paramedic. I am involved on campus as being the President of a student organization, I will have 2-3 years of research under my belt, and I also volunteer regularly. I have not taken my MCAT yet, but I will take it this May. I am wondering what my chances of getting accepted into an MD program are and I would also like to narrow my search for Medical schools. I am starting to worry about my GPA, and just would like some general advice. I have done lots for my resume over the years, but my GPA has dropped and I am concerned. Any help is appreciated!!

tantacles

It is absolutely impossible to stratify your chances without knowing your MCAT score. Your GPA itself will not stop you from getting into medical school, but you need to do well on the MCAT.


State school to ivy and now unhappy, should I go back?

So I've looked at a lot of posts and my situation seems a little unique and I'd appreciate any feedback that can be offered.

I decided to go to the University of Vermont when I graduated high school but I also had a guaranteed transfer offer to go to Cornell University my sophomore year and beyond. I had a great year at UVM, made amazing friends, was on the perfect dance team, great neuroscience club, thinking about doing the school EMT service, joined a sorority etc. But by the end of the year I decided to go to Cornell because I figured I would also have a great time there AND get an Ivy league degree, hopefully giving me an edge when it comes to applying for med school. Now it's been a month or two and I really don't like it at Cornell. I can't seem to make a good group of friends, everything is so competitive I can't get into anything to make me stand out, and my old sorority and dancing is completely different here. I'm really struggling to enjoy anything except my classes and am sorta home sick for my old school.

So all in all, I was much happier at UVM than I am currently at Cornell and I'm wondering if going back to Vermont for the next two years will lower my chances of getting into medical school or should i stick it out and potentially be unhappy for three years at Cornell but have an ivy degree?

Mr.Smile12

I don't know what advice you had when you first thought about transferring away from your state school. I'm sure the guarantee to transfer was very enticing, but I would have cautioned anyone who is thinking about transferring to understand whether there is enough support at the new university as it might be a different culture altogether. What was your goal when it came to transferring in the first place? If it was to be in a more challenging but rewarding situation , then understand that this is all to help you develop focus for your ultimate professional pursuit, to be surrounded by equally motivated students, and to get more support from people who want to go to medicine as you... then it was probably a rational decision.

Unfortunately just having a degree from an Ivy doesn't make you that much more desirable in the admissions process, and I hope many people were also telling you this as well. I do think that in your new environment, you need to challenge yourself to seek the resources, communities, and activities that can really help you through these difficulties. It is a myth that medical schools really want practically perfect people (though it always seems like those are the ones that are your ideal premeds). And there is something to be said about making adjustments because you'll make them again in medical school.

You do have freedom to choose, so I hope you can decide what location best puts you in a better situation academically and emotionally.


Please help! Currently freaking out about my grades!!

I am currently a first semester sophomore and I am very worried with how my grades are. My first semester of my freshman year was pretty good (all A or A-), but my grades dropped a bit second semester (a couple B) because of a couple stupid classes. My GPA as of now is around 3.65, mainly because of those Bs from last semester. Now, I am in organic chemistry and I am finding it very difficult. I am getting help, but my grades are not reflecting that and I am beginning to worry that I won't be able to make an A in this class. I know that organic chem is supposed to be hard and whatnot, but it is honestly making me hate being premed, even though I definitely want to become a doctor and practice medicine one day. Can someone please advise on what I can do? I am so worried and stressed about this right now and I don't know what to do.

tantacles

A 3.65 GPA is respectable, and with a good MCAT score, you will still be able to get into medical school provided all of the other parts of your application are solid. It is ok not to get an A in every course. I would suggest you do your best going forward, but cut yourself a little bit of slack; not everything in your application has to be perfect.


Do I have a chance?

I am a non-traditional student that attended university 10 years ago. In the 1 year i spent there I acted like an idiot and had a 0.8 GPA. I spent 6 years in the military after that and then went back to a different university. I now have a 3.96 GPA after 3 years. I received a 513 on the MCAT with decent shadowing experience. Will my 1 year of awful GPA ruin my chances?

Pathdocmd

Absolutely not. There are a lot of applicants. medical students, and physicians that had a rough start in college then grew up and did very well. I wouldn't worry.


Do I have to do extra curricular activities my freshman year of college?

I am majoring in Biomedical Engineering as my pre-med major and I was wondering if I have to join EC's my freshman year to get accepted into a top medical school like Harvard? I am adjusting to college right now and I do not think I can handle a club right now.

tantacles

It is ok not to do extracurricular activities in your first year of college if you do not have time. I would suggest that you expand your view of medical schools; every US MD school will train you to be a physician. Do your best in college and do well on the MCAT and you will be in a good place to get into a medical school, though it is impossible to predict where you will stand in terms of specific schools at this point.


Does it matter what kind of a Biochemistry course counts as a pre-requisite for medical school? Trying to decide between two biochem courses that are offered at my university (Biochemical Basis of Health & Disease vs Introduction to Molecular Biology)

There are some differences between the two courses, one is more biology-based biochem course versus the other, which is more biochemical concepts focused on physio, health, disease.

Mr.Smile12

I would want to know if you have a health professions advisor who had "official" word from the biochemistry course instructor or department hosting the class if either one would be rigorous for preparing for medical school. Usually the prehealth advising office collects the information on "acceptable" prerequisite classes for prospective applicants.


Medical School

Hai, I`m currently doing my first year of undergraduate degree. I`m aiming to become a psychiatrist, but not clear what class should i take to enter medical school. I`m doing a AA that has only chemistry, biology and psychology ( also abnormal psychology), just one math( statistics ) but no physics. Is that recommended that i only do few of these? Or should i change my courses to AS that has physics and lots of maths in it?

tantacles

If you wish to get into medical school, you will need a full year (two semesters) of inorganic chemistry, chemistry, physics, and biology at the bare minimum, but every school has its own requirements beyond those courses. I would look at the specific schools to figure out the requirements. Your major or concentration of study is not important as long as you do well and fulfill the pre-requisites for medical school, so study whatever interests you.


Do medical schools care where you get your undergraduate degree from?

I am currently at North Dakota State University, but I am considering switching to a smaller college, Bemidji State University. Should I stay at my current, larger, more well-known school or switch to a smaller school? I am a biochemistry major and both schools have this major, as well as a pre-medicine program.

tantacles

Some medical schools do care about where you did your undergraduate degree. Keep in mind, though, that the most important thing is to do well in all of your courses, fulfilling the pre-requisites for medical school. Your major itself is unimportant, and you can major in anything you wish as long as you do well in it, whether it be math, english, the sciences, the social sciences, the arts, or something else completely different.


Is calculus required for pre med / accelerated or combined bs/md or bs/do programs

I am a junior in highschool and was wondering if I have to take calculus as a pre med. i am applying to multiple bs/md and bs/do 8,7, and 6 year programs and am debating to take pre calculus this year or ap statistics. Thanks again for your help.

tantacles

Many programs require calculus, and many don't. I would look at the websites for individual programs to determine whether calculus is necessary. Some medical schools require calculus and some do not. My personal advice is that pre-calculus is a useful course, and if you think you may take calculus in college, you should study it.


Institutional Action AMCAS Application

Hi, I am a pre-med applicant with a conduct violation and want to know if I'm totally screwed. It was a really difficult time from me and I have gone through extensive therapy and learned from my mistakes.

I was involved in a Title IX investigation where someone I had a casual relationship with falsely accused me of awful things because his girlfriend found out and forced him to tell the university. I had a 2 year casual relationship with this person, but it got worse when I got pregnant and had to have an abortion, which really messed with me emotionally. I had two interactions with his girlfriend (once was before she became his girlfriend, when we found about each other and I apologized to her over FB because she thought they were exclusive and she told me to never talk to her again and blocked me). I kept hooking up with him because I was distraught about the abortion, he was the only one who knew, and hooking up with him was the only way I could see him. Fast forward 6 months, he raped me (I was drunk in my dorm room and said no) and I was so disoriented after the fact that I contacted her and told her again by text (yes i realize how stupid this was but i was just raped, wasn't thinking straight, and was intoxicated). He got angry I told his girlfriend and we cut off contact.

The guy and his girlfriend then proceeded to accuse me of sexual assault, stalking, and harassment. I was about to file a report myself about the rape but he and his girlfriend basically beat me to it with their lies. The guy lied to protect his relationship with his girlfriend and said we had only hooked up once but I was able to prove that he was lying through his teeth with text messages of him asking to hook up multiple times, etc. I was able to disprove stalking because, point blank, I was really afraid of seeing them on campus (conjured up a lot of emotions about the abortion) and absolutely did not stalk him or her. I was able to disprove sexual assault because, point blank, I never engaged in that. But, because I texted her after she told me on Facebook 6 months prior not to contact her, I was found guilty of harassment (not sexual harassment).

Bottom line, I was removed from housing because of alcohol (underage, and I was a resident assistant at the time), and found guilty of harassing the girlfriend (texted her once after she told me not to). I was not able to prove my rape. Sanctions: No contact letters and mandatory counseling to help me get over my grief over the abortion and my rape.

Am I screwed for med school? How deeply do they look into it if I check the box that says I have a conduct violation? I am an otherwise strong applicant. I know it reflects poorly, but I have learned a lot from that experience including to cut off unhealthy relationships and to look for help in the right places (counseling)

tantacles

This will absolutely be school dependent. My suggestion is that on your application you don't tell the full story. Simply admit to the parts that you were found guilty of and state what you have learned from the experience and how you will change your conduct in the future. Talking about finding counseling as a coping strategy is a great thing to mention as medical schools only want to accept people who have healthy ways of coping with their issues.


Would I significantly benefit from a postbac/SMP?

I'll be a senior and was wondering if, given my GPA and GPA history, it would be advisable to enroll in a postbacc/SMP program. Currently, my overall GPA is 3.48 and is projected to finish at a 3.55-3.60 assuming things go as planned. My MCAT score is also a 521 (taken earlier this summer).

My GPA history, separated by year and fall/spring semesters, is as follows:

Fr: 3.83 / 3.83 So: 3.17 / 2.81 Jr: 2.89 / 3.88 Sr: Projected around 3.8/3.8

After the fall of my junior year, I decided to see a psychiatrist and was diagnosed with persistent depressive disorder, and was prescribed medication for it, to which I attribute the jump in grades. Would med school adcoms be willing to give less weight to the 3 semesters in which I've had an extremely low GPA, given the explanation for it? Or should I enroll in a postbac program to raise it and improve my chances?

tantacles

You might benefit from a post-bacc. but given your stellar MCAT score when combined with your good GPA and known recovery from mental illness, I don't think you would need one to be accepted.


Secondary application essay asking about shadowing experience, can my answer be about scribing?

I have little volunteer shadowing experience (under 10 hours) but I have over 200 scribing hours. Can I answer an essay prompt about shadowing by describing my scribe experience?

tantacles

Your scribing experience is a bit different from shadowing as your job was not just to observe while you scribed. I would suggest you ask the physician you are working with if you can spend an extra day shadowing as that will count.


Chances of getting into med school after transferring

Hello! I am currently an undergraduate majoring in microbiology. My current GPA at my current school is a 3.3 but I expect it to go up to at least a 3.6 by the time I graduate in a year and a half. However, this school is my third, as I have transferred twice before. I transferred from the first school because I had trouble adjusting and left w 2.0 GPA. I have since retaken all the classes I did bad in and have gotten mostly A's and some B's in them. I left the second school with a GPA of 3.5. I am now at my third school. After my first year of college, I tutored chemistry for one semester at my second school. And currently I am working as a research assistant with plans to volunteer at a hospital pretty soon. My grade trend is upward. However, If I take all my classes from all three schools into account and calculate the cumulative GPA, I would be at about 3.1-3.2 by the time I graduate. With all this, and assuming I get a good MCAT Score, what would be my realistic chances of getting into medical school?

tantacles

Transferring to another school is all right in terms of admission. The bigger issue is your GPA. The following table illustrates percentage of accepted applicants by GPA and MCAT:

https://www.aamc.org/data/facts/applicantmatriculant/157998/factstablea24.html

Improving your GPA is important unless you get an absolutely stellar MCAT score as your GPA from all three schools will be counted by AMCAS.


3.2 Science, 3.1 cum chances of getting into med school

I'm worried about what my chances are of getting into med school with a 3.2 science and 3.1 cumulative GPA, assuming I get a high MCAT score. I had a tough beginning to college but have been recovering since. I am also a volunteer research assistant at my school.

Pathdocmd

You will have to improve your GPA regardless of what your MCAT turns out to be. With that GPA your chances of an allopathic school are diminished. However, if it was just a bad Freshman year then a dramatic increase, that is different. If the only medical/ volunteer experience is being a research assistant, then your chances are slim to none.


What are things you did to get into medical school besides academics?

I am a second year premedical student and I want to know what are things you guys have done to get into medical school. Any shadowing, outside clinic volunteering, etc.?

tantacles

Just about any extracurricular activity under the sun will increase your chances of getting into medical school. Of particular importance are showing your commitment to service by performing volunteering, performing research, and getting clinical experience in a hospital or clinic setting. Personally, I worked as a volunteer in an emergency department, recorded books on tape for the blind, and performed cardiology research on mice.


Should I become a 5th year senior to officially declare a pre-med concentration?

I’m a senior psychology major who will graduate this fall. I have not declared a pre-med concentration, but I am taking/have taken pre-med classes--i.e., general chemistry, organic chemistry, general biology, general physics, and biochemistry. I would need to take molecular biology, calculus, and either genetics or microbiology to meet the requirements for the pre-med concentration at my university. I did not want to delay my graduation any further, so that's why I have not taken those classes. My current dilemma is whether I should become a 5th year senior and declare the pre-med concentration or just graduate this semester. Additionally, I am unsure whether I should take an upper-level biology course this semester, or keep my course load light to preserve my GPA. I'm currently enrolled in biochemistry, physics 2, a psychology course, and applied medical physiology with a lab (which is a seriously intense course).

My current GPA is a 4.05--my university uses the plus/minus system. I've only earned a B+ in an art history class taken my freshman year. I have research experience, and I volunteer regularly. I work part-time as well. My plan was to study for the MCAT and shadow after my graduation in December until I take the MCAT in the summer of 2018.

Pathdocmd

A "pre-med" concentration on your diploma or transcript will make little, if any, difference to admissions committees. The additional science course (except calculus) would make you a slightly stronger candidate (depending on the school) and help you when you do get to medical school.


Retaking the MCAT with a good GPA?

So I have a double major and double minor with a 3.8 GPA. I just got my MCAT score back and it was a 496. Please don't ask if I even studied for the MCAT. I spent around 9 months with it and did a lot of practice tests (12) and was scoring well (around 510). I guess I had an MCAT I wasn't prepared for and psyched myself out. I have my primary application in and have gotten a secondary already. My question is do I need to withdraw my application or try applying DO? I would love to be a DO but I unfortunately don't know much about their schools. I also have great extracurriculars and references from doctors. I don't know what happened and I'm very discouraged and really debating withdrawing completely. I also know it's a little late in the game to even retake and work full time which I think was also my problem with why I did so poorly.

tantacles

Trying DO is a good idea, but I doubt that you will be successful with a 496 MCAT. My suggestion is that you withdraw this cycle and plan to apply the next cycle. Applying your first time when you are truly ready is extremely important.


pre-physician assistant advice?

I am applying to physician assistant schools this fall. I am wondering if i should attempt, or wait until next year. Currently CASPA has calculated my overall GPA as 2.8 and my science as 2.5. I have over 1500 hours of direct patient care. Additionally, i have shadowed and am actively volunteering. I feel like i meet prerequisites for some schools, but i may not get in. Should i not waste my money on more applications?

tantacles

Most of the advisers on this forum are not PAs, but my general suggestion is that if your GPA is below that of the average PA, work to improve your GPA and apply next year.


Can I major in child physiatrist as a pre med student?

I'm a pred med student and I'm a little conflicted as to what I would like to major in. I know I have a passion for kids and working with kids as well as helping others. I was wondering if as a pre med student if I can major in as a child physicshiatrist and minor in psychology? Would that be a good idea? I would like some advice and some guide in the right direction and perhaps some ideas in which route I should go in regarding my passions. Thank you in advice.

tantacles

As a pre-medical student, you can choose any major as long as you fulfill the pre-medical requirements in addition to that major. What is important is that you do as well as you can in your classes as you possibly can. When you apply to medical school, you will be applying for a general medical education, and you will specialize after.


Doing a post bacc vs second bachelors

I plan on applying next cycle in May 2018. Currently it is almost September 2017. During this time what do you recommend I do to improve my application?

I have low undergrad GPA in B.S Biology of 2.58 but a better M.S GPA in Biomedical Sciences of 3.5. My MCAT is a 505. I know I should retake the MCAT and if I score better around510 will this suffice to be competitive by May 2018 cycle?

It is too late to do a post bacc as the time to apply for this has passed. Do you recommend I just focus on the MCAT or consider starting a second bachelors in Spring semester?

Thank you.

tantacles

With a GPA of 2.58, it is unlikely that you will be competitive for ANY MD or DO programs without some kind of post-baccalaureate program regardless of your MCAT score. I would suggest that you hold off on applying until at the very least the 2019 cycle and do a post-baccalaureate program, retake the MCAT, and improve your chances maximally before you even consider an application.


Committee Letter by only academic professors

I realize that I only have my academic professor to write me a letter of recommendation as part of my committee letter, is that a bad thing? I did some volunteering, but they were for a short period of times (3 months tops) for each one and after a while, I stop because I was commuting from home.

Mr.Smile12

Admissions committees usually are much more flexible when it comes to institutional letters, as long as your letter conforms to the requirements set by the prehealth advising office. So it's only a problem if your prehealth advisors/committee requires specific letters that they can include for you, but usually the admissions committees will defer to the advisors.


Need expert advise deciding DO vs SMP/MD pathway

I am a senior in college with a GPA and sGPA of 3.2 and an MCAT of 512. I have leadership/ mentoring activities for a year, team building activities, 200+ hours of shadowing, some research experience and an upward trend in GPA.

I was contemplating either going to a DO school with my stats or try for an SMP into an MD school pathway. I want to go to a DO school because it saves time and money (no SMP) but I have heard that its hard for DO's to get competitive residencies and thus mostly are in primary care. However, I also want to go to an MD school because it is the traditional pathway and will provide broader avenues for specialties.

What do you suggest for me to do?

Mr.Smile12

Only you have an idea of what you want to do as a physician and whether you want to spend the money to get to that goal. You should check out resources from AACOM about residency matching, especially since a unified match is going to happen (if it hasn't already). DO's can get into specialties.


Post bacc?

Hi everyone,

I want to apply to med school but I don't know if i should do a post bacc first. I am a rising senior.. My GPA is 3.2 and MCAT is 506. I am a research intern at a hospital. I've shadowed doctors in two of the top hospitals in India. I've volunteered with a local non profit for 3 years now.

I'm not sure if my stats are good enough to get in. I was thinking of doing a post bacc, but some of them require you not to have taken the MCAT. And since they're expensive, i wanted to apply to a good one like Bryn Mawr, Goucher, Drexel... I also don't know if my chances of getting into those are good? any suggestions??

tantacles

Your stats right now are acceptable for DO school but not great for MD. I would absolutely do a post-bacc to improve your chances. It is hard to say what the chances are of your getting into a particular post-bacc program, and I would suggest e-mailing those programs or looking at their websites to get a better idea of your chances.


Would it be a very wise idea to take all the pre requisite course work for the Carver college of medicine at a four year university?

I am attending a community college in Iowa to get all of my general education requirements for my major (Biology) out of the way! I've heard that many medical schools want students to take pre requisites at a four year university. My plan was to wait until I transferred to a four year university to start my pre requisite courses needed for medical school since courses at a four year institution are more rigorous. Also during these years I plan on job shadowing and volunteering at a local hospital. Do I have a good plan so far? What else can I do? I really want to attend medical school at the Carver college of medicine.

tantacles

My suggestion is that you take a maximum of 1 course at a community college and take the rest (if not all) at a four year university. The courses will be more rigorous at a four year college, and medical schools are aware of this.


Late First Time Applicant-Wait or Apply Now?

So I am a rising senior in at a 4- year undergraduate program (business management major with chemistry minor). I am part of the honors program and will defend a thesis before graduation next semester (Spring 2018). I've hovered around a 3.65-3.7 GPA during the first three years, have decent (not great) EC's, and 100+ hours of shadowing and clinical experience. I am taking the MCAT on August 18th. The score release date for me would be September 19th, which I have realized is very late in the app cycle. Would it be better to sit on my MCAT score (and retake if the score isn't great) and apply next cycle, or just go ahead and give it a shot now? Any advice would be appreciated.

tantacles

If you haven't submitted your primary application yet, I would suggest that you wait until next year. Being complete September 19th is pretty late, and it would put you at a significant disadvantage. Waiting a year would allow you to retake the MCAT if necessary and continue to prepare your application.


Picking College for Pre-Med

So I just recently moved to Arkansas, from Texas and found myself in a depressing situation. I am currently in high school, and still deciding what college to go to. In Texas, I am probably going to find myself paying 4x the college cost to go to a decent college (University of Houston). Or I could go to state school over here for alot cheaper of a price (University of Arkansas). So here is the dilemna, Arkansas only has one medical school, which includes pharmacy and nursing. Meanwhile, in Texas there is alot more medical school as well as dental and programs like JAMP. Both school accept 90% instate students. My family is currently in a tough finicial position and I have 2 years left to decide where to go. Anybody have any advices?

tantacles

If you go to college and do extremely well and do well on the MCAT, it will put you in a great position to go to any medical school and potentially get a scholarship. I would suggest that you go to a school that makes sense for you financially and makes it possible for you to do your best in undergraduate. It is hard to say what the best choice would be, but it is an intensely personal decision.


Class Retake A Negative On My App??

Finished my undergrad with an overall GPA of 3.7. At the beginning of my senior year it was at 3.43 or something close to that. I retook a couple of classes that I got Cs in to get As (Ecology, Biochem, Physics 2, and Nutrition for Health Professionals) and one class to get an A after getting a B the first time (which I realize retaking it now was a mistake). On my transcript from my school it shows that those original class grades were dropped and my GPA is not the 3.7. HOWEVER, my new concern is that I was told that those Cs that I got ARE going to be used when med schools look at my overall GPA, and that my overall GPA is going to take a substantial hit. While this is a over site of my own fault, wouldn't adcoms want to see significant improvement in grades for core classes in regards to my major, and an improvement in GPA? Or is this something that is seriously going to hinder my application? I know I should have done better my first time around but I had some personal things seriously hindering my performance, and now I'm afraid that one bad year of undergrad, and trying to correct for that year has ruined my chances. Any and all help would be appreciated.

tantacles

Medical schools have very different ways of viewing retakes, but in terms of the calculation of your GPA for MD school, both classes will be used for the calculation. This does not mean your chances are gone of getting into medical school. However, I would suggest you do the best you can on your MCAT and apply broadly to increase your chances of acceptance.


Should I buy MCAT books freshman year of undergrad to read?

I am a freshman pre-med and I want to buy mcat prep books to read before I start each of my pre-med classes. I plan on learning the information in the books before I start studying for the mcat around Junior or Senior year so it is easier on me. I want to score in the 100th percentile so I can get into top medical schools for surgery. Is it a good idea to do this or should I wait to study?

tantacles

My suggestion is that you do as well as you can in your science courses, as those are the basis for the MCAT, and study for the MCAT during a dedicated period of time after you have finished your courses.


Can I talk about how having the Eating Disorder, Bulimia Nervosa, has encouraged and solidified my interest in becoming a medical doctor?

Hello, guys. I am applying to medical school this cycle, but I am yet to turn in my primary application; I haven't written my personal statement.

I was wondering if I could talk about how suffering from the ED, Bulimia Nervosa, for 4 years has shaped me and piqued my interest in pursuing a career in the medical field. Would talking about my ED be an absolute NO NO?

Thanks.

tantacles

Your experience with an eating disorder, when presented in the right light, could be very beneficial to your application and provide great insight into your motivation for pursuing medicine.

More importantly, though, we are approaching a late time to apply in this cycle. If you have not submitted your primary application, your application will be late. If it will be more than 10-12 days before your application, which must be fully complete with a fully realized personal statement, it submitted, I would suggest waiting until next year to apply.


Will my GPA from ten years ago haunt me when I apply, even if I have turned things around completely?

I am an extremely non-traditional student who originally went to undergrad in 2006. I was very immature, had no guidance from family, and came from an underperforming high school. That being said, I stuck around and floundered for 3 years, and I eventually left with a final GPA of 1.85. I did not graduate, but it should be noted that these were all business classes and I attempted no science courses.

After coming home and reevaluating my life, I became an EMT. I went to work full-time for a busy urban EMS agency, enrolled in the local community college and graduated with an Associate's degree in Emergency Medical Science. I continued to work as a paramedic, did very well, and eventually moved up the ranks to become a critical care flight paramedic with a large research based teaching hospital. In this position I have trained staff, written protocols, participated in research, and I have also taught ACLS, PALS, and BLS/CPR in the hospital's life support education department.

I made the decision to return to school and get a bachelor's degree two years ago. I finished my B.S. with a 3.6 and have a science GPA of 3.8 including all of the medical school pre-reqs, all while continuing to work full time. My question is this: is it worth a shot to apply to med-school now, given my extreme character change? Or will my mistakes as a 20 year old keep medical school out of reach?

tantacles

Your GPA from 10 years ago will be on your AMCAS application and may get you screened out of some medical schools, but given the amount of time you've spent in between undergraduate #1 and #2, it's likely that many will willfully overlook your GPA as a 20-year-old. Schools often award reinvention, and you've done a great job of redeeming yourself.


Getting a second bachelors.

I have a very low undergrad GPA in B.S Biology of 2.58 but a better M.S GPA in Biomedical sciences of 3.5. My MCAT is a 505. I know I should retake the MCAT but would it also help to do a second bachelors perhaps in biochemistry or something along those lines? I understand that some schools will like the improved masters GPA but they may not appreciate or reject me based upon my low undergrad gpa?

tantacles

Your undergraduate GPA along with your MCAT will get you screened out at many schools. I would suggest doing an a la carte post-baccalaureate program to re-do your pre-med pre-reqs and get better grades to prove to medical schools that you can handle the material.


Should i study medicine in Uk?

I have completed my A level and i am 20 years old. Can somebody please tell me if i should apply in a UK university for medicine. It is very expensive to study there as an international students, i dont know if i will be able to pay all the loans once i become a doctor.

tantacles

If you wish to practice medicine in the UK, studying medicine in the UK is your best option. If you wish to practice medicine outside of the UK, I would suggest going to medical school in the country you wish to practice. Insofar as financial issues are concerned, it would be worthwhile to compare the tuition of UK medical schools with the tuition of the medical schools in the country you are also considering practicing in. It is quite difficult to advise you without knowing anything else about where you wish to apply.


Which Kind of Post-Bacc Program?

I'm currently a rising senior biology major at a top 10 college looking to enroll in a post-bacc program after I graduate. My dream program is Scripps but I'm not sure if I should be looking at career-changer programs because I was a biology major. Over the course of my major, I took one year of general chemistry with lab, one year of biology without lab and one general physics class. Am I eligible for career changer because I didn't complete the pre-med coursework? Or am I ineligible because I have taken some of the reqs? Thank you so much!

tantacles

If you are able to complete the pre-medical coursework with excellent grades, the number of pre-requisites you took during undergraduate will not be important. As far as individual programs and whether they will accept you, I would reach out to the programs themselves and find out what they are looking for in terms of applicants as it is impossible to predict what individual programs are looking for.


Decent GPA, but downwards trend, can I still get into medical school?

I had a really strong start then had a few falls. Year-by-year my stats look like this FR 4.0 cGPA, 4.0 BCPM SO 3.72 cGPA 3.63 BCPM (C in Orgo I) JR 3.3 cGPA, 3.2 BCPM (W in Orgo II and C in PhysII) so overall... 3.7 cGPA, 3.6 BCPM

Some other aspects of my application are the following: -I have several scholarships including a full ride merit scholarship -I've been a CNA for 2 years, and will be transferring to a highly respected hospital soon -40 hours of shadowing -1 year of Clinical research where I hold a leadership position and my name was on a poster presentation at a medical conference -300 hours of non-cinical volunteering -I was a lab aide/student researcher at a biology lab for 1 year -I am in the Honors program at my school where I will be doing a senior year research project/presentation I am a Latina/Hispanic girl

My senior year I hope to: get straight A's and go on a week-long medical missionary trip. I have NOT yet taken the MCAT. I will take a gap year after undergrad...any suggestions on what I should do? I am really worried about my downward trend in GPA. What are my chances?

casedentalmed

It is a bit challenging to look at someone with just numbers without additional context, and such advice is best given by someone such as an academic or prehealth advisor at your school. I don't know if you have taken a lot of strong upper-level biomedical science classes or if your struggles are with coursework related to having to work. I will admit that general downward GPA trends with increasing rigor is a concern because it doesn't get easier in medical school. You'll need to ask advisors at your school and in various admissions offices on suggestions.


Am I in over my head?

Hello, so I wanted to get some advice on a difficult situation I have found myself in. I graduated this last year with my bio degree with the hope of taking my MCAT at the end of the summer after full-time studying to get it over with before taking my 2 gap years. I had a lot of hope after making a studying plan, purchasing all my AAMC material, and signing up for the July 28th MCAT. I had 2 hitches though, I hadn't taken a year of physics yet and I'm a terrible procrastinator. I thought though that since I'd blocked out my whole summer, then I'd be more responsible and have no excuses plus I really needed to do well with my 3.2 sciGPA weighing on me.

Well, I failed myself. I couldn't stay on track and spent most days too lazy or overwhelmed to either start studying or reviewing what little I did accomplish for the day. I ended up rescheduling my MCAT for August 24th and redoing my study schedule. I am 3 and half weeks out from my MCAT and I still haven't gone past the first 2 chapters of most of my books and I don't know what to do. I have been in a depression this summer as I felt like I graduated with no sense of true calling into medicine or any field, and I feel like that has killed my motivation for the MCAT even though I want to try, I just can't get myself to. I thought I wanted to be a doctor but I constantly saw others around me with a passion and discipline for their chosen paths that I didn't have. Although I've done the whole pre-med classes, shadowing, and experiences that qualify me to apply next year, I didn't do it with the best grades or same level of reassurance for pursuing medicine, but I also I feel like by giving up now, I am not giving my 100% and would be walking away with a lot of regrets.

At this point though, I don't know whether to try to cram and take it in August or even early September or reschedule it for next January. I've wanted advice all summer as I saw myself slowing slipping farther and farther behind, but I felt like the more the summer passed, the deeper my shame and guilt grew as I had been telling everyone that I was studying when really I was paralyzed by my own fears and doubts. Now I feel trapped because everyone(family, friends, mentors, pre-med advisor) expects me to take it and do well since I was supposedly studying all summer and if I change my date to January, the whole summer and my registration money feels like a waste and I wouldn't know what to tell people. Do you see my dilemma? And to be honest, I don't know if I need MCAT advice or psychological help more :(

tantacles

I would suggest that you reschedule your exam for when you are truly ready. You will ideally not take the MCAT more than once. It does not matter what your family thinks; if you do not do well on the MCAT you will be unable to get into medical school. Change your plans if necessary and let the people you love know that your plan has changed.


Became Premed Student Beginning of Sophomore Year

My initial question is: should I have a gap year between college and medical school?

I am currently a sophomore in college. I changed my major from Nursing to Community Health with a premed track. My question about having the gap year between college and medical school is because of the prereqs that I have not yet taken.

I still need to take: one year of general chem and one year of organic chem, one of physics, microbiology, biochemistry, statistics, and genetics. Would it be possible for me to take these classes before I take the mcat next year (2018-2019 school year)? In addition to these classes, I have to take all of the classes that I need for my major.

Do you think that it would be possible for me to complete this entire list of classes before I take the mcat next year? If not, would it be helpful for me to take a gap year?

Another question that I have is, would it be wise for me to take the mcat before i have completed all of my prereqs?


casedentalmed

Your question would be best answered by an academic advisor and a health professions advisor at your school. Note that the MCAT may cover some items you might have to learn in the courses you plan to take, and at least the health professions advisor can be able to determine this to help you make decisions on sequencing your courses. I would not take the MCAT before you know you have coverage most of the topics that you would need a solid foundation for the exam.


Can I apply to US medical schools after my second year of undergraduate studies?

I have finished my second year of undergraduate (in Canada) and I am starting my third year in September. Many Canadian medical schools accept students after their third year. Can I apply this application cycle to US medical schools?

Some info about me: GPA 3.95, MCAT 515, lots of extracurricular and research


casedentalmed

This is a question that requires you to contact the schools that you specifically want to focus on. There are some schools that may explicitly state how many completed class hours you must have at minimum to be considered. The more critical issue though is whether the schools will consider you if you are not a US citizen (you don't say that you are a US citizen going to college in Canada).


I am a senior Health Science major and I'm wondering if my goal of medical school is unattainable at this point in my career?

I have a science GPA of around 3.6, however, I have had to re-take Organic Chemistry twice (because I had a baby in the middle of the semester) and my GPA is only around a 3.2 right now. I do have an internship on my resume as well. What are more ways I can improve my chances of being accepted into medical school? My undergraduate degree was more difficult than I initially thought going in, but I love it. I love medicine, and I have been working towards becoming a doctor my entire life. Does anyone have any tips to improve my chances of medical school? I'm taking the MCAT this fall, and I graduate in May 2018 with a bachelors in Health Science (pre-professional) and minor in chemistry. And if you could cater your tips to a full-time student mother, that would be great. I'm blessed with my husband being able to work for the both of us at this time, but I am at school for around 30 hours a week already. Thank you so much for reading this far! And I appreciate all recommendations. I've gotten some negative responses in the past because I have a 1 year old, and apparently 21 year-old mothers don't "make it" in medical school, and it is so discouraging.

casedentalmed

For advice on how to approach your application being a full-time student mother, you may be able to find someone within the American Medical Women's Association or American Medical Students Association (http://www.amsa.org/members/benefits/amwa/). I am hopeful you will find someone who can be more help than those that have responded to you negatively.


Acceptance rate with online degree

I am finishing my undergrad with a degree from Penn State World Campus (online degree) studying Biotechnology. I understand I will most likely have to take post bac classes to fulfill pre-med course requirements. On average how many students are accepted to medical schools with undergrad degrees from online universities? I am 32, I have been a firefighter/paramedic for 6 years and I am currently working as a paramedic for a remote community in Tahoe National Forest; I love being a paramedic but I have always wanted more knowledge and authority to treat patients. I am hoping my background will help balance the fact that I have an online degree.

casedentalmed

I hope you have been able to make contact with admissions officers at the schools you are looking at seriously. They should be able to let you know if any students with online degrees similar to yours have ever applied or been admitted. That said, if you know you have to take postbac classes, I would focus on making sure those are NOT done online and presented to you in an intense science-oriented schedule similar to what you would experience in medical school.


Can one go into surgery while having sjogren's- as in punctal plug treatment--> watery eyes--> the need to wipe watery eyes ? I just realized this could be a problem with the whole goggles/vision thing.

So sjogren's can be treated with punctal plugs, which cause excessive tearing. This hasn't been a problem working in a lab/ with wearing glasses, but it is necessary to wipe away excess tears to maintain vision. Would this be a problem in pursuing surgery?

tantacles

If you are able to do the work, you will be able to go into surgery. I would suggest that you look into this while you are on your surgery rotation as this will give you insight into whether surgery as a career will be possible.


Multiple research positions vs continuing in one

Do top research schools prefer students who have done like 3 years of research at one lab over someone who did 3 years of research over 3 different labs? Or would that be not that big of a concern?

Also, how are summer research projects looked upon? Is doing 3 summers of intense research and none during the school year considered bad?

casedentalmed

I'm not sure if you have talked to anyone in admissions or students at what you rank as a "top research school". I would suspect that there are many applicants with significant research experience applying to those schools. I'm sure there are plenty of applicants that have been in only one lab over two years and others in multiple lab rotations over a similar period. If it were me, that wouldn't be the key issue as opposed to being able to talk about what you did in each lab and how you were involved.


Schools/programs (MD/PhD) to apply (3.67 cGPA, 3.57 sGPA, 525 on MCAT)

I would be incredibly happy going to any medical school but I'm curious if I stand a chance at the top tier schools where my GPA is in the 10th percentile of the accepted pool. And if so, is there a way to know which schools would be more lenient on below average GPAs.

I'm really interested in pursuing research and definitely considering the MD/PhD path. But I see most MD/PhD candidates have incredibly high GPAs. So would there be a chance of acceptance if I chose the MD route instead.

I do realize that schools have a holistic view but I feel like that occurs after getting to the interview.

To give you some basic info.

I'm a computer science major. Struggled a bit in my freshman and sophomore year (mostly math courses). But my junior and senior year, I have mostly A's and 2 B's. I've taken 20 credit hours every semester since sophomore year (which I don't necessarily think makes all that much of a difference).

Extracurricular Activities/Etc: Research - 1.5 years of computational biology research, half a year of theoretical CS research (I also will be doing full-time research for the next year) Volunteering - 2 years volunteering at a hospital (~150 hours) Regular community service (~250 hours) Leadership - 3 officer positions and founded one healthcare related organization Founded a non-profit (very small) Shadowing (a few weeks) 2 computer science internships IM tennis

tantacles

You have a chance of acceptance going both the MD/PhD routes, but I would suggest that you apply to both, particularly given your GPA. You likely have some chance at the top tier schools, but it's extremely hard to say, and I would recommend you consider those schools "extra" schools that you add on after the fact.

Wedgedawg, a prominent SDN user and moderator, has created a system to determine where you're most likely to have a chance at acceptance. See below:

https://forums.studentdoctor.net/threads/wedgedawgs-applicant-rating-system-updated-jan-2017.1131149/

Once you've come up with a list of schools to apply to based on that system (I would choose 15 based on your GPA and MCAT score), add on some reach schools and then post in the What Are My Chances section of SDN with your full list so that we can give you some guidance about what schools to add and subtract.


Low gpa 2.7 Community College . Will I still have a chance for medical school?

When I started community college, I had family struggles which impacted my gpa. My parent aren't supportive and I was struggling from lack of resources to navigate college and poverty.

My current gpa is 2.7 from community college and looking forward to transfer in Fall 2018 for undergraduate college.

Will I still have a chance to go to medical school when I transfer to an undergraduates college to reset gpa or will my community gpa still count against me?

tantacles

You still have a chance to get into medical school. Your community college courses will still count, but many medical schools reward an upward trend in grades. You will have to improve your grades and do your best to get all A's in the future.


Retaking Gen Chem II Three times...?

I took Gen Chem II the first time during the spring semester earlier this year and withdrew because I wasn't doing well. I'm currently retaking gen chem 2 the second time this summer and i've had my past two exams given back to me... they've both been terrible grades. I think I won't be able to make it to a C. (We only have 2 more exams until the final grade is out.) So i'm reconsidering on retaking gen chem 2 again next spring semester if I fail this summer. Not to put any blame on anyone, but I came from a really bad high school so my math foundation wasn't so strong. I now realize that the reason why I'm not doing well is because I suck at math. I I feel so bad because I still haven't made it to organic chemistry yet. Is it even possible to become a doctor anymore? I'm thinking about physician assistant school as well... I don't know what else to do with my life.

tantacles

My suggestion is that if you are failing, withdraw from the course before you get a grade. It is still possible to become a doctor, but you have to assess your abilities and figure out if you're capable of completing the requirements given that general chemistry is the first of many.


Optional essay on secondary apps

How important is it to write an optional essay on secondary applications when you have no terrible things you need to explain to the admissions committee (ex. bad grades, poor MCAT, time off, etc.)

tantacles

If your GPA and MCAT are appropriate for the school and your essays for mandatory sections are solid, there is often no need to fill in optional portions of the applications. Those portions are optional, and it is not a trap.


Would an Emergency Medical Care undergrad degree put me at a disadvantage when it comes to med school admissions?

I live in a very rural area of my state. At most of the colleges I have toured, I have been interested in a degree in Exercise Science/Kinesiology. These schools have been either out-of-state or at the other side of the state. However, at the closest state school, they do not offer this degree. If I pursued a degree in Emergency Medical Care would that be looked down upon by medical school admissions?

tantacles

Any major is fine for medical school admissions. The most important thing is that you get the best grades possible and do well on the MCAT.


adding new schools after you have been verified by AMCAS

Hey guys. I have been verified by AMCAS for the past two weeks and i am still working on some of them. If i send out my primaries to new schools will i get secondaries right away or will it come in waves. So is it better to wait until i have gotten through some of these secondaries first or just send my primaries now?

tantacles

It depends on the school and how they screen. Many schools will send their secondaries right away. There is no reason to wait if you know you will be applying to more schools.


Help with aspiring Dermatologist?

I am currently about to start my senior year of highschool (2017-2018) class of 2018, and I sorta messed up my GPA. Science isn't my favorite subject, I'm better in History or English but I don't like careers dealing with those studies. Everything I want to do is medical which includes science. Science isn't my favorite but if I pay real good attention I can pass the class with an A- or B+. I currently plan to retake and make up science courses that I messed up by failing by doing Credit Recovery while taking my regular senior classes. Can I still get into a decent college? I want to get into dermatology and I know it's a hard career to get into, I've heard it. I want to go to a decent college and earn my bachelors majoring in Science. This probably sounds silly since it's not the strong suit of a subject for me but I'm so determined to be a dermatologist that people say I probably won't make it though medical school but thinking into the future and picturing myself as a dermatologist, puts so much motivation and focus into the subject of Science. Is this possible? Can I get into a decent college? Get into medical school? Or even get a spot in dermatology residency? I'm so motivated but also scared that I won't be able too due to my current highschool grades (that I am currently fixing up asap) taking the SAT, college admissions and all. I want to try and maybe be a dental assistant to get by outside of school since in my state I only need months of training. Can anyone give me advice? What courses to take in college? How I can fulfill a career as a dermatologist? Get into a good medical school? Also, I want to move into cosmetic dermatology working with ageing skin, chemical peels, microdermabrasion and such, work with acne and stuff. I'm also okay with warts and such and skin diseases and all the other stuff as well but more into cosmetic dermatology. Please please please help me out, I tend to overthink and think far into the future and stress myself out.

tantacles

The most important thing is that you get into a four year college and do your best in your pre-medical courses. There is no need to be a science major unless you wish to, as your major itself is unimportant. Your high school record disappears once you enter college and you will have a clean slate. It might be useful also to open your mind to other fields of medicine as you will have to do rotations in multiple medical fields and a year of internal medicine residency to be able to pursue a career as a dermatologist.

Your first step, though, is getting into college, and after that you need to get into medical school. I would suggest that you focus on those two pieces of the puzzle for now.


Emergency Physician vs. Flight nurse

I'm currently a Pre-Med student AEMT and I'm actually doing a bachelors degree in which I will obtain my paramedic cert. I love EMS and Emergency Medicine. Lately I've met a lot of awesome flight nurses and I've just kind of been thinking of that as an option but I don't know if it would fulfill my lust for knowledge in Emergency Medicine and I've always wanted to be an Emergency Physician. I'm just looking for Pros and Cons as well as words of wisdom on this subject.

tantacles

I would suggest that you seek out people in each of these fields. The big differences are that in an emergency medicine residency, you learn to treat patients of all sorts in a hospital. Similarly, in general nursing training, your training will be similarly broad. The main difference is that as the physician, you will be the boss of any interaction, while as a nurse, you will always be tethered to a physician.


I'm a Pre-med student EMT that's trying to decide between EM physician or flight nurse, I love emergency medicine, and I want to know if anyone has any words of wisdom on this topic?

I'm currently a Pre-Med student and I'm actually doing a bachelors degree in which I will obtain my paramedic cert. I love EMS and Emergency Medicine. Lately I've met a lot of awesome flight nurses and I've just kind of been thinking of that as an option but I don't know if it would fulfill my lust for knowledge in Emergency Medicine and I've always wanted to be an Emergency Physician. I'm just looking for Pros and Cons as well as words of wisdom on this subject.

tantacles

I would strongly suggest that you find some emergency physicians to talk with as this will likely inform your decision. Emergency medicine physicians and flight nurses have very different roles, and an emergency medicine physician could potentially work as a flight physician, so I would suggest that you seek advice directly from sources who have pursued these fields.


Should I do an SMP?

I have cGPA 3.50 and sGPA 3.41 with MCAT score of 516 (1 year old). I got into Tufts MBS program but I am still uncertain on whether this would help me. I would like any advise on whether this SMP is worth it for me. I personally want to get into mid-tier school (like George Washington). Is that possible? I didn't submit my application yet, but I'm also considering whether I should still apply. Pretty much I want to know what is the best way to spend this one year.

tantacles

You have a good GPA and an excellent MCAT score. I would suggest you use the Wedgedawg Applicant Rating System (below) to help you choose schools. You would likely not benefit from an SMP as you are situated well to get into medical school given your stats. I would suggest you get a research or medically related job and make money rather than wasting your money on a special masters' program.

https://forums.studentdoctor.net/threads/wedgedawgs-applicant-rating-system-updated-jan-2017.1131149/


How will a lopsided MCAT score affect MD/PhD admissions?

I got a 520 on the MCAT with a 125 in CARS. The were (132/125/132/131). Does anyone know how important the CARS score is in MD/PhD admissions? I don't know that low CARS score will be heavily frowned on...

casedentalmed

There are a lot of factors with the admissions process for the MD/PhD programs, and certainly having high MCAT scores is part of the review. That said, review will likely revolve around multiple items, and just having a low subscore on the MCAT in-and-of-itself won't be a problem if there are other items in your application that could address your critical thinking and reading skills more positively.


Will my MCAT Accommodation Documentation be Reconsidered by the AAMC in Time?

So I complained and appealed the AAMC accommodation department's original response rejecting my application for double time on the MCAT. I subsequently received time and a half. This was upsetting because I had to then decide if I wanted to spend $1250 out of pocket for a re-evaluation with my psychologist. I am an online MS student studying medical microbio/biochem. I am completely running on federal aid at this point. My psychologist performed my original evaluation seven years ago and I felt it was best to go back to her. The only problem is that she doesn't accept medicaid. I started my re-evaluation appointments simply when I started them and decided to spend the money. They began in early July. My documentation would have been ready for mailing on 7-19-17. However, in my excitement with "finishing" my appts. I looked over the AAMC approval letter once more and found that I completely forgot to bring the Request for Reconsideration, Evaluator Checklist and What Does My Evaluator Need to Do? forms to the attention of my psychologist. I can tell that my psychologist didn't look over the approval letter I printed out for her when I asked if she read it. I have been so thorough in reading everything but this one time I got my arse kicked by missing these forms. Thus, I have to schedule one last test to be completed on 8-2-17 and my documentation will be ready for mailing on 8-7-17. My psychologist is a booked up specialist on vacation this week. I rescheduled my test again from 8-24-17 --> 8-25-17 to 9-8-17 --> 9-9-17. The Silver Zone deadline by which a decision on my documentation must be reached is 8-25-17. I will FedEx overnight the documentation so that it is received on 8-8-17. This gives the AAMC 17 days out of a normal 30 re-review cycle to approve my documentation. The AAMC cannot state if this is enough time. Is this enough time? For the initial 60 day review, it took 37 days to get an answer. This sounds nuts but if we multiply this ratio toward 30 it equals ~19 days. This occurred during a less busy time. I submitted additional requested documents on 1-11-17 and was approved on 2-17-17. July-September are peak testing months and accommodation applications are reviewed in the order they are received. I cannot take the MCAT without double time. I am not going to allow schools to filter me out based on MCAT score when I was unable to perform my absolute best. I won't let AAMC win this ridiculous fiasco and I don't want to delay my application to 2018.

As a side note -- I'm quite annoyed by ETS and the AAMC. ETS gave me a horrible testing experience on the GRE at one of their testing locations. I had to fight a credit card dispute for a test fee refund. The AAMC ignores most of my questions in email responses and responds in a very vague, general and robotish manner. There is a form for EVERYTHING that you try to do with them. You have to submit a form just to get permission to do something else that requires a form to be filled out! All this takes time -- often up to 30 days per form. It is so darn hard to get accommodations on this exam. I understand they are making sure that only those who deserve them get them. Still, I have received accommodations for nearly a decade for college exams at two different universities and the GRE without hassle the first time around. The AAMC is going overkill big time. This is a deterrent from applying but biomedical research and clinical work in infectious diseases with an MD is my goal. I am not competitive enough for an MD/PhD program and I feel I'd be better off with an education in medicine over research skills lost without a PhD at this point. I want an education in medicine.

tantacles

I would suggest that you take the time that you need to have the best application possible. If that means delaying your application for a year, I would suggest that you do this. Getting extra time on your MCAT will be important, and you want to do your best.


Secondaries Question

I'm a premed in my freshman year of undergrad, and I'm just looking into the application process early. And I was wondering if there is a way to look at the secondary applications of primary schools that I want to apply to? And I mean an actual copy of it I can look at on my laptop. Or will they only be available in secondary threads on the premed forum?

casedentalmed

In general, most of the secondary prompts probably have been pretty stable over a set number of years except for a couple of schools. Regardless, many medical schools have their own application portals that they only send out on a limited basis, so I am not sure you will have much success accessing the secondary applications themselves unless those links are somehow public.


What should my plan be as a transfer and pre med?

I am doing an associates in Biology at a community college, starting the fall of 2017 and will complete it the spring of 2020. I am planning to transfer to a four year university thr Fall of 2020. When should I apply to trasnfer? When should I begin studying for the MCAT and when should I take the MCAT? When should I be working on a clinical/medical experience? I had a plan for all of this but now I am confused because I thought I would finish my associates earlier. Please help! I need a plan!!

casedentalmed

Success in the route to medical school relies on assistance from your staff advisors and faculty mentors. It's just as important to rely on their expertise in your journey as it is to get great grades. There should be many counselors who can give you insight into your coursework and your plans to transfer to a four-year institution, and to that effect, they should be able to help you with your plan.


What are examples of academic honors/awards that most secondaries ask for, and what advice would you give to a pre-medical student to enhance this section of the application?

I'm an incoming Sophomore in undergrad, and I'm just skimming through secondary apps for some of my prospective medical schools. One of the questions that is common to most secondaries (from the ones that I've read through) is a list of academic achievements and scholarships. This question just kind of threw me off guard and made me realize that I hadn't really received any awards/scholarships during my freshman year of college. I was just wondering if you could possibly list a few academic honors/types of awards that you received during undergrad and put on your secondary (if this applies to you).

tantacles

Academic achievements can be anything from presenting a poster at a conference to graduating with honors from college. There is no need to fill this section up with extra material. If you have not received any awards, typically you will not be hurt by this; many students are accepted to medical school without any awards.


AP credits when creating undergrad schedule?

Hi,

I am an incoming college freshman, starting undergrad. I recently received my AP scores and I am so happy I got a 5 on AP English Lit and Comp. I would love to totally transfer these credits and not have to deal with comp in college. However, I am a bit worried if medical schools will accept this? Do medical schools accept the AP cred from high school or do they require that you retake the course at your college? idk what the smartest thing to do is? Thank you

tantacles

Many medical schools may require you to take an advanced level course to cover your english requirement. I would suggest you check the websites of medical schools you plant to apply to to see if you should take 1-2 more English courses to fulfill their requirements. Every school is different, so this will be on a case by case basis.


Route change from PhD to MD late in the game. NEED HELP ASAP

Hello all ! So I was originally planning on doing a PhD program (microbiology) once I graduated. I have since graduated and while I was volunteering in a hospital in my down time, I had the privilege of shadowing a doctor that has both his MD and PhD. So I was able to go and see his lab work and also shadow him clinically with patients. I have since decided to change things up and pursue an MD instead, and couldn't be more excited about this transition! I have quite a bit to ask so bear with me!!

Hence forth, I need insight on the process. Does the volunteer work I'm doing in the hospital (patient transport, mailing work, paper work, news paper/flower delivery, and other delivery services) count as clinical or community service experience? Should I look for something more?

I graduated from my Biology Human Physiology emphasis program with overall GPA of 3.65, I'm currently in line to shadow some more doctors across multiple specialties, and take the MCAT in August. Since I wouldn't be applying for the start of this fall, I would be applying for next fall. Would a basic EMT course in between help or hinder me? Or was looking to do a masters of physiology or masters of medical science at Loyola.

Also, how does the cycle process work? like if I applied for next fall semester, would I have to apply sooner than later? or would applying next spring be enough? I'm new to all this so any and all input would be helpful!!

tantacles

Your volunteer experience is both volunteering and clinical experience.

It sounds like you are preparing yourself well for medical school. I would not suggest an EMT course; unless you plan to practice as an EMT, this course will not be helpful to you; EMT training does not prepare you for medical school and will not put you in a better place. Rather, I would suggest that you start getting research together, take the MCAT, and find a job where you will be involved in the medical field. That could be as an EMT, but it might also be in a doctor's office or research lab. There is no reason to do an extra master's degree when you could simply work for a year or two in a relevant field and make money while you do it.

It is almost certainly too late to apply for the fall semester of this year, so what I would suggest is that you take a year or two to strengthen your application, find an adviser, and look into what it will take you to apply to medical school. You may have missed some prerequisites in college, or you may need more relevant extracurriculars, and you definitely need to study for and take the MCAT, and all of these things take time and money. Take your time and only apply once.


Can I see still improve my grades?

I completed my second year of uni with a gpa of 2.0. I'm pretty disturbed as my grades went downhill. I got a C in gen chem 1/2, a D but it's a pass on my transcript in calc 2. I also didn't do well in retaking gen bio and organic 1 in my first two years. The first time I took gen bio I got a D and the second an F. I failed organic chem two times. I feel like idk what's the trick to get through my last two years of college by sailing through, but I don't know if I can make it to organic chem 2. However, I can make it to physics and mammalian biology. I feel like my transcript is ruined. Please share some insight.

tantacles

It is possible to improve your grades, and an upward trend will be looked upon well. Just keep in mind, though, that you will have to do exceptionally from this point on to have a chance of getting into medical school, and your MCAT score must be stellar.


Do schools look into institutional action

Do Medical schools look request to the the disciplinary record of all of the students they send offers to? In order to verify that a student was not lying on their application?

Could a student get away with not disclosing their institutional actions and not get caught? Or is the risk too high?

Please let me know, thanks!

Pathdocmd

95% of institutional actions are minor alcohol infractions. One is no big deal. You need to disclose any and all institutional actions and any criminal record, so matter how small you think they are or old they are. The schools WILL find out when they get transcripts and/or committee letters. AMCAS sends out reports to all medical schools if they find a problem. It is much to risky to try to hide it. If you get caught your chances of getting accepted, let alone an interview, are zero. If they find out after you are accepted, they will rescind your acceptance or if you somehow matriculate, kick you out.


Physics confusion!

I have the option to take a calculus based physics even though my program (molecular Biology B.A to Biomolecular Science M.S.) only requires algebra based. I heard from one of my friends that upper level Medical school will not consider applicants that haven't taken Calculus based physics. I was wondering if this was true or not?

tantacles

Whether a medical school will take someone who hasn't taken calc-based physics is extremely variable. I would suggest that you look on the websites of individual schools you are interested in to see if they specify.


Continue taking second series ochem?

I am in a weird situation. I have taken my ochem class at a community college but have heard many times that its looked down upon. I have graduated from a university with a BA in English and have already completed my Bio and Gen chem series and because of money situation decided to finish my prerequisites at a community college. My question is should I invest in going to a state university and dropping this class (class starts next month but resisted already) or continue the second series in this college? (also, I do plan on going to a post-bacc or doing a masters) Also, non-related but I'm also taking a calculus course and was wondering if it mattered where its taken since it's not really part of the prerequisites.

tantacles

Many schools may not take pre-requisites that come from a community college, but most are ok with other courses being taken at a community college. I would strongly suggest that any pre-requisites you take be at a four year college.


Overlooked Disciplinary Action

So during my first semester of freshman year I was caught in a room with alcohol and received a written warning for alcohol underage. I was not drinking or intoxicated but was in the room where the alcohol was. I was told that this would not appear on my record by the hall director since it was a warning. I am now receiving secondary applications from schools and have just become aware that the alcohol charge is reported on my disciplinary action record but not my transcript. I am wondering if you think I should just report it in secondary applications? Go back and call AMCAS since I said no to IA on the primary? Or not mention it at all? Any input would be appreciated.

tantacles

I would not report this in your secondary applications but rather e-mail schools to have them put a note in your file. Most schools will look over an alcohol infraction, but if it appears on your transcript or record and you have not been transparent, they may care more. Thus, I would contact the schools individually instead of writing anything in your secondary.


If I am going against an undergraduate school's anti-gay policies, how honest can/should I be about it with medical school I am applying too? How will it be seen?

I currently attend Brigham Young University. I came to terms with my sexuality during my junior year and had a change of religious belief away from the Mormon religion which runs the school. The school prohibits any kind of homosexual behavior including dating, holding hands, hugging etc... Not wanting to give up my research and take longer to graduate by transferring schools, I have been dating guys, but keeping it quiet. I have been open about my sexuality in my applications to medical school, and my transition to accepting myself and my sexuality is a big part of who I am and something I would like to include in my application. Not to mention schools ask about sexual orientation in secondaries. I am worried about how to approach this issue in secondaries and interviews. I would love to just be completely honest, but I am worried that violating the schools policies will be seen very negatively even if the policies aren't agreeable.

tantacles

About 99% of schools will mostly be on your side. I would recommend that you do whatever is most comfortable for you in terms of interviews and translate that to your primary. If this is something you'll want to talk about, it will most likely be well received. On the other hand, if this is something you're uncomfortable speaking frankly about, it would be poor form to mention something on your application that you didn't want to speak about as the application gives many interviewers a chance to get to know you before you come to visit their school, and they will likely want to ask about questions in your applications.


low gpa post bacc

im a junior with a very low gpa. most probably would go for a post bacc or smp after bachelors. i was researching on post bac/smp programs that has linkage with med school. what i was able to see was that most post bacs require you to be a career changer(no or few science courses completed). if you have completed all prerequisites you are not even eligible to apply to those programs. since i haven't completed all of my science prerequisites should i change my major to a non science / non premed major? thank you.

tantacles

Many places allow you do to do a la carte post baccs, where you take the courses and are not part of a formal linkage program. I would suggest this if you are too far along in your major to switch course. You will need to repair your GPA, and a linkage with a medical school is not necessary, if you can do very well in your postbacc.


Disclosure of Sealed Criminal Record on Secondary

Hello,

I was interested in applying to medical school, however, I have a sealed criminal record that will be 5 years old by the time I apply (2019-20). This won't show up on a state background check or most commercial (in the event it does I challenge it with the court order for sealing). The incident involved a childish, immature argument over shoes with my sister that escalated to calling the police. I was ultimately convicted of disorderly conduct (a fourth-degree misdemeanor) in 2014, before I formally started college.

Otherwise, I'm hoping to craft an exemplary application to show that I have since matured exponentially and grown into a better person after this turning point in my life. Hopefully, my credentials offset this black mark in my past that I fully own.

How should I approach answering the questions on secondaries with a sealed conviction?

Follow up accordingly, and thanks for taking the time to read!

casedentalmed

Full disclosure is always your best strategy, especially if the misdemeanor will ultimately get expunged from your record. Read the instructions carefully and be prepared to tell them the satisfactory minimum required for your disclosure.


AMCAS hours mistake in activities.

I messed up my hours on my activities sheet. I had a job that I miscalculated hours for saying I worked 1400 hours, but I really worked around 800. My question is how big of a deal is this? I want to be honest and I did NOT mean to falsify those hours. I didn't update it before I submitted. I called AMCAS and they told me I can't do anything about it and I'll possibly be able to adjust it with secondary applications. I'm worried because that is a lot of hours and I would want to check that if I was reviewing. My application is also not verified yet.

tantacles

I would not mention it unless it is brought up by an interviewer. If someone asks you about it, be honest and say you miscalculated. If not, you need not say anything.


Canadian, 3.83 GPA, 510 MCAT, help!!

~100h shadowing, one publication, Started Charity Organization Own a small transcription company and I do transcriptions for a family doc. Worked/Volunteered at a clinic and pharmacy for 4+ years

I would really like to go to a top tier American University or something like Albert Einstein. Aside from retaking the MCAT next year is there anything else I can do? (Special Masters Program etc.?) What should I do next year if I don't get into any schools?? (Masters etc.?) I am also Hispanic/Latina, Female. Struggled a bit in first year but consistently got higher GPA: 3.73 in first year to 3.83 in second and 3.93 in third year. The reason I'd like to apply to American schools is because I am Cuban so I would be able to become an American resident in a year of living there. Would this give me an advantage or is it based on me having a Canadian Undergraduate degree?? Thanks!! I'm a bit lost when it comes to American Universities

tantacles

I would suggest that if you are absolutely intent on going to a top tier university, you retake the MCAT. However, I would strongly suggest that you apply broadly and if you are accepted to ANY US MD school, you take the acceptance. All of them will grant you a fantastic degree and make you able to apply for residency. If you do well in medical school, you will set yourself in to go into any specialty.

You can still add schools at this point of your application, and if you are worried about getting in, you may wish to add some lower tier MD schools to prevent yourself from having to apply again; it is still early in the application cycle so you have a great chance of getting an acceptance, particularly given your stats.


Pre med req situation

I enrolled into a gen chem 1 course while I was at my previous school, and I had filled out a transfer into anther school that has prestigious biology program and made it. Before I received the acceptance letter from my new school, I enrolled at a CC to take gen chem to get ahead for the semester. While I was at orientation two weeks ago, I was told by my advisor they wouldn't accept my credits. So now I'm left with an option to withdraw and get my money back but at the visiting school I will have a W grade for a pre med req. is it possible that I won't ever have to mention this grade of a W to anyone ? Like med schools. I'm a stellar student with a 3.9 gpa. And this is so unfortunate, I literally feel my life coming to an end. Please get back to me I would love to hear your advice. Thank you

tantacles

Even if your school won't accept your credit, the credits may still be worthwhile. Your grade, while not accepted by your school, will definitely be accepted by AMCAS, and if you get an A, it will positively impact your GPA. I would avoid dramatics as your life definitely will not end over one course. If you get a W, your life still will not end, and you won't be heavily impacted. You are over-thinking the situation, and you don't need to worry about either scenario.


Will this ruin my chances of med school?

I am currently going into sophomore year with outstanding grades 3.82 gpa. During this past year at my college I wasn't happy with the biology department so I decided to fill out a transfer to a very well known biology program at another school. Since I didn't take chemistry at the college I attended as a freshman I decided to take it as a summer course at a CC. When the course started I received a letter saying I was accepted into the other school(yay), so I went to orientation and told my pre med advisor I'm taking chem at another school as a visiting student and she tells me they won't take my credits for the course. I'm doing well at the CC but I contacted their financial office and their willing to refund me 50% of tuition and leave with a W grade. I'm in a messed up situation and I don't know what to do. I am scared that this W will cost me med school.

tantacles

One W will not cost you medical school, and if you take this course, it will still be accepted by AMCAS.


What does it take to become a plastic surgeon? still considering what to specialize on.

I still don't know what to go for, I will be becoming a high school senior this august and after fixing some grades in the community college I will go into pre medical to then get started in medical school. Also how long would this take?

tantacles

I would suggest that you focus on getting into medical school first. That will be the most important step. I would suggest you do your best to get as close to an A as you can in all of your courses, and then study for the MCAT and do well. Along the way, make sure that you do plenty of volunteering. I would use SDN judiciously, particularly the forums, as there are many skilled and wise advisers there who are physicians and admissions committee representatives who I'm sure would be happy to advise you.


2 Gap Years or 1?

Currently debating whether it would be wise of me to take another gap year and would greatly appreciate any input.

Just graduated from a top 12 university, average cumulative GPA (3.6), below average science GPA (3.45), solid research experience, worked part-time during the entirety of school, solid ecs. However, my clinical experience is lacking (shadowed around 45 hours total 3 different physicians). I plan to work full-time in a hospital during my gap year after re-taking the MCAT (originally received a 29).

My question: given my stats aren't eye-opening and I lack clinical experience, do you recommend I take an extra year to apply so that I can include my full-time medical employment on my primary AMCAS next year, or will doing it for a few months prior to interviews (and continuing afterwards) be sufficient? Would of course rather apply this cycle, but am open to delaying if absolutely necessary! Thank you!

tantacles

I would recommend that you take the full year off. You want to have the best application possible and only apply once, and it sounds like you need an extra year to brush things up and be ready.


I took 2 years of community college classes while still in high school but ended up with an unsatisfactory GPA?

I took roughly 80 credits through a dual enrollment program at a local community college but I ended up getting a 3.0 (!) GPA. I wasn't really sure what career i wanted to go into until the end of my senior year in hs, but the damage has already been done. Now I am a college freshman starting fall 2017 with a direct acceptance into their biochem (bs) and performance music (bm) majors. How much will this impact my medical school application/acceptance? How can I compensate for the mishap?

tantacles

Medical schools understand that classes taken during high school might not go well. My suggestion is that you look forward and do your best in the courses you have going forward and do your best on the MCAT as well.


Looking for any medical programs across the U.S. Would extremely like help!

I see myself in the future doing emergency medicien. I'm currently trying to set my path towards becoming a E.R physcian (Trama doctor), and i'm hitting a couple of road blocks. First of which is that due to my age(17yrs), the programs in my area(Homestead FL), all pretty much require you too be 18. Which is fustrating to me because i really wanted to dedicate this summer to advancing my carrer and im two weeks in with absolutley nothing. I even have skipped partys and didnt hang out with friends just as a caution too not get tied up and prevent me missing any calls. So as a result of getting fed up with the situation of programs near home so i decided to branch out more. So i decided that i'll seek programs abroad. One caught my eye, the program is called "Gap medicien", and it was pretty much perfect in terms of what i was looking for, it had a emergecy medicien class, i would've been able to travel to republic dominica, and apprently this program allowed you too actually preform care under the guidence of the doctor, which i thought was really awsome, it was really hands on which i really appreciated, so being excitied i showed my parents. Who are definitely supportive and encouraging of what i want, but i ended up with a lecture from my dad about how dangerous dominica or any 3rd world country can be????. And that he wants me too be a more worldy person but it'll have to basicly be at his pace. So whatever i didnt fight him on it. We ended the discussion on the basis that it has to be in America, So being that im dissopointed with the results of my search and summer so far, i ask preferably doctors or anyone whom truley knowledgeable on the subject, would see this and be so nice and caring to donate their time too point me in the right direction being; if theirs any programs in your area that would fit my progress, it doesnt even have to be strictly emergency medicien but something hands on preferably. And that also goes for any internships or shadowings, i just want to get out there and take my first step already. I'm very eager to get started and thank you for the consideration too reply it is defiantly appreciated????. If anyone wants to talk more in depth about it then this post allows ill leave my snapchat too thoes who would ask. Thanks again.

casedentalmed

You have a lot of time before you should consider opportunities outside the US. Since you haven't even hit 18, you will still face some restrictions on what you are able to participate in, and even when you get into college, hands-on experience in a clinical setting is highly frowned upon until you actually get into medical school. I'm sure there are many outside-of-US mission trips organized by various faith-based groups you can be part of (as long as your adult guardian or parent goes with you).

Realize that health is just one of many needs that most people in a destitute position face. The more you learn what struggles they go through with their lives, the easier you can understand what you are able to do as a physician and what you cannot do. ER is not the glamorous life you may see in TV shows or movies. Watch a few documentaries like "The Waiting Room" (you can stream this) to really understand.


Burn out vs second guessing med school

Ever since I was 12, I've always wanted to become a physician. I truly felt it was my calling in life. I couldn't imagine doing anything else (not in a stubborn way; I always kept my Options open). Sadly as I grew older, becoming a doctor became part of who I was. Everyone would associate me with medicine. My dreams of being a physician were always met with sincere encouragement and support even from established doctors.


I graduated in 2016 and decided to take some time off. I took a job as a PCA in my college's cancer hospital which is really prestigious (HR placed me there I had no say).

I always knew oncology wasn't "my thing" but I decided to give it my all.However, as time went on, I became worn down. The HR assigned me to pre op so I had to be there at 5 Am every shift. I had to work 40 hours/week, 10 hour shifts, with one 45 min break. Bear in mind this was my first real job, and I went from being a full time student to working full time. I became extremely exhausted. I'd do my job well and would receive compliments from patients and nurses but inside I disliked it. Also like i said earlier I hated anything oncology. Although I had great empathy for the patients, I resented being in a cancer related environment where all the patients had cancer and nothing but it. I felt like if I had been placed in the heart hospital (I love anything cardiology) I would have enjoyed it better.

Basically I'm wondering if what I'm feeling is just burn out or if it's really second guessing. I grew up with an OB/GYN father And a RN mother so it's not like I had idealized views of medicine. I know what it takes. However I'm wondering if this is an accurate assessment bc my pca job is more along the lines of nursing than medicine. I basically did all the duties a nurse does (make beds, transport patients, clean) except for administering drugs and talking to the Dr. I always knew I wasn't cut out for nursing and this job confirmed this but I'm wondering if it's accurate to transfer this and conclude that I'm not cut out for medicine. On one hand I wonder if I can handle even more rigorous schooling (it would be the rigor that would stress me out not the actual content), but on the other side, I know plenty of physicians who hated medical school and residency who now love their jobs as attendings.


I also realized in my job that if I did become a physician I wouldn't want to do 24/7 patient care, meaning that I wouldn't always want to be around patients in the hospital. I've been considering the idea of conducting research so that my time would be split instead of constantly in the hospital all day. In wondering if i should consider that road.

I apologize for the length but I'm very torn here Ana would appreciate some advice. In case you were wondering I have all the research, volunteering,shadowing (all which I enjoyed) etc so I definitely don't need to do anything else like that.

Thank you!

tantacles

You are currently not doing the job of a physician. There are many fields in medicine where you are not always required to provide patient care, and you might consider one of these: Radiology and pathology are two examples. There are also positions in medicine where you can do substantial research and not spend all of your time in clinical practice. The options are as varied as you can imagine.

It sounds as if your job just isn't ideal and that's why you're second guessing yourself. Consider finding a new clinical position that suits you better. Consider a position outside of oncology.


Is June 25th still early to submit applications?

tantacles

Yes. June 25th is an early time to submit medical school applications.


My Cars score is lower than I would like.

My MCAT score is 132/128/131/128. Do you think my CARS score being unbalanced will negatively affect my chances of getting into a top tier school? My GPA is 3.94 and extracurriculars and research are pretty good to go.

tantacles

It seems like you have a great chance of getting into medical school. it is hard to stratify your chances without other information, but your GPA and MCAT score put you in the running for many fantastic schools.


I am a new graduate and need advise on how to maximize my gap year for mcat preparation and medical school admissions.

I just graduated with a BS in Chemistry (Biochemistry focus) from the United States. My cumulative GPA is 3.54 and my in -major GPA is 3.48. During my undergrad, I volunteered at a children's museum (1st year only) , was a club officer for two years , and have 1 year research experience in computational chemistry and 1 year in biochemistry. I was unable to gain clinical experience or do more volunteering in my senior year to due extenuating family circumstances which I plan to explain in my personal statement. I plan to take the MCAT between Jan-Mar 2018 and apply for the 2019 cycle. I am applying to MD and DO schools as well Canadian schools.

So, my question is what should my priorities be before the application opens to get into med school? At the moment, I am somewhat overwhelmed and would really appreciate any advise.

So far, I plan to prepare for the MCAT fully from the Start of September and begin shadowing/ volunteering at a hospital with it. I also lack work experience but do not want to overwhelm myself. Should I volunteer, work part time & prepare for MCATs? or Just do MCAT preparation & gain clinical experience until MCAT is over.

tantacles

My suggestion to you is that you do not try to explain away your lack of clinical experience; rather, you should take extra time to help you feel less overwhelmed and get the appropriate experience. This way, you can spend the time you need to take the MCAT and then work while doing some light but continuous volunteering for a year to prepare for medical school.

Medical school is a marathon and not a race, and I would suggest that you take the time you need to have the best application possible. That might mean applying in the 2020 cycle instead of the 2019 cycle.


Chances of getting into a PA program

Hello, i was wondering if i can get help regarding my chances of being accepted into a PA program. I messed up my first few semesters in college but stepped it up before i graduated. I really want to become a physician assistant. i planning on taking the GRE after July and will also be taking microbiology in the month of july (confidant ill get an A). I know i can score really good in the GRE, I dont care which PA program i get into, as long as i can get into one. Ill have my personal statement reviewed by professionals .

CHEM 1: W, retake B+ CHEM 2: C+ BIO 1: W, retake C BIO 2: D, retake B- Physics 1: W, retake A Physics 2: A- Organic Chemistry: F, W, D+, A Organic chemistry 2: B Biochemistry: C+ Anatomy and physiology: B Anatomy and physiology 2: B+

Resume: TA for organic chemistry 1 and 2 for a year Research associate for chemistry department for about 400 hours Volunteered in a hospital for about 350 hours (shadowed a nurse) Worked (currently) as a Medical assistant and scribe in a private practice for about 300 hours ( i work with a NP and Doctor)

tantacles

It is hard to determine admissions chances, particularly without a GRE score and a full GPA as many schools have GPA minimums. I would look at the specific requirements from schools to figure out your chances and speak to applicants from SDN so that you understand your chances better.

Consider also having people from SDN review your personal statements; professional personal statement reviewers are great, but may not necessarily be worth your money.


What else can I do to improve my chances of getting accepted? From a current Masters student

I received my B.S. in Biology (concentrating in Molecular and Cell Biology) at Cal Poly San Luis Obispo with just a 3.3 GPA. I was an RA (Resident Advisor) for two years, which kind of affected my grades more than it should have, but I also had some bad winter blues. I only did two semesters of undergrad research. I have just one more semester to complete of my Masters in Biomedical Sciences at Hebrew University in Israel, which is considered a very good university and research school. I will most likely finish with a 3.76 GPA, and there is a chance that my work will be published, but probably not before I finish my degree. However, at least I will have a nice thesis from work with a great PI. I also currently work part time for an Israeli startup that is a database for online research tools. Does my lack of a lot of things (such as a lot of undergrad research, a good GPA, a lot of volunteer hours and shadowing) completely ruin my chance of getting accepted into a medical school in the US? Or, if I do well on the MCAT, could I possible get in? I just want to know if I should do more things before I start studying for the MCAT or while I study for it?

tantacles

I would suggest that you work on remedying your lack of research, volunteer hours, and shadowing before you apply to medical school. I would still take the MCAT and do your best, but just realize that you may need to spend a year or two figuring out your extracurriculars before you apply. If you do well on the MCAT, you still have a good chance of getting in provided everything else is in order.


Already sent in application, but want to retake MCAT and resubmit score this cycle

I have already submitted my AMCAS application into schools (submitted early on June 2) and I just received me MCAT score back. However, it is not strong enough nor where I need to be. I plan to retake it early August... I am doing serious work in preparing differently (CARS killed my score!!!!!!!) Is it possible to let schools I am retaking it and submit my second score in time for this application cycle's consideration?

tantacles

It is possible to do this. Many schools will wait until they receive the second score if you tell them you are retaking. However, many schools may just evaluate you with what is on your application. This will likely be on a school-specific basis.


Research Experience on CASPA Application

On the CASPA Application (PA School), what exactly can you include as Research Experience?

The CASPA website doesn't clarify other than "preferrably outside of classroom work".

I'm wondering whether to include a Sociology research piece I did for my undergrad Senior Thesis. This was a project that was part of my classroom coursework, but I executed the research myself, and it was granted Distinction and published.

The reason I want to include it somewhere on my application is because it was related to the medical field (it was Medical Sociology research).

tantacles

If you did research, it is absolutely relevant to include on an application.


Chances of MD or DO w/ cGPA 4.1 sGPA 3.9 MCAT TBD

Hi Guys, I'm really having trouble deciding a couple of things. I hope you guys can provide some guidance. 1. My practice MCAT exams are pretty low. My real exam is coming close, june 17th. Should I just take it and risk the fact that I may have a low MCAT score on record? 2. Should I apply this cycle or wait until next cycle? I really wanted to make it on time for this cycle, but how are my chances looking... especially since my MCAT exam will not go so well.

NM Resident Vietnamese; Female

cGPA: 4.13 sGPA: 3.9 MCAT: 495 on practice exams. Real exam to be taken June 17, 2017

Medical Volunteering: - over 120 hours

Non-Medical Volunteering: - over 300 hours

Shadowing: - over 230 hrs MD/DO Radiology - 120 hrs MD Ortho - 15 hrs MD Emergency

Research (3 projects) - 2 Paid and One on my own -Consistency of MRI during right ventricular function -Cancer Center Overview -The incidence of Injury to the Brachial Plexus During Traditional vs Hawk Tackling.

Employment: - Radiology/Faculty Assistant at UNM Hospital - Teleradiology specialist

tantacles

Please wait to take the MCAT until you are ready. An MCAT score of 495 will tank your chances of getting into school. If you need more time, take it, do more studying, and do your best. It is impossible to stratify your chances without that score, but at a glance, your extracurriculars look good.


Which medical schools are known to send out secondaries before primary application is verified?

I have heard of some medical schools sending secondary applications to students before their primary application is even verified. Do you know schools, or have a list of schools that are known to do this?

tantacles

Many schools do this. That being said, until your primary application is verified, submitting the secondary application will serve little purpose given that almost no school will offer an interview without having a verified AMCAS application.

I think the best source for this information would be your colleagues on the SDN forums who are applying to these schools; most schools do not release this data.


Can I become an orthopedic plastic surgeon

Hi I was having trouble picking between ortho and plastics I really love them both so I saw something online about a orthopedic plastic surgeon and was wondering if this was true.

tantacles

If you wish to do both, you may be able to do both residencies. I would suggest that you speak with your adviser in medical school. If you are not in medical school yet, I would focus on gaining an acceptance to medical school.


What should my objective be on my resume as a student?

I want to gain clinical experience as well as research experience or a job related to what I like, I am Interested in anything related to neurology, neuroscience, Neuroradiology and diseases such as Alzheimer's

tantacles

You do not necessarily need to have an objective on your resume. If you feel you must put something, you can customize your resume based on the job you are applying to. For example, if you are applying to a neuroscience job, you can write "To gain experience in neuroscience research." You can use this same strategy for every job. be sure to customize your cover letter as well!


I was averaging 504 on my practice MCAT exams, I won't know my score until July 5th 2017, should I just submit now, or wait until I get my score? I felt good on all sections except for CARS.

I felt better than usual on CP, BB, and PSs, but I did not feel good on CARS, my average on CARS was a 124, and I am mostly applying to DO schools.

tantacles

I would suggest submitting with just one school, and then you can customize your school list based on how you do or choose to wait to apply until next year if your scores are too low to gain acceptance.


512 MCAT (124, 130, 127, 131). Will that C/P kill my app?

GPA 3.7, sGPA 3.5. I got As in Chemistry and Physics in college, but just bombed the MCAT section.

tantacles

It is possible that it will hurt your application, but if you apply broadly to many schools that fall within your MCAT range, you stand a good chance of getting interviews. I would suggest you purchase the MSAR to help you choose schools appropriately.


Institutional Action

I had made a prank as a sophomore (last year) where I posted a staff member's picture and phone number on a public intimacy website. I didn't know that this was an offense until I found out from the school that the staff member received nudes, and this caused her emotional distress. I was charged by the university with sexual harassment. I didn't know that I could get in trouble for such a stupid prank, and when I found out, I was shocked and just apologetic. I never meant to cause a person emotional distress. I thought it was merely a prank. I'm about to apply to medical school. How bad are my chances with this on my record? I understand I have to report it to AMCAs and a reflection on how this has influenced me for the better. I just want to know whether it's worth applying still or not.

tantacles

It is hard to say how much this will affect your record, but it is likely that it will negatively impact your chances of receiving an acceptance. I would suggest that if you apply this year, you focus on what you learned from the experience and how you grew, and express how sorry you are that you did this.


Coursework AMCAS

In the AMCAS handbook, it states:

"The most common reasons for a returned application include: failure to list coursework in chronological order." and "Enter courses in chronological order exactly as the appear on the official transcript of the school where they were originally attempted."

I have completed inputting my coursework, and I did it in the exact order that is on my transcript, but it still did not save in chronological order. I made sure to do so. The semesters themselves are organized, but the courses are not in exact chronological order. Will this be an issue, and if so, how can I fix it?

Thank you

tantacles

I would submit your AMCAS application and allow AMCAS to determine if your application is appropriate. If they return it, you will have your answer.


Non-science letter of recommendation

I participated in an elective program through the University of Tennessee College of Medicine that I was able to shadow physicians of many different specialties and I gained one hour of credit for each semester. The associate dean of the medical school who was over the program wrote me a letter of recommendation for my application. Will this count as my non-science since I received credit for it? Or do I need to ask a professor who I had a lecture with?

tantacles

I would try to find a professor with whom you actually took a course for your non-science letter. This letter, while it may be strong, will not count for much in terms of medical school applications because it shows little about your academic potential; shadowing is an activity where you do little but watch, and this letter, while it won't hurt your application, likely won't count towards that requirement.


Really, how important is undergraduate research?

So I have heard people say "do undergraduate research if you want to, if not, it's ok not to". But then I have also heard that it is an unwritten requirement at most schools these days. The MSAR indicates that at most schools, 80-90% of students have done some research, and this is even higher at the top ranked schools.

So basically, I have been working in a lab as a volunteer for a few weeks now, and I honestly don't enjoy it. My question is, would I be better off continuing with research this summer just to have a few months of it on my application, or would it be better for me to use this time to gain additional shadowing experience/volunteering (things that I do actually enjoy but already have a fair amount of). I find that the two options are mutually exclusive, as the research is taking up a large portion of my time along with class. So assuming I am not interested in going to a top 50 research ranked university, how would not having research experience affect my app? I have a solid GPA (~3.8) and assume I will get a solid MCAT score as well.

tantacles

At many institutions, research is not valued strongly. Clinical experience and service may be valued more. I would suggest that you look for schools whose missions are more focused on service to apply to as those schools would be more likely to review your application positively, particularly if you have an appropriate GPA and MCAT score. In addition, a few months of research will likely not be a significant positive factor in your application due to the short duration; it will likely count for little.

I will also recommend, though, that if you don't enjoy scientific research that you participate in scholarly work elsewhere; if you are able to find a position doing humanities or social sciences research, it may hold just as much weight.


Graduated Pharmacy student

I have completed my 6 year Pharmacy degree at Rutgers University and have graduate with a doctor of pharmacy. I want to continue to enhance my knowledge within the healthcare profession and apply for medical school (MD or DO). I graduated with a GPA of 3.066 from Rutgers University.

I am not sure how to calculate my science GPA but If i count my GPA for BIO, CHEM,ORGO, AND pHYSICS it is only 2.5.... I struggled my first 2 years of school. I have many other science classes in graduate years but i am not sure if they count.

I just wanted to know what path I should take. I have a lot of clinical and research experience of working in multidisciplinary teams in the hospital.

I still have to take the MCAT and I understand I have to score high.

What are my chances for a Post-bacc program for DO schools or even MD schools. any help would be appreciated.

tantacles

Mathematics courses apply as well for your science GPA. There are many calculators available online.

The competitiveness for post-bacc programs is hard to gauge, but you can also do a post-bacc a la carte at your state school if you are not accepted to a formal program, and you will likely need to participate in one for GPA repair. If you can correct your cumulative GPA and science GPA and do well on the MCAT, you will have a chance at medical school, though DO will likely be a safer bet.


More suited for MD/DO or PA?

Hi, 34 yr old male here that just graduated from Berkeley MCB program with a degree in cell and developmental biology with an emphasis in physiology and medical biology. I was a transfer from San Jose City College and previously attended North Idaho College for a year. I have calculated my BCPM (3.49), AO (3.27) and Cum (3.42) GPA as best possible using the AMCAS guidelines. I have not yet taken MCAT because I'm not sure if I am competitive with these scores. I did not do research at Cal, but I do have lots of clinical exp. working in hospital settings as a CNA (2000+ hrs) on the nursing resource team (AKA Float Pool) so I've worked in all departments: ED CCU PCU ICU GEN MED SURG REHAB ect. I have completed technical training and obtained certificates in Nurses Aid, Phlebotomy, and EKG tech. Can someone in the know tell me if I would be a candidate, if got a decent MCAT score? I'm also considering PA school. Would I be a better candidate for PA school vs Med school given my current background?

Best to all! and Thanks!

tantacles

Without an MCAT score, it is impossible to stratify you, but I would estimate that if you were to receive an MCAT score of 512 or above you would be competitive for MD and DO schools and 508 or above you would be competitive for DO schools. You would also be a candidate for PA school. I would suggest that you take the MCAT and purchase the MSAR, which will tell you the MCAT and GPA combinations for students at all US medical schools. Then, I would make a post in WAMC AFTER you have received your MCAT score for help building an appropriate school list.


Academic probation do I still have a chance to attend med school?

My story: my freshman year first semester, I started off really well I was doing on my work, handing it in on time, was getting the grades I wanted . Then I had got into a really bad car accident. This car accident caused me to stress and kind of lose focus on all of my work. I was not trying to lose focus on my schoolwork but it was something that just kind a happen the stress just took over my whole body I had to go to court it was just a big mess. During finals week of my first semester I thought I did good but then again I know I didn't because of how much stress I was in. I ended up failing I believe three classes and that caused my GPA to go down to a 1.0 when it was originally a 3.2. When I came back for the second semester of my freshman year I was placed on academic probation and I met all the requirements but I still had to back out of two of my classes because my GPA was so low I ended up going from a full-time student and taking six classes to going to a part-time student taking three classes. This semester my academic advisor had changed and he asked me what I wanted to be a major in? and I explained to him that I wanted to go to med school to be an anesthesiologist and things of that nature. He was just kind of putting me down telling me I couldn't be a doctor and I would have to work really hard which I already knew . I have never failed any class in my whole entire life and this was kind of depressing for me but I knew if I wanted to get where I wanted to go I would have to work my butt off. I made sure that I didn't let what he told me put me down so much because I know if I wanted to do something I could and it wasn't going to stop me. so I made sure that this semester I was working the hardest trying to get all A's and that unfortunately happen for one of my classes but I got a B in my other class and a C+ and my third class this raised my GPA to a 1.9. I'm not sure how other schools work but for my university in order to get off of academic probation you have to have a 2.0 and higher. I'm kind of upset about that I have a 1.9 because I'm literally like one point away from a 2.0. I registered for classes like biology and applied pre-Cal because I knew in my gut that I was going to get this 2.0. So I could take these classes but now that I am a continued academic probation student and going into my sophomore first semester I will not be able to be taking these classes which I'm kind upset about but I know that I can take other classes so that something that I am going to have to do until my sophomore second semester. I am planning and I am going to be working really really hard this semester and I will be taking winter classes after my sophomore for semester aswell because I really do want to get caught up and I am planning to transfer at the end of my sophomore year. so I just really really really want this for myself and I am determined to get into med school. I just was wondering if it would still be possible I know it's going to be really hard but is it going to be possible is basically my question ?

tantacles

If you are able to get all A's from now on and do extremely well on the MCAT, you may have a chance at medical school. Your academic record as it stands is troubling and you will not have success unless you show significant improvement. Right now, it will be an uphill battle. Do your best, and take time off to improve yourself if necessary.


Low GPA welp

Here are my stats!

Cumulative GPA: 2.9 Idk my science GPA yet

MCAT-- aiming for a high score

Founder and President of a non-profit organization to help developing countries with medical equipment

Held various leadership positions on cultural boards

On Dance team for 1 year

Research for 2.5-3 yrs working on my own project

Volunteered at hospital for 5 semesters

On APO for service hours besides the hospital hours

Do I have a chance at MD or DO or Post bacc?

tantacles

I would strongly suggest that you do a post-baccalaureate program (either formal or a la carte through a local university) to improve your GPA. Right now, you will have very little chance at admission to any MD or DO school regardless of your MCAT score.


Do schools screen based on unfulfilled prerequisite coursework?

I plan on fulfilling my biology requirement and possibly other courses that may help my application in the fall. Although I am enrolled at a community college, course registration for the fall has not opened yet, so I cannot guarantee I will be taking the class in August.

Should I still enter the class I intend to take on the AMCAS?

tantacles

If you are going to be taking the course in your next semester, you should enter the course on your AMCAS. If your pre-requisite coursework is not complete, some schools may interview you solely based on your prior coursework.